Sei sulla pagina 1di 215

!

1. AL AUSCULTAR A UN PACIENTE QUE SE QUEJA DE FATIGA PROGRESIVA Y


ORTOPNEA, ESCUCHA UN SOPLO HOLOSITÓLICO, SEGUIDO DE UN
DESDOBLAMIENTO AMPLIO DEL SEGUNDO TONO Y UN TERCER TONO BAJO TRAS EL
CIERRE DE LA VÁLVULA AÓRTICA. EL SOPLO LO ENCUENTRA IRRADIADO A LA AXILA.
LO MÁS PROBABLE ES QUE SE TRATE DE:

1. 1. ESTENOSIS PULMONAR.
2. 2. ESTENOSIS AÓRTICA.
3. 3. INSUFICIENCIA MITRAL.
4. 4. INSUFICIENCIA AÓRTICA.
Gráfico de respuestas
Comentario

La estenosis mitral y la insuficiencia aórtica producen un soplo diastólico. La estenosis aórtica, la


estenosis pulmonar, y la insuficiencia mitral producen un soplo sistólico. Los dos hechos que deben
hacer inclinarnos hacia la insuficiencia mitral serían la irradiación hacia la axila y el carácter
holosistólico del soplo. El desdoblamiento amplio del segundo ruido aparece frecuentemente por la
hipertensión pulmonar, y el tercer ruido puede indicar que la insuficiencia mitral es severa.(R3)

2. Masculinol de 25 años presenta máculas eritematosas con descamación en cuero


cabelludo, áreas superciliares y surcos nasogenianos desde hace 1 mes. Refiere prurito
sobre todo en cuero cabelludo. ¿Cuál sería su tratamiento tópico?

1. 1. Lindano.
2. 2. Mupirocina.
3. 3. Ketoconazol.
4. 4. Acido retinoico.
Gráfico de respuestas
Comentario

Se trata de un caso clínico muy sencillo. Recuerde siempre que debe fijarse en cómo describen la
lesión y su localización típica. En este caso son lesiones eritemato-descamativas (psoriasis, el
liquen ruber plano y la dermatitis seborreica como las más importantes). La distribución de las
lesiones sigue zonas del cuerpo con abundantes glándulas sebáceas: cuero cabelludo, región
centrofacial, línea media del tronco, axilas e ingles. Con todo esto, deberemos sospechar una
dermatitis seborreica, cuyo tratamiento puede hacerse con antifúngicos tópicos (opción 3) o con
corticoides tópicos suaves, aunque actualmente el tratamiento de elección es el pimecrolimus en
crema.(R3)

3. Recién nacido producto de un parto traumático presenta a la exploración física: reflejo


de Moro asimétrico, brazo izquierdo en aducción y rotación interna con el antebrazo en
pronación y extensión ¿Qué tipo de lesión presenta?

1. 1. Fractura de clavícula izquierda.


2. 2. Fractura de húmero izquierdo.
3. 3. Lesión de nervios C5 y C6.
4. 4. Lesión raquídea con hemiparesia izquierda.
Gráfico de respuestas
Comentario

!
!
!
!
Existen dos parálisis braquiales en el reción nacido.

1) ERB DUCHENNE: raices (C4) C5 y C6: brazo en aducción y rotación interna, reflejo de moro no
presente o asimétrico, reflejo de presión palmar presente y se puede asociar a parálisis frénica.

2) KLUPMKE: C7, C8, (T1): mano caida con reflejo de Moro presente y prensión palmar abolida, se
puede asociar a síndrome de Horner.(R3)

4. La ictericia fisiológica del recién nacido es:

1. 1. Una hiperblirrubinemia no conjugada.


2. 2. Es más común en aterminos que en preterminos.
3. 3. Es poco común en niños de raza negra.
4. 4. Generalmente asociado a anemia moderada y reticulocitosis.
Gráfico de respuestas
Comentario

La ictericia fisiológica del RN es a expensas de B. indirecta. Hay que tener en cuenta que cualquier
aumento de B. directa en el RN se debe considerar patológico y no fisiológico.(R1)

5. Respecto a la ictericia por lactancia materna es CIERTO que:

1. 1. Suele debutar entre el 5º y el 7º día.


2. 2. Debe suspenderse la lactancia materna de forma definitiva.
3. 3. Aparece en 1 de cada 50 niños.
4. 4. El tratamiento más indicado consiste en la administración de fenobarbital.
Gráfico de respuestas
Comentario

La ictericia por leche de madre o síndrome de Arias es una forma de hiperbilirrubinemia no


colestática que aparece en 1/200 RN alimentados al pecho, entre los días 5-7. Se puede perpetuar
hasta los días 20-30 de vida. No suele haber riesgo de kernicterus. Para tratarlo, basta con explicar
a la madre lo benigno de esta ictericia, y recomendarle que siga ofreciéndole el pecho a
pequeño.(R1)

6. ¿Cuál de los siguientes, no representa un beneficio de la cirugía ambulatoria?

1. 1. Beneficios psicológicos, en especial en niños.


2. 2. Menor infección de la herida.
3. 3. Un menor consumo de analgésicos.
4. 4. Reducción de la lista de espera.
Gráfico de respuestas
Comentario

Una pregunta sobre un aspecto muy particular de la cirugía, Se puede responder con sentido
común.

!
!
!
!
•! El régimen de ingreso es hostil para cualquier paciente, sobre todo en la infancia, por lo
que la respuesta 1 es correcta.
•! La forma más normal de transmisión de las infecciones nosocomiales son las manos del
personal sanitario, por lo que en casa serán menos probables (respuesta 2 correcta).
•! Lógicamente, si no es necesario ingresar al paciente, la lista de espera será menor, ya que
no existirá el factor limitante de la disponibilidad de camas (respuesta 4 correcta).

Sin embargo, la pauta analgésica no tiene por qué ser distinta, por lo que la opción correcta es la
3.(R3)

7. Una estudiante universitaria de 19 años acude a la consulta acompañada por sus


padres refiriendo sentirse en los últimos dos meses progresivamente más asténica, con
pérdida de apetito, de peso y con mayores dificultades para concentrarse en los estudios.
En la anamnesis también destaca que ha perdido interés en salir con las amigas, presenta
ideas de muerte sin ideación autolítica y cogniciones pesimistas de futuro. Su peso es el
90% del considerado ideal por edad y género. No presenta fobia ponderal, ni distorsión
de la imagen corporal. El diagnóstico más adecuado es:

1. 1. Anorexia nerviosa.
2. 2. Trastorno límite de la personalidad.
3. 3. Distimia.
4. 4. Trastorno depresivo mayor.
Gráfico de respuestas
Comentario

Un caso clínico fácil. Una chica de 19 años lleva dos meses con astenia, falta de apetito (ha
perdido peso), problemas cognitivos (no se concentra al estudiar), pérdida del interés en las
relaciones sociales, ideas de muerte y pensamientos pesimistas hacia su futuro. Expresamente,
nos dicen que su peso está dentro de unos límites razonables, que no presenta distorsión de su
imagen corporal y que no desea perder peso, con lo que descartamos anorexia. La duración
recortada (2 meses) nos hace descartar distimia y trastorno de la personalidad, y en las descripción
clínica no cuentan ningún síntoma ansioso, por lo que asumimos que se trata de un síndrome
depresivo.

Según el modelo de clasificación actual, un síndrome depresivo de intensidad relevante y duración


superior a dos semanas es un EPISODIO DEPRESIVO MAYOR. Aunque la DSM y la CIE difieren
ligeramente en su forma de denominar a los pacientes que sufren su primer episodio depresivo,
parece sencillo aceptar que la opción TRASTORNO DEPRESIVO MAYOR es la más ajustada.

La DSM clasifica como trastorno depresivo mayor a los pacientes que sólo han tenido un episodio
depresivo, y como trastorno bipolar a los pacientes que sólo han tenido un episodio maníaco. La
CIE 10 prefiere considerar a estos pacientes como episodios y reservar el término trastorno para
los pacientes con dos o más episodios afectivos.(R4)

8. A los 8 años de realizar una nefrectomía radical a un paciente por un adenocarcinoma


renal, se detecta un nódulo pulmonar en el lóbulo superior derecho, confirmando el
mismo origen celular tras punción del mismo. El resto del rastreo en busca de metástasis
es negativo. La actitud terapéutica más aconsejable sería:

1. 1. Lobectomía pulmonar o nodulectomía.


2. 2. Radioterapia y cirugía posteriormente.

!
!
!
!
3. 3. Interleukina-2 y vinblastina.
4. 4. Bloqueo androgénico completo y progestágenos.
Gráfico de respuestas
Comentario
El adenocarcinoma renal tiene la particularidad de poder causar metástasis incluso 20 años
después del diagnóstico. Ante la aparición de una metástasis única esta recomendado claramente
el tratamiento quirúrgico de la misma. Además parece aumentar la supervivencia, especialmente
cuanto mayor es el tiempo transcurrido entre la extirpación del tumor primario y la aparición de las
metástasis.(R1)

9. What is the intensely pruritic skin disorder that presents with grouped vesicles with
cutaneous IgA deposits and is associated with gluten-sensitive enteropathy?

1. 1. Seborrheic dermatitis.
2. 2. Hailey-Hailey disease.
3. 3. Atopic dermatitis.
4. 4. Dermatitis herpetiformis.
Gráfico de respuestas
Comentario

Nos comentan las características clínicas (vesículas agrupadas, intensamente pruriginosas,


localizadas en superficies de extensión, como codos, rodillas y región lumbar) y las características
histológicas (ampolla subepidérmica, con microabscesos de polimorfonucleares en las papilas
dérmicas, en la IFD aparecen depósitos granulares de IgA +/- C3 en el vértice de las papilas
dérmicas). Pero lo que nos da la clave en esta pregunta para diferenciarla del resto de
enfermedades ampollosas es la enteropatía por sensibilidad al gluten, que habitualmente es
asintomática.

El tratamiento se apoya en dieta exenta de gluten y sulfona. La dermatitis seborreica y la dermatitis


atópica no plantean duda (respuestas 1 y 3 incorrectas).

La enfermedad de Haley-Haley, o pénfigo benigno familiar, es una genodermatosis autosómica


dominante caracterizada por defecto en el cemento intercelular, de manera que aparecen vesículas
recidivantes que dejan erosiones y costras en áreas de roce como axilas o caras laterales de cuello
(respuesta 2 incorrecta).(R4)

10. Las bacterias del género Vibrio, presentan las siguientes características, EXCEPTO:

1. 1. Móviles.
2. 2. Bacilos gram-negativos curvados.
3. 3. Algunas especies necesitan sal para crecer.
4. 4. Oxidasa negativa.
Gráfico de respuestas
Comentario

Los aspectos microbiológicos de los agentes implicados en las gastroenteritis no son un tema muy
preguntado en el ENARM. A modo de "curiosidad" comentamos esta pregunta; los vibrios son
bacilos gramnegativos, curvos, móviles, anaerobios facultativos, algunos de los cuales crecen en
medios con NaCl (halófilos). Lo que sí que nos tiene que quedar claro es que ciertos vibrios, como
el Vibrio cholerae, son productores de enterotoxinas, las cuales actúan a nivel de la superficie de

!
!
!
!
los enterocitos sin destrucción de la mucosa, dando lugar a una diarrea acuosa, sin productos
patológicos y sin leucocitos al analizar las heces con el microscopio. La opción falsa es la 4, ya que
no es característica propia de los vibrios el ser oxidasa negativos.(R4)

11. En referencia a la endocarditis en drogadictos intravenosos NO es correcto:

1. 1. El germen más frecuente es Estafilococo aureus.


2. 2. Cursa con ausencia de soplo, habitualmente.
3. 3. Entre los bacilos gramnegativos, el más frecuente es E. coli.
4. 4. La válvula más frecuentemente afecta es la tricúspide.
Gráfico de respuestas
Comentario

La microorganismos de las drogas más frecuentes en adictos a drogas vía parenteral (ADVP) son
los gérmenes de la piel, y menos frecuentemente que la infección provenga de las drogas. El
germen más frecuente es el S. aureus (opción 1), con 60% de los casos, después estreptococos y
enterococos (20%). De los gramnegativos (6%) el más habitual es la Pseudomonas (opción 3). Es
frecuente la infección por múltiples agentes. La válvula más frecuentemente afectada es la
tricúspide (opción 4), si lo razona verás qué fácil: si la fuente de infección viene de la piel, la válvula
a la que primero llega es ésta, que por cierto al ser afectada no suele soplar por lo que es una
endocarditis en la que no hay soplo (opción 2), salvo si se afectan otras válvulas.(R3)

12. Una mujer de 31 años ha sido diagnosticada recientemente de infección por VIH. Está
asintomática. Acude a la consulta con una prueba de PPD de 11 mm de diámetro, un
recuento de 550 linfocitos CD4+/mm 3 y una carga viral de 80,000 copias/ml. ¿Qué
medidas deben recomendarse?

Tratamiento de infección tuberculosa latente, profilaxis primaria de Pneumocystis jiroveci


1. 1.
y tratamiento antirretroviral.
Tratamiento de infección tuberculosa latente y profilaxis primaria de P. jiroveci. No tiene
2. 2.
criterios de inicio de tratamiento antirretroviral.
Tratamiento de infección tuberculosa latente. No tiene criterios de inicio de profilaxis de
3. 3.
P. jiroveci ni de tratamiento antirretroviral.
4. 4. Seguimiento sin ninguna intervención específica en este momento.
Gráfico de respuestas
Comentario

Una pregunta muy interesante. Para resolverla debemos saber qué hacer respecto a tres
circunstancias: una prueba de PPD positiva, criterios de tratamiento antirretroviral y cuándo estaría
indicado realizar profilaxis frente a P. jiroveci. Veamos una a una:

•! Respecto a la tuberculosis, en todo paciente VIH con PPD positivo, independientemente de


su edad, estaría indicado utilizar quimioprofilaxis con isoniacida (duración entre 9 y 12
meses, según autores).
•! El tratamiento antirretroviral, en el caso de esta paciente, no estaría indicado por el
momento. Actualmente, existen cuatro razones para emplearlo: embarazada VIH+,
exposición accidental al VIH, pacientes con recuento inferior a 350 CD4 y pacientes
sintomáticos.
•! Respecto al Pneumocystis, la indicación para establecer profilaxis primaria sería una cifra
de CD4 inferior a 200.

!
!
!
!
Por tanto, la respuesta correcta es la 3.(R3)

13. Una niña de 3 años con SIDA desarrolla un cuadro de fiebre, sudoración nocturna,
anorexia, pérdida de peso y linfadenopatías generalizadas. Continúa con la profilaxis de
TMP-SMX; el recuento de células CD4 es de 50/mm3. La causa más probable de la
enfermedad es:

1. 1. Neumonía por Pneumocystis jirovecii.


2. 2. Sepsis por Staphylococcus aureus.
3. 3. Infección diseminada por complejo Mycobacterium avium.
4. 4. Retinitis por CMV.
Gráfico de respuestas
Comentario

La clínica nos haría sospechar un cuadro similar a la tuberculosis y dado el grado de


inmunosupresión de la paciente deberíamos pensar en la respuesta número 3.(R3)

14. En una paciente de 65 años. ¿Dónde le buscaría un NIC o cáncer de cérvix?

1. 1. En el exocérvix.
2. 2. En la zona interna del cérvix.
3. 3. En la horquilla vulvar.
4. 4. En el fondo de saco vaginal.
Gráfico de respuestas
Comentario

El cáncer de cérvix suele asentar en la zona de transición entre el exocérvix y el endocérvix, por
este motivo debemos buscar las lesiones en la zona interna del cérvix. Una colposcopia solo será
satisfactoria si logra ver la zona de transición en el orificio cervical externo.(R2)

15. Un paciente de 56 años es visto por episodios frecuentes de fibrilación auricular


paroxística. No toma tratamiento habitual ni tiene otras enfermedades asociadas. Los
exámenes de laboratorio son rigurosamente normal. Se realiza un ecocardiograma que
muestra un corazón estructuralmente normal. ¿Cuál de las siguientes medidas
terapéuticas aconsejaría inicialmente?:

1. 1. Amiodarona.
2. 2. Ablación percutánea con catéter de las venas pulmonares.
3. 3. Ablación quirúrgica mediante técnica de MAZE.
4. 4. Propafenona.
Gráfico de respuestas
Comentario

En un paciente joven, sin cardiopatía estructural y con FA paroxística habitualmente se inicia un


tratamiento para control del ritmo cardiaco. Entre las diferentes opciones para conseguir esto son
de elección los antiarrítmicos de la clase Ic (flecainida y propafenona). Como segunda línea de
tratamiento estarían la amiodarona, el dofetilide y la ablación percutánea. La ablación quirúrgica se
suele realizar únicamente en aquellos casos que ya tengamos que intervenir al paciente del
corazón por otra causa (por ejemplo una valvulopatía).(R4)

!
!
!
!
16. El manejo de un recién nacido con hipoglicemia sintomática es:

1. 1. Dextrosa 10% VO 10 mg / kg / dosis.


2. 2. Formula artificial del 1er semestre 80 ml / kg / dosis dividido cada 3 horas.
3. 3. Dextrosa al 10% 2 ml / kg vía IV.
4. 4. Proporcionar un dextrosa al 5% de 5mg / kg / minuto.
Gráfico de respuestas
Comentario

En casos de hipoglicemia en RN el tratamiento más indicado es solución glucosada al 10% y


administrar 2 mL/kg vía IV. Respuesta 3 correcta. Cuidado con la respuesta 1, ya que detecte que
la vía de administración es VO.(R3)

17. Femenino de
32 años, gestante de 10.3 semanas, que acude a urgencias por sangrado genital menor
que regla de varios días de evolución y que actualmente es muy escaso. Presenta
también molestias abdominales tipo dismenorrea que han ido aumentando los últimos
días, a pesar del tratamiento con paracetamol oral que le prescribió su médico de
Atención Primaria y mal estado general. Antecedentes personales: sin interés.
Antecedentes gineco-obstétricos: menarquia 13 años, primigesta, usuaria de
anticonceptivos orales hasta hace 5 meses. Exploración general: TA 93/65, FC 99 lpm, Tª
36.7ºC. Consciente, orientada y colaboradora. Palidez cutáneo-mucosa. En la exploración
ginecológica se observan genitales externos y vagina con restos hemáticos oscuros en
cantidad escasa, cérvix de nulípara con eritroplasia periorificial y sin sangrado activo en
el momento actual, útero aumentado unas 8-9 semanas, dolor a la movilización cervical
y ambos anejos no se palpan aumentados, aunque el derecho es doloroso. El abdomen
está blando y depresible salvo por la FID, donde el signo de Blumberg es positivo.
Biometría hemática: Hb 9.1g/dl; Htco 29%; VCM 78 fl; HCM 25 pg; plaquetas 299,000/mm3,
leucocitos 12,579/mm3 (77% neutrófilos, resto de la fórmula normal). Coagulación:
normal salvo fibrinógeno > 500mg/dl. Al realizarle un USG pélvico vía vaginal, usted
observa la imagen que se muestra. Con todos estos datos, señale la respuesta
CORRECTA:

La paciente necesita valoración urgente mediante laparotomía urgente en conjunto con


1. 1.
cirugía general, ya que la paciente presenta un abdomen agudo quirúrgico.
La paciente necesita valoración urgente por cirugía general, ya que si se descarta
2. 2. apendicitis aguda, el diagnostico de sospecha más probable es enfermedad inflamatoria
pélvica.

!
!
!
!
La paciente necesita la realización de una laparoscopía (que será diagnostica y terapéutica)
3. 3.
y, según los hallazgos, se decidirá la técnica a aplicar sobre la trompa derecha.
Realizaría un legrado evacuador, ya que no se han expulsado todos los restos abortivos
4. 4.
intrauterinos.
Gráfico de respuestas
Comentario

Las imágenes del USG nos muestran un útero vacío rodeado de líquido libre, junto con una
formación anexial compatible con vesícula gestacional por lo que el diagnóstico es de gestación
ectópica tubárica complicada. El resto de los datos clínicos y de laboratorio que nos aporta el caso
clínico son compatibles con este diagnóstico. En el diagnóstico de gestación ectópica es útil la
titulación de HCG pero fundamentalmente para aquellas pacientes en las que la ecografía no nos
permite hacer un diagnóstico definitivo. Recuerda que esta HCG tendrá valores inferiores y con un
ritmo de crecimiento menor que en el embarazo intrauterino.(R3)

18. En relación con la paciente del caso clínico anterior, señale la opción terapéutica de
primera elección:

Laparotomía urgente: apendicetomía y anexectomía si el ovario torsionado no recupera su


1. 1.
vitalidad tras 30 minutos de detorsión intraoperatoria.
Ingreso hospitalario, ayunas y suplementos de hierro oral. Administración de clindamicina
2. 2.
y gentamicina intravenosa hasta que la paciente permanezca afebril 48 horas.
Si en la titulación seriada de B-HCG presenta un crecimiento escaso, después de
3. 3. comprobar la normalidad de las enzimas hepáticas y creatinina, se le ofrecería tratamiento
con metotrexate como primera opción terapéutica.
Laparoscopía urgente: según el estado de la trompa, se realizaría salpingostomía líneal con
4. 4.
aspiración de contenido ovular o salpinquectomía total.
Gráfico de respuestas
Comentario

Al tratarse de un embarazo ectópico debemos elegir entre 3 opciones terapéuticas: observación,


tratamiento con metotrexate o cirugía. Para elegir tratamiento con metotrexate la paciente debe
reunir una serie de características como: ectópico menor 4 cm, títulos de HCG bajos o en descenso
y ausencia de líquido libre abdominal y que le puedes dar seguimiento estrecho. Si se fija en el
USG se visualiza líquido libre por lo que debemos desestimar el tratamiento con metotrexate,
eligiendo el tratamiento quirúrgico. La cirugía fundamentalmente se realiza por vía laparoscópica
siempre que la paciente esté hemodinámicamente estable como es nuestro caso.(R4)

19. En relación a los test de bienestar fetal ante parto, es FALSO que:

La amnioscopia está actualmente cuestionada, ya que no está clara la repercusión del


1. 1. hallazgo accidental de meconio en el líquido amniótico y además es una prueba no exenta
de complicaciones.
El RCTG no estresante se basa en la premisa de que la frecuencia cardiaca del feto no
2. 2. acidótico y neurológicamente íntegro, reaccionará con aceleraciones transitorias a los
movimientos fetales.
La prueba de Pose valora la respuesta de la frecuencia cardíaca fetal al estrés simulado por
3. 3.
el trabajo de parto, siendo una prueba muy sensible pero poco específica.
La estimulación vibroacústica consigue disminuir la tasa de patrones no reactivos, la
4. 4.
duración del test no estresante y la frecuencia de utilización de la prueba estresante.
Gráfico de respuestas
Comentario

!
!
!
!
La amnioscopia está actualmente cuestionada, ya que no está clara la repercusión del hallazgo
accidental de meconio en el líquido amniótico y además es una prueba no exenta de
complicaciones. Aunque en general la aparición de meconio depende de la edad gestacional y
refleja una madurez neurológica e intestinal, en algunas circunstancias podría ser manifestación de
un compromiso fetal.

El RCTG no estresante se basa en la premisa de que la frecuencia cardiaca del feto no acidótico y
neurológicamente íntegro, reaccionará con aceleraciones transitorias a los movimientos fetales.
Aporta un margen de seguridad de una semana, siendo el periodo de mayor estabilidad de las
variables analizadas, el de 48 horas.

La prueba de Pose valora la respuesta de la frecuencia cardíaca fetal al estrés simulado por el
trabajo de parto, siendo una prueba muy específica pero poco sensible. Tiene un valor predictivo
negativo del 99.8% y un valor predictivo positivo del 8.7- 14.9%

La estimulación vibroacústica consigue disminuir la tasa de patrones no reactivos, la duración del


test no estresante y la frecuencia de utilización de la prueba estresante. Se considera que es un
método que mejora la eficacia de la monitorización fetal no estresante sin modificar su valor
predictivo. El perfil biofísico fetal nos da información sobre el compromiso fetal crónico y traduce
integridad de la función cerebral fetal, lo que prueba la ausencia de hipoxemia sistémica.(R3)

20. En relación a las taquicardias supraventriculaes, NO es cierto:

Si un paciente ha presentado un episodio, es muy raro que vuelva a aparecer otro en el


1. 1.
seguimiento.
2. 2. Las reentradas más frecuentes son las intranodales.
3. 3. Tienen un inicio y fin de forma brusca.
4. 4. En algún caso se puede presentar poliuria.
Gráfico de respuestas
Comentario

La respuesta falsa es claramente la 1. Las taquicardias supraventriculares de forma frecuente


recurren durante el seguimiento. Suele manifestarse en forma de palpitaciones, cefalea, dolor
torácico o disnea. Otros síntomas como el síncope, son poco frecuentes en esta entidad, siendo
más frecuentes en las arritmias ventriculares. Las reentradas más frecunetes son las intranodales
(aproximadamente un 60%), seguidas por las reentradas aurículo-venticulares. Las reentradas
suelen desencadenarse de foma brusca precipitadas por una extrasístoles auricular o venticular, a
diferencia de la taquicardia sinusal, que se inicia de forma progresiva. No es frecuente, pero en
algunos de los casos en que la arritmia se mantiene de forma prolongada puede aparecer poliuria,
relacionada con la liberación de factor natriurético auricular. El tratamiento en fase aguda consiste
en masaje del seno carotideo, adenosina o verapamilo. En caso de tratamiento definitivo, la
ablación con radiofrecuencia tiene unas probabilidades de éxito superiores al 90%.(R1)

21. Las pacientes con síndrome de ovarios poliquísticos presentan un aumento de riesgo
a largo plazo para los siguientes procesos EXCEPTO:

1. 1. Diabetes mellitus tipo 2.


2. 2. Osteopenia.
3. 3. Hipertensión arterial.
4. 4. Coronariopatía.
Gráfico de respuestas

!
!
!
!
Comentario

Estas pacientes tienen resistencia a la insulina, con lo cual se incrementa el riesgo de respuesta 1,
3 y 4, por lo que la respuesta que debe elegir es la número 2.(R2)

22. En los
exámenes de laboratorio de un masculino de 47 años, en seguimiento en urología por
impotencia, se detectan niveles de testosterona de 1 ng/ml (normal 3-10 ng/ml), siendo
remitido a endocrinología para estudio. El reso de laboratorios destaca PRL de 1,759
ng/ml, solicitándose a continuación una RMN (ver imagen). Dada la patología que
presenta el paciente señale lo CORRECTO:

Si el paciente estuviera tomando un fármaco que produjera hiperprolactinemia, sería


1. 1.
recomendable retirarlo y reevaluar antes de emitir el diagnóstico.
Dado el tamaño de la lesión y los niveles hormonales, lo más probable es que el tumor
2. 2.
produzca hiperprolactinemia por compresión.
Si el paciente presentara ginecomastia, se debería a la acción de la prolactina elevada
3. 3.
sobre el tejido glandular mamario.
Aunque no existan déficit visuales en la anamnesis, será necesario derivarlo a
4. 4.
oftalmología para realización de campimetría.
Gráfico de respuestas
Comentario

Dada la clínica del paciente (impotencia), las cifras de prolactina y la imagen radiológica, habría
que considerar un macroprolactinoma como primer diagnóstico. Recuerde que los prolactinomas
son más frecuentes en mujeres, pero cuando aparecen en varones suelen ser mayores, ya que
tardan más tiempo en producir síntomas, por lo que tienden a consultar más tarde.

Dado el enorme tamaño de la lesión, habría que descartar afectación del campo visual (respuesta
correcta 4). Es posible que el paciente no las refiera, pero eso no quiere decir que no existan, ya
que la compresión del quiasma óptico suele producir una hemianopsia heterónima bitemporal, es
decir, una pérdida de campo periférico. En muchos casos, estas alteraciones campimétricas pasan
desapercibidas o son muy sutiles, al menos de inicio, por lo que debemos descartarlas mediante
una campimetría.(R4)

23. Respecto a la patología del paciente anterior, señale lo FALSO:

Siempre se deben tratar aquellos tumores mayores de 1 cm, independientemente de las


1. 1.
manifestaciones clínicas.
2. 2. La radioterapia puede ser un tratamiento coadyuvante a la cirugía.

!
!
!
!
Recientemente se ha descrito la presencia de insuficiencia valvular cardiaca en pacientes
3. 3. tratados con dosis altas de cabergolina, aunque su aparición en el caso del tratamiento de
la hiperprolactinemia no está clara.
En aquellos pacientes con respuesta al tratamiento médico es rara la recurrencia de la
4. 4.
enfermedad.
Gráfico de respuestas
Comentario

El tratamiento de elección de los prolactinomas es inicialmente médico, mediante análogos de la


dopamina. Recuerde que la dopamina era el enigmático PIF (factor inhibidor de la prolactina), del
que se hablaba antiguamente, y que después resultó ser esta sustancia. Lo habitual es que el
tratamiento médico sea suficiente para controlar la enfermedad, pero eso no significa que las
recurrencias no sean posibles. De hecho, son relativamente frecuentes y, por ello, debemos
realizar determinaciones periódicas de prolactina, para detectar precozmente una posible recidiva.

El resto de las opciones son ciertas y representan principios básicos a tener en cuenta en el
tratamiento del prolactinoma. No se preocupe si no sabía lo que dice la respuesta 3. No obstante,
ya sabe que, en Medicina, un “puede”, “se ha descrito”, o “algunos autores consideran” suelen ser
ciertas.(R4)

24. En relación con el lupus eritematoso sistémico hay una afirmación que es FALSA:

1. 1. La nefropatía se asocia a mal pronóstico.


2. 2. El lupus cutáneo subagudo es un signo de mal pronóstico.
3. 3. La artritis no suele ser erosiva.
4. 4. La alopecia es una manifestación cutánea inespecífica del LES.
Gráfico de respuestas
Comentario

Veamos las respuestas que nos dan en la pregunta: 1) La nefritis lúpica es un marcador de mal
pronóstico de la enfermedad al igual que lo será la afectación de otros órganos vitales como el
sistema nervioso o el pulmón. 2) La aparición de lupus eritematoso cutáneo subagudo suele ocurrir
en pacientes con escasa actividad inflamatoria a nivel de órganos vitales, por lo que esta es la
respuesta que debe elegir. Se asocia con frecuencia a los anticuerpos antiRo. 3) A diferencia de la
artritis reumatoide en el LES la artritis no es erosiva salvo excepciones. 4) Existen múltiples
manifestaciones cutáneas inespecíficas en el LES entre las que se encuentra la alopecia.(R2)

25. A 10-day-old full-term infant is brought to the emergency department presenting with
uncontrollable crying, abdominal distension, bilious vomiting and bloody stools. The
child doesn't have a fever. Elimination of meconium occured during the second day of
life. What is the most likely diagnosis?

1. 1. Pyloric stenosis.
2. 2. Duodenal atresia.
3. 3. Cystic fibrosis.
4. 4. Midgut volvulus.
Gráfico de respuestas
Comentario
Midgut volvulus. The child had a normal transit at birth (meconium was adequately eliminated) so
duodenal atresia and cystic fibrosis can be easily ruled out. Intussusception occurs in children

!
!
!
!
between the ages of three to six months and pyloric stenosis around the third week of life. Bloody
stools indicate a compromise and ischemia of the mucosa. Midgut volvulus occurs earlier.(R4)

26. ¿Cuál es la causa más frecuente de hipoxemia clínica?

1. 1. Hipoventilación.
2. 2. Efecto shunt.
3. 3. Efecto espacio muerto.
4. 4. Alteraciones en la concordancia ventilación/perfusión.
Gráfico de respuestas
Comentario

Conceptos básicos de fisiología poco preguntados en el ENARM.

La causa más frecuente de hipoxemia clínica son las alteraciones de la ventilación/perfusión.(R4)

27. Señale la respuesta errónea respecto al sistema inmune:

1. 1. La inmunidad innata es inespecífica y no genera memoria.


Dentro de los efectores celulares de la inmunidad innata se encuentran los neutrófilos, NK,
2. 2.
células presentadoras de antígenos, macrófagos, monocitos y basófilos entre otros.
3. 3. La inmunidad específica presenta adaptabilidad al antígeno, además de memoria.
4. 4. Los factores humorales participan tanto en la inmunidad innata como en la específica.
Gráfico de respuestas
Comentario

Pregunta de dificultad baja sobre los distintos tipos de inmunidad, sus características y las células
en ellos implicadas, tema poco importante en el ENARM aunque conviene que lo tenga claro. A
grandes rasgos podemos definir dos tipos de inmunidad: innata o natural y la específica. La
inmunidad innata está siempre presente y dispuesta a actuar inmediatamente, sin tiempo de
latencia, pero carece de especificidad y memoria; dentro de sus elementos encontramos las
barreras epiteliales, elementos celulares como fagocitos (monocito-macrófagos y PMN) y células
agresoras naturales (NK), y elementos humorales como lisozima, complemento e interferones. Por
otro lado la inmunidad específica es mucho más compleja y necesita un período de latencia, posee
adaptabilidad, especificidad y memoria; en ella participan también tanto elementos celulares como
linfocitos T y B o células presentadoras de antígenos como humorales, principalmente Igs y
citoquinas. Como ve las células presentadoras de antígeno pertenecen a la inmunidad específica
no a la innata.(R2)

28. Señalar cuál de estas afirmaciones NO es cierta:

1. 1. El nervio facial entra en el oído por el conducto auditivo interno.


2. 2. El nervio facial sale del temporal por el agujero estilomastoideo.
3. 3. El nervio facial tiene fibras motoras sensitivas y parasimpáticas.
4. 4. La cuerda del tímpano no es una rama del facial.
Gráfico de respuestas
Comentario

!
!
!
!
El nervio chorda timpani, o cuerda del tímpano, es una rama que el facial emite a la altura de su
tercera rodilla, todavía en su porción intrapetrosa. Este nervio se anastomosa con el nervio lingual
y, por lo tanto, tiene función gustativa.(R4)

29. Secundigesta de 32 años, con un aborto diferido previo. Gestación actual controlada
presentando diabetes gestacional en tratamiento con dieta. Ingresa en la semana 40+ 5
de parto. A lo largo de la dilatación, el feto va desarrollando progresivamente caput, se
encuentra algo deflexionado (presentación bregma), en OIDP y Iº plano. Ya alcanzada la
dilatación completa, se decide pasar a la paciente a una prueba de parto. Tras 60 minutos
en expulsivo, el feto no pasa de un 2º plano, presenta un caput importante y
acabalgamiento de parietales. Señale la respuesta CORRECTA:

1. 1. Se trata ya de un expulsivo prolongado, por lo que debemos abreviarlo con un fórceps.


Se trata de una desproporción pelvicocefálica, por lo que prepararemos a la mujer para una
2. 2.
cesárea.
Se trata de un expulsivo prolongado, por lo que debemos abreviarlo aplicando unas
3. 3.
espátulas.
Se trata de un expulsivo prolongado, por lo que debemos abreviarlo aplicando una
4. 4.
ventosa.
Gráfico de respuestas
Comentario

Cuando la presentación fetal no pasa del II plano de Hodge, nos permite diagnosticar una
desproporción pelvicocefálica y deberemos indicar una cesárea. Esperar más tiempo sólo
aumentará la morbilidad materna y fetal, sin contribuir a conseguir un parto vaginal. Para indicar
una ventosa, un fórceps o unas espátulas, la presentación tiene que haber alcanzado un plano en
concreto, específico para cada uno. Todos ellos se emplean por debajo del II plano, por lo que en
este caso no son respuestas correctas.(R2)

30. Cuál de los siguientes parámetros NO está incluido en la prueba de Apgar:

1. 1. La frecuencia cardiaca.
2. 2. Tensión arterial.
3. 3. Tono muscular.
4. 4. Color.
Gráfico de respuestas
Comentario

El test de Apgar valora cinco parámetros: esfuerzo respiratorio, respuesta a la estimulación (o a la


introducción de una SNG), frecuencia cardíaca, tono muscular y color. NO la TA. Respuesta 2
falsa.(R2)

31. Mujer de 32 años que acude a urgencias por presentar fiebre de 39.5 ºC, acompañada
de astenia y anorexia. En la placa de tórax no se aprecian anormalidades
parenquimatosas, pero sí un derrame pleural unilateral, por lo que se decide hacer una
toracocentesis diagnóstica que demuestra una relación LDH en líquido pleural/LDH
sérica mayor de 0.6; además existe un predominio de polimorfonucleares en el líquido
pleural. No se detectan microorganismos visibles en la tinción de Gram del líquido
pleural. Carece de antecedentes de interés, aunque fue operada de apendicitis hace dos
meses. ¿Cuál sería su primera sospecha diagnóstica?:

!
!
!
!
1. 1. Derrame pleural inducido por nitrofurantoína.
2. 2. Tuberculosis.
3. 3. Absceso intraabdominal.
4. 4. Síndrome de Meigs.
Gráfico de respuestas
Comentario

El tema del derrame pleural es importante conocerlo porque siempre suele caer alguna pregunta
en el ENARM. Aspectos a destacar son la etiología según las características del líquido y el
tratamiento. Existen múltiples causas de derrame pleural, de tal manera que hay que buscar
aquellas pistas que nos llevan directamente al diagnóstico. Como en este caso, si el paciente está
febril, muestra un predominio de polimorfonucleares en el líquido y no presenta anomalías
parenquimatosas (que nos pudieran hacer pensar en una neumonía y un derrame paraneumónico)
se debe pensar en un absceso intraabdominal, lo que se apoya además en el antecedente de
cirugía. En cuanto a la opción 1, el derrame asociado a fármacos suele presentar eosinófilos en el
líquido. Las infecciones víricas suelen acompañarse de predominio linfocitario, no presentan fiebre
tan alta y se resuelven de modo espontáneo. Aunque en las fases iniciales de la TB podemos
encontrar polimorfonucleares en el líquido, lo característico es el predominio linfocitario también. El
síndrome de Meigs es la asociación de ascitis y derrame pleural en relación con un tumor ovárico
(normalmente, tumor de la teca-fibroma).(R3)

32. Masculino de 9 años de edad que acude a Urgencias por presentar un cuadro de dolor
abdominal que ha comenzado en epigastrio, para posteriormente localizarse en la fosa
ilíaca izquierda, de 3 horas de evolución. En la exploración, el paciente presenta una
temperatura de 38.8º C. El abdomen es doloroso de forma difusa, más localizado en la
fosa ilíaca izquierda, con importantes signos de irritación peritoneal. Ante tales datos,
deberemos pensar que el paciente puede presentar los siguientes diagnósticos,
EXCEPTO:

1. 1. Apendicitis aguda.
2. 2. Neumonía basal izquierda.
3. 3. Invaginación intestinal.
4. 4. Salpingitis aguda.
Gráfico de respuestas
Comentario

Pregunta muy sencilla sobre el abdomen agudo, que se resuelve fácilmente leyendo con atención
la primera palabra del enunciado.

El término de abdomen agudo se aplica a cualquier dolor abdominal de inicio repentino que
presente signos de irritación peritoneal, siendo imprescindibles ambos datos para el diagnóstico.

Nos describen el caso clínico de un hombre, y como los hombres NO tienen trompas de Falopio, el
diagnóstico menos probable es una salpingitis aguda. Sentimos decepcionaros, pero no hay
salpingitis sin trompas.(R4)

33. ¿Cuál de los siguientes casos dejaría evolucionar por vía vaginal?

1. 1. Feto en situación transversa.


2. 2. Gemelar, primero en podálica.
3. 3. Feto con meningocele.

!
!
!
!
4. 4. Primípara, nalgas con feto muerto.
Gráfico de respuestas
Comentario

Vamos a analizar cada una de las opciones que nos presentan:

- Situación transversa. Cuando nos encontramos con un feto en una situación no longitudinal
(transversa u oblicua), es indicación de cesárea.

- Gemelar. Para admitir un parto vaginal en el embarazo gemelar debe ser biamniótica y estar el
primero en cefálica. Si no se cumple alguno de estos criterios como es el caso de la respuesta,
deberemos hacer una cesárea.

- Meningocele. Cuando el feto presenta alguna malformación que pueda requerir una actuación
inmediata por parte de los pediatras (por ejemplo hernia diafragmática) o en las que el parto
vaginal pueda ser lesivo, como es el caso de la respuesta, indicaremos una cesárea.

- Parto en podálica. Para admitir un parto vaginal se deben reunir unos criterios. Son útiles en
casos de fetos vivos ya que lo que buscamos disminuir la morbilidad fetal, pero en el caso que nos
presentan con feto muerto preferiremos un parto vaginal que para la madre tiene menor morbilidad,
ya que no hay interés fetal.(R4)

34. Mujer de 54 años que, en una mastografía de screening, se detecta nódulo espiculado
de 6 mm, en cuadrante superoexterno de la mama izquierda. Biopsia con aguja gruesa:
carcinoma ductal infiltrante. Los receptores hormonales resultan ser positivos. Her 2-neu
negativo. Exploración clínica: nódulo no palpable, axila libre. Ultrasonido axilar: sin
ganglios sospechosos. Señale la afirmación INCORRECTA respecto a este caso:

Dado el tamaño tumoral, sería preferible una resección conservadora frente a una
1. 1.
mastectomía radical.
2. 2. Sería conveniente añadir tratamiento local con radioterapia.
Sería conveniente el estudio de los ganglios linfáticos, mediante la técnica del ganglio
3. 3.
centinela.
4. 4. Sería aconsejable el uso de trastuzumab como parte del tratamiento.
Gráfico de respuestas
Comentario

El trastuzumab sólo tiene uso en el cáncer de mama metastásico que expresa el gen Her2 neu. En
este caso no hay ningún indicio de que se trate de un cáncer diseminado y, sobre todo, el gen Her
2 neu es negativo, por lo que la respuesta correcta es la 4.(R4)

35. ¿En cuál de los siguientes síndromes NO esperaría encontrar pólipos


hamartomatosos?

1. 1. Síndrome de Peutz-Jeghers.
2. 2. Síndrome de Muir-Torre.
3. 3. Poliposis juvenil.
4. 4. Neurofibromatosis.
Gráfico de respuestas
Comentario

!
!
!
!
El síndrome de Muir-Torre es un trastorno autosómico dominante. Se caracteriza por el desarrollo
de menos de 100 pólipos de tipo adenomatoso en el colon proximal, con riesgo de cáncer. El
síndrome de Peutz-Jeghers, la poliposis juvenil y el síndrome de Cowden cursan típicamente con
pólipos hamartomatosos. La neurofibromatosis puede cursar con neurofibromas submucosos en el
tracto gastrointestinal. Aunque pueden presentar alguno adenomatoso, no son en absoluto el tipo
predominante.(R2)

36. A 9-year-old Senegalese girl with a previous diagnosis of sickle cell anemia comes to
the pediatrician's consultation for presenting persistent pain on her left knee. Initially she
had joint pain only when she played tennis at school, but it got worse and now she feels
it almost every day, so she has discontinued sport activities. Physical examination of the
left knee shows absence of tenderness at palpation, but there is a marked reduction in
the range of motion. She is afebrile. What is the most likely diagnosis?

1. 1. Rheumatoid arthritis.
2. 2. Stress fracture.
3. 3. Osteosarcoma.
4. 4. Avascular necrosis.
Gráfico de respuestas
Comentario
Avascular necrosis. Sickle cell anemia can cause hyperviscosity syndrome. This background is
essential to answer the question and guide the diagnostic suspicion. Hyperviscosity cause a
blockage of the arteries supplying the femoral head. The symptoms are progressive, starting with
mechanical joint pain during exercise until symptoms at rest. Fever is not usually present and
should prompt us to think about an infectious cause.(R4)

37. Respecto a la neumonía por citomegalovirus en pacientes inmunodeprimidos, señale


la respuesta FALSA:

1. 1. En trasplantados renales, el periodo de mayor riesgo es el primer mes postrasplante.


2. 2. Cursa con infiltrados intersticiales en la periferia de los lóbulos inferiores.
3. 3. El diagnóstico en pacientes con SIDA requiere biopsia pulmonar.
4. 4. No es frecuente el patrón nodular en la radiografía de tórax.
Gráfico de respuestas
Comentario

El CMV, cuando produce neumonía, da lugar a infiltrados pulmonares bilaterales, de tipo


intersticial, en ocasiones en forma de vidrio deslustrado. La neumonía por CMV es casi exclusiva
de pacientes inmunodeprimidos (SIDA, trasplante de órgano sólido, enfermedades tumorales
hematológicas…). En los pacientes trasplantados, el periodo de máximo riesgo de infección por
CMV es entre el segundo y el sexto mes postrasplante (respuesta 2 falsa).(R1)

38. Respecto a las complicaciones de varicela, señale lo FALSO:

1. 1. La más frecuente es la sobreinfección bacteriana de las lesiones cutáneas.


2. 2. La neumonía por varicela es más frecuente en niños que en adultos.
3. 3. La queratitis y conjuntivitis vesicular es una complicación rara y en general benigna.
4. 4. Se puede producir un síndrome hipotalámico con obesidad y fiebre recurrente.
Gráfico de respuestas
Comentario

!
!
!
!
La opción correcta es la 2, puesto que la neumonía por varicela es más frecuente en los adultos
que en los niños.

Esta complicación puede aparecer en el 30% de los adultos que contraen la varicela y cursa con
una recuperación clínica rápida, pero con una evolución radiológica más tardía.

De las otras opciones cabe destacar la 1, por ser la complicación más frecuente de este cuadro y
mencionar que la afectación cerebelosa es un dato de buen pronóstico en la encefalitis
postinfecciosa.(R2)

39. A possible etiologic agent in pelvic inflammatory disease, typically associated with
intrauterine device (IUD) use is:

1. 1. Serratia.
2. 2. Pseudomonas.
3. 3. Neisseria gonorrhoeae.
4. 4. Actinomyces israelii.
Gráfico de respuestas
Comentario

Pregunta de guardia de medicina interna, no está demás saberlo de complicaciones del uso de
DIU, es enfermedad pélvica inflamatoria por Actinomyces israelii.(R4)

40. Niña de 7 años que consulta por cuadro catarral de 5 días de evolución con aparición
en las últimas 48 horas de lesiones cutáneas muy pruriginosas. A la exploración se
encuentra febril, con temperatura de 39.2ºC, presenta buen estado general, la
auscultación cardiopulmonar es normal, el abdomen es blando y depresible y no se
palpan masas ni visceromegalias, no presenta rigidez nucal y los signos meníngeos son
negativos, la faringe está intensamente hiperémica pero no presenta placas ni exudados
y la otoscopia bilateral es normal. En la piel se aprecian múltiples lesiones máculo-
papulares eritematosas, en algunas zonas vesiculosas. Respecto a la enfermedad que
sospecha, señale la afirmación FALSA:

La principal vía de transmisión del germen es la respiratoria, aunque también se contagia


1. 1.
por contacto con las lesiones cutáneas.
La complicación más frecuente es la neumonía y debemos sospecharla ante el
2. 2.
empeoramiento de la tos y de la fiebre.
El tratamiento con fármacos antivirales se recomienda en los casos complicados,
3. 3.
neonatales o en inmunodeprimidos.
Puede asociarse a una encefalopatía aguda con degeneración grasa hepática si se
4. 4.
administra ácido acetil salicílico.
Gráfico de respuestas
Comentario

Cuadro clínico típico de VARICELA y muy preguntado en el ENARM. La pista la da el hecho de


presentar un exantema POLIMORFO (es decir, que coexisten lesiones en distinto estado
evolutivo).

La principial vía de transmisión de la varicela es la vía respiratoria, por inhalación de partículas tras
accesos de tos o estornudos de un huésped infectado. Su periodo de máxima contagiosidad
abarca desde las 24 horas antes de la aparición del exantema hasta que todas las lesiones se

!
!
!
!
encuentren en fase de costra. El tratamiento es sintomático, y SOLO está indicado en uso de
aciclovir (u otros antirretrovirales) en caso de inmunodeprimidos y neonatos (como dice la
respuesta 3).

Dentro de las complicaciones debemos recordar que la más frecuente es la sobreinfección


bacteriana de las lesiones cutáneas por S.pyogenes o S.aureus; esta frase contradice por tanto lo
que dice en la respuesta 2 (que la complicaciones más frecuente es la neumonía, siendo ésta muy
rara en niños). Como anotación mencionar que la respuesta número 4 se refiere al síndrome de
Reye (asociado al tratamiento con AAS).(R2)

41. Señale lo FALSO en relación con la enfermedad de Behçet:

1. 1. Se asocia a prueba de patergia positiva.


En mujeres es frecuente la presencia de úlceras recidivantes en la boca, siendo extraña la
2. 2.
aparición de úlceras vulvares, que de estar presentes suelen dejar cicatriz.
Asocia lesiones oculares tales como: iritis, uveítis anterior y posterior, hipopion, con
3. 3.
riesgo incluso de ceguera.
Las úlceras características son las úlceras aftosas: úlceras dolorosas con bordes rojos
4. 4.
elevados y base necrótica amarillenta.
Gráfico de respuestas
Comentario

Las aftas orales recidivantes (100% de los casos) son dolorosas, y tienen un tamaño y aspecto
variable curándose solas en 1 a 3 semanas, sin dejar cicatrices. Las aftas genitales (60-80%) se
localizan en el glande y escroto en el varón y en la vulva, vagina y cervix en la mujer, siendo
dolorosas y tardando en cicatrizar en el varón, y mucho menos molestas en las mujeres. Se trata,
por lo tanto, de un problema frecuente (respuesta 2 falsa). La uveítis, bilateral casi siempre está
presente en el 60-70% de los pacientes. A veces, se asocian coroiditis, hemorragias del cuerpo
vítreo, neuritis óptica, alteraciones vasculares retinianas y otras que pueden conducir a la ceguera
si no se trata la enfermedad.(R2)

42. Señale la FALSA sobre la artritis crónica juvenil (ACJ):

1. 1. En la ACJ de comienzo sistémico, los ANA son positivos.


2. 2. En la ACJ de comienzo sistémico de larga evolución puede haber amiloidosis.
3. 3. La uveítis crónica condiciona el pronóstico en la forma oligoarticular precoz.
En la forma poliarticular seropositiva, el 50% sigue un patrón destructivo similar a la
4. 4.
artritis reumatoide del adulto.
Gráfico de respuestas
Comentario

Esta pregunta “toca” algún aspecto de las diferentes formas de ACJ, pero son más o menos
conocidos.

La forma de comienzo sistémico (enfermedad de Still) se diagnostica por la clínica, ya que ninguna
determinación analítica es propia de la enfermedad (no presentan ni FR ni ANAs). La complicación
más grave de esta forma de comienzo es, efectivamente, la amiloidosis.

Dentro de las formas de comienzo oligoarticular (4 o menos articulaciones afectadas en los


primeros 6 meses de la enfermedad), distinguimos la de comienzo precoz (niñas de menos de 6
años, con frecuencia con ANAs + y DR5), cuya principal complicación es la uveítis posterior, y la de

!
!
!
!
comienzo tardío (varones a partir de los 10 años), que muestran una distribución articular
predominantemente en miembros inferiores, como las espondiloartropatías y, al igual que ellas, se
asocian con frecuencia al HLA B27.

Las formas de comienzo poliarticular (más de 4 articulaciones afectadas en los primeros 6


meses) se diferencian entre sí, en función de que presenten o no factor reumatoide, en formas
seronegativas y seropositivas. Estas últimas son muy parecida a una AR, mostrando una
agresividad articular comparable.(R1)

43. Ante un paciente con amnesia de fijación, apatía, hiperfagia e hiperactividad sexual,
¿dónde localizaría la lesión responsable?:

1. 1. Lóbulo frontal derecho.


2. 2. Ambos lóbulos frontales.
3. 3. Ambos lóbulos temporales.
4. 4. Lóbulo parietal derecho.
Gráfico de respuestas
Comentario

Esta es una pregunta de cierta dificultad sobre las lesiones temporales, no os preocupéis si no la
habéis hecho bien porque no ha sido preguntado en el ENARM. De todas formas, por intuición,
podemos pensar que la lesión se localiza en esta región, ya que el lóbulo temporal participa en la
conducta emocional y afectiva. La afectación temporal bilateral puede conducir a un síndrome
amnésico de Korsakoff, síndrome de Klüver-Bucy (apatía, placidez, incremento de la actividad
sexual y falta de reconocimiento de objetos comestibles o amnesia de fijación) y sordera
cortical.(R3)

44. Embarazada de 32 semanas que acude a su consulta para el control de la gestación


en visita programada. En el último mes ha ganado 4 kg, de peso. Presenta edemas en
miembros inferiores y una TA de 140/90 en ese momento y tras repetir la toma 30 minutos
después. Realiza un USG en el que observa un feto con una biometría acorde con la
amenorrea, placenta y líquido amniótico normales. Cuál de las siguientes decisiones
tomará a continuación:

1. 1. Pautar tratamiento con alfametildopa.


2. 2. Diagnóstico de preeclampsia. Reposo y tratamiento antihipertensivo.
Dado que no existe patología, se debe continuar con los controles normales de la
3. 3.
gestación.
4. 4. Solicitar examen de orina.
Gráfico de respuestas
Comentario

Tema de vital importancia en el ENARM sobre los estados hipertensivos en la gestación.

La paciente presenta hipertensión, edemas y aumento de 4 kg de peso en un mes, en la semana


32 de gestación, lo que nos hace sospechar una preeclampsia. Para confirmarlo (y descartar que
se trate una hipertensión preexistente u otros procesos) debemos realizar un análisis de orina para
ver la existencia de proteinuria (respuesta correcta 4).

Las demás opciones se descartan con facilidad. Las respuestas 1 y 2, que dan por hecho la
existencia de una preeclampsia y aplican su tratamiento, son además de incorrectas, contrarias a

!
!
!
!
la 4 (no sabemos que tenga una preeclampsia todavía). La respuesta 3 es falsa, pues es obvio que
la mujer no está completamente sana: ganancia de 4 kg en un mes, edemas en MMII, TA
140/90...(R4)

45. La triada clásica del síndrome de Ziéve la forman:

1. 1. Anemia hemolítica, ictericia e hiperlipemia.


2. 2. Ascitis, ictericia y encefalopatía hepática.
3. 3. Anemia macrocítica, ictericia y ascitis.
4. 4. Trombocitopenia, anemia, leucopenia.
Gráfico de respuestas
Comentario
El síndrome de Zieve es una complicación extremadamente infrecuente de la esteatosis hepática
secundaria al alcohol (hígado graso alcohólico). Se define por la asociación de hipertrigliceridemia,
anemia hemolítica e ictericia en un paciente con esteatosis hepática alcohólica. Los pacientes
suelen tener además dolor abdominal. La ascitis, encefalopatía, anemia macrocítica, trombopenia,
leucopenia e hiperesplenismo podemos encontrarlas como hallazgos asociados en el mismo
paciente pero no definen este síndrome.(R1)

46. Recién nacido de 2 días de vida que comienza con ictericia y lesiones petequiales en
tronco y raíz de extremidades. Es una niña, de 33 semanas de edad gestacional y peso
de 2050 gr. Tiene hepatomegalia de 3 cm y esplenomegalia de 5. Le llama la atención su
gran microcefalia y la presencia de leucocoria. Además presenta somnolencia excesiva,
tiraje subcostal moderado, taquicardia, taquipnea y edemas en ambos párpados. Se le
realiza ecografía cerebral (calcificaciones periventriculares), los exámenes de laboratorio
muestran anemia, eritroblastosis, leucocitosis con neutrofilia y trombocitopenia en la
biometria hemática y en la bioquímica hiperbilirrubinemia y aumento de transaminasas.
Tras cuatro días de evolución fallece por insuficiencia cardíaca. ¿Cuál es su sospecha
diagnóstica?:

1. 1. Isoinmunización Rh.
2. 2. Infección congénita por toxoplasma.
3. 3. Infección perinatal por citomegalovirus.
4. 4. Infección connatal por citomegalovirus.
Gráfico de respuestas
Comentario

Tema muy importante que debe dominar. Sospecha que un neonato padece una infección connatal
si presenta antecedente de retraso del crecimiento intrauterino, hiperplasia retículo-endotelial
(hepatoesplenomegalia, adenopatías), anemia y trombopenia. Parece que el niño tiene una
infección por CMV porque presenta microcefalia, coriorretinitis y calcificaciones
periventriculares.(R4)

47. Desde el punto de vista clínico, la duración del embarazo se mide desde:

1. 1. El primer día de la última regla.


2. 2. El final de la última regla.
3. 3. El coito fecundante.
4. 4. La primera falta menstrual.
Gráfico de respuestas

!
!
!
!
Comentario

Pregunta muy fácil sobre la cronología del embarazo. Recuerde que los obstetras a la hora de
datar la gestación, contamos como primer día de amenorrea el primer día de sangrado menstrual
de la última regla (respuesta correcta la 1). A partir de ahí, la duración normal son 40 semanas de
gestación.(R1)

48. A 15-year-old Senegalese female is brought to the doctor's office presenting with pain
on her right knee that keeps her from sleeping at night. She refers no fever. She denies
recent trauma and overstraining. The joint shows no swelling or redness. Hemoglobin
level is 8 g/dL. Which of the following conditions would explain this case?

1. 1. Osteonecrosis.
2. 2. Chronic osteomyelitis.
3. 3. Ewing's sarcoma.
4. 4. Osteomalacia.
Gráfico de respuestas
Comentario
Osteonecrosis. A painful joint with no signs of inflammation at physical examination is very
suggestive of osteonecrosis. Osteonecrosis may be due to bone infarcts in children affected by
sickle cell anemia, which is the main suspicion in this case (Senegalese origin). Of the remaining
options, osteomyelitis and Ewing's sarcoma would be expected to be accompanied by a systemic
inflammatory response. Osteomalacia produces bone deformities and Osgood schlatter disease is
more typical of younger boys.(R1)

49. Un paciente de 25 años acude al hospital con un síndrome febril, alteraciones de la


conducta, confusión, desorientación y alucinaciones. Poco después tiene varios
episodios convulsivos, tras lo cual entra en coma. En la exploración se aprecia rigidez de
nuca y hemiparesia izquierda. El estudio del LCR se caracterizó por tener un aspecto
ligeramente hemorrágico, 450 leucocitos/mm3, glucosa 60 mg/dl y proteínas 560 mg/dl.
El EEG mostró actividad difusa de ondas lentas y en la TAC se evidenció la presencia de
lesiones hemorrágicas con edema cerebral a nivel del lóbulo temporal. ¿Cuál sería su
tratamiento de elección?

1. 1. Ribavirina.
2. 2. Aciclovir.
3. 3. Ganciclovir.
4. 4. Zidovudina.
Gráfico de respuestas
Comentario

La encefalitis por VHS ha sido preguntada en múltiples ocasiones. Las ideas más importantes al
respecto son:

- Es la causa más frecuente de encefalitis vírica.

- Si se acompaña de focalidad neurológica, es de predominio temporal.

!
!
!
!
- El diagnóstico es inicialmente clínico. Actualmente, la confirmación se realiza mediante la
demostración del ADN del virus en LCR. En el pasado, era precisa una biopsia cerebral.

- Dada la lentitud de las pruebas diagnósticas, está justificado el tratamiento empírico ante la
sospecha de esta entidad (aciclovir i.v.)(R2)

50. Respecto a Citrobacter freundii, es FALSO:

1. 1. Es un bacilo gramnegativo.
2. 2. Utiliza el citrato.
3. 3. Reduce los nitratos.
4. 4. Es oxidasa positiva.
Gráfico de respuestas
Comentario

Pregunta muy rebuscada, para fines del ENARM. El género Citrobacter pertenece a la familia
Enterobacteriaceae - enterobacterias- que incluye muchos géneros de interés médico; entre sus
géneros más relevantes se encuentran: Escherichia, Shigella, Salmonella, Citrobacter, Klebsiella,
Enterobacter, Hafnia, Serratia, Proteus, Morganella, Providencia, Yersinia, Edwardsiella. Todas las
enterobacterias son bacilos gramnegativos, aerobios, anaerobios facultativos, no esporulados,
fermentan la glucosa, reducen los nitratos a nitritos, y son oxidasa negativa. Los Citrobacter crecen
en agar citrato, esta cualidad la comparten con algunas otras bacterias de la familia
Enterobacteriaceae.(R4)

51. Recién nacido a término, de 4 kg de peso, nacido de parto eutócico con líquido
meconial, que presenta desde el nacimiento cianosis, tiraje intenso, aleteo nasal y
rinorrea mucosa bilateral. Es incapaz de alimentarse y mejora su situación con el llanto
intenso. La radiografía de tórax es normal. ¿Qué exploración será más útil para el
diagnóstico?

1. 1. Biometría hemática y hemocultivos.


2. 2. Radiografía lateral de faringe.
3. 3. Pasar una sonda por la nariz hasta la faringe.
4. 4. pH y gases capilares.
Gráfico de respuestas
Comentario

Pregunta clásica que no debe plantearle ningún problema. El caso que nos describen corresponde
con una atresia de coanas. Es importante que aprenda a identificarlo.

Los recién nacidos, igual que los adultos, respiran por la nariz la mayor parte del tiempo. Sin
embargo, en algunsa ocasiones lo hacen por la boca, por ejemplo durante el llanto. Por ello, en la
atresia de coanas, característicamente, mejoran durante el llanto, porque si respiran bucalmente el
flujo aéreo no encuentra ningún obstáculo. Por el contrario, cuando se alimentan, al estar
empleando la boca en la succión, se asfixian y tienen que parar para poder respirar. De las
exploraciones que nos mencionan, la correcta sería la sonda hasta faringe, lo que confirma o
descarta la obstrucción de las coanas.(R3)

52. ¿Cuál de los siguientes enunciados es FALSO con respecto al tratamiento del brote
de esclerosis múltiple?

!
!
!
!
1. 1. El tratamiento de los brotes sensitivos se realiza con corticoterapia oral.
2. 2. El tratamiento corticoideo logra acortar la duración del brote.
3. 3. El tratamiento corticoideo está contraindicado en caso de pseudobrote.
4. 4. El tratamiento corticoideo mejora el pronóstico de recuperación funcional.
Gráfico de respuestas
Comentario

Recuerde que la intención al tratar los brotes de esclerosis múltiple no es mejorar el pronóstico de
recuperación funcional, sino acortar el tiempo hasta la máxima recuperación, que el paciente de
todas formas conseguiría sin tratamiento, aunque tardaría más tiempo.(R4)

53. El vaso que se encuentra envuelto en el ligamento de Hesselbach y que nos sirve para
diferenciar las hernias inguinales directas de las indirectas es:

1. 1. Arteria hipogástrica inferior.


2. 2. Arteria hipogástrica superior.
3. 3. Arteria epigástrica superior.
4. 4. Arteria epigástrica inferior.
Gráfico de respuestas
Comentario

Es habitual encontrar preguntas sobre hernias. Es importante que conozca todos los tipos posibles
y además añada algunos datos epidemiológicos que seguro serán de mucha utilidad. El ligamento
de Hesselbach es un engrosamiento de la fascia transversalis que forma uno de los límites del
orificio profundo del conducto inguinal. Dentro de este engrosamiento se localiza la arteria
epigástrica inferior que asciende por él para anastomosarse con la arteria mamaria interna. Esta
estructura es importante porque sirve también para clasificar la hernias inguinales en directas
(internas al vaso) e indirectas (externas al vaso).(R4)

54. A 6-year-old girl is brought to the office presenting with easy fatigability and easy and
excessive bleeding after minor injuries. Physical examination is unremarkable except a
delayed physical development compared to girls of same age (lower than percentile 3).
She was previously healthy and has no medical history. Blood laboratory tests are:
Hemoglobin 7 g/dL, MCV 104 fl, Leukocyte 3000/mm3, Platelets 70,000/mm3. Which of the
following is the most likely cause of this condition?

1. 1. Chromosomal breaks.
2. 2. Congenital viral infections.
3. 3. Thymic neoplasm.
4. 4. Celiac disease.
Gráfico de respuestas
Comentario
Chromosomal breaks. Chromosomal breakage syndromes are a group of genetic disorders typically
autosomal recessive in transmission. The high rates of chromosomal breakage or instability, lead to
chromosomal rearrangements. In this case, the involvement of bone marrow stem cells has led to
aplastic anemia. Celiac disease and iron deficiency will only affect the red cell count. In this case
the three series are affected. Congenital infections have manifestations at an earlier age.(R1)

55. Señale la afirmación CORRECTA en relación con la infección por sarampión en los
niños con SIDA:

!
!
!
!
1. 1. La vacuna del sarampión está contraindicada.
2. 2. La neumonía por sarampión tiene una alta mortalidad.
3. 3. El exantema aparece siempre y es típico.
4. 4. La vitamina A oral reduce las complicaciones.
Gráfico de respuestas
Comentario

Debe saber las características clínicas más importantes que le permitan diferenciar las distintas
enfermedades exantemáticas.

El sarampión es una enfermedad eruptiva febril de carácter agudo. Habitualmente es una


enfermedad benigna, de curación espontánea, pero puede dar lugar a distintas complicaciones.

La neumonía intersticial de células gigantes es una complicación rara, pero de elevada mortalidad.
Aparece en niños con procesos sistémicos graves, como leucemia, inmunodeficiencia 1ª o
adquirida o desnutrición intensa. Se caracteriza por síntomas respiratorios graves, infiltraciones
pulmonares y células gigantes multinucleadas en el parénquima pulmonar. La neumonitis puede
aparecer faltando el exantema típico.

En pacientes con inmunodeficiencia, los complementos de vitamina A administrados 2 días


consecutivos en cuanto se diagnostica el sarampión puede disminuir la incidencia de
complicaciones graves.

Puede vacunarse a pacientes infectados por VIH siempre que conserven una inmunidad
“aceptable”. La administración de gamma-globulina está indicada cuando no es posible garantizar
el desarrollo de la inmunidad después de la vacuna, como ocurre en pacientes VIH.(R2)

56. Ante una paciente amenorreica con test de gestágenos positivo, ¿cuál de las
siguientes etiologías de la amenorrea es la más factible?:

1. 1. Anovulación crónica.
2. 2. Síndrome de Asherman.
3. 3. Disgenesia gonadal.
4. 4. Fallo ovárico precoz.
Gráfico de respuestas
Comentario

Ante una amenorrea secundaria, lo primero que debe solicitarse es un test de embarazo pues es la
primera causa de amenorrea secundaria. En caso de ser positivo, ya tendríamos cuál es la causa
de su amenorrea. En caso de ser negativo, se debe practicar una determinación de TSH y PRL. Si
son normales, el siguiente paso consiste en administrar progesterona.

El test de la progesterona consiste en administrar una pequeña cantidad esta hormona, durante 5
días. Si la paciente ha producido con normalidad la primera fase del ciclo (proliferativa), pero no
llega a ovular y por tanto no llega a producir progesterona, al administrarle esta progesterona que
le faltaba, tendrá la regla y concluimos que la causa era la anovulación (respuesta 1 correcta).

En caso de no conseguirse la regla con la administración de progesterona, el paso siguiente es


combinar estrógenos y progestágenos para continuar el estudio.(R1)

57. Respecto al hijo de madre diabética mal controlada, señale la opción VERDADERA:

!
!
!
!
La hiperglucemia materna protege en las primeras horas de la vida de hipoglucemias al
1. 1.
feto.
Dentro de las malformaciones cardiacas, la hipertrofia septal asimétrica es la más
2. 2.
prevalente.
3. 3. No se han descrito casos de trombosis de vena renal en estos pacientes.
4. 4. La agenesia lumbosacra no es un hallazgo característico de esta entidad.
Gráfico de respuestas
Comentario

El hijo de la madre diabética es tema importante. Recuerde que el principal riesgo que presentan
es la aparición de hipoglucemia en el periodo neonatal inmediato, debido a la falta de aportes
maternos tras el parto y la respuesta insulínica excesiva del neonato derivada de la hiperglucemia
mantenida durante la época fetal (opción 1 falsa). Dentro de las malformaciones asociadas las más
frecuentes son las cardiacas, siendo, entre ellas, la hipertrofia septal asimétrica la principal (opción
3 verdadera), dentro de las digestivas, la más frecuente es el colon izquierdo hipoplásico, y la más
característica es la agenesia lumbosacra o síndrome de regresión caudal (opción 4 falsa). Por otro
lado existen casos descritos de trombosis de vena renal debido a la policitemia que presentan
estos pacientes (opción 3 falsa).(R2)

58. Acude a su consulta una paciente de 48 años con diagnóstico de cáncer de mama
hace 3 años, cuyo tratamiento fue mastectomía mas ganglio centinela, radioterapia y
quimioterapia posterior. El tumor presentaba receptores hormonales positivos por lo que
se añadió tamoxifeno como tratamiento hormonal. La paciente refiere un sangrado
irregular en los últimos meses, al que no le ha dado importancia. ¿Cuál sería su actitud?:

1. 1. Histerectomía radical.
2. 2. Petición de marcadores tumorales.
3. 3. Ultrasonido transvaginal.
4. 4. Histeroscopía diagnóstica.
Gráfico de respuestas
Comentario

La paciente que nos describen es una mujer que ha recibido tratamiento para su cáncer de mama
incluyendo tratamiento hormonal dado que los receptores hormonales eran positivos. Al tomar
tamoxifeno debemos tener especial cuidado en la vigilancia del endometrio, ya que, aunque actúa
como antiestrogénico en la mama, su acción sobre el endometrio es una estimulación estrogénica,
por ello se considera factor de riesgo de cáncer de endometrio. No olvide que ante cualquier
sangrado vaginal en estas pacientes debemos realizar despistaje de un posible carcinoma de
endometrio para lo cual la prueba de elección es la histeroscopía. Recordemos que el raloxifeno, a
diferencia del tamoxifeno, es antiestrogénico en el endometrio.(R4)

59. Como consecuencia de la esplenectomía pueden encontrarse en la sangre todos los


datos siguientes SALVO:

1. 1. Cuerpos de Heinz en los hematíes.


2. 2. Hematíes en forma de diana.
3. 3. Aumento importante de Ig, como compensación .
4. 4. Hematíes nucleados.
Gráfico de respuestas
Comentario

!
!
!
!
El bazo tiene dos grande funciones una No inmunológica que incluye: el reconocimiento y la
eliminación de hematíes con alteraciones; la eliminación de inclusiones intraeritrocitarias rígidas,
como los cuerpos de Heinz y de Howell- Jolly; la filtración y fagocitosis de partículas no
opsonizadas aún; maduración de los reticulocitos; y el almacenamiento de plaquetas y
granulocitos. Las funciones inmunológicas son igual de importantes, incluso el riesgo de sepsis
postesplenectomia es muy elevado, esto se debe a la ausencia de eliminación de bacterias
recubiertas de anticuerpos y a una disminución de la producción de IgG e IgM.(R3)

60. A 76-year-old woman, with parity story 3-0-1-3 and last menstrual period when she
was 52 years old comes to her gynecologist complaining of a 5 year history of vulvar
pruritus. She says that she has tried several over-the-counter therapies. For the past 4
months, she has started feeling a little lump on her labia majora that bleeds easily. Further
questioning reveals frequent dysuria. Which of the following is the most likely diagnosis?

1. 1. Genital herpes infection.


2. 2. Vulvar Paget's disease.
3. 3. Urethral caruncle.
4. 4. Vulvar squamous carcinoma.
Gráfico de respuestas
Comentario
El cáncer escamoso vulvar es una neoplasia relativamente rara que suele aparecer entre los 65 y
75 años, siendo excepcional antes de los 30 años. El síntoma más frecuente es el prurito vulvar
que suele ser de larga evolución como es el caso que nos presentan. Se asocia a tumoraciones en
genitales externos y ulceraciones junto a otros síntomas tardíos como la secreción serohemática o
dolor por sobreinfección o extensión a tejidos vecinos. Los síntomas urinarios, como la disuria que
refiere la paciente del caso clínico, aparecen cuando hay extensión al meato uretral. La lesión se
localiza de forma más frecuente en los labios mayores, seguida de los labios menores.(R4)

61. Una mujer de 24 años acude a urgencias por dolor abdominal de 24 horas de evolución
localizado en fosa iliaca derecha (FID), acompañado de vómitos que aparecieron después
del dolor. La exploración es difícil por la poca colaboración de la paciente, que está muy
nerviosa, y por la obesidad que presenta, pero parece focalizar el dolor en FID con
Blumberg positivo. En los exámenes de laboratorio destaca leucocitosis con desviación
izquierda y elevación de PCR. Niega relaciones sexuales; no obstante, es sometida a una
ecografía ginecológica donde sólo se aprecia pequeña cantidad de líquido libre en
Douglas. ¿Cuál le parece la mejor opción en este caso?:

1. 1. Alta y revisión de nuevo en 24 horas.


2. 2. Mantener en observación hospitalaria para ver evolución.
3. 3. Laparotomía, porque parece que tiene una apendicitis aguda.
4. 4. Laparoscopia diagnóstica y actuar según los hallazgos.
Gráfico de respuestas
Comentario

El abdomen agudo en la mujer presenta un manejo más dificultoso que en el hombre. Este mismo
cuadro en un hombre joven sería susceptible de cirugía urgente por la alta probabilidad de que se
tratara de una apendicitis aguda. En el caso de una mujer joven, ante este tipo de cuadros siempre
hemos de descartar primero patología ginecológica (enfermedad inflamatoria pélvica, quiste
torsionado…), como se hizo en urgencias en el caso que planteamos. Una vez descartado, existen
varias opciones válidas: por un lado la realización de un escáner, que presenta el inconveniente de
la radiación pélvica a una mujer fértil, por otro, la vigilancia y seriación de exámenes de laboratorio,

!
!
!
!
ya que la paciente no presenta ningún signo de alarma de sepsis grave. La laparoscopía
diagnóstica sería en este caso la técnica de elección ya que está indicada fundamentalmente en
mujeres jóvenes y pacientes obesos que presentan cuadros compatibles con apendicitis
aguda.(R4)

62. Señale la opción FALSA respecto a la hipertensión pulmonar:

La hipertensión pulmonar se define como la elevación de la presión en la arteria pulmonar


1. 1.
media por encima de 20 mmhg en reposo.
La hipertensión pulmonar postcapilar o venosa se caracteriza por la presencia de una
2. 2. presión capilar mayor de 18 mmhg, aunque inicialmente el gradiente entre la presión de
enclavamiento y la diastólica pulmonar es despreciable.
La hipertensión pulmonar suele aparecer en el seno de enfermedades respiratorias crónicas
3. 3. que cursan con hipoxemia o en caso de enfermedades cardíacas, siendo más raro la
hipertensión pulmonar primaria.
La gravedad del cor pulmonale, que puede ser secundaria a hipertensión pulmonar, viene
4. 4. dada por el grado de aumento de la precarga ventricular derecha, es decir, la presión
arterial pulmonar.
Gráfico de respuestas
Comentario

Es una pregunta díficil, en la que hay que manejar diferentes conceptos hemodinámicos de la
hipertensión pulmonar. En la opción 1 encontramos la definición clásica de hipertensión arterial
pulmonar, que tal y como dice la opción 3, puede tener un origen precapilar, postcapilar o ser un
proceso primario; el principal parámetro para diferenciar una hipertensión precapilar de una
postcapilar es la presión de enclavamiento, estando normal y aumentada respectivamente. Por lo
tanto, opción 4 falsa; el cor pulmonale se define como un aumento del tamaño del ventrículo
derecho secundario a enfermedades pulmonares, torácicas o de la circulación pulmonar, que
puede ir acompañado de insuficiencia ventricular derecha. La gravedad del cor pulmnonale viene
determinada por el aumento de la postcarga ventricular derecha o presión arterial pulmonar y no
por la precarga como dice el enunciado.(R4)

63. En relación al hígado graso macrovesicular, ¿cuál de las siguientes NO se encuentra


entre las causas habituales?:

1. 1. Malnutrición calórico-proteica.
2. 2. Síndrome de Reye.
3. 3. Nutrición parenteral total.
4. 4. Diabetes mellitus.
Gráfico de respuestas
Comentario

De entre las que se citan, el síndrome de Reye es la única que no cursa con estaetosis
macrovesicular. La obesidad y el alcohol son las principales causas de esteatosis por lo que
deberán eliminarse. Los pacientes con diabetes mal controlada, sobre todo la tipo 2 que cursa con
resistencia a la insulina, pueden producir esteatosis. La nutrición parenteral total supone un
aumento del pool de grasas que llegan al hígado pudiéndose producir su acúmulo que como en las
anteriores, cursa en forma de grandes gotas lipídicas. Sin embargo el síndrome de Reye produciría
de tipo microvesicular. (R2)

64. ¿Cuál de los siguientes criterios NO se considera actualmente por la ACR para el
diagnóstico de artritis reumatoide?:

!
!
!
!
1. 1. Rigidez superior a una hora.
2. 2. Artritis de tres o más articulaciones.
3. 3. Factor reumatoide positivo.
4. 4. Biopsia sinovial compatible.
Gráfico de respuestas
Comentario

El diagnóstico de AR se establece por la coexistencia de 4 de los 7 criterios diagnósticos


propuestos por el ACR para el diagnóstico de la enfermedad. Estos son : - Rigidez articular
depués de la inactividad de al menos 1 hora de duración. - Afectación poliarticular (al menos tres
áreas articulares de forma simultánea) - Distribución simétrica de la afectación articular.
- Participación de las articulaciones de las manos (MCF, IFP, Carpo). - Nódulos subcutáneos.
- Presencia de Factor Reumatoide. - Alteracion radiológica (erosiones, osteoporosis yuxtaricular.
La histologia no es un criterio ya que los hallazgos anatomopatológicos de la AR son inespecíficos,
mostrando sinovitis crónica y aguda indistinguible de la que aparece en otras enfermedades que
cursan con afectación inflamatoria articular (artropatía psoriasica, etc...); sin embargo en el 2010
dichos criterios han sido actualizados.(R4)

65. Le remiten desde la consulta de Reumatología a un hombre de 25 años, en estudio


desde hace años por artritis de grandes articulaciones no deformante. Actualmente
presenta fiebre y diarrea acompañada de importante pérdida de peso y signos de
malnutrición. La exploración es anodina, salvo por la existencia de adenopatías axilares
e inguinales rodaderas de pequeño tamaño. ¿Cuál es el tratamiento de elección para la
enfermedad que usted sospecha?:

1. 1. Dieta sin gluten de por vida.


2. 2. Trimetoprim-sulfametoxazol.
3. 3. Dieta baja en grasas y aporte de triglicéridos de cadena media.
4. 4. Nutrición parenteral.
Gráfico de respuestas
Comentario

Nos están describiendo un caso típico de enfermedad de Whipple de larga evolución (varón,
artritis, fiebre, diarrea, pérdida de peso, adenopatías). En estos casos, para confirmar el
diagnóstico debería realizarse una biopsia (por ejemplo, intestinal) en la que se observaran
macrófagos con inclusiones PAS+ en su interior (Tropheryma whipplei): Al ser una enfermedad
bacteriana, el tratamiento debe hacerse con antibioterapia: TMP-SMX, duarnte al menos 1
año.(R2)

66. A 6-year-old boy who has a history of well controlled asthma is brought to the doctor's
office presenting with a runny nose, nasal congestion, sneezing and a mild fever. His
mother adds that the child has been recently having episodes of uncontrollable coughing
which bring up thick phlegm and a 'whooping' sound with each sharp intake of breath
after coughing. Which of the following is the most suitable treatment for this patient?

1. 1. Ampicillin.
2. 2. Amoxicillin.
3. 3. Erythromycin.
4. 4. Isoniazid.
Gráfico de respuestas
Comentario

!
!
!
!
Erythromycin. Whooping cough starts with a catarrhal phase, followed by a paroxysmal phase. The
first line of antibiotics recommended are macrolids (erythromycin, azithromycin).(R3)

67. Un lactante a término de 2100 gramos presenta irritabilidad, febrícula y temblor amplio
a las 36 horas de nacer. Se alimenta mal y tiene diarrea y obstrucción nasal. ¿Cuál es el
diagnóstico más probable?:

1. 1. Hipocalcemia.
2. 2. Hipomagnesemia.
3. 3. Déficit de piridoxina.
4. 4. Abstención de heroína en una madre drogadicta.
Gráfico de respuestas
Comentario

El síndrome de abstinencia a opiáceos puede cursar clínicamente con gran variedad de


sintomatología, que abarca desde la irritabilidad (llanto, temblores, crisis comiciales, hiperfagia,
taquicardia, polipnea, sudoración) hasta síntomas menos específicos como el despeño diarreico, la
inestabilidad térmica y la rinorrea. Recuerde que el manejo de este síndrome consiste en la
administración de fenobarbital.(R4)

68. Uno de estos tumores pulmonares NO es susceptible de tratamiento quirúrgico con


intención curativa:

1. 1. T3 N1 M0.
2. 2. T2 N1 M0.
3. 3. T1 N2 M0.
4. 4. T4 N1 M0.
Gráfico de respuestas
Comentario

Pregunta importante sobre el tratamiento del cáncer no microcítico de pulmón. Recuerde que la
cirugía con intención curativa no se intenta en los estadios T4 y M1.(R4)

69. En relación a la vulvovaginitis, indique cuál de las siguientes alternativas no guarda


correspondencia:

1. 1. Infección por monilia - gestación.


2. 2. Infección por gonococo - flujo vaginal sanguinolento.
3. 3. Infección por Tricomonas - flujo vaginal espumoso.
4. 4. Infección por Herpes - ulceración.
Gráfico de respuestas
Comentario

Pregunta de un tema muy importante. La que no guardia correspondencia es la 2: la gonorrea en la


mujer suele ser asintomática, o como mucho dar cierto flujo purulento.

Recuerden que moniliasis (respuesta 1) es otra forma de llamar a la candidiasis.(R2)

!
!
!
!
70. Hombre de 80 años, que consulta por un primer episodio de hematoquecia de 7 días
de evolución, sin otros síntomas asociados. En el momento de la exploración física
llevaba dos días asintomático. El tacto rectal no mostró alteraciones relevantes, y una
sigmoidocolonoscopia que llegó hasta el ángulo esplénico del colón fue normal. No
existían alteraciones en la BH, la VSG era normal. El diagnóstico más probable en este
paciente sería:

1. 1. Angiodisplasia.
2. 2. Carcinoma de colon ascendente.
3. 3. Hemorragia diverticular.
4. 4. Tiflitis.
Gráfico de respuestas
Comentario

La causa más frecuente de hemorragia digestiva baja en un anciano es el divertículo sangrante.


Debe tener cuidado con esta pregunta, porque encierra una trampa muy sutil.

Aunque la localización más frecuente de los divertículos es a nivel del colon descendente-sigma, la
mayoría de los divertículos sangrantes se localizan en el colon derecho. Dado que la colonoscopia
no ha progresado más del ángulo esplénico, no habríamos tenido opción de verlos, con lo que la
respuesta correcta sería la 3.

Tenga cuidado con la respuesta 2. La angiodisplasia de colon es la causa más frecuente de


hemorragia digestiva baja RECIDIVANTE en el anciano, y aquí nos hablan de un solo
episodio.(R3)

71. Femenino de 38 años con nódulo mamaría indoloro, de bordes imprecisos. La


mastografía revela imagen nodular, con espículas en todos sus márgenes, y 10
microcalcificaciones finas agrupadas en el interior. El diagnóstico más probable entre los
que se citen es:

1. 1. Fibroadenoma.
2. 2. Quiste.
3. 3. Carcinoma.
4. 4. Mamografía normal para la edad del paciente.
Gráfico de respuestas
Comentario

Las calcificaciones finas agrupadas en una mamografia son altamente sugestivas de malignidad.
Este dato, junto a un nódulo en mama con imagen espiculada nos debe hacer pensar directamente
en un carcinoma de mama como primera opción diagnóstica.(R3)

72. Una mujer de 37 años con fenómeno de Raynaud se queja de debilidad progresiva
con incapacidad para incorporarse a partir de la posición de sentada sin ayuda. En la
exploración, la paciente tiene unos dedos tumefactos "en salchicha", alopecia, zonas
parcheadas de eritema en los nudillos, telangiectasias faciales y debilidad muscular
proximal. Las pruebas de laboratorio incluyen una biometría hemática y una bioquímica
normal, excepto por una creatina fosfoquinasa de 4.5 µkat/L (270U/L) y una aldolasa de
500 µkat/L (30 U/L). Se encuentra el siguiente perfil serológico: factor reumatoide positivo
a 1:1 600; ANA positivo a 1:1 600 con un patrón moteado y títulos muy altos de
anticuerpos frente al
!
!
!
!
componente ribonucleoproteico sensible a ribonucleasa del antígeno nuclear extraíble.
Esta paciente tiene probablemente:

1. 1. Artritis reumatoide precoz.


2. 2. Esclerosis sistémica.
3. 3. Dermatomiositis.
4. 4. Enfermedad mixta del tejido conectivo.
Gráfico de respuestas
Comentario

Este tipo de enunciados asustan a la vista, pero resume a la perfección lo que es la EMTC, aunque
no sea muy rentable. Es una paciente con anti-RNP positivos a títulos altos (los anticuerpos
imprescindibles en el diagnóstico de esta entidad, lo más importante a recordar), y
secundariamente otros anticuerpos que normalmente asociamos a AR y LES (FR y ANAs
respectivamente). Tiene síntomas de varias enfermedades que podemos diferenciar en ese cóctel:
1) de esclerodermia limitada: Raynaud, afectación cutánea con edema en manos y alopecia
(pudiera tener esclerodactilia en el futuro?) y telangiectasias; 2) de miopatía inflamatoria: CPK
elevada con debilidad muscular proximal y eritema en los nudillos (pápulas de Gottron); 3) de
Lupus; 4) de artritis reumatoide (de estas últimas no menciona síntomas, pero con las otras cumple
criterios de EMTC: anti- RNP, edema de manos, miositis y Raynaud).(R4)

73. ¿Cuál es el tratamiento médico de elección del hipertiroidismo materno durante el


embarazo?:

1. 1. Metimazol.
2. 2. Propiltiouracilo.
3. 3. Yoduro.
4. 4. Propranolol.
Gráfico de respuestas
Comentario

La enfermedad de Graves es autoinmune y el hipertiroidismo no sigue un curso paralelo al de


oftalmopatía en muchos casos. El tratamiento debe individualizarse dependiendo del grupo de
edad (niños y jóvenes, adultos o ancianos) y ante la presencia de embarazo en una mujer. En
general, en nuestro medio se recomiendan tratamiento con antitiroideos prolongados en niños,
jóvenes y adultos (entre 12 y 24 meses) para conseguir un efecto inmunomodulador y así evitar
brotes de hipertiroidismo futuros. En el caso de la mujer embarazada se emplean antitiroideos,
preferiblemente propiltiouracilo, a la dosis mínima eficaz para mantener los niveles de las
hormonas tiroideas de la madre en el límite alto de la normalidad y así evitar el hipotiroidismo fetal.
El I-131 está contraindicado en embarazadas. En caso de falta de control con antitiroideos podría
estar indicada la cirugía.(R2)

74. Lactante de 3 meses que acude a su consulta porque hoy ha comenzado con tos,
mocos y ruidos respiratorios. Hace bien las tomas. Tiene un hermano de dos años que
ha estado acatarrado los días previos. En la exploración encontramos espiración
alargada en ambas bases pulmonares con algún crepitante aislado y ruidos de
secreciones de vía aérea superior. Tiene una temperatura de 37.8ºC. ¿Cuál de los
siguientes gérmenes se ha asociado con mayor frecuencia con el cuadro clínico que
sospecha?:

1. 1. Adenovirus.

!
!
!
!
2. 2. Virus parainfluenzae.
3. 3. Mycoplasma.
4. 4. Enterovirus.
Gráfico de respuestas
Comentario

Esta pregunta es muy importante nos presenta una caso clínico de bronquiolitis, algo que debemos
saber reconocer sin problemas. El agente causal más frecuente es el virus respiratotio sincitial
seguido del parainfluenzae, que es el germen que nos están preguntando esta vez. Recuerde que
es el responsable de la mayoría de los casos de laringitis aguda.(R2)

75. ¿Cuál de los siguientes enunciados NO es contraindicación de tratamiento


trombolítico?:

1. 1. Diátesis hemorrágica.
2. 2. Diabetes mellitus.
3. 3. Antecedentes de hemorragia cerebral.
4. 4. Intervenciones mayores recientes.
Gráfico de respuestas
Comentario
Pregunta sencilla pues si a todos los diabéticos que tienen un IAM no los podemos fibrinolisar,
estos medicamentos se usarían muy poco. Las contraíndicaciones absolutas de la fibrinolisis son
las siquientes: - Hemorragia activa. - Sospecha de rotura cardiaca. - Disección aórtica. -
Antecedentes de ictus hemorrágicos. - Cirugía o traumatismo craneal <2 meses. - Neoplasia
intracraneal, fístula o aneurisma. - Ictus no hemorrágico <6 meses. - Traumatismo importante <14
días. - Embarazo. - Hemorragia digestiva o urinaria <14 días.(R2)

76. ¿Qué síndrome paraneoplásico NO es típico del neuroblastoma?

1. 1. Hipertensión arterial sistémica.


2. 2. Diarrea secretora.
3. 3. Poliglobulia.
4. 4. Hematoma palpebral lineal.
Gráfico de respuestas
Comentario

El tumor extracraneal sólido más frecuente es el neuroblastoma, siendo el tumor abdominal


principal. Deriva de cresta neural, por lo que no es de extrañar que libere entre otras aminas:
catecolaminas y origine HTA, VIP con aparición de diarrea secretora.

El síndrome de Kinsbourne es el síndrome opsoclono-mioclono, caracterizado por ataxia,


mioclonías con movimientos conjugados de ambos ojos y desarrollo de demencia progresiva.

La poliglobulia es más típica del nefroblastoma o tumor de Wilms (opción 3 falsa).(R3)

77. De los siguientes factores hay uno que no se ha descrito como etiología del retraso
de crecimiento intrauterino. Indíquelo:

1. 1. Heroína.
2. 2. Preeclampsia.

!
!
!
!
3. 3. Amenaza de aborto en el primer trimestre.
4. 4. Diabetes gestacional.
Gráfico de respuestas
Comentario

Pregunta difícil sobre la etiología del CIR. Recuerde que la causa más frecuente de CIR es la
alteración de los flujos utero-placentarios. Así, si el aporte sanguíneo al feto está restringido, habrá
un retraso en su crecimiento. Pueden ser causa de esta alteración de flujos todas aquellas
patologías que afecten al tejido vascular como son las toxicomanías (opción 1), la preeclampsia
(opción 2) y enfermedades metabólicas con afectación vascular como puede ser una diabetes
gestacional mal controlada (opción 4). Fíjese que la diabetes gestacional puede ser causa tanto de
CIR como de macrosomía fetal. Lo que no es causa de CIR es que durante el primer trimestre
ocurra un episodio de amenaza de aborto, por lo que la opción que debemos señalar es la nº
3.(R3)

78. ¿Cuál es el tratamiento indicado en un paciente con enfermedad de Paget localizada


y poco sintomática?

1. 1. Calcitonina.
2. 2. Vigilancia sin tratamiento (controlando fosfatasa alcalina).
3. 3. Etidronato.
4. 4. Indometacina.
Gráfico de respuestas
Comentario

La enfermedad de Paget es más frecuente en hombres y su prevalencia aumenta con la edad,


alcanzando su pico máximo hacia los 65 años. Hay una fuerte tendencia a la agregación familiar.

La característica principal es el aumento en la resorción ósea, seguido de un incremento en la


síntesis compensatorio. El recambio óseo puede ser 20 superior al normal.

Lo más frecuente es que sea asintomática. El dolor óseo primario es la manifestación clínica más
habitual. Puede provocar deformaciones progresivas, cefalea, pérdida de audición (por afectación
de la cadena de huesecillos o compresión del VIII par) o paraplejia (por compresión medular). Las
complicaciones más importantes son IC de alto gasto, cálculos urinarios por hipercalciuria y
sarcoma (1% de los pacientes con Paget).

Laboratorio: lo más importante es la elevación de fosfatasa alcalina e hidroxiprolina. La


gammagrafía se utiliza para conocer la extensión de la enfermedad.

Solo requieren tratamiento aquellos pacientes sintomáticos o aquellos que se van a someter a una
Qx ortopédica.(R2)

79. Paciente primigesta a término de 39 años, con IMC de 31, a la que, tras 12 horas de
bolsa rota, se decide inducción del parto con oxitocina, terminando el parto mediante
cesárea por fracaso de inducción. A los 4 días de la cesárea comienza con fiebre,
afectación del estado general, taquicardia y dolor hipogástrico. La exploración física
pone de manifiesto una subinvolución uterina, sensibilidad y dolor uterinos con loquios
malolientes. Ante la sospecha clínica que usted tiene, ¿cuál de las siguientes medidas le
parece ERRÓNEA?:

!
!
!
!
1. 1. Gentamicina y clindamicina iv.
2. 2. Oxitocina iv.
3. 3. Legrado uterino bajo control por ultrasonido.
4. 4. Paracetamol iv.
Gráfico de respuestas
Comentario

La paciente presenta un claro caso de endometritis, siendo más fecuente en paciente con bolsa
rota de muchas horas de evolución, partos prolongados o cesáreas (más frecuentes que en el
parto vaginal). El tratamiento consiste en antibioterapia junto con uterotónicos (oxitócicos,
ergotamínicos) y medidas de soporte como antipiréticos. El legrado uterino se realiza en casos de
retención de restos trofoblásticos. En la endometritis no se realiza legrado, es más, se puede
producir una diseminación de la infección y mayor riesgo de perforación uterina.(R3)

80. ¿Cuál de los siguientes procesos NO es una complicación de la inmovilización


crónica en un paciente encamado?

1. 1. Diarrea.
2. 2. Depresión.
3. 3. Amiotrofia.
4. 4. Neumonías o atelectasias pulmonares.
Gráfico de respuestas
Comentario

Pregunta muy fácil. Se intuye que por inmovilización, al no usar los músculos corporales, dé lugar a
amiotrofias, úlceras por decúbito por ectasia venosa, neumonías o atelectasias pulmonares por no
movilización del aire y depresión debido a las limitaciones de estar en cama.

Lo que no es una complicación es la diarrea, ya que precisamente el guardar cama produce el


efecto opuesto, el estreñimiento debido a una disminución de peristalsis. De hecho, nos
encontramos un cuadro denominado síndrome de Ogilvie, que cursa como una pseudoobstrucción
en este tipo de circunstancias.(R1)

81. Paciente femenino de 42 años que acude al servicio de urgencias con un cuadro de
obstrucción intestinal. En su historia sólo tiene como antecedentes de interés una
apendicectomía hace 5 años. La clínica y la radiografía simple orientan más hacia una
obstrucción de intestino delgado. ¿Cuál suele ser la causa más frecuente que origina
este cuadro?:

1. 1. Hernia estrangulada.
2. 2. Invaginación intestinal.
3. 3. Carcinoide de íleon.
4. 4. Adherencias postcirugía.
Gráfico de respuestas
Comentario

Pregunta que se contesta de forma directa si se conocen las dos principales causas de obstrucción
intestinal. Saber este tipo de datos resulta importante de cara al ENARM. La causa más frecuente
de obstrucción del ID son las adherencias, seguidas de las hernias (que son la primera causa en
pacientes sin cirugía previa). Ya de paso, cabe mencionar que la causa más frecuente de la
obstrucción del intestino grueso es el cáncer colorrectal, sobre todo a nivel de recto-sigma.(R4)

!
!
!
!
82. Un niño capaz de correr, subir escaleras solo, sin alternar los pies, hacer una torre de
seis cubos y elaborar frases de tres palabras, presenta un desarrollo psicomotor propio
de la siguiente edad:

1. 1. 15 meses.
2. 2. 18 meses.
3. 3. 24 meses.
4. 4. 48 meses.
Gráfico de respuestas
Comentario

Pregunta de crecimiento y desarrollo. Puede ser un poco difícil si no se ha estudiado el tema con
anterioridad, pero se puede intuir de forma general.

A los dos años, el niño debe ser capaz de correr, subir escaleras y patear balones sin perder el
equilibrio, bailan al ritmo de la música, contruyen frases sencillas de más de tres palabras, con
incontinencia verbal y preguntas continuas, así como desarrollo motriz con capacidad motora
compleja como construir torres de más de 4 cubos.(R3)

83. Mujer de 55 años, intervenida de apendicitis hace 24 años, que acude porque, estando
previamente bien, ha comenzado con vómitos de repetición tras desayunar, hace unas
12 horas, asociado a distensión abdominal. Ha presentado una evacuación diarreica a las
pocas horas de comenzar el cuadro. En la exploración se observa distensión abdominal,
timpanismo y aumento de ruidos intestinales, pero no irritación peritoneal. Los
laboratorios no presenta alteraciones. En la radiografía de abdomen aparece dilatación
de asas de intestino delgado, sin poder observarse gas en el ámpula rectal. Respecto a
este caso, es cierto que:

Se trata de un cuadro de obstrucción intestinal por bridas que requiere laparotomía de


1. 1.
urgencia.
Lo más probable es que nos encontremos ante un caso de obstrucción a nivel de intestino
2. 2.
grueso.
El hecho de que la paciente haya presentado una evacuación diarreica nos permite excluir
3. 3. el diagnóstico de obstrucción intestinal. Son necesarios más estudios de imagen para
asegurar un diagnóstico.
El manejo inicial debe ser conservador, con sueroterapia, aspiración nasogástrica y
4. 4.
vigilancia clínica, de laboratorio y radiográfica periódica.
Gráfico de respuestas
Comentario

Caso clínico representativo de un cuadro de obstrucción intestinal a nivel de intestino delgado. El


cuadro es típico: antecedente de intervencón previa que se relaciona con la principal causa de
obstrucción de intestino delgado (las adherencias); cuadro clínico con dolor abdominal, vómitos,
distensión abdominal, hiperperistaltismo, timpanismo y radiografía compatible (dilatación de asas
con el colon deprovisto de gas). Respecto al manejo la mayoria se benefician de un tratamiento
conservador con SNG y reposición hidroelectrolitica por lo que la respuesta 4 es la correcta. Puede
que las otras sean parcialmente ciertas, pero la 4 es clarísima: razonable, abierta, incluye
seguimiento y, sobretodo, resume perfectamente el manejo inicial de una obstrucción
intestinal.(R4)

!
!
!
!
84. A 16-year-old female comes to the pediatrician's office presenting with fever, sore
throat, odynophagia and malaise. Physical examination shows bilateral axillary, cervical
and inguinal lymphadenopathies. Oral examination shows pharyngeal erythema and
swollen tonsils with a witish covering. Vital signs are within normal limits. She self-
prescribed amoxicillin presenting with a maculopapular erythematous generalized rash
after administration. Which is the most likely causative agent related to this clinical
manifestation?

1. 1. Epstein Barr Virus.


2. 2. Human Immunodeficiency Virus.
3. 3. Neisseria meningitidis.
4. 4. Togavirus.
Gráfico de respuestas
Comentario

Epstein Barr Virus. The symptoms presented are very suggestive of a viral infectious disease.
Differential diagnosis of acute infectious mononucleosis should include acute Cytomegalovirus
infection and Toxoplasma gondii infections. However, the rash after antibiotic intake (penicillins or
related) should make us suspect an infection caused by Epstein Barr Virus.(R1)

85. Respecto a la cirugía de derivación coronaria ("by pass") es FALSO que:

1. 1. Los riesgos quirúrgicos son muy pocos, con una mortalidad inferior al 1%.
La permeabilidad a largo plazo es más alta con los injertos de vena safena que con los de
2. 2.
la arteria mamaria interna.
La intervención reduce la mortalidad en los pacientes con estenosis del tronco principal
3. 3.
izquierdo
4. 4. El alivio de la angina se obtiene en el 85% de los pacientes.
Gráfico de respuestas
Comentario

Es una pregunta sencilla en la que único que tenemos que saber es que los injertos con arteria son
mejores porque duran más. El flujo que proporcionan y las dificultades quirúrgicas son similares a
las de los injertos venosos. Sin embrago, los injertos venosos presentan un 10-20% de oclusión al
año de su implante. Los años posteriores, la incidencia de oclusión es del 2% anual. Los injertos
arteriales de mamaria interna, por el contrario, permanecen permeables en un 90% a los 10 años
de la intervención. En la práctica, se suele reservar la arteria mamaria interna [normalmente la
izquierda] para la anastomosis con la arteria descendente anterior, utilizando injertos venosos de
safena para el resto de vasos afectados. De todos modos, la tendencia debe ser a intentar
revascularizaciones arteriales completas [esto es, que todos los puentes se hagan con arterias],
para lo que además de las arterias mamarias internas, se suele utilizar la arteria radial. Por tanto, la
respuesta 2 es la falsa. El resto de opciones son correctas, y nos exponen la efectividad y relativo
bajo riesgo de la cirugía de revascularización.(R2)

86. A 32-year-old homeless man is brought to the ER by the police. He was found in poor
condition, sweating and drooling. Physical examination reveals: BP 130/70 mm Hg, pulse
110/min and temperature 38.5ºC. His mouth is swollen and red, and there is a tender,
indurated swelling with palpable crepitus in the submandibular area. What is the most
likely diagnosis?

1. 1. Acute sialadenitis.

!
!
!
!
2. 2. Diphtheria.
3. 3. Ludwig's angina.
4. 4. Parotid gland cancer.
Gráfico de respuestas
Comentario

Pregunta sencilla sobre la angina de Ludwig, respuesta 3 correcta.

Los datos clave de esta pregunta son los siguientes: vagabundo, en malas condiciones generales,
con diaforesis y babeando. Los signos vitales con fiebre y taquicardia. La boca edematosa y roja,
asociado a crepitación subamndibular.

Es un tipo de celulitis que compromete el piso de la boca, debajo de la lengua. A menudo ocurre
después de una infección de las raíces de los dientes (como absceso dental) o una lesión en la
boca.

El tratamiento consiste en antibioterapia parenteral cubriendo anaerobios y Gram negativos y en


ocasiones es necesario el drenaje quirúrgico.(R3)

87. Un niño de 18 meses de edad llega a Urgencias después de una convulsión tónico
clónica que duro menos de 15 minutos Presenta fiebre desde las últimas 24 horas. Su
primera decisión es:

1. 1. Buscar la causa de la fiebre.


2. 2. Realizar punción lumbar.
3. 3. Administrar metamizol.
4. 4. Iniciar una vía intravenosa.
Gráfico de respuestas
Comentario

Pregunta sencilla sobre crisis febriles. La respuesta correcta es la número 1.

Las crisis febriles son un proceso típico de la edad infantil entre los 3 meses y los 5 años de edad,
que se relaciona más frecuentemente con el aumento de temperatura, lo que da lugar a una crisis
el primer día de un proceso febril, independientemente del origen del mismo.

Las crisis febriles simples son generalizadas, duran menos de 15 minutos, presentan buena
recuperación posterior y los hallazgos en el periodo intercrítico son normales o negativos. Con
frecuencia existen antecedentes familiares de crisis febriles o de epilepsia; son recurrentes en un
tercio de los casos, aunque sólo el 10% de los pacientes sufre más de dos episodios, de forma
más probable si las crisis se producen en el primer año de vida; no se relacionan con un mayor
riesgo de presentar epilepsia.

Las crisis febriles complejas son las que tienen signos focales, una duración superior a 15 minutos,
o se repiten en el curso del mismo episodio febril; se relacionan con un 2-5% de incremento del
riesgo de sufrir epilepsia posteriormente.

Las crisis febriles pueden tratarse con diazepam vía rectal o IV, aunque dado que ceden
espontáneamente, el manejo más adecuado es el control de la temperatura, preferiblemente con
paracetamol. En pacientes con crisis febriles típicas recurrentes puede administrarse diazepam oral
o rectal en situaciones

!
!
!
!
de ascenso térmico. No está indicado el tratamiento continuado con anticomiciales como profilaxis
de crisis febriles.(R1)

88. Paciente de 26 años, fumadora, sin hijos


ni pareja estable, es remitida a su consulta de ginecología por citología positiva para HSIL
(lesión intraepitelial de alto grado). Usted realiza una colposcopia ante semejante
hallazgo, obteniéndose la imagen que se muestra. ¿Cuál es la conducta más indicada a
continuación?:

1. 1. Tratamiento con antiinflamatorios locales.


2. 2. Estudio de extensión de la enfermedad.
3. 3. Biopsia de la lesión.
4. 4. Conización cervical.
Gráfico de respuestas
Comentario

En la imagen se ve ina lesión en "mosaico", con áreas blancas separadas por vasos, dando ese
aspecto característico. Es un patrón sugestivo de lesión neoplásica o preneoplásica de cérvix; por
tanto, el primer paso es biopsiarlo para confirmar su naturaleza maligna.(R3)

89. Con respecto a la patología que presenta la paciente, ¿cuál de los siguientes
hallazgos NO sería sugestivo de patología cervical?:

1. 1. Eritroplasia cervical.
2. 2. Áreas de neoformación vascular.
3. 3. Superficie ulcerada.
4. 4. Leucoplasia.
Gráfico de respuestas
Comentario

A continuación, te recordamos los hallazgos colposcópicos anormales:

1.- Epitelio acetoblanco.

!
!
!
!
2.- Epitelio yodonegativo.

3.- Cambios de coloración (leucoplasia).

4.- Neoformación vascular.

5.- Superficie irregular, con pérdida del epitelio normal, con ulceración.

La ectopia o eritroplasia cervical consiste en la presencia de epitelio cilíndrico por debajo del orificio
cervical externo, visible en la colposcopia desde la vagina, quedando en contacto con el medio
vaginal hostil. Aunque la mayor parte de las veces son lesiones asintomáticas, pueden originar
leucorrea y hemorragia postcoital. El diagnóstico se realiza de visu mediante colposcopía, y
pueden eliminarse si producen clínica (a veces produce sangrado). No obstante, no se relaciona
con patología cervical maligna (respuesta 1 correcta).(R1)

90. Un niño con tetralogía de Fallot, tras haber estado llorando, comienza a mostrarse
agitado, taquipneico y cianótico, llegando a perder el conocimiento. ¿Cuál de las
siguientes medidas sería INCORRECTA?:

1. 1. Administración de morfina.
2. 2. Colocación en posición genupectoral.
3. 3. Beta-bloqueantes.
4. 4. Efedrina.
Gráfico de respuestas
Comentario
En casi todos los pacientes con tetralogía de Fallot se suele realizar una corrección quirúrgica de la
malformación. No obstante, hasta el momento de la cirugía el tratamiento de las crisis hipoxémicas
que sufren estos pacientes es médico. Se debe evitar que el niño realice esfuerzos intensos y
estados de agitación que contribuyan al espasmo infundibular. Si el niño presenta una crisis
hipoxémica aguda, se le debe colocar en posición genupectoral (acercar las rodillas al pecho) y
debe ser trasladado rápidamente al hospital para ofrecerle tratamiento urgente a base de oxígeno,
sedación con barbitúricos o morfina, bicarbonato si existe acidosis y betabloqueantes para relajar
el infundíbulo pulmonar.(R4)

91. ¿Cuál de las siguientes afirmaciones sobre el linfoma de Burkitt es INCORRECTA?:

1. 1. Existe una traslocación del oncogén c-ras.


Se trata de un linfoma difuso de células pequeñas no hendidas, con imagen en "cielo
2. 2.
estrellado".
3. 3. En los países occidentales afecta con frecuencia la cavidad abdominal.
4. 4. Debe efectuarse profilaxis del SNC como parte del tratamiento.
Gráfico de respuestas
Comentario

El oncogén relacionado con el linfoma de Burkitt es el c - myc t(8;14). Respuesta incorrecta la 1. El


resto de respuestas son correctas.(R1)

92. An 82-year-old male is being discharged tomorrow after a 10-day hospital stay for
acute respiratory failure. The nurse reports that the patient appears to be in an altered
mental state and has been incontinent for the past 24 hours. His medical history is

!
!
!
!
irrelevant. His blood pressure is 155/95 and his pulse 110/min. His respirations are 16/min.
The patient's pulse oximetry shows 99% on room air. The abdomen is soft. Rectal
examination shows no abnormalities. Neurological examination is normal. Mental status
examination shows disorientation to time, space and person. The patient's labs reveal:
urine: blood negative, ketones negative, glucose negative, leukocyte esterase postivive,
nitrites positive, WBC 40-50+/hpf, RBC 10-20+/hpf, bacteria: many. Which of the following
is the best next step?

1. 1. Leave a Foley catheter.


2. 2. Start trimethoprim and sulfamethoxazole.
3. 3. Give a dose of haloperidol.
4. 4. NSAIDs.
Gráfico de respuestas
Comentario

Start trimethoprim and sulfamethoxazole. Lab results show a urine infection. Treatment is
intravenous antibiotics.(R2)

93. A 2-year-old boy is brought to the emergency department because his parents have
noted that he tends to take his hands to his ears and is a little bit more irritable than usual.
Physical examination shows no fever or inflammatory signs. Ear examination reveals
amber liquid behind eardrums. Further questioning to his parents reveals that he had an
acute episode of otalgia and fever a month ago, for which he was prescribed a 7-day
course of amoxicillin. Which of the following is the most likely diagnosis?

1. 1. Recurrent Acute Otitis Media.


2. 2. Otitis media with effusion.
3. 3. Chronic Suppurative Otitis Media.
4. 4. Cholesteatoma.
Gráfico de respuestas
Comentario

Pregunta sencilla sobre las complicaciones de las otitis media (respuesta correcta 2). La clave es el
líquido ámbar que se observa, sin datos de fiebre o signos inflamatorios.(R2)

94. ¿Cuál es la indicación fundamental del tratamiento hormonal sustitutivo del


climaterio?

1. 1. Aliviar los sofocos y síntomas climatéricos.


2. 2. Retrasar las lesiones del tejido conjuntivo (prolapso, incontinencia).
3. 3. Prevenir las lesiones vasculares.
4. 4. Prevenir la osteoporosis.
Gráfico de respuestas
Comentario

El motivo fundamental del tratamiento hormonal sustitutivo (THS) no es la prevención


cardiovascular ni la de la osteoporosis, sino aliviar los sofocos y el resto de los síntomas
relacionados con el climaterio.

!
!
!
!
En el pasado, se creía que la THS podría tener algún papel protector desde el punto de vista
cardiovascular, pero algunos estudios demostraron que esto no era así, e incluso se asociaban a
un incremento del riesgo. Por otra parte, la THS tampoco ha demostrado mayores beneficios en la
prevención de la osteoporosis.(R1)

95. Paciente de 64 años que acude a consulta refiriendo una historia de dificultad
respiratoria, progresiva con el ejercicio, a la que en los últimos meses se ha unido dolor
opresivo centrotorácico irradiado al cuello que aparece a los cinco minutos de un
ejercicio y cede con el reposo. En la última semana ha perdido dos veces el conocimiento
mientras corría a tomar el autobús. Respecto a la patología que probablemente padece
nuestro paciente, señale la FALSA:

1. 1. El pulso arterial probablemente muestre las características de pulso Corrigan.


2. 2. En la auscultación probablemente destaque un soplo sistólico irradiado a carótidas.
3. 3. Puede auscultarse un cuarto tono en el ápex.
La hipertensión arterial significativa es poco probable si la patología que padece es muy
4. 4.
marcada.
Gráfico de respuestas
Comentario

Por la historia que se nos cuenta (disnea + síncopes + angina), lo más probable es que el paciente
tenga una estenosis aórtica. Es típico de esta enfermedad:

- Soplo sistólico irradiado a carótidas

- Cuarto tono en ápex

- Pulso parvus et tardus

La hipertensión es rara. El pulso de Corrigan es característico de la insuficiencia aórtica, no de la


estenosis.(R1)

96. ¿Cuál de los siguientes microorganismos infecta principalmente las células del
endotelio vascular?

1. 1. Salmonella typhi.
2. 2. Rikettsia typhi.
3. 3. Haemophilus influenzae.
4. 4. Streptococcus agalactiae.
Gráfico de respuestas
Comentario

Pregunta directa de dificultad alta.

La R. typhi ocasiona el tifus endémico o murino, que es transmitido por una pulga. No debe olvidar
que las fiebres manchadas están producidas por Rickettsias, son transmitidas por vectores y que
procuran un cuadro clínico gripal junto con un exantema generalizado que, característicamente,
afecta a palmas y plantas. Las Rickettsias tienen tropismo por el endotelio vascular (opción 2
cierta), y en casos de infecciones muy graves, son capaces de lesionarlo y producir edemas,
hemorragias, fracaso renal, edema pulmonar no cardiogénico o encefalopatía por edema cerebral.

!
!
!
!
El diagnóstico es serológico y el tratamiento de elección es la doxiciclina, asociada a corticoides en
las formas graves.

La S. typhi origina la fiebre tifoidea. Esta bacteria se ingiere con agua o alimentos contaminados y
penetra la mucosa intestinal intacta, alcanza las placas de Peyer y los ganglios linfáticos y de ahí
pasa a la sangre dando el cuadro clínico característico (fiebre, dolor abdominal, roséola tifoidea,
cefalea, esplenomegalia, bradicardia...). No debe confundirla con la Salmonella que coloniza los
aneurismas (no propiamente el endotelio vascular) y produce aneurismas micóticos, que es la
Salmonella enteritidis (opción 1).

H. influenzae causa, sobre todo, infecciones en los bebés y en los niños, y los cuadros más
frecuentes son otitis medias, conjuntivitis, sinusitis e incluso neumonía o meningitis (opción 3).

El S. agalactiae, o estreptococo beta-hemolítico del grupo b, es causa significativa de sepsis


neonatal y de infecciones en gestantes y adultos inmunocomprometidos (opción 4) (diabéticos,
alcohólicos, etc.). Esta bacteria es un coco grampositivo aeróbico y encapsulado. Comúnmente
coloniza la vagina y la región anorrectal de la población femenina, sobre todo en mujeres
embarazadas. En los neonatos causa neumonía, sepsis y meningitis con una mortalidad alta. Para
la prevención de la sepsis neonatal, se administra a la madre antibioterapia intravenosa intraparto
(generalmente ampicilina o penicilina). La profilaxis se hace en pacientes con cultivo rectovaginal
positivo a las 35 semanas de embarazo o en las que, teniendo cultivo negativo o desconocido,
tienen factores de riesgo. Para el tratamiento de la infección por S. agalactiae se requieren dosis
altas de penicilina, muchas veces asociada a aminoglucósidos.(R2)

Diferencias entre fiebre botonosamediterránea y fiebre Q

!
!
!
!
97. El niño cuadruplica su peso de nacimiento aproximadamente a los:
1. 1. 4 meses.
2. 2. 1 año.
3. 3. 2 años.
4. 4. 3 años.
Gráfico de respuestas
Comentario

El tema de crecimiento y desarrollo es muy preguntado en el ENARM. En cuanto al peso del niño
es importante que recuerde algunos datos básicos: El peso al nacimiento: 3.5 Kg, lo duplica a los 5
meses, lo triplica al año y cuadruplica a los 2 años. (El peso se cuadruplica a los 2 años y la talla
se duplica a los 4 años).(R3)

98. ¿Por qué en la atelectasia NO suele ser importante la desaturación de O2?

1. 1. Broncoconstricción.
2. 2. Circulación colateral.
3. 3. Vasoconstricción selectiva.
4. 4. Cortocircuito a nivel central.
Gráfico de respuestas
Comentario

La atelectasia consiste en una zona del pulmón que no está ventilada, pero sí perfundida. Por
tanto, el problema que produce sería un shunt, y la sangre que pasase por los vasos de esta zona
no se oxigenaría. Sin embargo, también es cierto que la mayoría de las atelectasias no dan
demasiados problemas. Esto es debido a que la hipoxia alveolar produce vasoconstricción de los
vasos de la zona, con lo que apenas pasará sangre por ellos, y el shunt será menor (respuesta 3
correcta). Si se trata de una gran atelectasia, entonces el problema sería evidente, pero las
atelectasias pequeñas y medianas pueden pasar desapercibidas por este mecanismo.(R3)

99. En un paciente atendido seis horas después del inicio de peritonitis en diseminación
por diverticulitis perforada, cabría esperar los siguientes signos, EXCEPTO:

1. 1. Íleo adinámico.
2. 2. Trasudado de líquido con bajo contenido de proteínas.
3. 3. Acidosis metabólica.
4. 4. Choque hipovolémico.
Gráfico de respuestas
Comentario

Esta pregunta no es muy difícil teniendo en cuenta las características diferenciales entre la
peritonitis bacteriana primaria y secundaria.

Tras la perforación de una diverticulitis, así como en cualquier otra lesión de la pared intestinal, el
colon queda funcionalmente abolido (opción 1) y el consumo de oxígeno es menor. Además, la
perforación de una víscera hueca se sigue de una peritonitis secundaria con sus signos
abdominales característicos, los cuales preceden a los sistémicos como son: fiebre, taquicardia,
deshidratación, oliguria, choque y acidosis (opciones 3 y 4). El hallazgo de un líquido peritoneal
bajo en contenido proteico es un dato característico, así como factor de riesgo de una peritonitis
bacteriana espontánea.(R2)

!
!
!
!
100. ¿Qué técnica es la empleada más frecuentemente en el diagnóstico microbiológico
de la bronquiolitis por VRS?:

1. 1. Inmunofluorescencia directa en muestra de moco nasal.


2. 2. Cultivo viral en moco nasal.
3. 3. Inmunofluorescencia directa en muestra de frotis faríngeo.
4. 4. Inmunofluorescencia indirecta en muestra de moco nasal.
Gráfico de respuestas
Comentario
El fallar esta pregunta sobre diagnostico de bronquiolitis no tiene importancia. El diagnostico de
bronquiolitis es clínico y el etiológico se realiza a través de diversas técnicas: a) técnicas de
diagnostico rápido (24h): ELISA, IFD y IFI que permiten la identificación de antígenos del virus en
el moco nasal. La técnica mas empleada es la inmunofluorescencia por ser más sensible que el
ELISA b) el cultivo de las secreciones nasofaringeas se reserva para pacientes con test rápidos
negativos, pues el resultado tarda 1 semana c) Las pruebas serológicas se puede utilizar para
confirmar la infección pero su sensibilidad es baja en lactantes d) La PCR es mas sensible que la
inmunofluorescencia, pero no está disponible en la mayoría de los hospitales.(R1)

101. Regarding HIV infection in childhood, which of the following statements is FALSE?:

1. 1. All children born to HIV-positive mothers have antibodies against HIV at birth.
2. 2. Vertical transmission occurs only during labor.
3. 3. Maternal antibodies against HIV persist in the newborn for about ten months.
4. 4. HIV-infected children should be routinely vaccinated, when indicated.
Gráfico de respuestas
Comentario

La transmisión del VIH se produce durante el parto en la mayor parte de los casos. Sin embargo,
también es posible la transmisión transplacentaria (respuesta 2 falsa) y a través de la lactancia.

El resto de las opciones son correctas:

1.- Si la madre es seropositiva, el recién nacido también lo será, aunque no haya sido infectado
por el VIH, ya que los anticuerpos maternos atraviesan la placenta y serían detectados mediante el
ELISA.

3.- Los anticuerpos maternos persisten durante varios meses en sangre fetal, ya que la vida media
de las inmunoglobulinas es muy prolongada.

4.- Los niños infectados por el VIH, mientras no tengan inmunosupresión manifiesta, deben seguir
el mismo calendario vacunal que el resto de los niños. Mientras la inmunidad sea suficiente,
pueden y deben recibir vacunaciones como la triple vírica o la varicela, que reducirán la gravedad
de estas enfermedades en caso de padecerlas en fase de SIDA.(R2)

102. Un RN pretérmino de 29 semanas de edad gestacional y 950 g de peso es alimentado


por sonda nasogástrica y tratado con antibióticos por riesgo infeccioso. Presenta, a los
12 días de vida, distensión abdominal muy llamativa y heces sanguinolentas. ¿Qué
prueba le podría dar el signo diagnóstico de la patología que debe sospechar?

1. 1. Pruebas de coagulación.

!
!
!
!
2. 2. Biometría hemática con plaquetas.
3. 3. Radiografía simple de abdomen.
4. 4. Ecografía abdominal.
Gráfico de respuestas
Comentario

El caso que nos describen es muy importante: corresponde con una enterocolitis necrotizante. Esta
enfermedad afecta fundamentalmente a íleon distal y colon proximal. Suele relacionarse con
situaciones de hipoxia y bajo gasto. Clínicamente, debe sospecharla ante la presencia de
distensión abdominal y evacuaciones sanguinolentas. Existen varios signos radiológicos, pero el
más sugestivo es laneumatosis intestinal, que es diagnóstico de esta enfermedad. Por ello, en este
caso habría que solicitar una radiografía de abdomen (opción 3 correcta). Con respecto a este
hallazgo, es interesante que recuerde que, en el pasado, era considerado como un signo de muy
mal pronóstico. Sin embargo, hoy día se sabe que, en realidad, aparece en muchos más casos de
los que se pensaba y que por tanto no se le puede dar un significado tan ominoso.

El tratamiento consiste en dieta absoluta, sonda nasogástrica y sueroterapia. Debe administrase


además antibioterapia, cubriendo gérmenes anaerobios y gramnegativos. En caso de perforación
intestinal o refractariedad a antibióticos, se plantearía la cirugía. En el paciente de esta pregunta,
no nos han mencionado datos de perforación, por lo que aún no estaría indicada.(R3)

103. Cuál de las enfermedades de transmisión sexual produce infertilidad:

1. 1. Trichomoniasis.
2. 2. Gonorrea.
3. 3. Candidiasis.
4. 4. Condilomatosis.
Gráfico de respuestas
Comentario

Pregunta sencilla sobre ETS. De las opciones que dan ¿Cuál puede producir con mayor frecuencia
EPI y por consiguiente esterilidad? La respuesta correcta es la 2, el gonococo.(R2)

104. En la cirugía del cáncer de esófago es importante conocer bien la anatomía


esofágica. ¿Cuál de las siguientes afirmaciones es CORRECTA?

1. 1. El esófago abdominal es más largo que el cervical.


2. 2. El diámetro máximo del esófago normal es de 5 cm.
3. 3. El esófago torácico pasa por detrás del cayado aórtico.
4. 4. El epitelio esofágico normal es cilíndrico.
Gráfico de respuestas
Comentario

Una pregunta de cierta dificultad, ya que nos exige conocer ciertos detalles anatómicos con
bastante precisión.

•! R1: el esófago abdominal es considerablemente más corto que el cervical.


•! R2: el diámetro máximo del esófago, según Sabiston, alcanza los 2.5 cm, medidos en un
esofagograma con bario.
•! R4: el epitelio esofágico normal es escamoso estratificado, no cilíndrico. Si fuese cilíndrico,
obedecería a fenómenos de metaplasia (esófago de Barrett).

!
!
!
!
La única respuesta correcta es la 3. Efectivamente, el esófago torácico pasa por detrás del cayado
aórtico, como puede verse en la figura adjunta.(R3)

Nervios laríngeos recurrentes

105. Pablo es un paciente de 8 años con diagnostico de asma severa que utiliza
corticoides inhalados por periodo prolongados, dentro de los posibles hallazgos que se
podrían encontrar en Pablo tenemos:

1. 1. Supresión del eje hipotálamo hipofisario.


2. 2. Disminución en su talla final en forma marcada.
3. 3. Disminución de su talla final en forma moderada.
4. 4. Mayor riesgo de desarrollar cataratas.
Gráfico de respuestas
Comentario

La asministración prolongada de corticoides hace que estos ejerzcan un mecanismo feedback


negativo sobre el eje hipotalamo-hipofisario-suprarrenal de tal forma que se inhibe la liberación de
ACTH y de corticoides endógenos.(R1)

106. En relación a la evaluación del espermatograma. Marque lo CORRECTO:

1. 1. La licuefacción del semen ocurre entre 60 – 120 minutos.


2. 2. El pH seminal es de 7.5.
3. 3. Se considera espermatozoides móviles > 20%.
4. 4. Se denomina oligozoospermia a la falta de movilidad.
Gráfico de respuestas
Comentario

!
!
!
!
Pregunta muy difícil, que no debe preocupar demasiado. Se estudia la concentración, la movilidad,
vitalidad y la morfología de los mismos. Se considera normal a partir de 15 millones de
espermatozoides/mL, de los cuales deben ser móviles progresivos al menos el 32%. La
disminución en el número de espermatozoides se denomina oligozoospermia, y la reducción en la
movilidad se denomina astenozoospermia. También se analiza la densidad y el número de
leucocitos presentes en la muestra de semen. El pH del semen es de 7.5. Respuesta 3
correcta.(R3)

107. Después de la menarquía, el himen imperforado puede asociarse a los siguientes


signos y síntomas, EXCEPTO:

1. 1. Hematocolpo.
2. 2. Hipermenorrea.
3. 3. Hematometra.
4. 4. Retención urinaria.
Gráfico de respuestas
Comentario

Entre las opciones que nos dan, los efectos del himen imperforado son sobre todo derivados del
efecto masa por acúmulo de sangre, como el hematocolpos y la hematometra. Lo que no se
encuentra, dado que se encuentra imperforado, es la hipermenorrea.(R2)

108. Con respecto a los secuestros pulmonares, señale cuál de las siguientes
afirmaciones es FALSA:

1. 1. Es una zona de tejido pulmonar embrionario separada del árbol bronquial.


2. 2. Los secuestros intralobares drenan en la circulación pulmonar.
3. 3. Los secuestros siempre son irrigados por arterias que proceden de la circulación sistémica.
4. 4. Su complicación más frecuente es la malignización.
Gráfico de respuestas
Comentario

Las malformaciones pulmonares son poco relevantes, excepto el secuestro pulmonar, que es lo
único que merece la pena revisar de este tema. El secuestro pulmonar consiste en la presencia de
una masa de tejido pulmonar separada del parénquima sano, sin comunicación con la vía aérea
(opción 1 cierta). Todos los secuestros reciben irrigación sistémica (opción 3 cierta), pero el drenaje
es diferente. El extralobar, que tiene pleura propia, drena a la circulación sistémica, mientras que el
intralobar lo hace a los vasos pulmonares (opciones 2 cierta). Clínicamente, la mayoría de los
secuestros pulmonares son asintomáticos, hasta que se complican con una infección, no con una
malignización (opción 4 falsa). Cuando esto ocurre, la clínica consiste en hemoptisis y
expectoración purulenta. Recuerde que, anatómicamente, el 90% está en contacto con el
hemidiafragma izquierdo.(R4)

109. Mujer que acude a su ginecólogo para su revisión anual. En la exploración se


encuentra el anexo derecho aumentado de tamaño. Está prácticamente asintomática,
salvo que comenta que soporta mal el calor y que su apetito ha aumentado, aunque ha
perdido peso. ¿Qué tumor ovárico podría explicar este cuadro?:

1. 1. Tumor de Brenner.
2. 2. Disgerminoma.
3. 3. Struma ovarii.

!
!
!
!
4. 4. Tumor del seno endodérmico.
Gráfico de respuestas
Comentario

La paciente parece contar un cuadro de tirotoxicosis. Y, ¿hay algun tumor ovárico que pueda
producir tirotoxicosis?.

Pues sí, hay uno, el struma ovarii. Es un tipo especial de tumor germinal, que consiste en un tumor
formado por tejido tiroideo normal funcionante, que libera hormonas tiroideas.

Si no conocía su existencia, espero que después de esta pregunta lo recuerde para siempre de
este tumor tan singular.(R3)

110. ¿En cuál de estos pacientes que acudieron a un hospital con diagnóstico de
SCASEST estaría MENOS indicada una estrategia invasiva precoz?:

Masculino de 55 años con único episodio de dolor torácico de 30 minutos de duración y


1. 1.
elevación de troponinas.
Masculino de 70 años con episodio de dolor torácico de reposo claramente isquémico, sin
2. 2. alteraciones en el ECG relevantes, troponinas seriadas de cero sin recurrencia del dolor
desde el ingreso con tratamiento médico.
Masculino de 70 años, con triple by pass hace 3 años, asintomático hasta hace 1 mes, en
3. 3.
que comienza con episodios de angina
Masculino de 63 años con dolor torácico, insuficiencia cardíaca y FEVI moderadamente
4. 4.
deprimida.
Gráfico de respuestas
Comentario

Ante un paciente con sospecha de SCASEST hay que hacer una estratificación del riesgo de
muerte y eventos agudos a corto y largo plazo. Para ello existen diferentes tablas o scores, de
forma que a más puntos, mayor riesgo y por ello mayor beneficio de una estrategia invasiva
precoz. En todas los scores que tratan de predecir ese riesgo se incluye de forma sistemática las
siguientes variables: elevación de troponinas, DM, los descensos del ST con cada episodio de
dolor, los antecedentes previos de cardiopatía isquémica y la IC y/o FEVI deprimida. Si no tiene
antecedentes de cardiopatía isquémica, no es diabético, el ECG y las troponinas seriadas son
normales se considera un SCASEST de bajo riesgo y sería apropiada una estrategia inicial
conservadora empezando con tratamiento médico.(R2)

111. ¿Cuál es el fármaco de elección en los pacientes con angina de esfuerzo de grados
II a IV?:

1. 1. Mononitrato de isosorbide.
2. 2. Parches de nitroglicerina.
3. 3. Betabloqueantes.
4. 4. Diltiazem.
Gráfico de respuestas
Comentario

El tratamiento antianginoso de la angina estable crónica se puede realizar con los siguientes
fármacos: - Betabloqueantes: son los fármacos de 1ª elección en ausencia de contraindicaciones.

!
!
!
!
- Nitratos: orales, sublinguales o transdérmicos. - Calcioantagonistas: muy útiles si hay
vasoespasmo asociado.(R3)

112. ¿Cuál de las siguientes estructuras es más precoz en el desarrollo embrionario?:

1. 1. El blastocele.
2. 2. La blástula.
3. 3. La mórula.
4. 4. La notocorda.
Gráfico de respuestas
Comentario

La mórula es una fase del desarrollo embrionario que incluye la fase de 16, 32 y 64 células. Es una
sólida, no como la blástula, el siguiente paso, que es un balón hueco.(R3)

113. Paciente laparotomizado por un abdomen agudo. A los tres días del postoperatorio,
tras un ataque de tos, se le rompen todos los planos de la herida quirúrgica, asomando
parte del epiplon y algunas asas de intestino delgado. Esto se conoce como:

1. 1. Eventración.
2. 2. Evisceración.
3. 3. Intususcepción.
4. 4. Hernia posquirúrgica.
Gráfico de respuestas
Comentario
Cuando se produce una evisceración se ponen en contacto las vísceras intraabdominales con el
medio externo. Las evisceraciones suelen producirse en el postoperatorio inmediato, en general
por una fallo parcial o total del cierre de una laparotomía que conlleva la exposición de las vísceras
al medio externo, en general asas de intestino delgado, sin que estén contenidas en un saco con
envuelta peritoneal, aunque en ocasiones es sólo la piel la que contiene las vísceras en lo que se
suele denominar evisceración cubierta. El riesgo de estrangulación no suele ser alto pues el
defecto del cierre suele ser amplio. Por el contrario las eventraciones también llamadas hernias
incisionales, suelen producirse de una forma tardía por el fallo del cierre de una laparotomía y las
vísceras están contenidas en un saco con recubrimiento peritoneal. El riesgo de estrangulación
será mayor en aquellos casos en los que el defecto sea de menor tamaño. En ambos casos el
tratamiento definitivo es quirúrgico si bien las evisceraciones requieren cirugía urgente.(R2)

114. En relación a la adolescencia, señale lo FALSO:

1. 1. El “estirón” del crecimiento se inicia distalmente por manos, pies y extremidades.


Los cambios puberales de las fases 1 y 2 de Tanner coinciden con la adolescencia
2. 2.
temprana.
En la adolescencia tardía se prefiere la relación de grupo, a la relación con personas del
3. 3.
sexo opuesto.
La activación neurohormonal para el inicio de la pubertad es dependiente de los factores
4. 4.
culturales.
Gráfico de respuestas
Comentario

Cuestión de lógica, las hormonas se despiertan y a los 15-19 años prefieres la compañía del sexo
opuesto. Respuesta 3 correcta.(R3)

!
!
!
!
115. ¿Cuál es el perfil lipémico de los pacientes afectos de disbetalipoproteinemia
familiar?:

1. 1. Elevación del colesterol y de los triglicéridos en proporción 2:1 a favor del colesterol.
2. 2. Elevación del colesterol y de los triglicéridos a cifras similares (relación 1:1).
3. 3. Elevación del colesterol con cifras normales de triglicéridos.
Elevación de los triglicéridos y del colesterol en proporción 3:1 a favor de los
4. 4.
triglicéridos.
Gráfico de respuestas
Comentario
La disbetalipoproteinemia familiar es un trastorno hereditario en el que se eleva la concentración
de colesterol y de triglicéridos con un cociente entre ambos que se aproxima a 1. La mutación
responsable afecta un gen que codifica la apoproteína E. Clínicamente aparecen 2 tipos de
xantomas: xantoma estriado palmar, que aparece como un cambio de coloración naranja de los
surcos palmares y digitales, y los xantomas tuberosos o tubero eruptivos que son xantomas
cutáneos prominentes que varían de tamaño. Se observa también aterosclerosis grave a edad
temprana.(R2)

116. La fórmula más idónea para calcular el peso medio(Kg) en un niño de 4 años es:

1. 1. Edad (años ) x 7-5 / 5.


2. 2. Edad (años) x 2+8.
3. 3. Edad (años) x 7+15.
4. 4. Ninguna de las anteriores
Gráfico de respuestas
Comentario

Concepto memorístico. La fórmula para calcular el peso es la que señala la opción 2.(R2)

117. Paciente femenino con diabetes tipo 2 da a luz a las 40 semanas a un RN normal,
pero a las 30 horas le encontramos pálido, tiene hematuria y se le palpa una masa en el
flanco izquierdo. ¿Cuál es el diagnóstico más probable?

1. 1. Hematoma renal.
2. 2. Tumor de Wilms.
3. 3. Trombosis de la vena renal.
4. 4. Riñón poliquístico.
Gráfico de respuestas
Comentario

La trombosis de la vena renal produce una masa en flanco y hematuria, siendo especialmente
frecuente en hijos de madre diabética y en recién nacidos con deshidratación. En el adulto,
recuerde su relación con glomerulonefritis que producen síndrome nefrótico, y también en
pacientes con hipernefroma, por invasión tumoral de las venas renales.(R3)

118. Una madre trae a su hijo recién nacido de 7 días por presentar coloración amarillenta
en la piel desde hace 48 horas. Al exámen físico sólo se encuentra ictericia generalizada.
Exámen de laboratorio: bilirrubina total: 12 mg/dl de predominio indirecto. ¿Cuál es el
diagnóstico más probable?

!
!
!
!
1. 1. Incompatibilidad ABO.
2. 2. Ictericia por lactancia materna.
3. 3. Ictericia fisiológica.
4. 4. Enfermedad hemolítica del recién nacido.
Gráfico de respuestas
Comentario

Ictericia fisiológica: es la ictericia neonatal más frecuente. Es el resultado de la inmadurez de los


distintos pasos del metabolismo de los grupos hem y de un mayor turnover de éstos. Se
caracteriza por ser monosintomática, fugaz (desaparece antes de una semana), poco intensa, sin
afectación del estado general y aparece despuès de las primeras 24 horas de vida. Esta ictericia
no recibe tratamiento médico.

Ictericia patológica. Esta ictericia sí que recibe tratamiento médico y se caracteriza por aparecer
durante las 24 horas de vida, la bilirrubina total aumenta más de 5 mg/dl al día y es por lo general
superior a 12 mg/dl en términos o 15 mg/dl en preterminos. Según su aparición se puede clasificar
en:

Temprana (antes de las 24 h): en donde la principal causa es la anemia hemolítica por
incompatibilidad del grupo Rh.

Intermedia (24 h a 10 días): en donde destaca la anemia hemolítica por grupo AB0, ictericia por
leche materna, hipotiroidismo e infecciones.

Tardía (posterior a los 10 días): en donde aparecen las hiperbilirrubinemias de predominio directo
(hepatitis y atresia de las vías biliares).(R3)

119. Paciente de 74 años acude a Urgencias por una sensación intensa de cansancio y
malestar desde hace unos días. Está diagnosticado de mieloma múltiple hace 6 meses,
pero refiere que desde hace unos días su situación ha empeorado bastante. La EF es
normal y en los exámenes de laboratorio se encuentra hipofosfatemia, hipopotasemia e
hipouricemia. Ante la sospecha de que el problema pueda ser renal, se realiza análisis de
orina que presenta glucosuria, fosfaturia, uricosuria, hipercalciuria, bicarbonaturia y
aminoaciduria. ¿Qué trastorno presenta este paciente?

1. 1. Riñón de mieloma.
2. 2. Síndrome de Fanconi secundario.
3. 3. Amiloidosis renal.
4. 4. Metástasis renales del mieloma múltiple.
Gráfico de respuestas
Comentario

Aunque el paciente padece un mieloma múltiple, el diagnóstico no es el de riñón de mieloma.


Tenga en cuenta que esta entidad se debe al daño renal ejercido por las cadenas ligeras, cuya
confirmación es, por otra parte, histológica (se verían moldes proteicos, cilíndricos, a nivel renal,
con un infiltrado inflamatorio alrededor).

Lo que sí podemos decir es que el paciente padece un síndrome de Fanconi. Recuerde que puede
llegarse a este síndrome de varias formas, entre ellas el mieloma múltiple, y consiste en una
disfunción generalizada del túbulo contorneado proximal. Normalmente, a este nivel se reabsorben

!
!
!
!
multitud de solutos (aminoácidos, glucosa, bicarbonato…), que podemos encontrar en orina debido
a este problema en la reabsorción.(R2)

120. Las siguientes medidas, EXCEPTO una son útiles en el tratamiento de la displasia
broncopulmonar:

1. 1. Diuréticos.
2. 2. Adrenalina nebulizada.
3. 3. Evitar la sobrecarga hídrica.
4. 4. Oxigenoterapia.
Gráfico de respuestas
Comentario

La displasia broncopulmonar es la consecuencia de un trípode: prematuridad, toxicidad por


oxígeno y baro-volutrauma. De ahí la importancia de limitar los parámetros del respirador en cuanto
sea posible. En estos niños hay que restringir el aporte de líquidos y administrar diuréticos, para
evitar el edema pulmonar. Además de estas medidas, no ha de olvidar los broncodilatadores y el
oxígeno, pero no la adrenalina nebulizada, respuesta 2.(R2)

121. Niño de 8 años que acude a su consulta por presentar estomatitis angular labial de
forma crónica. En la exploración, usted nota que el niño tiene un cráneo con numerosos
abultamientos en regiones fronto-parietales y un engrosamiento de la cara anterior de la
tibia. Tiene la nariz en silla de montar y un pequeño derrame simétrico y no doloroso en
ambas rodillas, que es fluctuante en el tiempo. ¿Sobre qué incidiría en la anamnesis para
intentar llegar a un diagnóstico?

1. 1. Dieta del niño, fundamentalmente ingesta de lácteos.


2. 2. Si ha tenido fracturas de repetición ante traumatismos mínimos.
3. 3. Si la madre padeció alguna enfermedad infecciosa durante el embarazo.
4. 4. Consumo de tóxicos durante el embarazo.
Gráfico de respuestas
Comentario

La sífilis congéntia tardía produce un cuadro clínico muy típico: queratitis intersticial + hipoacusia
neurosensorial + deformidades óseas.

Éstas pueden producirse a distintos niveles (ensanchamiento de la clavícula, frente, tibia,


deformidades en las piezas dentarias, alteración de la pirámide nasal). Por otra parte, es frecuente
que también asocien patología neurológica (tabes dorsal, paresia juvenil).

Por ello, el cuadro que nos describen debemos enfocarlo hacia una posible sífilis materna.
Respuesta 3 correcta.(R3)

122. En la mastografía rutinaria de una paciente de 52 años, sin factores de riesgo,


aparece un grupo de 5 microcalcificaciones en un área de 2 cm2, sin que se palpe ningún
nódulo en esa zona. El ultrasonido mamario y la termografía son normales. ¿Qué debe
hacer?:

1. 1. Tranquilizar a la paciente.
2. 2. Repetir la mastografía en 6 meses.

!
!
!
!
3. 3. Hacer una biopsia con arpón.
4. 4. Actuar según los marcadores tumorales.
Gráfico de respuestas
Comentario

Esta pregunta no es difícil si razonamos las posibilidades diagnósticas que disponemos. Cuando
en una mastografía nos encontramos un signo de malignidad, debemos obtener un diagnóstico
anatomopatológico (que nos dirá si hay benignidad o malignidad, y realizaremos un tratamiento en
función a dicho diagnóstico), por lo que descartamos las opciones 1, 2 y 4. Un dato muy importante
es que no se palpa ningún nódulo, con lo que el método de biopsia no puede ser a ciegas y
utilizaríamos una biopsia con arpón, colocado previamente con control radiológico (opción 3
correcta).(R3)

123. Mujer de 20 años de edad, sexualmente activa, presenta fiebre, dolor abdominal
inferior, náuseas y vómitos. Posteriormente, dolor en hipocondrio derecho. Al examen:
cervicitis mucopurulenta, leucocitosis y aumento de la velocidad de sedimentación
globular. ¿Cuál del siguiente microorganismo causa con más frecuencia este cuadro
clínico?:

1. 1. Treponema pallidum.
2. 2. Chlamydia trachomatis.
3. 3. Mycoplasma hominis.
4. 4. Ureaplasma urealyticum.
Gráfico de respuestas
Comentario

Pregunta clásica en el ENARM. Describen un caso típico de enfermedad pélvica inflamatoria, que
puede estar complicada por una perihepatitis (síndrome de Fitz-Hugh-Curtis). En nuestro país,
México, las etiologías más frecuentes son Chlamydia trachomatis y Neisseria gonorrhoae
(gonococo).

Luego entonces, la respuesta correcta es la 2.(R2)

124. La GN postestreptocócica se caracteriza por depósitos electrodensos de


localización típica:

1. 1. Subepitelial.
2. 2. Membranosos.
3. 3. Cápsula parietal de Bowman.
4. 4. Subendotelial.
Gráfico de respuestas
Comentario

La microscopía electrónica es la técnica que nos indica la localización de los depósitos, y en este
caso muestra, como hallazgo más característico y constante, la presencia de depósitos nodulares
voluminosos, electrónicamente densos, situados en la vertiente externa de la MBG (localización
subepitelial, RESPUESTA 1 CORRECTA). Estos depósitos, denominados "humps" en inglés, se
observan con preferencia en biopsias tempranas y desaparecen por lo general 4 a 8 semanas
después del comienzo clínico, pero no son patognomónicos.(R1)

!
!
!
!
125. En el examen clínico de mama de una paciente se encuentra un tumor que es clasificado como
T2-N0. Esta situación corresponde a:

1. 1. Cualquier tamaño del tumor con dos ganglios positivos.


2. 2. Tumor avanzado inoperable.
3. 3. Tumor entre 2 a 5 cm de diámetro, sin ganglios.
4. 4. Tumor menor de 2 cm de diámetro sin ganglios.
Gráfico de respuestas
Comentario

Pregunta del TNM de Cáncer de mama, la verdad es que no son tan preguntados las
clasificaciones del TNM sólo da una repasada días previos. Recuerde: T tumor, N nódulos, M
mestástasis. Al ser T2 N0, puede descartar la 1 y 2. T1 sería un tumor menor de 2 cm y
obviamente el T2 de 2 a 5 cm(R3)

Repasa el TNM

!
!
!
!

126. A young and healthy medical student wants to jog around a park. While he will be
exercising, some hemodynamic changes will happen to him. Mark the FALSE answer:

1. 1. Stroke volume and heart rate will increase.


2. 2. Preload will increase, due to the enhanced venous tone and skeletal muscle pump.
While exercising, there is an elevated release of catecholamines that helps keeping the
3. 3.
myocardial contractility.

!
!
!
!
Due to the increased stroke volume and muscular vasoconstriction, a significant elevation
4. 4.
of blood pressure occurs.
Gráfico de respuestas
Comentario

Durante el ejercicio aeróbico (como correr en un parque), la presión arterial se eleva levemente. El
aumento del gasto cardíaco y de la contractilidad contribuyen a su incremento, pero la
vasodilatación periférica se opone a que se produzca un aumento importante.(R4)

127. Un hombre de 50 años acude al Servicio de Urgencias presentando un cuadro de


dolor abdominal súbito. ¿Cuál de las siguientes causas de dolor abdominal se plantearía
en último lugar?

1. 1. Infarto agudo de miocardio.


2. 2. Diverticulitis.
3. 3. Perforación de úlcera péptica.
4. 4. Embolia mesentérica.
Gráfico de respuestas
Comentario

Esta pregunta requiere más atención que conocimientos teóricos para ser resuelta. La clave está
en la palabra “súbito” que utilizan en el enunciado. De las opciones que nos presentan, a primera
vista puede resultar sorprendente que mencionen el infarto agudo de miocardio como causa de
dolor abdominal, pero sabemos que es posible, ya que los infartos inferiores pueden producir dolor
en hemiabdomen superior. Sin embargo, el dolor de la diverticulitis es más insidioso y, por tanto,
sería la última opción que deberíamos considerar.(R2)

128. Mujer de 27 años que consulta por amenorrea de 3 meses. Un embarazo y parto
normales hace 5 años. Ex usuaria de drogas por vía inhalatoria, negando el uso de la vía
parenteral. Su pareja sexual es adicto a drogas parenterales y VIH positivo en estadio C3.
En la serología del primer trimestre se diagnostican anticuerpos anti-VIH positivos en la
gestante. Debemos informarle de todo lo siguiente, EXCEPTO:

La transmisión vertical o materno-fetal de la infección por VIH-1 constituye el principal


1. 1.
modo de adquisición de la infección en los niños.
La existencia de otras ETS asociadas, pueden incrementar el riesgo de transmisión
2. 2.
vertical.
Deberá renunciar a la lactancia materna, puesto que esta duplica el riesgo de transmisión
3. 3.
madre-hijo.
4. 4. Está aceptada la cesárea electiva de forma sistemática como única vía del parto.
Gráfico de respuestas
Comentario

Las complicaciones infecciosas durante el embarazo es un tema poco preguntado. No obstante,


conviene repasar la relación del VIH y el embarazo.

En nuestro medio, la práctica totalidad de las infecciones por VIH en menores de 13 años se debe
a transmisión vertical, que puede ser:

- Vía transplacentaria: supone 30-40% de los casos.

!
!
!
!
- Transmisión durante el parto: es la vía más importante. Depende de distintos factores, siendo el
más importante la rotura de membranas > 4 horas. También influye la prematuridad, bajo peso al
nacimiento, asociación a otras ETS, recuento de CD4 materno bajo o viremia elevada.

El uso de zidovudina intraparto y su administración al RN ha demostrado disminuir el riesgo de


infección (hoy día, la triple terapia lo disminuye todavía más). La cesárea es el método de elección
para finalizar el parto, pero no el único, ya que en algunos casos también se admite el parto vaginal
(respuesta falsa 4), si la carga viral es lo suficientemente baja y hemos empleado la triple terapia..

- Transmisión postnatal: a través de la leche materna. En países desarrollados se desaconseja la


lactancia materna.(R4)

129. ¿Cuál es el papel de la hormona FSH en el ciclo genital femenino?:

Estimula el crecimiento de la teca, induce actividad aromatasa y aumenta los receptores de


1. 1.
FSH en la teca.
Estimula el crecimiento de la granulosa, induce actividad aromatasa y aumenta los
2. 2.
receptores de FSH en la granulosa.
Estimula el crecimiento de la teca, aumenta la producción de andrógenos y el número de
3. 3.
receptores de FSH en la granulosa.
Estimula el crecimiento de la granulosa, induce actividad aromatasa y disminuye los
4. 4.
receptores de FSH en la granulosa.
Gráfico de respuestas
Comentario

Pregunta importante de ginecología.

La FSH tiene dos picos de liberación el primero en forma de meseta durante la fase proliferativa
tiene como misión estimular el crecimiento de la cohorte folicular y la selección del folículo
dominante. El segundo pico sucede justo antes de la ovulación. Sus acciones principales son las
descritas en esta pregunta: la estimulación del crecimiento de la capa granulosa en el folículo que
ha seleccionado, inducir actividad aromatasa en la granulosa que convierte los andrógenos en
estradiol (por tanto la FSH estimula la producción de estrógenos en el folículo ovárico) y aumentar
los receptores de FSH en la granulosa. La FSH es inhibida por la inhibina folicular y los estrógenos.
Es decir, los estrógenos producidos gracias a la FSH, inhiben a la propia FSH mediante un
feedback negativo.(R2)

130. De los siguientes factores, ¿cuál NO se considera de riesgo para el cáncer de


mama?:

1. 1. Fallo ovárico precoz.


2. 2. Síndrome de Li-Fraumeni.
3. 3. Raza blanca.
4. 4. Historia familiar de cáncer mamario.
Gráfico de respuestas
Comentario

El síndrome de Li-Fraumeni implica una mutación en el gen que codifica la proteína p53, lo que
predispone en gran medida al padecimiento de tumores muy diversos. Las lesiones proliferativas
con atipias son las que más predisponen al cáncer de mama en la mastopatía fibroquística. Sin

!
!
!
!
embargo, las proliferativas sin atipias también implican cierto riesgo, aunque no tan elevado. Las
que no predisponen al cáncer de mama son las lesiones no proliferativas.

La solución correcta es la 1. El fallo ovárico precoz no se ha relacionado con el cáncer de mama,


ya que supone un acortamiento de la edad fértil y, por ello, menor exposición a estrógenos por
parte de la glándula.(R1)

131. Masculino de 56 años, con historia de alcoholismo crónico. Dos días después de
suspender bruscamente el consumo de alcohol se instaura un cuadro caracterizado por
disminución del nivel de conciencia, desorientación temporoespacial, ilusiones visuales
y alucinaciones con visión de animales pequeños e insectos. Presenta importante
inquietud, sudoración profusa y elevación de la temperatura. ¿Cuál de los siguientes
fármacos tiene menos interés en el manejo de esta situación?

1. 1. Benzodiacepinas.
2. 2. Tiamina.
3. 3. Pimozide.
4. 4. Sulfato de magnesio.
Gráfico de respuestas
Comentario

El tratamiento del delirium tremens se ha preguntado en el ENARM en repetidas ocasiones. Esta


pregunta es muy importante y no se debería fallar el día del examen.

En el tratamiento del delirium, entendiendo como tal el síndrome confusional agudo, se utilizan
neurolépticos como el haloperidol o el pimozide. Sin embargo, en el caso concreto del delirium
tremens, no se deben emplear tales fármacos, siendo preferibles las benzodiacepinas (respuesta 4
falsa). Esto se debe a que se trata de pacientes alcohólicos, predispuestos por ello a las
convulsiones… Y el pimozide, como el haloperidol, disminuye el umbral convulsivo, por lo que no
se debe utilizar.(R3)

132. Mujer joven de raza negra que acude a consulta quejándose de dolor y tumefacción
en las manos, también ha padecido en los últimas semanas lesiones dolorosas en boca
y nariz que resultan ser úlceras a la exploración. En la enfermedad que Ud. sospecha
¿cuál de los siguientes se correlaciona con su actividad y la presencia de nefritis?:

1. 1. Anticuerpos anti-La.
2. 2. Anticuerpos anti-RNP.
3. 3. Anticuerpos anti-P.
4. 4. Anticuerpos anti-histona.
Gráfico de respuestas
Comentario

La presencia en una mujer joven de lesiones ulceradas en la cavidad oral asociada a artritis es
sugestivo del diagnóstico de lupus eritematoso sistémico (LES). Además, las mujeres de raza
negra tienen una mayor predisposición a presentar esta enfermedad. Dentro del LES se ha
observado que existe relación entre distintos anticuerpos que aparecen en la enfermedad con
algunas manifestaciones clínicas. Así por ejemplo los anticuerpos antirribosomales (anti- P) se
relacionana con la psicosis lúpica. Los anti-La disminuyen el riesgo de nefritis, mientras que los
anti-DNAds aumentan este riesgo y se relacionan con una mayor actividad de la enfermedad. Los

!
!
!
!
anti-RNP se asocian a la aparición de fenómeno de Raynaud y los anti-histona aparecen tanto en
el lupus inducido por fármacos como en el LES.(R1)

133. La más alta incidencia de hemorragia intraventricular en los prematuros ocurre:

1. 1. Durante las primeras 24h de vida.


2. 2. Entre el primer y tercer día.
3. 3. Entre los 10 y 30 días de vida.
4. 4. Después de los 30 días de vida.
Gráfico de respuestas
Comentario

La hemorragia intraventricular es un sangrado dentro de las áreas llenas de líquido (ventrículos) en


el cerebro y se observa con más frecuencia en prematuros. Cuanto más pequeño y más prematuro
sea el bebé, mayor será el riesgo de presentar hemorragia. Rara vez está presente al nacer. Si
ocurre, normalmente será en los primeros días de vida, sobre todo las primeras 72 hrs. La HI es
infrecuente después de 1 mes de edad, sin importar qué tan prematuro haya sido el bebé al
nacer.(R2)

134. En la gastritis eosinofílica es FALSO que:

1. 1. Se presente en ocasiones como una obstrucción intestinal.


2. 2. Se asocie en el 50% de los casos a infección por H. pylori.
3. 3. El diagnóstico sea fundamentalmente histológico.
4. 4. Pueda provocar diarrea crónica.
Gráfico de respuestas
Comentario

La etiología de la gastritis eosinofílica es desconocida. Puede cursar dependiendo de la severidad


y extensión de las lesiones, bien como diarrea, o ascitis eosfinofílica, o cuadros obstructivos.

El diagnóstico requiere de histología, que demuestra una masiva proliferación de eosinófilos de


aspecto maduro en estómago y/o intestino delgado, excluyendo otras causas de eosinofilia,
fundamentalmente las parasitosis. Por el momento, no se ha asociado a la infección por
H.pylori.(R2)

135. ¿Cuál es el síntoma más característico de la osteomalacia?:

1. 1. Hipotonía.
2. 2. Fractura patológica.
3. 3. Dolor óseo difuso.
4. 4. Debilidad muscular distal.
Gráfico de respuestas
Comentario

En la osteomalacia, lo más característico es el dolor óseo difuso en todo el cuerpo, no localizado,


pues afecta a toda la ergonomía, y se exacerba a la palpación. También existe calor local. Más
características que las fracturas en sí, son las pseudofracturas o líneas de "Looser-Milkman". La
hipotonía y la tetania son más típicas del raquitismo; en los niños se manifiesta así la hipocalcemia
severa, pero no así en el adulto, que genera el hiperparatiroidismo compensatorio eficazmente y

!
!
!
!
los niveles de calcio no descienden drásticamente. Sí existe debilidad muscular de predomiio
proximal, en cinturas escapular y pelviana.(R3)

136. Masculino de 19 años de edad que es ingresado tras un accidente de tráfico


presentando múltiples fracturas de los últimos arcos costales, hematoma, y dolor en
flanco izquierdo, así como hematuria, con estabilidad hemodinámica. Sobre el
diagnóstico más probable, seleccione la respuesta CORRECTA.

La hematuria después de un traumatismo indica lesión del aparato urinario, y debe aparece
1. 1.
en todos los casos.
2. 2. El grado de hematuria se correlaciona con el grado de afectación.
3. 3. Las lesiones renales son las más frecuentes del aparto urinario tras traumatismo.
4. 4. Los traumatismos renales menores son poco frecuentes.
Gráfico de respuestas
Comentario

Los traumatismos renales no son muy preguntados, pero es conveniente conocer algunas
generalidades sobre la actitud terapéutica. Las lesiones renales son las más frecuentes del aparto
urinario tras traumatismo. La hematuria después de un traumatismo indica lesión del aparato
urinario, aunque no aparece en todos los casos. El grado de hematuria no se correlaciona con el
grado de afectación. Por otro lado, los traumatismos renales menores son con mucho los más
frecuentes (85%) de todos los tipos de traumatismo renal.(R3)

137. ¿Cuál de los siguientes NO es un predictor clínico de buena respuesta a los


antidepresivos heterocíclicos de una depresión?:

1. 1. Insomnio terminal.
2. 2. Ideación no congruente con el humor.
3. 3. Anhedonia generalizada.
4. 4. Pérdida importante de peso.
Gráfico de respuestas
Comentario
La incongruencia con el humor de fondo de las indeaciones sobrevaloradas o delirantes durane un
episodio afectivo (ya sea manía o depresión) indica peor pronóstico y peor respuesta a
tratamientos puramente "afectivos" (antidepresivos y estabilizadores del humor), y suele requerir la
asociación al menos temporal de un antipsicótico.(R2)

138. Mujer de 22 años que viene al médico a causa de una historia de dolor al orinar hace
2 días, comezón vaginal intensa y una descarga de aspecto espeso. Ella no tiene
antecedentes de enfermedad grave. Es sexualmente activa y usa un anticonceptivo oral.
Su temperatura es de 37°C. El examen abdominal no muestra anormalidad alguna.
Examen genitourinario muestra eritema de la vulva y vagina con descarga que tiene un
olor similar al requesón. El cuello uterino parece normal. Examen bimanual no muestra
anomalías. El pH de la secreción vaginal es 4; se obtienen preparados de montaje con
solución salina y KOH; la lamina muestra células epiteliales y escamosas y múltiples
levaduras con pseudohifas en ciernes. ¿Cuál de las siguientes es la posibilidad
diagnóstica más probable?

1. 1. Vaginosis bacteriana.
2. 2. Infección por Chlamydia trachomatis.

!
!
!
!
3. 3. Candidiasis.
4. 4. Liquen escleroso.
Gráfico de respuestas
Comentario

El cuadro clínico es compatible con candidiasis vulvovaginal, con flujo blanquecino, grumoso y con
olor a requesón, con pH que se mantiene dentro de los parámetros normales o menor de 4.5. La
reacción del exudado vaginal con KOH no genera ningun olor y en el examen directo al
microscopio se obervan pseudohifas en ciernes, que es típico de muchas especies de
Cándida.(R3)

139. ¿Cuál de las siguientes situaciones NO contraindica la práctica de un trasplante


hepático?:

1. 1. Edad superior a 60 años.


2. 2. Insuficiencia hepática aguda grave.
3. 3. Sepsis.
4. 4. Insuficiencia respiratoria crónica grave.
Gráfico de respuestas
Comentario

Esta pregunta no entraña mucha dificultad si razonas que una persona que tenga una insuficiencia
hepática aguda lo que necesitaría es precisamente un trasplante hepático, con lo cual no puede
ser una contraindicación, sino una indicación. La indicación de trasplante hepático más frecuente
es la atresia de las vías biliares en el niño y la cirrosis etílica en abstinencia y la viral en el adulto.
Recuerde que el colangiocarcinoma no es indicación de trasplante debido a sus malos resultados.
Es el trasplante más difícil de realizar. Las contraindicaciones absolutas si la piensa son las
generales a cualquier trasplante: enfermedades sistémicas severas, enfermedades bacterianas o
micosis extrahepáticas incontroladas, enfermedad cardiopulmonar preexistente avanzada,
anomalías congénitas múltiples e incorregibles, tumor metastático, adicción a alcohol o drogas de
forma activa. La infección VIH antes se consideraba una contraindicación pero actualmente se está
realizando trasplante a seropositivos con VHC en los que la enfermedad hepática limita el uso de la
terapia antirretroviral.(R2)

140. La neumonía intersticial linfoide en los niños con SIDA:

1. 1. Es menos frecuente en niños que en adultos.


2. 2. Se ha relacionado etiológicamente con una infección persistente por CMV.
Su comienzo suele ser gradual, a diferencia de la neumonía por P. jirovecii, que presenta
3. 3.
un comienzo brusco.
4. 4. Para su tratamiento y profilaxis se utiliza el trimeptroprim-sulfametoxazol.
Gráfico de respuestas
Comentario

Pregunta de dificultad media sobre el SIDA pediátrico. La neumonía intersticial linfoide (NIL) es un
cuadro característico del SIDA en los niños y por el contrario es muy poco frecuente en adultos. En
comparación con la neumonía por P. jirovecii tiene mejor pronóstico, ya que ésta última es la causa
más frecuente de muerte en estos pacientes. Se ha relacionado etiológicamente con el propio VIH
y también con el virus de Epstein- Barr y para su tratamiento se utiliza el oxígeno, los corticoides y
los broncodilatadores. Por lo tanto la opción correcta es la 3, se presenta de forma gradual como

!
!
!
!
dificultad respiratoria e hipoxemia moderada. No está demostrado que la NIL esté asociado a un
deterioro de la enfermedad.(R3)

141. A 9-year-old male visits the pediatrician’s office for a routine health check-up. His
mother is worried about sunlight damage of his skin and eyes. The boy has a type 2
phototype with blonde hair, blue eyes and some junctional acquired nevi. Which of the
following is the best advice in this case?

1. 1. To avoid sun exposure between 12 am and 5 pm.


2. 2. Employment of total sunscreen protection.
3. 3. Vit A supplements.
4. 4. Dermatoscopic examination at least once a year.
Gráfico de respuestas
Comentario
To avoid sun exposure between 12 am and 5 pm. The risk of sun exposure begins early in life, and
children need extra protection from the sun. The most effective methods are basic: keep out of the
midday sun. The rest of the listed measures are complementary. The child described in this case
does not have an albinism phenotype.(R1)

142. ¿Cuál es la modificación de laboratorio típica de la premenopausia?:

1. 1. LH alta.
2. 2. Testosterona ligeramente elevada.
3. 3. FSH alta.
4. 4. LH baja.
Gráfico de respuestas
Comentario

En la premenopausia se produce un incremento de la FSH, probablemente debido al descenso de


inhibina, consecuencia del escaso número folicular. El resto de hormonas se mantiene en niveles
de normalidad. Recuerde que la presencia de síntomas climatéricos suele ir asociado al descenso
en los niveles de estrógenos.(R3)

143. Which of the following findings is not observed in Crohn's disease?

1. 1. Cobblestone appearance that occurs as a result of inflammatory changes.


2. 2. Thickening of the entire intestinal wall and mesentery and enlargement of lymph nodes.
3. 3. The mucous membrane involvement is normally continuous from the rectum to the colon.
Any part of the gastrointestinal tract can be affected, although ileocolic involvement is the
4. 4.
most common.
Gráfico de respuestas
Comentario

Tema importante para el ENARM, debe saber diferenciar muy bien entre CUCI y Crohn.

En el Crohn es típica la afectación de la zona ileocólica. Es característica la afectación parcheada


(no continua - respuesta 3 falsa-), que puede afectar a todo el tubo digestivo (desde boca hasta
ano), respetando típicamente el recto. Esta lesión suele ser transmural, es decir, que afecte a todo
el espesor de la pared del tubo digestivo, lo que favorece la formación de fístulas. Estas dos
características le confieren a la enfermedad de Crohn la clásica imagen en empedrado. Lo que es

!
!
!
!
más sugestivo de enfermedad de Crohn es la presencia de granulomas no caseificantes (no se ven
prácticamente nunca en la colitis ulcerosa).

A modo de regla nemotécnica puede ser útil pensar en:

- COlitis ulcerosa: Afecta COlon y es COntinua (para diferenciarlo de la E. de Crohn que afecta
todo el tubo digestivo de forma parcheada).(R3)

144. El síndrome de Eagle-Barret o "prune-belly" cursa de forma habitual con todo lo


siguiente, EXCEPTO:

1. 1. Cardiopatías congénitas en el 50% de los casos.


2. 2. Malrotación intestinal.
3. 3. Deficiencia de la musculatura de la pared abdominal.
4. 4. Criptorquidia bilateral.
Gráfico de respuestas
Comentario

El 10% de los casos de síndrome de prune belly cursan con cardiopatía congénita, y no el 50%
como aparece especificado en la opción 1 de esta pregunta. El resto de las opciones son
rigurosamente correctas, pues este síndrome puede acopañarse de malroatción intestinal, atrofia
de los rectos abdominales, criporquidia y ureterohidronefrosis.(R1)

145. Atiende a un vagabundo traído a urgencias por la policía; está inconsciente e


impresiona de gravedad. En la exploración destaca un amplio número de adenopatías
junto a una gran esplenomegalia. El paciente tiene una frecuencia cardíaca de 97 lpm,
con una tensión arterial de 90/70 mmHg. En los exámenes de laboratorio, el paciente
presenta datos de deshidratación, con aumento de PDF, trombopenia y aumento de los
tiempos de coagulación. Señale la FALSA respecto a la patología que sospecha:

1. 1. El agente etiológico es un gram negativo aerobio con tinción bipolar en "imperdible".


2. 2. El paciente tiene una clínica de sepsis y CID.
El tratamiento de elección es la estreptomicina, salvo en el caso de la forma meníngea que
3. 3.
es la doxicilina.
4. 4. La forma meníngea es la forma más rara.
Gráfico de respuestas
Comentario

Con los datos que nos dan, es complicado reconocer la enfermedad, pero las opciones de las
respuestas pueden orientarnos incluso más que el enunciado. Quizás la opción 2 es la que nos
puede resultar clave para reconocer que estamos ante un caso de peste, si recordamos que esta
enfermedad se transmite a través de la picadura de la pulga de la rata, la Xenopsylla cheopis. El
germen causal es la Yersinia pestis, un bacilo gramnegativo, aerobio, inmóvil y con una tinción
bipolar característica en imperdible. La peste tiene una forma adenopática, una septicémica, una
neumónica y otra meníngea, mucho más rara. Si no se trata, puede evolucionar hacia un cuadro
como el del enunciado de septicemia y CID (aumento de PDF, trombopenia e incremento de
tiempos de coagulación). El tratamiento de elección es la estreptomicina, excepto en la forma
meníngea, donde se emplea cloranfenicol.(R3)

!
!
!
!
146. De los siguientes enunciados, señale la respuesta VERDADERA respecto al lugar en
el que se encuentra la alteración patogénica principal en los síndromes mielodisplásicos.

1. 1. En el microambiente de la médula ósea.


2. 2. En la célula germinal pluripotencial o célula stem cell.
3. 3. En alguna de las células “comprometidas” hacia una línea celular concreta.
La alteración principal es en los receptores de los factores de crecimiento celular
4. 4.
(citoquinas).
Gráfico de respuestas
Comentario

Pregunta de dificultad media acerca de los síndromes mielodisplásicos (preguntan un concepto


explicado en clase).

Los síndromes mielodisplásicos son panmielopatías, por probable mutación de la célula germinal
pluripotencial mieloide, que ocasiona una expansión clonal con eritropoyesis ineficaz, dando lugar
a citopenias en sangre periférica, con gran frecuencia pancitopenia (opción 2 correcta).

Una característica fundamental de los síndromes mielodisplásicos, conforme indica su nombre, es


la anomalía morfológica de los precursores de la médula ósea (displasia medular). En el 90% de
los casos no existen factores etiológicos conocidos. En el restante 10% existen enfermedades o
factores asociados a la mielodiaplasia, por ejemplo, citostáticos, radiaciones, SIDA y un porcentaje
importante son trastornos en la biosíntesis del hem y las porfirinas, lo que ocasiona un incremento
en la captación celular de hierro y da lugar a precursores eritroides anormales.

Generalmente, son enfermedades que comienzan con curso clínico indolente y progresivo.
Aparece VCM normal o aumentado, y una anemia progresiva que se caracteriza por ser refractaria
a la mayoría de los tratamientos. Su único tratamiento definitivo es el trasplante de médula
ósea.(R2)

147. Mujer de 47 años que acude a consulta de ginecología remitida desde su médico de
atención primaria. Refiere menstruaciones en cantidad mayor a lo habitual en los últimos
meses. Su médico le realizó una biometría hemática, donde presentaba hemoglobina de
10 g/dL, por lo que le pautó tratamiento con hierro oral y decidió remitirla al ginecólogo.
Para ampliar el estudio, se decide solicitar, entre otros, un estudio hormonal. Señale cuál
es el resultado que esperaría encontrar con mayor probabilidad:

1. 1. Aumento de LH, con FSH normal y leve disminución de estradiol.


2. 2. GnRH y estrógenos disminuidos, con FSH y LH normal.
Elevación de FSH, con GnRH y estrógenos normales y LH normal o levemente
3. 3.
aumentada.
4. 4. Disminución notable de estradiol, con FSH, LH y GnRH normales.
Gráfico de respuestas
Comentario

La respuesta correcta es la número 3, ya que en el período de premenopausia- perimenopausia se


produce un incremento de FSH, que es el cambio endocrino más precoz y, que suele acompañarse
de niveles de LH normales o aumentados, siendo los estrógenos y la GnRH normales. El
incremento de la FSH parece estar en relación con el descenso de la inhibina folicular.(R3)

!
!
!
!
148. Está revisando los datos prenatales de la madre mientras completa el examen de
admisión del recién nacido de 8 horas de vida extrauterina. La madre fue evaluada para
virus de hepatitis B y el antígeno de superficie HB (AgsHB) resultó positivo. La acción
más acertada sería administrar:

Inmunoglobulina antihepatitis B (IGHB) en las primeras 12 horas post parto y vacunación


1. 1.
en la consulta de control del niño sano a las 2 semanas de vida.
2. 2. Vacuna anti-VHB en las primeras 12 horas del parto.
3. 3. Vacuna anti-VHB e IGHB en las primeras 12 horas del parto.
Vacuna anti-VHB en las primeras 12 horas post parto e IGHB en la consulta de control de
4. 4.
niño sano a las 2 semanas.
Gráfico de respuestas
Comentario

En cuanto a la prevención del RN de hijo de madre con Ag de VHB positivo, se debe:

-Administrar IG contra hepatitis B durante las primeras 12 horas de vida.

-Administrar la primera dosis de vacuna de VHB.

Tras realizar estas maniobras se puede inciar la lactancia materna.

Posteriormente hay que seguir una pauta de vacunación en el segundo mes y en el sexto mes.(R3)

!
!
!
!

149. A previously healthy 82 year-old-man


comes to the physician because of a 2-month-history of vision loss on the right eye. Both
eye fundus are shown, Mark the CORRECT answer:

1. 1. Central and peripheral visual field loss is probably referred by this patient.
2. 2. This illness is caused by a neovascularization process.
3. 3. A visual field testing will help with the diagnosis.
4. 4. Laser photocoagulation would be very useful in the treatment of the right eye.
Gráfico de respuestas
Comentario

El caso que nos ofrecen corresponde a una degeneración macular senil (DMAE). Van a su favor la
edad del paciente, la frecuencia del proceso en ancianos y, sobre todo, la presencia de drusas en
ambos fondos de ojo. Normalmente, la DMAE produce una pérdida lenta, progresiva, a lo largo de
años. Es la evolución típica en la forma más frecuente de DMAE, que es la seca o atrófica. Sin
embargo, en algunos pacientes se produce otro tipo de DMAE, que es la llamada forma húmeda o
exudativa, bastante menos común. Aparte de encontrarnos las típicas drusas, podemos hallar
membranas fibrovasculares, neovasos, hemorragias subretinianas y desprendimientos de retina
exudativos. La evolución es mucho más rápida y de peor pronóstico, como sucede en este
paciente. En el fondo de ojo derecho (imagen inferior), vemos una imagen en la mitad inferior, que
corresponde a un importante exudado junto con tejido neovascular (respuesta 2 correcta). El resto
de las opciones no son correctas, por los motivos siguientes: R1: La afectación será principalmente
central, no periférica. R3: La campimetría nos mostraría un campo periférico prácticamente
preservado. Recuerde que, en la DMAE, se afecta sobre todo el área macular, por lo que la
campimetría sería normal. Lo que sí se pierde es agudeza. R4: Desgraciadamente, la
fotocoagulación láser no es tan eficaz como se dice en esta opción. Puede frenar parte de la
evolución en las formas exudativas, pero eso no significa que este paciente, cuya visión ya está
muy alterada, tenga un buen pronóstico visual por emplear este tratamiento.(R2)

!
!
!
!
150. Se observa en una mujer lesiones dérmicas en la vulva que son compatibles con
molusco contagioso, esta es una enfermedad causada por un miembro de la familia
Papovavirus, en su tratamiento se incluye:

1. 1. Podofilotoxina.
2. 2. Observación.
3. 3. Cidofovir.
4. 4. Retinoides tópicos.
Gráfico de respuestas
Comentario

Entre las terapias de primera línea para tratar el molusco contagioso se encuentran: crioterapia,
curetaje, cantaridina y podofilotoxina; y dentro de las opciones de segunda línea se encuentra el
imiquimod, hidróxico de potasio, ácido salicílico, retinoides tópicos. En caso de pacientes
inmunosuprimidos se podría utilizar interferón alfa o cidofovir.

La decisión de tratar tiene ventajeas ya que limita la diseminación a otro sitios, reduce el riesgo de
transmisión a otras personas y disminuye el prurito.

La podofilotoxina (podofilina) es un agente antimitótico que se utiliza en forma de solución o gel.


Puede producir eritema local, prurito, quemaduras, inflamación y erosiones.

El uso de retinoides tópicos como tretinoina, adapaleno y tazaroteno es limitada, pero se han
utilizado en su tratamiento. El mecanismo de acción es a través de la irritación local que lesiona la
membrana proteico-lipídica viral.

Cidofovir se utiliza en pacientes inmunocomprometidos con molusco refractario y severo.(R1)

151. Uno de los siguientes hallazgos endoscópicos NO se corresponde con la


enfermedad de Crohn:

1. 1. La afectación gastroduodenal puede simular una úlcera péptica.


La imagen en empedrado, con una afectación continua de todo el intestino grueso es
2. 2.
característica de la enfermedad.
La fibrosis secundaria a la inflamación transmural puede producir una obstrucción
3. 3.
intestinal.
Los pacientes con afectación severa de la mayor parte del intestino pueden llegar a sufrir
4. 4.
un cuadro de malabsorción.
Gráfico de respuestas
Comentario
En la enfermedad de Crohn es típica la afectación parcheada (no continua), que puede afectar a
todo el tubo digestivo (desde boca hasta ano), respetando típicamente el recto. Sin embargo, esta
lesión suele ser transmural, es decir, que afecte a todo el espesor de la pared del tubo digestivo, lo
que favorece la formación de fístulas. Estas dos características le confieren a la enfermedad de
Crohn la clásica imagen en empedrado.(R2)

152. Señale la respuesta FALSA acerca de las complicaciones renales de la HTA:

En una minoría de los pacientes existe hiperuricemia que puede ser signo temprano de
1. 1.
nefroangioesclerosis.

!
!
!
!
La microalbuminuria es el signo más precoz de la nefroangioesclerosis y constituye por sí
2. 2.
mismo un factor de riesgo cardiovascular.
Con el tiempo la microalbuminuria evoluciona típicamente a proteinuria de rango
3. 3.
nefrótico.
Los cambios vasculares producen un aumento de la resistencia vascular renal, con
4. 4.
disminución del flujo plasmático renal y posteriormente del filtrado glomerular.
Gráfico de respuestas
Comentario

No se preocupe si ha fallado esta pregunta porque es difícil. Lo que debe saber es que la
nefroangioesclerosis es el cuadro histológico y clínico condicionado por la HTA a nivel renal. Se
asocia a otros trastornos metabólicos como la dislipemia y la hiperuricemia y es mucho más
frecuente en los individuos de raza negra. Es causa habitual de insuficiencia renal que progresa
lentamente y se acompaña de proteinuria ligera-moderada. La microalbuminuria en pacientes con
HTA es el marcador clínico principal de la nefroangioesclerosis pero la evolución de ésta hacia una
proteinuria franca en rango nefrótico es más característica de la nefropatía diabética.(R3)

153. Indique el tratamiento de elección, en un centro de referencia para cirugía


coloproctológica, para un paciente de 45 años con un tumor de recto situado a 8 cm del
margen anal, que ha sido evaluado con RMN y ultrasonido endorrectal, y en el que se ha
visto que llega a la submucosa, sin afectación ganglionar ni enfermedad metastásica:

1. 1. Resección local por vía anal.


2. 2. Cirugía endoscópica microscópica transanal.
3. 3. Resección anterior baja.
4. 4. Neoadyuvancia con quimiorradioterapia.
Gráfico de respuestas
Comentario

El TEM (cirugía endoscópica microscópica transanal) se ha convertido en el procedimiento de


elección para la exéresis de tumores de recto en estadios iniciales T1N0, ya que gracias a su mejor
exposición, permite una extracción más correcta de la pieza.

El mayor inconveniente, es que es una técnica que precisa un entrenamiento adecuado y un


instrumental específico del que no se dispone en todos los centros. Si no se dispusiera de TEM, la
cirugía convencional para la resección trasanal sería la respuesta adecuada en este estadio.(R2)

154. ¿Cuál es el efecto secundario más frecuente del tratamiento con interferón en la
hepatitis crónica por virus B?:

1. 1. Alopecia.
2. 2. Síndrome gripal.
3. 3. Depresión de la médula ósea.
4. 4. Erupciones cutáneas.
Gráfico de respuestas
Comentario
El interferón presenta múltiples efectos secundarios entre los que se encuentran todos los citados
en las distintas opciones pero sin duda el más frecuente es el cuadro pseudogripal que aparece
tras la inyección de dicho fármaco y que se caracteriza por malestar general, artro- mialgias,
cefalea, astenia, fiebre y escalofríos.(R2)

!
!
!
!
155. Adolescente de 16 años de edad, quien es visto en la sala de urgencias por presentar
desde hace 3 semanas un cuadro caracterizado por fiebre entre 38.3°C, letargia, pérdida
de 2 kilos de peso, al examen físico marcada adenopatía cervical posterior, amigdalas
con exudado y aumento de volumen, pequeñas hemorragias en paladar blando y
esplenomegalia. En la BH se observa linfocitosis (10% atípicos) ¿el diagnóstico probable
sería?

1. 1. Mononucleosis infecciosa.
2. 2. Enfermedad de Kawasaki.
3. 3. Infección estreptocócica.
4. 4. Enfermedad por VIH.
Gráfico de respuestas
Comentario

Cuadro típico de una mononucleosis infecciosa que no debe olvidar.

El síndrome mononucleósico se caracteriza por la triada de fiebre, faringitis y adenopatías. Se


dividen en mononucleosis infecciosa, causado por VEB y los otros que incluyen infección por CMV,
Toxoplasmosis aguda, primoinfección por VIH, entre otros.

El virus del Epstein Bar constituye el agente etiológico de la mononucleosis infecciosa con
anticuerpos heterófilos positivos, también denominada la enfermedad del beso, afecta
habitualmente a sujetos entre 15 y 25 años y se trata de una infección de los linfocitos B. El perido
de incubación es de 30-45 días, comienza con síntomas gripales que duran 7-14 días, seguido del
cuadro florido durante dos a cuatro semanas y caracterizado por fiebre alta, astenia, anorexia,
dolor faríngeo intenso, mialgias, cefalea, adenopatías de predominio cervical,
hepatoesplenomegalia y exantema cutáneo maculopapular especialmente con la administración de
ampicilina. Se caracteriza por presentar linfocitosis y 10-20% de linfocitos atípicos.(R1)

156. ¿A qué se denomina hemólisis gamma de un estreptococo?:

1. 1. Hemólisis parcial de hematíes de caballo.


2. 2. Hemólisis parcial de hematíes de carnero.
3. 3. Ausencia de hemólisis.
4. 4. Presencia de un halo ventral alrededor de las colonias.
Gráfico de respuestas
Comentario

Se denomina hemólisis gamma de un estreptococo cuando, ante el cultivo de las especies de este
género en sangre, hay ausencia de hemolisis.

Alfa: halos verdosos (reducción de la hemoglobina de los glóbulos rojos a metahemoglobina en el


medio).

Beta: halos incoloros (hemolisis total).

Gamma: inexistencia de halos (sin hemolisis).(R3)

157. El síntoma habitual de unas hemorroides internas es:

!
!
!
!
1. 1. Prurito anal.
2. 2. Rectorragia dolorosa con la defecación.
3. 3. Rectorragia indolora con la defecación.
4. 4. Supuración anal.
Gráfico de respuestas
Comentario

Tema preguntado con bastante frecuencia en el bloque de cirugía. Las hemorroides internas se
producen por dilatación del plexo hemorroidal interno formado por venas rectales superiores y
media. Este plexo se encuentra en un espacio submucoso por encima de las válvulas de Morgagni.
Los síntomas más comunes son la rectorragia y el dolor (especialmente cuando se trombosan,
pero la trombosis de las internas es mucho menos frecuente que la de las externas por lo cual se
suele producir rectorragia indolora). Se dividen en 4 grados y según ellos se establece el
tratamiento.(R3)

158. Señale lo FALSO con respecto a la serología de la mononucleosis infecciosa por


virus de Epstein-Barr:

1. 1. En la fase aguda, se detectan IgM frente al antígeno de la cápside viral.


Los anticuerpos anti-ANEB son los que primero se detectan y su presencia indicaría que la
2. 2.
infección ha ocurrido unos días antes (menos de 1 semana).
Prueba de Paul-Bunell, que detecta IgM que reaccionarían frente a antígenos de hematíes
3. 3.
de carnero o de caballo, suele ser negativo en menores de 5 años.
4. 4. Los anticuerpos anti-ANEB duran toda la vida.
Gráfico de respuestas
Comentario

Efectivamente los anticuerpos heterófilos, detectados por la reacción de Paul-Bunnel o por el


Monospot test, son positivos con menor frecuencia en los niños.

La IgM ACV (contra la cápside viral) es propia de la primoinfección por VEB, como también lo es la
posterior seroconversión de los anticuerpos anti antígeno nuclear (ANEB), que se produce entre la
3ª y 6ª semana (opción 2 es falsa por tanto).

Los anti-ANEB permanecen durante toda la vida. Los títulos elevados de anticuerpos frente al
antígeno precoz restringido se asocian a formas graves de infección, como puede ser el linfoma de
Burkitt (también se usan para predecir el riesgo de carcinoma de cavum en poblaciones de
riesgo).(R2)

159. ¿Cuál de los siguientes fármacos es el responsable de la fase 0 (fase rápida) de


despolarización de un miocardiocito ventricular?

1. 1. Aumento de conductancia para el sodio (gNA).


2. 2. Aumento de conductancia para el potasio (gK).
3. 3. Aumento de conductancia para el cloruro (gCl).
4. 4. Aumento de conductancia para el calcio (gCa).
Gráfico de respuestas
Comentario

!
!
!
!
Pregunta fácil del tema de electrofisiología. En esta pregunta hay que recordar las fases del
potencial de acción de una célula cardíaca y su correspondencia con las corrientes iónicas que son
responsables de dichas fases:

•! Fase 0: fase de despolarización rápida. Aumento de la conductancia para el sodio y


apertura de canales rápidos de sodio.
•! Fases 1 y 2: fase de meseta. Producida por salida de potasio y entrada lenta de calcio.
Mantiene el potencial ligeramente positivo durante un tiempo.
•! Fase 3: repolarización. Salida de potasio y restablece el potencial de reposo.
•! Fase 4: la célula recupera el equilibrio iónico por la bomba sodio-potasio ATP
dependiente.(R1)

160. Una de las malformaciones más frecuentes del intestino delgado es el divertículo de
Meckel. Señale cuál de las siguientes afirmaciones acerca de esta entidad es FALSA:

1. 1. Con relativa frecuencia presenta mucosa ectópica gástrica o pancreática.


2. 2. Es un divertículo falso, ya que su pared carece de muscular.
Es un vestigio del conducto onfalomesentérico o vitelino, que no se reabsorbe de forma
3. 3.
completa.
4. 4. Su complicación más frecuente es la hemorragia.
Gráfico de respuestas
Comentario

El divertículo de Meckel es un verdadero divertículo, ya que está compuesto por las tres capas de
la pared intestinal. Si fuera un pseudodivertículo, sólo lo formarían una o dos. Este divertículo es un
vestigio del conducto onfalomesentérico, que no se reabsorbe en su totalidad durante la época
neonatal, con lo que no desaparece por completo. Suele localizarse a menos de 1 metro de la
válvula ileocecal. Puede estar revestido de mucosa ileal, o contener mucosa gástrica, lo que podría
justificar ulceración en las inmediaciones, si produce ácido. Es raro que produzca síntomas
después de los 5 años de edad, pero si lo hace suele ser la hemorragia.(R2)

161. ¿Cuál de los siguientes hallazgos no forma parte del diagnóstico del síndrome de
HELLP en la embarazada?

1. 1. Elevación de enzimas hepáticas.


2. 2. Trombocitopenia.
3. 3. Hipertensión arterial.
4. 4. Anemia hemolítica.
Gráfico de respuestas
Comentario

Tema básico para el ENARM, debe dominar el tema en todos sus aspectos. El síndrome HELLP
es la asociación de hemólisis, elevación de las enzimas hepáticas y plaquetopenia. A nivel
cerebral, se manifiesta el vasoespasmo como alteraciones occipitales tipo cefalea frontal, fotopsias,
escotomas y ceguera cortical transitoria y no como una verdadera encefalopatía hipertensiva, sin
que el grado de vasoespasmo se correlacione directamente con la gravedad (hasta 20% de
eclampsias debutan con TA normal o límite).(R3)

162. ¿Con cuál de las siguientes vasculitis se ha relacionado la leucemia de "células


peludas"?:

!
!
!
!
1. 1. PAN clásica.
2. 2. Vasculitis por hipersensibilidad.
3. 3. Arteritis de células gigantes.
4. 4. Angeítis y granulomatosis alérgica.
Gráfico de respuestas
Comentario

Existen diversas enfermedades asociadas con determinados tipos de vasculitis, conviene mucho
recordar las más características: la asociación de la panarteritis clásica con la hepatitis B sobre
todo, y, en algunos casos con la leucemia de células peludas; la crioglobulinemia mixta se asocia,
en altísimo porcentaje con el virus de la hepatitis C, mientras que la crioglobulinemia tipo I se
asocia con la macroglobulinemia de Waldestrom y el mieloma múltiple. Existen muchas
asociaciones con las vasculitis por hipersensibilidad, como el desarrollo de estas enfermedades
por fármacos, por infecciones (VEB, VIH, endocarditis bacterianas subagudas, hepatitis crónicas),
por otras enfermedades autoinmunes ( LES, Sde Sjögren artritis reumatoide) y neoplasias.(R1)

163. Ante una imagen mastográfica de microcalcificaciones agrupadas y moldeadas, sin


tumoración mamaria palpable, ¿cuál es el diagnóstico más probable?

1. 1. Carcinoma ductal infiltrante.


2. 2. Tumor filodes.
3. 3. Fibroadenoma.
4. 4. Carcinoma intraductal.
Gráfico de respuestas
Comentario

Ante una imagen mastográfica de microcalcificaciones agrupadas, siempre debe descartar la


presencia de un carcinoma de mama. La palabra “moldeadas” es clave en esta pregunta, y denota
que el tumor está en el interior de un conducto. Por tanto, lo más probable es que se trate de un
carcinoma intraductal.(R4)

164. En qué circunstancias indicaría Ud. quimioprofilaxis antituberculosa en un niño,


marque la o que considere apropiada.

1. 1. Niños con PPD mayor 9 mm sin enfermedad activa y menor de 5 años sin BCG.
2. 2. Niños con PPD › 15 mm, sin enfermedad activa.
3. 3. Niños con VIH positivo sin TB.
4. 4. Todas las anteriores.
Gráfico de respuestas
Comentario

Antes de empezar una quimioprofilaxis siempre se debe descartar emfermedad activa. Cuando una
persona está vacunada, el diametro de la induración debe superar los 15 mm para considerarse
significativo. Los paciente VIH tienen más riesgo de infección por TB, tambien por otras causas de
immunosupresión.(R4)

165. Decir cuál de los siguientes hechos del vértigo posicional paroxístico NO es cierto:

1. 1. Se diagnostica mediante las maniobras de Dix-Hallpike.


Tiene una latencia de unos pocos segundos y, una vez que comienza, dura unos 30
2. 2.
segundos.

!
!
!
!
3. 3. La base del tratamiento consiste en el uso de fármacos antivertiginosos.
4. 4. Normalmente, el vértigo no se asocia a hipoacusia.
Gráfico de respuestas
Comentario

El vértigo periférico tipo “Vértigo Posicional Paroxístico Benigno”, es un cuadro muy sencillo de
recordar. Se diagnostica con maniobras provocadoras como la que nos indican en la opción 1. Es
un cuadro que se desencadena con los cambios posturales del paciente y que tiene una corta
duración –de unos segundos- . Si repetimos la maniobra, presenta el fenómeno de fatiga o
habituación. Recuerde que el vértigo que se asocia a la hipoacusia es el Meniere y no el VPPB.
Por último, el tratamiento de elección no es con fármacos antivertiginosos, sino con maniobras de
reubicación canalicular.(R3)

166. Which of the following statements is correct regarding human egg implantation in
the uterus?

1. 1. Implantation usually occurs between 3 and 4 days after ovulation.


2. 2. Implantation usually occurs between 6 and 7 days after ovulation.
3. 3. Implantation usually occurs between 12 and 14 days after ovulation.
4. 4. Implantation usually occurs between 16 and 18 days after ovulation.
Gráfico de respuestas
Comentario

Sabemos que exiten personas que no tienen ningún problema con inglés pero también otras que
si, debe tratar de acostubrarse en caso de que le cueste trabajo leer con calma y no apresurase
recuerde que son 45 preguntas en el examen que pueden repuercutir en el puntaje final,
generalmente no son nada dificiles y vinenen intercaladas con unas con otras de forma aleatoria.

Pregunta sobre la fisiología de la fecundación e implantación. Teniendo en cuenta que el óvulo


fecundado tarda 6-7 días en recorrer la trompa e implantarse en el endometrio; y que el óvulo sólo
puede ser fecundado en las primeras 24 horas después de la ovulación, la implantación ocurrirá
entonces a los 6-7 días después de la ovulación (en fase de blastocisto).(R2)

167. Lactante de 2 meses de edad de origen asiático cuyos padres traen al servicio de
urgencias por aparición de coloración amarillenta progresiva desde el nacimiento. El
embarazo fue normal, con parto a las 41 semanas, eutócico y en cefálica, con peso al
nacimiento de 3.020 kg. Alimentada con lactancia materna exclusiva con buena ganancia
ponderal. A la exploración presenta ictericia verdínica generalizada, abdomen globoso,
hepatomegalia de 2 cm y se palpa el polo del bazo. Usted indaga sobre el aspecto de las
heces y los padres comentan que son más claras de lo normal, en ocasiones
blanquecinas. Se realizan estudios de laboratorio que reportan lo siguiente: bilirrubina
total 28.1 mg/dl, bilirrubina directa 25.9 mg/dl, AST 338 U/l, ALT 170 U/L, GGT 233 U/l,
fosfatasa alcalina 350 U/l y LDH 675 U/l. Respecto a la patología que usted sospecha,
señale la afirmación FALSA:

1. 1. Se trata de una ictericia colestática.


Con frecuencia se observa dilatación del colédoco en el ultrasonido abdominal con
2. 2.
vesícula biliar dilatada.
3. 3. Existe un trastorno en la excreción de bilis.
4. 4. El diagnóstico se confirma con biopsia hepática.
Gráfico de respuestas

!
!
!
!
Comentario

Esta pregunta pertenece al tema de ictericia neonatal. Para saber cuál es la opción falsa en primer
lugar debemos saber cuál es la enfermedad que padece esta paciente.

Si la ictericia aparece desde el nacimiento, es progresiva y verdínica a expensas de bilirrubina


directa no puede ser otra cosa que una ictericia colestática.

Dentro de éstas sobre todo existen dos patologías importantes: la hepatitis neonatal y la atresia de
vías biliares.

La hepatitis neonatal es un cuadro transitorio que se presenta con mayor frecuencia en recién
nacidos prematuros y de bajo peso, lo cual no es el caso que nos describen, por lo que en primer
lugar debemos sospechar una atresia biliar.

En esta entidad se produce una estrechez de la vía biliar que dificulta la excreción de la bilis, lo que
condiciona una ictericia colestática con coluria y acolia, y lo que hace muy improbable que en el
ultrasonido se reporte una dilatación de la vía biliar.

El diagnóstico definitivo es anatomopatológico y su tratamiento es quirúrgico con


portoenterostomía de Kasai y trasplante a largo plazo, siendo la principal indicación de trasplante
hepático en la infancia.(R2)

168. De las siguientes intervenciones quirúrgicas, todas se clasificarían como limpias,


SALVO:

1. 1. Mastectomía simple.
2. 2. Herniorrafia.
3. 3. Safenectomía.
4. 4. Gastrectomía.
Gráfico de respuestas
Comentario
Teniendo en cuenta la clasificación de las heridas quirúrgicas en función del grado de
contaminación debemos recordar que se consideran cirugías limpias aquellas en las que no existe
apertura del tracto gastrointestinal, ni respiratorio ni tampoco genitourinario, además de aquellas en
las que no existe infección previa ni son traumáticas. En virtud de esta clasificación no podemos
considerar la gastrectomía como cirugía limpia ya que para extirpar el estómago debemos contar el
tubo digestivo en algún momento así como para la realización de las anatomosis pertinentes
utilizadas en la reconstrucción del transito digestivo.(R4)

169. ¿Cuál de las siguientes asociaciones fármaco antihipertensivo - efecto indeseado es


INCORRECTA?:

1. 1. Hidroclorotiazida - hipopotasemia.
2. 2. Enalaprilo - hiperpotasemia.
3. 3. Doxazosina - hipotensión ortostática.
4. 4. Verapamilo - taquicardia.
Gráfico de respuestas
Comentario

!
!
!
!
El verapamilo, por su acción de antagonista del calcio, sobre las células del corazón reduce la
contractibilidad y la frecuencia cardiaca. Por lo que como efecto adverso se asocia con bradicardia,
no taquicardia. El resto de las opciones son correctas.(R4)

170. La mielofibrosis con metaplasia mieloide puede presentar las siguientes


características, con la EXCEPCION de:

1. 1. Esplenomegalia sintomática.
2. 2. Diátesis hemorrágica en un 20% de los casos.
3. 3. Hiperuricemia en un 25-30%.
4. 4. Poliadenopatías en un 50%.
Gráfico de respuestas
Comentario

Pregunta de dificultad intermedia. Solo con un poco de sentido común puede responder
correctamente esta pregunta. La mielofibrosis con metaplasia agnogénica forma parte de los
síndromes MIELOproliferativos crónicos, y no se produce en principio una alteración de la serie
linfoide. Luego no es característica de esta enfermedad la presencia de poliadenopatias (opción 4
falsa), ya que es típica de los síndromes LINFOproliferativos.(R4)

171. La colitis ulcerosa y la enfermedad de Crohn o colitis granulomatosa son dos


entidades a veces no bien diferenciadas. Señale cuál de las siguientes afirmaciones,
referentes a estas dos entidades, es FALSA:

En la colitis ulcerosa, la afección del recto es prácticamente constante, mientras que en la


1. 1.
enfermedad de Crohn sólo se presenta en el 50% de los casos o menos.
La colitis ulcerosa sangra poco, y las hemorragias son muy frecuentes en la enfermedad de
2. 2.
Crohn.
La enfermedad de Crohn es una afección transmural, mientras que la colitis ulcerosa es
3. 3.
una enfermedad de la mucosa.
La enfermedad de Crohn suele cursar con extensas e importantes lesiones del intestino
4. 4. delgado, mientras que en la colitis ulcerosa la afección fuera del intestino grueso es
excepcional.
Gráfico de respuestas
Comentario

Una buena pregunta para revisar las diferencias entre estas dos entidades. Es la colitis ulcerosa la
que tiene mayor tendencia al sangrado, mientras que las hemorragias son bastante más
infrecuentes en la enfermedad de Crohn.(R2)

172. Mujer con antecedente de insuficiencia renal crónica secundaria a nefropatía túbulo-
intersticial por reflujo vesicoureteral. En la revisión realizada destaca los siguientes
labortorios: Hb 12.5 g/dl, Hto 35%, VCM 93, Fe sérico 70 mg/dl, saturación de transferrina
30% y ferritina sérica de 120 ng/dl, Cr sérica 3 mg/dl, BUN 40 mg/dl, CCr de 40 ml/min, Ca
9.2 mg/dl y P de 7 mg/dl. ¿Qué tratamiento pautaría usted?:

1. 1. Eritropoyetina subcutánea.
2. 2. Quelantes del fósforo.
3. 3. Dieta rica en productos lácteos.
4. 4. Administración de hierro intravenoso.

!
!
!
!
Gráfico de respuestas
Comentario

Debe tener una idea del tratamiento conservador del paciente con insuficiencia renal crónica. Nos
encontramos con una ligera anemia normocítica no ferropénica, pero has de saber que el
tratamiento con EPO se pauta para mantener el hematocrito entre 33-36% y la hemoglobina entre
11-12g/dl, y esta paciente todavía tiene estos parámetros. No necesita aporte de hierro porque la
normalidad de la sideremia en mujeres está entre 35-140 mg/dl y el índice de saturación de
transferían entre 20-50%. La paciente todavía conserva un aclaramiento de creatinina de 40
ml/min, y la indicación de diálisis está en 5 ml/min (aunque no es una respuesta en esta ocasión).
El Ca++ está dentro de la normalidad 8.5- 10.5mg/dl por lo que no necesita aportes
suplementarios. Sin embargo el fósforo está por encima de la normalidad que es 2.6- 4.5 mg/dl,
con lo cual es necesario administrar quelantes de fósforo como carbonato cálcico por vía oral para
que el metabolismo fosfocálcico se aproxime a la normalidad. Recuerde que cuando el producto
Ca++ x P = 70 puede haber calcificación metastásica en partes blandas (y en este caso está muy
cerca). No nos dan ningún dato del K+, y aunque lo dieran en la IRC lo primero que se hace es
reducir el aporte antes que dar resinas de intercambio iónico. (R2)

173. En la valoración en el Servicio de Urgencias hospitalario de un paciente con dolor


abdominal agudo, ¿cuál de las siguientes afirmaciones es INCORRECTA?

1. 1. El inicio, la localización y severidad de dolor son útiles en el diagnóstico diferencial.


2. 2. La palpación es el aspecto más importante de la exploración física.
El ultrasonido es una prueba de imagen útil y cada vez más utilizada en la valoración del
3. 3.
dolor abdominal.
No debe administrarse medicación analgésica hasta que el cirujano valore al paciente
4. 4.
porque puede oscurecer el diagnóstico.
Gráfico de respuestas
Comentario

Cuidado con esta pregunta ya que es un concepto que ha cambiado hace poco. Existen estudios
publicados recientemente que demuestran que la administración de analgésicos mejora la
asistencia a estos pacientes con abdomen agudo, sin dificultar el diagnóstico. En este estudio, se
empleaban fármacos pertenecientes al primer escalón de la escala analgésica de la OMS
(paracetamol, AINE).(R4)

174. En el diagnóstico de las infecciones por Helicobacter pylori se emplean los


siguientes métodos, EXCEPTO:

1. 1. Tinción de Giemsa de las biopsias gástricas.


2. 2. Test del aliento con urea 13C.
3. 3. Cultivo de heces.
4. 4. Prueba de la ureasa sobre muestra biópsica.
Gráfico de respuestas
Comentario

La forma de cultivar el H. pylori sería a través de una muestra tomada directamente de la mucosa
gástrica, y por supuesto en medios selectivos, por los particulares requerimientos nutricionales de
esta bacteria. Sin embargo, el coprocultivo carece de interés para el diagnóstico de esta
infección.(R3)

!
!
!
!
175. El tumor de Krukenberg del ovario:

1. 1. Es metastático de origen pulmonar.


2. 2. Es metastático de origen en vejiga.
3. 3. Es metastático de origen en aparato digestivo.
4. 4. Sólo se presenta pasada la menopausia.
Gráfico de respuestas
Comentario

Esta pregunta de tumores de ovario no es muy importante, pero no está de más que sepa el tumor
de Krukenberg es la metástasis ovárica de un adenocarcinoma del aparato digestivo, normalmente
estomago, que se caracteriza por células repletas de moco con aspecto de anillo sello. No todo
tumor prodecedente del aparato digestivo es Krukenberg ya que la denominación tiene una base
estrictamente histológica.(R3)

176. Paciente de 80 años que consulta por la lesión


en cara que se muestra en la imagen nº 2. ¿Cuál es el diagnóstico más probable?:

1. 1. Léntigo maligno.
2. 2. Melanoma lentiginoso acral.
3. 3. Léntigo actínico.
4. 4. Queratosis actínica pigmentada.
Gráfico de respuestas
Comentario

Ante una lesión pigmentada grande, policroma e irregular en la cara de una paciente mayor, como
la que se muestra en la foto (lesión pigmentada de color marrón claro, marrón oscuro y negro, de
contorno irregular, de unos 2 x 2.5 cm de diámetro, localizada en la mejilla) se debe descartar la
presencia de un léntigo maligno melanoma.

Clínicamente, puede ser difícil diferenciar un léntigo actínico de una queratosis actínica pigmentada
y de un léntigo maligno. Por eso, se hace indispensable el estudio histológico de estas lesiones. El
melanoma lentiginoso acral aparece, como indica su nombre, en zonas acras (palmas y plantas) y

!
!
!
!
es más frecuente en pacientes de raza negra. El epitelioma basocelular pigmentado también puede
simular a un melanoma, aunque si uno se fija bien, puede reconocerse el característico margen
perlado.(R1)

177. Un lactante de 7 meses, no vacunado, presenta tos en accesos, cuadro catarral y


febrícula. En los exámenes de laboratorio se reporta 25,000 leucocitos/ml con predominio
de linfocitos. Una muestra faríngea para detección rápida de Bordetella pertussis por
inmunofluorescencia resulta positiva. ¿Qué actitud le parece la más CORRECTA?:

1. 1. Administrarle inmunoglobulina específica.


2. 2. Tratar con eritromicina oral.
3. 3. Tratar con penicilina oral.
4. 4. No precisa ninguna medida terapéutica.
Gráfico de respuestas
Comentario

De esta enfermedad debemos recordar los datos típicos que son la presencia de accesos de tos
con "gallo" al final, el hallazgo de leucocitosis con linfocitosis y su tratamiento, que es la
eritromicina. Por lo tanto, la opción correcta es la 2. También debemos recordar que administrada
dentro de los primeros 14 días de evolución del cuadro, la eritromicina tiene capacidad curativa,
pero si se hace posteriormente únicamente produce disminución del período de contagio sin influir
en la evolución clínica.(R2)

178. ¿Qué alteración ecocardiográfica es sugestiva de taponamiento cardiaco?:

1. 1. Alternancia eléctrica.
2. 2. Alternancia mecánica.
3. 3. Colapso diastólico de cavidades derechas.
4. 4. Colapso sistólico de cavidades derechas.
Gráfico de respuestas
Comentario

Las respuesta 4 no puede ser, porque obviamente en sístole por definición las cavidades siempre
estan colapsadas. La alternancia eléctrica es una alteración electrocardiográfica y la alternancia
mecánica inidica severidad del derrame, pero no taponamiento. La respuesta correcta es la 3:
durante la diástole ventricular, la presión que el derrame pericárdico hace sobre el ventrículo hace
que éste no se pueda llenar. (R3)

179. Lactante de 21 días de vida que presenta, desde hace 4 días, vómitos postpandriales
que cada vez son más intensos y frecuentes. Señale cuál de las siguientes afirmaciones
es CIERTA en relación a la enfermedad del niño:

1. 1. Esta enfermedad es más frecuente en niños.


2. 2. Suele cursar con acidosis metabólica.
3. 3. La enfermedad está producida por una bacteria.
4. 4. Es frecuente el antecedente de polihidramnios
Gráfico de respuestas
Comentario

Se trata de una estenosis hipertrófica del píloro. Se presenta normalmente entre los 20 y 45 días.
Cursa con ALCALOSIS metabólica y se debe a una predisposición multifactorial (genética y

!
!
!
!
ambiental). No es frecuente el antecedente de polihidramnios porque el cuadro no aparece hasta
pasados unos días tras el nacimiento (20 días por norma general).(R1)

180. Given the aforementioned data, which of the


following is the most appropriate treatment for this patient?

1. 1. Intravenous penicillin G Sodium.


2. 2. Doxycycline
3. 3. Cotrimoxazole.
4. 4. Gentamicin.
Gráfico de respuestas
Comentario

La enfermedad que se describe es una Fiebre Botonosa Mediterránea. Es coherente con los
factores de riesgo: ocurre en verano y en medio rural. Es coherente con el cuadro clínico: fiebre
alta, cefalea intensa, alteración leve de enzimas hepáticos y exantema cutáneo que
característicamente afecta a las palmas y a las plantas. También es coherente con la fotografía
que se facilita en la cual se ve una lesión compatible con mancha negra: úlcera necrótica de
pequeño tamaño rodeada de un halo eritematoso. Por tanto la respuesta correcta es que esta
enfermedad está producida por Rickettsia conorii y que el tratamiento de primera elección es
doxiciclina.

Treponema pallidum es el agente causal de la sífilis. También produce fiebre y exantema que
afecta a palmas y plantas en la sífilis secundaria pero no se describe factor de riesgo para
enfermedad de transmisión sexual y no tendría que tener mancha negra.

Borrellia burdogferi es el agente causal de la enfermedad de Lyme. Se transmite por picadura de


garrapata, al igual que la Fiebre Botonosa Mediterránea, pero no tiene mancha negra y la
afectación cutánea inicial no es en forma de exantema generalizado. La afectación cutánea de la
primera fase es una lesión "en diana" denominada eritema cronico migrans.

!
!
!
!
Bartonella henselae produce la enfermedad por arañazo de gato y lesiones cutáneas de tipo
"vascular" en pacientes infectados por VIH (la denominada angiomatosis bacilar). No es compatible
con el cuadro clínico y fotográfico que se describe.

Coxiella burnetii produce la denominada Fiebre Q: no tiene lesiones cutáneas. El cuadro agudo
suele ser una neumonía atípica acompañada de hepatitis y el cuadro crónico una endocarditis
sobre válvula aórtica con hemocultivos negativos.(R2)

181. A 46-year-old woman comes to the emergency department in the middle of summer
with a 3-day history of high fever (40 ºC) and severe headache. She has returned from a
vacation trip from Europe. Today, a generalized rash appeared on her skin, involving her
palms and soles (as seen in the picture). The upper part of the image shows another skin
lesion, on the posterior part of her thigh. Physical examination reveals no other
significant findings. Laboratory tests show: Hemoglobin 14.1 gr/dL, WBC count: 4300
mm3, lymphocytopenia, AST 68 U/L, ALT 47 U/L. X-ray of the chest shows no
abnormalities. Which of the following options most likely caused her current condition?

1. 1. Treponema pallidum.
2. 2. Borrelia burgdorferi.
3. 3. Bartonella henselae.
4. 4. Rickettsia conorii.
Gráfico de respuestas
Comentario

La enfermedad que se describe es una Fiebre Botonosa Mediterránea. Es coherente con los
factores de riesgo: ocurre en verano. Es coherente con el cuadro clínico: fiebre alta, cefalea
intensa, alteración leve de enzimas hepáticas y exantema cutáneo que característicamente afecta
a las palmas y a las plantas. También es coherente con la fotografía que se facilita en la cual se ve
una lesión compatible con mancha negra: úlcera necrótica de pequeño tamaño rodeada de un halo
eritematoso. Por tanto la respuesta correcta es que esta enfermedad está producida por Rickettsia
conorii y que el tratamiento de primera elección es doxiciclina.

Treponema pallidum es el agente causal de la sífilis. También produce fiebre y exantema que
afecta a palmas y plantas en la sífilis secundaria pero no se describe factor de riesgo para
enfermedad de transmisión sexual y no tendría que tener mancha negra.

Borrellia burdogferi es el agente causal de la enfermedad de Lyme. Se transmite por picadura de


garrapata, al igual que la Fiebre Botonosa Mediterránea, pero no tiene mancha negra y la
afectación cutánea inicial no es en forma de exantema generalizado. La afectación cutánea de la
primera fase es una lesión "en diana" denominada eritema cronico migrans.

Bartonella henselae produce la enfermedad por arañazo de gato y lesiones cutáneas de tipo
"vascular" en pacientes infectados por VIH (la denominada angiomatosis bacilar). No es compatible
con el cuadro clínico y fotográfico que se describe.

Coxiella burnetii produce la denominada Fiebre Q: no tiene lesiones cutáneas. El cuadro agudo
suele ser una neumonía atípica acompañada de hepatitis y el cuadro crónico una endocarditis
sobre válvula aórtica con hemocultivos negativos.(R4)

182. La mortalidad de la enfermedad de Kawasaki es del:

!
!
!
!
1. 1. 0.5-3%.
2. 2. 10%.
3. 3. 25%.
4. 4. No existe mortalidad asociada a la enfermedad.
Gráfico de respuestas
Comentario

La enfermedad de Kawasaki es un síndrome agudo febril que afecta a niños caracterizado por
adenopatías cervicales no supuradas, fiebre, lesiones de la mucosa oral y conjuntival y
alteraciones de la piel como exantema y descamación de la piel de los dedos. Suele ser una
enfermedad benigna que se resuelve espontáneamente, pero en ocasiones se complica con el
desarrollo de aneurismas coronarios en un 25%, presentando la enfermedad una mortalidad de
entre un 0.5 y un 2.8%. El tratamiento precoz con aspirina y gammaglobulinas disminuye la
frecuencia de aparición de aneurismas.(R1)

183. Paciente de 2 días de vida extrauterina, con antecedente de depresión respiratoria


severa de recuperación lenta secundaria a prolapso de cordón. Recibe maniobras de
reanimación durante 10 minutos y hoy presenta convulsiones y flacidez persistente.
Usted pantearía el siguiente diagnóstico.

1. 1. Encefalopatía hipóxica isquémica.


2. 2. Muerte cerebral.
3. 3. Persistencia de conducto arterioso.
4. 4. Enterocolitis necrotizante.
Gráfico de respuestas
Comentario

Una de las causas más frecuentes de crisis convulsivas en RN y retraso mental es secundario a
encefalopatía hipóxico-isquémica y más conociendo el antecedente de prolapso del cordón.
Respuesta 1 correcta.(R1)

184. Ante un paciente ADVP activo que presenta lesiones pustulosas en áreas pilosas,
endoftalmitis y artritis condrocostal, usted deberá sospechar:

Que el paciente consume además anfetaminas, por lo que padece una infección por
1. 1.
Eikenella corrodens.
Que el paciente ha utilizado agua de charcos para disolver la droga, por lo que padece una
2. 2.
infección por Pseudomonas.
3. 3. Que el paciente consume heroína marrón y padece una candidiasis diseminada.
Que el paciente presenta una bacteriemia por S. aureus a partir del punto de inyección de
4. 4.
la droga.
Gráfico de respuestas
Comentario

Pregunta fácil acerca de infecciones en el paciente ADVP. Debe saber reconocer este cuadro
clínico: foliculitis + endoftalmitis + afectación condrocostal = candidiasis diseminada. Normalmente,
te lo presentan en un ADVP adicto a la heroína marrón. No es confunda con la afectación
esternoclavicular del S. aureus, que también puede dar foliculitis, pero no asociaría el resto de la
clínica.

!
!
!
!
La candidiasis diseminada se caracteriza porque tras una fase febril, el paciente desarrolla una
triada clásica de lesiones cutáneas (pústulas y nódulos subcutáneos en zonas pilosas), oculares
(coriorretinitis o endoftalmitis) y osteoarticulares. La endocarditis es excepcional. La principal
secuela de esta infección es la pérdida de agudeza visual. La mortalidad es prácticamente nula.

El tratamiento de elección es la anfotericina B por vía intravenosa, pudiendo ir asociada a 5-


flucitosina si no existen contraindicaciones. La cirugía es necesaria en la afectación ocular y en los
raros casos de endocarditis.(R3)

185. Cuando acude a la consulta un paciente joven con una masa cervical en la línea
media que en la exploración si le hacemos deglutir observamos que se produce su
ascenso, hemos de pensar que puede tratarse de:

1. 1. Adenopatía.
2. 2. Fibroma.
3. 3. Quiste tirogloso.
4. 4. Neurinoma.
Gráfico de respuestas
Comentario

Los quistes cervicales más frecuentes en la línea media son los procedentes del conducto
tirogloso. Suelen aparecer en la primera década de la vida, como una masa única en línea media.
Típicamente, ascienden con la deglución, tal como nos describen en este caso clínico.

Una de las posibles complicaciones es la sobreinfección, con dolor local y ocasional formación de
fístulas cutáneas. Aunque no es frecuente (1% de los casos) puede malignizar, produciéndose un
carcinoma papilar. Los procedimientos diagnósticos más comunes son la PAAF, la ecografía y el
TAC. Antes de la extirpación debemos realizar una gammagrafía tiroidea para descartar que sea el
único tejido tiroideo funcionante.

El tratamiento quirúrgico es la cirugía, extirpándose también el cuerpo del hueso hioides. El motivo
de hacerlo es que suelen estar en contacto con este hueso. Por ello, si no se extirpa, existe cierto
riesgo de recidiva.(R3)

186. A 54-year-old Caucasian man has an ulcer on the right foot, which you find
surrounded by erythema and edema. He is diabetic and hypertense. An x-ray reveals
some grade of underlying osteomyelitis. The most likely mechanism for bone involvement
in this patient is:

1. 1. Contiguous spread.
2. 2. Lymphogenous.
3. 3. Hematogenous.
4. 4. Iatrogenic contamination.
Gráfico de respuestas
Comentario
Contiguous spread. Diabetic ulcers occur as result of different factors: changes in the bony
architecture, arterial and neural disease, etc. Nonenzymatic glycation predisposes ligaments to
stiffness. Neuropathy causes loss of protective sensation and loss of coordination of muscle groups
in the foot and leg, both of which increase mechanical stresses during ambulation.(R1)

!
!
!
!
187. A un hombre de 74 años de edad con historia de enfermedad coronaria se le ha
practicado un triple by-pass coronario hace 3 días. Se le extubó en el primer día
postoperatorio. Ahora requiere valoración porque está agitado a pesar de antipsicóticos
y sujección mecánica. Se quita el tubo de oxígeno a pesar de sujeción en muñecas.
Debido a su confusión se ha limitado la deambulación y mantiene una sonda vesical. Los
signos vitales son normales y el médico recomienda quitar la sonda vesical y realizar una
análisis de orina con cultivo. ¿Cuál de los siguientes puede significativamente disminuir
la agitación y ayudar a resolver el estado confusional agudo?

Limitar la movilidad a transferencia al sillón hasta que se resuelva el estado confusional


1. 1.
agudo.
2. 2. Retirar las sujeciones mecánicas de las muñecas.
3. 3. Ayudarle en la ingesta.
4. 4. Aumentar la dosis de antipsicóticos.
Gráfico de respuestas
Comentario

El caso clínico describe un delirium (síndrome confusional agudo) en el posoperatorio de una


cirugía cardíaca extracorpórea, posiblemente una de las intervenciones médicas con mayor tasa
de delirium.

El paciente está agitado y ha sido necesario administrarle antipsicóticos (probablemente


haloperidol) y aplicar contención mecánica. Por lo que parece su situación médica ha ido
mejorando y ha sido posible extubarle, planteándose incluso la retirada del sondaje vesical.

Nos preguntan cuál sería la medida que podría contribuir más a la resolución de la agitación y del
síndrome confusional.

No hay que olvidar que conforme el delirium va resolviéndose, algunas de las decisiones tomadas
inicialmente para proteger al paciente de accidentes derivados de la agitación pueden ser
contraproducentes, aumentando la inquietud del paciente.

Se ha comprobado que la inmovilización prolongada es un factor importante en la aparición de un


delirium. Por tanto, es necesario intentar la movilización precoz de los pacientes mediante la
deambulación supervisada o ejercicios de activación motora en la propia cama; asimismo habrá
que minimizar aquellas medidas que inmovilizan, como sondajes innecesarios, correas de sujeción
o sistemas de suplementación de oxígeno.

La medicación sedante debe ser utilizada en la mínima dosis y el mínimo tiempo posibles para
evitar que empeore el estado mental del paciente, eligiendo siempre los medicamentos menos
tóxicos para el funcionamiento cognitivo.

Otras medidas que se han demostrado útiles son las dirigidas a una estimulación cognitiva
(orientación en la realidad con relojes y calendarios), el mantenimiento de un ritmo de sueño
aceptable (control del ruido nocturno, planificación horaria de las intervenciones sobre el paciente),
facilitación de ayudas sensoriales (gafas, audífonos), cuidados médicos generales, etc.(R2)

188. ¿Qué cuadro clínico NO se ha relacionado con las infecciones por enterovirus en la
infancia?:

1. 1. Poliomielitis.

!
!
!
!
2. 2. Fiebre reumática.
3. 3. Pericarditis.
4. 4. Linfoma de Burkitt.
Gráfico de respuestas
Comentario

Los enterovirus son gérmenes implicados con frecuencia en las infecciones infantiles; un
enterovirus es el agente responsable de la poliomielitis. Pero, además de éste, ha de conocer otros
enterovirus, como el Coxsackie A (responsable de a herpangina), el Coxsackie B (responsable de
la mialgia epidémica de Bornholm) y los echovirus (implicados en miocarditis y pericarditis). El
linfoma de Burkitt puede tener relación etiológica con el VEB, que es un herpesvirus.(R4)

189. El Kwashiorkor es un síndrome clínico resultado de una grave deficiencia en:

1. 1. Proteínas.
2. 2. Grasas.
3. 3. Niacina.
4. 4. Tiamina.
Gráfico de respuestas
Comentario

Esta pregunta es importante no fallarla Es un tema básico para el ENARM. Debemos tener claros
los conceptos de Kwashiorkor malnutrición proteínas y marasmo o malnutrición calórica. Recuerde
que el déficit de vitamina A produce ceguera nocturna y sequedad de piel; el de niacina pelagra y
el de tiamina, beriberi.(R1)

Revise la siguinete guía de práctica clínica.

http://www.cenetec.salud.gob.mx/descargas/gpc/CatalogoMaestro/119_GPC_DESNUTRIC

190. ¿Cuál sería el diagnóstico más probable ante la aparición de una pápula color carne,
bien delimitada, blanda y grasienta al tacto, en la región pectoral de un paciente de 53
años?

1. 1. Queratosis seborreica.
2. 2. Quiste epidérmico.
3. 3. Xantoma eruptivo.
4. 4. Queratoacantoma en fase inicial.
Gráfico de respuestas
Comentario

Aunque la descripción dermatológica del enunciado no es demasiado afortunada, la pregunta se


puede contestar sin problemas.

Las queratosis seborreicas son tumores benignos de la piel sumamente comunes. De hecho, se
puede decir que cualquier persona desarrollará una a lo largo de su vida. Sólo con este dato, se
podría contestar la pregunta, porque la queratosis seborreica es muchísimo más prevalente que el
resto de opciones. Más claves para contestar adecuadamente serían la edad (típicamente, se da

!
!
!
!
en pacientes mayores de 40 años) y la localización (el tronco es un lugar muy frecuente). Se les da
el nombre de “seborreicas” por su aspecto, pero no tienen nada que ver con las glándulas
sebáceas. Muestran una topografía verrucosa, con múltiples folículos obturados y arborescencias
(de ahí que en el enunciado te hablen de “ grasienta al tacto”).

Las verrugas vulgares son pápulas ásperas, escamosas y espinosas (no blandas). Lo más
frecuente es que se presenten en las manos.

El quiste epidérmico, como su nombre indica, se presenta como un tumor sólido intradérmico (no
se trata de una pápula). Los veremos como una protuberancia de la piel en forma de cúpula.

Los xantomas son lesiones infrecuentes que se producen como consecuencia de alteraciones en el
metabolismo de las grasas. Característicamente, presentan un color amarillento (no color carne).
En concreto, los xantomas eruptivos son lesiones múltiples, características de la
hiperquilomicronemia severa.

El queratoacantoma es un tumor benigno que, en su mayoría, se localizan en piel expuesta al sol


(por tanto, no en el área pectoral). Su apariencia y descripción es muy característica; nódulo
cupuliforme con cráter queratósico central. La mayoría de los queratoacantomas regresan de forma
espontánea.(R1)

191. Un paciente de 78 años de edad, previamente sano, que vive en una residencia de
ancianos bastante masificada y con insuficientes recursos higiénicos, padece un cuadro
diarréico desde hace 6 semanas. Refiere molestias abdominales tipo retortijón, febrícula
ocasional y 4-6 deposiciones diarias, alguna de ellas nocturna, con mucosidad y, en
ocasiones, con hebras de sangre. Entre los diagnósticos que se enumeran a continuación
seleccione el que le parece MENOS probable:

1. 1. Cáncer de colon.
2. 2. Infección por Clostridium difficile.
3. 3. Enfermedad de Crohn.
4. 4. Salmonelosis.
Gráfico de respuestas
Comentario

Es una pregunta bastante difícil y confusa.

Se trata de un varón sin factores de riesgo que vive en una residencia (factor epidemiológico), con
diarrea crónica mucosa y, en ocasiones, sanguinolenta. La opción 1 puede ser verdadera, por el
aumento de incidencia con la edad, la febrícula (típica del cáncer de colon) y la diarrea secretora
(como en los pólipos adenomatosos vellosos).

La opción 2 puede ser verdadera: vive en una residencia masificada y el tipo de diarrea es
característico del Clostridium. En cuanto a la opción 3 hay un segundo pico de incidencia de EII en
ancianos y puede cursar con diarrea crónica secretora y sanguinolenta. La colitis isquémica se
produce en pacientes con factores de riesgo cardiovascular (que nos faltarían en este caso) y
también puede cursar de forma subaguda.

Por consiguiente, la opción 4 sería la menos probable, ya que cursa de forma aguda con diarrea y
afectación general importante, fiebre elevada y, en ancianos, incluso con signos de
deshidratación.(R4)

!
!
!
!

192. Hombre de 2 años que acude por fiebre


de 39ºC de 1 semana de evolución y reacción cutánea que se observa en la figura. El
exantema apareció hace 2 días, inicialmente en el tronco y el área del pañal. Niegan
síntomas gripales. En la exploración presenta una hiperemia faríngea marcada, sin
exudados, cierta irritación conjuntival y exantema de distribución generalizada con
afectación de palmas y plantas, que además están algo tumefactas. Resto sin interés.
Señale la opción que considere CORRECTA:

1. 1. La aparición de lengua aframbuesada nos dará el diagnóstico de escarlatina.


El caso es compatible con enfermedad de Kawasaki, derivaría a hospital para realización
2. 2.
de pruebas complementarias y tratamiento con gammaglobulina iv.
El caso es compatible con sarampión, extraería serologías, investigaría contactos y en caso
3. 3.
de ser positivo, reportaría la enfermedad.
4. 4. Estamos ante una varicela. Advertiría de evitar aspirina por el riesgo de síndrome de Reye.
Gráfico de respuestas
Comentario

El caso descrito corresponde con una enfermedad de Kawasaki ya que reúne los 5 criterios
clínicos: Fiebre de al menos 4-5 días con la aparición (criterio obligatorio) y al menos 4 de los
siguientes criterios: alteraciones en mucosa orofaríngea (eritema faríngeo, labios eritematosos o
fisurados, lengua aframbuesada), conjuntivitis bilateral no exudativa, alteraciones en zonas acrales
(edema-eritema de manos y pies, descamación periungueal), exantema polimorfo, no vesicular y
linfadenopatía cervical unilateral.

El caso presenta hiperemia faríngea marcada y labios eritematosos, exantema, conjuntivitis


bilateral y edema en manos y pies; por ello están presentes 4 de esos 5 criterios, además de la
fiebre.

La principal complicación de esta enfermedad sería la formación de aneurismas coronarios y por


ello es necesaria la instauración precoz de gammaglobulina intravenosa , dentro de la 1º semana.

La opción 1 no es correcta ya que la escarlatina, puede presentarse con lengua aframbuesada y


exantema, pero no hay que olvidar que suele presentarse con faringoamigdalitis exudativa y sin
presencia de hiperemia conjuntival.

La opción 3 (sarampión) es improbable por la ausencia de síntomas respiratorios.

L opción 4 (varicela) faltarían las lesiones definitorias: las vesículas, que en la figura no están
presentes.(R2)

!
!
!
!
193. Señale entre las siguientes la complicación más infrecuente en el curso de esta
enfermedad:

1. 1. Meningitis.
2. 2. Artritis.
3. 3. Neumonía.
4. 4. Cardiopatía.
Gráfico de respuestas
Comentario

El caso descrito corresponde con una E. Kawasaki, donde la principal complicación, por su
gravedad, es la vasculitis coronaria y formación de aneurismas (25% de los casos), cuya
prevención dependerá del empleo de gammaglobulina intravenosa en estadios precoces de la
enfermedad.

Se han descrito más complicaciones derivadas de la vasculitis sistémica: meningitis aséptica,


artralgias y artritis, dolor abdominal, proteinuria y piuria estéril debido a inflamación glomerular,
hipertransaminasemia, anemia, trombocitosis; sin embargo la neumonía no es una de ellas
(OPCIÓN 3 CORRECTA). No obstante, la neumonía es una complicación potencial de
enfermedades exantemáticas como la varicela y el sarampión.(R3)

194. ¿Cuál de las siguientes NO es indicación absoluta de cirugía de la hiperplasia


benigna de próstata?:

1. 1. Urgencia miccional.
2. 2. Retención urinaria.
3. 3. Hidronefrosis.
4. 4. Infecciones de orina de repetición.
Gráfico de respuestas
Comentario

Pregunta relativamente sencilla, que debemos contestar correctamente. Las indicaciones absolutas
son: · Persistencia de sintomatología a pesar de correcto tratamiento médico · Hidronefrosis o
datos de uropatía obstructiva supravesical · Hematuria intensa y de repetición · ITU#s de repetición
· Episodios repetidos de retención urinaria aguda o imposibilidad de retirada de sonda urinaria ·
Litiasis vesical (dato de residuo postmiccional) Por tanto la opción 1 en incorrecta, porque lo que
deberíamos hacer es instaurar tratamiento médico adecuado y esperar a ver la evolución.(R1)

195. Cuál de las siguientes complicaciones no es frecuente en los pacientes que han
recibido un trasplante hepático alogénico?

1. 1. Rechazo hiperagudo.
2. 2. Trombosis de la arteria hepática.
3. 3. Infecciones por citomegalovirus.
4. 4. Enfermedad linfoproliferativa.
Gráfico de respuestas
Comentario

El trasplante hepático es un tema poco preguntado. Debe centrarse en las indicaciones y


contraindicaciones.

!
!
!
!
Las complicaciones y, por lo tanto, las causas de fracaso del trasplante varían a lo largo del
tiempo. En los tres primeros meses se suelen deber a complicaciones quirúrgicas de tipo técnico, o
infecciones postoperatorias. En este periodo de tiempo se suele desarrollar la trombosis de la vena
porta o de la arteria hepática.

Posteriormente, los fracasos se relacionan más con infecciones por la inmunosupresión, rechazo o
recidiva de la enfermedad primaria.

En el primer mes suelen ser infecciones bacterianas y, a partir del segundo, secundarias a la
inmunosupresión, apareciendo infecciones oportunistas como CMV, herpes, hongos… La
inmunosupresión también facilita el desarrollo de enfermedades linfoproliferativas. Y en cuanto al
rechazo, el más frecuente es el rechazo agudo celular, que es reversible con corticoides. Por el
contrario, el rechazo hiperagudo es poco frecuente. Se debe a incompatibilidad ABO, y el hecho de
seleccionar a pacientes compatibles antes de realizar el trasplante hace que su frecuencia sea
escasa. Se debe a la existencia de Ac contra las células endoteliales y no tiene tratamiento, la
única solución es el retrasplante.(R1)

196. En un paciente diagnosticado de traumatismo hepático mediante TC, el criterio más


importante para suspender el tratamiento conservador y proceder a la intervención
quirúrgica es:

1. 1. Que se evidencie dolor, íleo paralítico y distensión abdominal.


2. 2. Que se produzca inestabilidad hemodinámica del paciente.
3. 3. Presencia de leucocitosis.
4. 4. Presencia de hematocrito inferior a 30%.
Gráfico de respuestas
Comentario

Ante un traumatismo abdominal cerrado, está indicado realizar una TC cuando el paciente se
encuentra hemodinámicamente estable (si está inestable, se prefieren pruebas más rápidas como
el ultrasondio o el lavado peritoneal), permitiendo diagnosticar posibles roturas o hematomas
viscerales, y siendo el hígado el segundo órgano más afectado después del bazo.

La lesión hepática más frecuente es el hematoma subcapsular sin hemorragia activa. Como nos
dicen en el enunciado, el manejo del paciente de la pregunta es inicialmente conservador, y la
cirugía estaría indicada en caso de inestabilidad hemodinámica (respuesta 2 correcta), evidencia
de hemorragia activa en la TC o ante la aparición de signos de irritación peritoneal en la
exploración. En este caso la respuesta era deducible aunque no se supiera en profundidad del
tema porque el resto de opciones están referidas a hallazgos que no se corresponden
necesariamente con el fracaso del tratamiento conservador.(R2)

197. What is the first symptom to appear in primary biliary cirrhosis?

1. 1. Pruritus.
2. 2. Ascites.
3. 3. Xanthomas.
4. 4. Digestive bleeding due to esophageal varices.
Gráfico de respuestas
Comentario
Los síntomas de presentación más frecuentes de la cirrosis biliar primaria son el prurito y la
astenia. La bilirrubina es normal en las fases iniciales de la enfermedad y se eleva a medida que

!
!
!
!
ésta progresa, por lo que no es frecuente que sea el síntoma de presentación. Los xantomas
aparecen también más tardíamente. Es excepcional que la cirrosis biliar primaria se presente en su
estadio final, es decir, con síntomas de cirrosis franca, como ascitis o varices esofágicas.(R1)

198. Una limitación para el Lavado Peritoneal Diagnóstico en el trauma abdominal es:

1. 1. No poder descartar lesiones internas ni de órganos retroperitoneales.


2. 2. Alta incidencia de complicaciones.
3. 3. Tiempo requerido para su realización.
Posibilidad de introducir aire, lo que interfiere con la posible localización de un
4. 4.
neumoperitoneo.
Gráfico de respuestas
Comentario

El Lavado Peritoneal Diagnóstico se utiliza sobre todo en casos de traumatismos abdominales


cerrados en pacientes con inestabilidad clínica (al igual que el ECO-FAST). Su limitación es que, al
puncionar la fosa iliaca para determinar si existe líquido libre, no será capaz de detectar lesiones
retroperitoneales o internas que no se manifiesten con líquido libre peritoneal (respuesta 1
verdadera). Por lo demás es una técnica con pocas complicaciones y rápida (por eso se utiliza en
pacientes intestables).(R1)

199. Una paciente de 47 años ha sido diagnosticada de esofagitis por reflujo


gastroesofágico y está en tratamiento con omeprazol. Le consulta para saber su opinión
sobre el tratamiento y porque ha oído que ese fármaco produce tumores. ¿Cuál de las
siguientes afirmaciones NO sería correcta?:

1. 1. Efectivamente, hay casos de tumores carcinoides por la hipoclorhidria mantenida.


2. 2. Es el mejor tratamiento médico para su enfermedad.
3. 3. Es un tratamiento que cura la esofagitis.
Para prevenir la recaída de su esofagitis, necesitaría tomar la misma dosis que para el
4. 4.
tratamiento.
Gráfico de respuestas
Comentario

Nos planteaban un paciente en tratamiento con omeprazol por úlcera gástrica, que tenía
hipergastrinemia. Tal como se explicó, esta hipergastrinemia se debe al aumento del pH del
contenido intestinal, al producirse en el estómago una menor cantidad de ácido. Esto condiciona
una hiperactividad de las células G, que harán más gastrina, tratando de compensar este déficit de
ácido.
En tomadores crónicos de omeprazol podría producirse una HIPERPLASIA de las células G. Sin
embargo, de TUMORES afortunadamente todavía no se han descrito como consecuencia de este
tratamiento. La respuesta incorrecta, por tanto, es la 1. El paciente está mal informado, cuando
dice haber oído rumores de este tipo.(R1)

200. ¿Cuál de los siguientes casos dejaría evolucionar por vía vaginal?

1. 1. Presentación de cara, mentoposterior.


2. 2. Gemelar, primero en podálica.
3. 3. Feto con meningocele.
4. 4. Primípara, nalgas con feto muerto.

!
!
!
!
Gráfico de respuestas
Comentario

Vamos a analizar varias posibilidades que podrían existir:

- Situación transversa. Cuando nos encontramos con un feto en una situación no longitudinal
(transversa u oblicua), es indicación de cesárea.

- Cara mentoposterior. Dentro de las presentaciones cefálicas, todas pueden ser un parto normal
menos ésta y la variedad de frente, en las que tendremos que hacer una cesárea.

- Gemelar. Para admitir un parto vaginal en el embarazo gemelar debe ser biamniótica y estar el
primero en cefálica. Si no se cumple alguno de estos criterios como es el caso de la respuesta,
deberemos hacer una cesárea.

- Meningocele. Cuando el feto presenta alguna malformación que pueda requerir una actuación
inmediata por parte de los pediatras (por ejemplo hernia diafragmática) o en las que el parto
vaginal pueda ser lesivo, como es el caso de la respuesta, indicaremos una cesárea.

- Parto en podálica. Para admitir un parto vaginal se deben reunir unos criterios. Son útiles en
casos de fetos vivos ya que lo que buscamos disminuir la morbilidad fetal, pero en el caso que nos
presentan con feto muerto preferiremos un parto vaginal que para la madre tiene menor morbilidad,
ya que no hay interés fetal.(R4)

201. La hipótesis del "doble vínculo" como forma de comunicación entre madre y niño ha
sido estudiada como factor etiológico de la:

1. 1. Depresión reactiva.
2. 2. Depresión endógena.
3. 3. Esquizofrenia.
4. 4. Neurosis obsesiva.
Gráfico de respuestas
Comentario

Pregunta muy difícil sobre la etiopatogenia de la esquizofrenia. Por supuesto, no debe preocuparle
si la ha fallado, porque nunca se ha podido demostrar científicamente su veracidad.

Existen dos teorías psicológicas que trataron de explicar el origen de la esquizofrenia. Una fue la
hipótesis de la “madre esquizofrenógena”. Según ésta, un sistema patológico de crianza provoca
cambios psicológicos de extrema gravedad en sus hijos. Hoy en día, se defiende que muchas de
esas madres padecían en realidad formas atenuadas de la enfermedad esquizofrénica. Por otra
parte, los estudios de gemelos adoptados por familias diferentes no demostraron un aumento de la
incidencia cuando un gemelo “sano” era adoptado por una familia “enferma”. La otra teoría es la
hipótesis del “doble vínculo”, que de nuevo sugería el carácter patógeno de un estilo anormal de
educación, basado en órdenes contradictorias que nunca se podrían satisfacer enteramente.

Lo que sí parece claro es la influencia de los factores psicológicos en la evolución de la


enfermedad. Por un lado los pacientes son más sensibles al estrés psicosocial, que favorece sus
recaídas. De ahí que con frecuencia se recurra a distintas medidas para alejar temporal o

!
!
!
!
definitivamente al paciente de un entorno altamente estresante (centros de día, talleres protegidos,
pisos o pensiones protegidas, residencias…).(R3)

202. Masculino de 25 años con hematuria recurrente y una creatinina de 2 mg/dl. Su padre
y su tío han sido transplantados por insuficiencia renal terminal. Nuestro paciente está
siendo visto en el servicio de ORL por pérdida progresiva de la audición. ¿Cuál de las
siguientes afirmaciones son FALSAS respecto a la nefropatía que padece nuestro
paciente?:

1. 1. La etiología puede deberse a alteraciones de algún componente de la membrana basal.


2. 2. Al microscopio electrónico hay un adelgazamiento y delaminación de la membrana basal.
3. 3. El tratamiento de elección son dosis elevadas de prednisona más ciclofosfamida.
4. 4. No se suele reaparecer tras el transplante.
Gráfico de respuestas
Comentario

Esta pregunta es de dificultad media. La nefropatía ha sospechar en este caso es un síndrome de


Alport: un defecto hereditario del gen que codifica el colágeno IV de las membranas basales del
cristalino, la cóclea y el riñón.

La imagen al microscopio electrónico muestra una membrana basal glomerular deslaminada "en
capas de hojaldre". Lleva a la insuficiencia renal terminal en la segunda o tercera década de la vida
por lo que la evolución hacia la diálisis o el trasplante es la regla sin existir otro tratamiento
alternativo. No suele reaparecer tras trasplante pero puede desarrollarse una enfermedad de
Goodpasture.(R3)

203. ¿Cuál de las siguientes infecciones requiere tratamiento sistémico?:

1. 1. Candidiasis vaginal.
2. 2. Tricomoniasis vaginal.
3. 3. Condilomatosis vulvar.
4. 4. Cervicitis por flora mixta.
Gráfico de respuestas
Comentario

La tricomona vaginalis es un protozoo que se transmite durante las relaciones sexuales. El


tratamiento gold standard es con Metronidazonl por via oral y hay que tratar a la paciente y a su
pareja.(R2)

204. En relación con el tratamiento hormonal sustitutivo combinado con estrógenos y


progestágenos de forma prolongada, las siguientes afirmaciones son ciertas, EXCEPTO
una. Señale ésta:

1. 1. Disminuye la descalcificación propia de la menopausia.


2. 2. Mejora el trofismo vaginal.
3. 3. Aumenta el riesgo de cáncer de endometrio.
4. 4. Disminuye los síntomas del climaterio.
Gráfico de respuestas
Comentario

!
!
!
!
Pregunta sencilla sobre tratamiento hormonal, sus efectos y sus complicaciones.

La indicación principal para el tratamiento hormonal sustitutivo con estrógenos y progesterona es


el síndrome climatérico, dado que éste está ocasionado por un déficit de estrógenos. Asimismo, la
suplementación con estrógenos inhibe la reabsorción ósea, por lo que es útil en la osteoporosis.

Una de las contraindicaciones para el tratamiento hormonal sustitutivo es el antecedente de


tumores estrógeno-dependientes. Sin embargo, los gestágenos se usan para compensar el efecto
proliferativo de los estrógenos en el ámbito del endometrio y así evitar el cáncer de
endometrio.(R3)

205. Acude a Urgencias un paciente de 75 años de edad diagnosticado de EPOC tipo


bronquitis crónica. Refiere cuadro de 48 horas de evolución de aumento de su disnea
hasta hacerse de reposo, aumento de la tos y aumento de su expectoración, que se ha
vuelto purulenta. No refiere fiebre. En la auscultación pulmonar presenta roncus y
sibilancias diseminadas. La Rx de tórax es normal. En la gasometría arterial basal se
observa pH 7.38, PaCO2 55 mmHg, PaO2 51 mmHg, HCO3- 35. Indique cuál es la pauta
terapéutica inicial más adecuada:

Oxigenoterapia al 28%, corticoides inhalados, ipratropio, salbutamol y


1. 1.
amoxicilina/clavulánico.
Oxigenoterapia al 50%, corticoides sistémicos, ipratropio, salbutamol y amoxicilina-
2. 2.
clavulánico.
3. 3. Oxigenoterapia al 28%, salbutamol, corticoides sistémicos e ipratropio.
Oxigenoterapia al 28%, corticoides sistémicos, amoxicilina-clavulánico, salbutamol e
4. 4.
ipratropio.
Gráfico de respuestas
Comentario

El tratamiento básico de la agudización de EPOC es el uso de broncodilatadores de acción corta


(beta2-agonistas y anticolinérgicos) junto con corticoides sistémicos. Además será necesaria la
oxigenoterapia siempre que exista insuficiencia respiratoria (inicialmente con una FiO2 baja en
caso de que el paciente presente hipercapnia crónica). La antibioticoterapia empírica es necesaria
siempre que se cumplan dos de los siguientes criterios: 1) aumento de la disnea, 2) aumento en el
volumen de la expectoración, 3) purulencia de la expectoración.(R4)

206. En relación a la evaluación del factor tubárico, marque lo CORRECTO:

1. 1. Se puede hacer por histerosonografía


2. 2. Sólo se debe hacer por laparoscopía
3. 3. La histerosalpingografía debe de hacerse entre el 7-8 día post-menstruación
4. 4. La histerosalpingrafía puede hacerse en cualquier día del ciclo luego de la menstruación
Gráfico de respuestas
Comentario

La mejor forma de evaluar el factor tubárico es la histerosalpingografía. Por tanto, las respuestas 1
y 2 son incorrectas.

La histerosalpingografía se debe realizar entre el día 7-8 postmenstruación: la respuesta 3 es


correcta. Así, al hacerlo antes de la ovulación, nos aseguramos de que la mujer no está
embarazada. Si se hiciera más tarde, correríamos el riesgo de que estuviera embarazada, y

!
!
!
!
además el grueso endometrio de la fase secretora puede ocluir la entrada a las trompas y dar la
falsa impresión de que hay una obstrucción tubárica causando la infertilidad.(R3)

207. Un paciente de 29 años de edad, sin antecedentes personales de interés, ingresa en


el servicio de Urgencias por un cuadro de cefalea, acompañado de náuseas y vómitos sin
causa aparente, con adormecimiento progresivo del brazo derecho, hasta la paresia que
el paciente presenta al llegar a Urgencias, no habiendo transcurrido en todo el proceso
más de treinta minutos. Con el paso de los minutos el paciente está estuporoso. Al
realizar un TAC craneal se aprecia la existencia de una hemorragia intraparenquimatosa.
Pensando en una malformación vascular cerebral como primera causa del cuadro clínico,
¿cuál de las siguientes malformaciones vasculares cerebrales con producción de
sintomatología es la más frecuente?

1. 1. Angioma venoso.
2. 2. Malformación arteriovenosa.
3. 3. Telangiectasias capilares.
4. 4. Fístulas arteriovenosas durales.
Gráfico de respuestas
Comentario

La RESPUESTA CORRECTA a esta pregunta es la NÚMERO 2 (MAV).

El angioma venoso es la malformación vascular más frecuente, pero suele ser asintomática. En
cambio la MAV es la anomalía vascular sintomática más frecuente, presentando focalidad
neurológica (como en este caso) o crisis epilépticas.(R2)

208. Una mujer de 35 años presenta una historia de un mes de prurito y astenia. En los
últimos días se ha hecho evidente un tinte ictérico. Se constatan xantelasmas
palpebrales. Refiere asimismo sequedad ocular desde hace algunos meses. En el
diagnóstico de su probable patología, NO participa uno de los siguientes criterios:

1. 1. AMA (+).
2. 2. Biopsia hepática compatible.
3. 3. Clínica de colestasis.
4. 4. Elevación de la bilirrubina.
Gráfico de respuestas
Comentario

Lo más importante, en esta pregunta, es que sospeche el diagnóstico correcto: la cirrosis biliar
primaria. La clave para acertarla está en saber que, en la mayor parte de los casos, se diagnostica
en fase asintomática, donde casi nunca existirá elevación de la bilirrubina, sino únicamente
enzimas de colestasis y AMAs positivos (Ac antimitocondriales). La bilirrubina comienza a elevarse
más tardíamente, y tiene valor pronóstico, más que diagnóstico. Recuerde que está demostrado un
paralelismo entre la evolución de la enfermedad y la elevación de la bilirrubina sérica. De hecho, es
el mejor parámetro de laboratorio para el control evolutivo de esta patología.(R4)

209. Ante una radiografía en la que observamos un arrancamiento de espina ilíaca


anteroinferior, sospecharemos arrancamiento del músculo:

1. 1. Isquiotibiales.
2. 2. Recto anterior.

!
!
!
!
3. 3. Sartorio.
4. 4. Psoas ilíaco.
Gráfico de respuestas
Comentario
Para contestar esta pregunta es fundamental el conocimiento detallado de la anatomía de la
cadera. Las avulsiones de los puntos de inserción es una lesión frecuente en pacientes jóvenes,
generalmente secundario a trauma o esfuerzos deportivos; su tratamiento suele ser conservador
excepto en los casos de mucho desplazamiento y en las avulsiones de la tuberosidad posterior del
calcáneo. En este caso la respuesta correcta es la 3, el recto anterior del muslo tiene su origen en
la espina iliaca anteroinferior para insertarse a nivel del fémur distal en su cara anterior. El gluteo
medio tiene su origen a nivel de la cresta iliaca y su inserción en la región del trocanter mayor,
pudiendo también provocar avulsiones a ese nivel. Sartorio: origen, espina iliaca anterosuperior,
inserción a nivel de la región proximal y medial de la tibia formando la parte mas anterior de la pata
de ganso Psoas iliaco: origen a nivel de las apofisi transversas de vértebras lumbares y cresta
iliaca, inserción a nivel de trocanter menor(R2)

210. Un recién nacido de 39 semanas y 3 días de edad gestacional presenta a las 3 horas
de vida una hipertonía marcada con clonus y alteración del nivel de conciencia. Durante
el parto se observó un líquido amniótico meconial. Al nacimiento el niño presentó un
Apgar al primer minuto de 4 y a los 5 minutos de 7, precisó reanimación con presión
positiva intermitente durante 2 minutos y masaje cardíaco durante 30 segundos. El pH de
cordón fue de 7.02 y en la primera hora de vida de 7.15. ¿Cuál de las siguientes es FALSA?

1. 1. Se trata de una encefalopatía hipóxico-isquémica.


2. 2. Se debe a una lesión focal de la corteza temporal.
3. 3. Su principal secuela es la parálisis cerebral.
Son factores de riesgo el desprendimiento de placenta, la procidencia de cordón, la
4. 4.
insuficiencia placentaria o la hipotensión materna grave.
Gráfico de respuestas
Comentario

La encefalopatía hipóxico-isquémica es la afectación difusa de la corteza cerebral por hipoxemia


y/o disminución del flujo sanguíneo cerebral en el recién nacido a término. Se manifiesta por
alteración del nivel de conciencia, en el tono muscular, movimiento anómalos y convulsiones. Su
principal secuela es la parálisis cerebral. En los últimos años se ha visto que la inducción de
hipotermia en el niño puede actuar como neuroprotector.(R2)

211. La diálisis peritoneal, que ha mejorado enormemente la calidad de vida de los


enfermos con insuficiencia renal, se basa en el transporte bidireccional que puede
llevarse a cabo a través de la membrana peritoneal. Además, permite aclarar multitud de
drogas y medicamentos; este transporte, sin embargo, no puede realizarse para cualquier
sustancia. De entre las siguientes, señale cuál NO puede ser eliminada mediante diálisis
peritoneal:

1. 1. Antihistamínicos.
2. 2. Aspirina.
3. 3. Penicilina.
4. 4. Fenobarbital
Gráfico de respuestas
Comentario

!
!
!
!
Pregunta muy dificil, que si no sabe la respuesta no se estrese.

Son muchas las sustancias que son dializables mediante el sistema de diálisis peritoneal y no
resulta muy rentable estudiar este tipo de listas. De todas las opciones la única que no puede ser
eliminada por este dispositivo son los antihistamínicos.(R1)

212. En la enfermedad ósea de Paget se pueden observar todas las complicaciones que
se citan EXCEPTO una:

1. 1. Fracturas patológicas.
2. 2. Policitemia.
3. 3. Impresión basilar.
4. 4. Degeneración maligna.
Gráfico de respuestas
Comentario

Recuerda las complicaciones típicas de la enfermedad de Paget del hueso:

- Dolor óseo y fracturas patológicas

- Cefaleas y pérdida auditiva por afectación del cráneo

- Compresión nerviosa por afectación del cráneo y la columna

- Somnolencia ó parálisis por síndrome del robo del cráneo o las vértebras

- Enfermedad vascular

- Degeneración maligna

- Impresión basilar

La policitemia no se asocia a la enfermedad de Paget.(R2)

213. ¿Cuál es el tratamiento empírico de la sepsis neonatal precoz sin meningitis?:

1. 1. Ampicilina más gentamicina.


2. 2. Vancomicina más gentamicina.
3. 3. Ampicilina más fluconazol.
4. 4. Vancomicina más cefotaxima.
Gráfico de respuestas
Comentario
Los agentes etiológicos más frecuentemente implicados en la génesis de la sepsis neonatal precoz
son dos: Streptococcus agalactiae y Escherichia coli. Además, has de tener en cuenta la
posibilidad etiológica de otros gérmenes, como Listeria sp y enterococo. Por tal motivo, la totalidad
de estos agentes patógenos queda cubierto con la combinación de ampicilina (que cubre muy bien
la listeria) y gentamicina (que cubre muy bien los gram negativos). Si hubiera meningitis, tendrías
que sustituir el aminoglucósido (que no cruza la BHE) por una cefalosporina de tercera
generación.(R1)

!
!
!
!
214. Secundípara en la 38 semanas de gestación que comienza con dinámica de parto.
Entre sus antecedentes obstétricos destaca la presencia de cesárea anterior por
sufrimiento fetal agudo. Se hace un registro cardiotocográfico que demuestra una
frecuencia cardíaca de 105 lpm, con baja variabilidad y deceleraciones tardías en el 10%
de las contracciones. Se hace una microtoma de sangre fetal con resultado de 7.25. ¿Cuál
de las siguientes actuaciones está contraindicada?:

1. 1. Toma de pH materno para descartar acidosis materna.


2. 2. Aumentar el goteo de oxitocina para abreviar el expulsivo.
3. 3. Repetir microtoma en 15-20 minutos.
4. 4. Ayudar instrumentalmente en el expulsivo para acortarlo.
Gráfico de respuestas
Comentario

Caso clínico fácil sobre la microtoma fetal. Se nos presenta a una gestante a término que está de
parto con cesárea anterior. Ante la presencia de un RCTG con sospecha de sufrimiento fetal
(bradicardia fetal, deceleraciones tardías y baja variabilidad) se procede a la microtoma que nos da
un valor prepatológico (entre 7.20 y 7.25). Para descartar que una de las causas de acidosis fetal
no sea una acidosis materna, debemos estudiar el pH materno. Como el resultado del pH es
prepatológico, siempre debemos obtener una nueva toma para observar una recuperación o un
empeoramiento fetal. Colocar a la gestante en decúbito lateral, preferiblemente izquierdo siempre
es aconsejable ya que así evitamos la compresión de la cava. Otra actitud correcta es abreviar el
periodo expulsivo cuando se alcance un tercer plano con un fórceps, para así evitar mayor
sufrimiento fetal. La opción que debemos elegir es la 2, ya que si aumentamos la dosis de
oxitocina, aumentarán las contracciones y por consiguiente el flujo uteroplacentario disminuirá y
aumentará la acidosis fetal.(R2)

215. Un niño de 8 años presenta un cuadro de cefalea, fiebre, exantema maculopapular


generalizado y dolor centrotorácico punzante. En las exámenes complementarios se
aprecia en el LCR una pleocitosis linfocitaria con glucorraquia y proteinorraquia
normales, y en el electrocardiograma se observa una elevación generalizada del
segmento ST con concavidad hacia arriba. ¿Qué diagnóstico es el más probable ante
dicha constelación de hallazgos?

1. 1. Infección por Haemophilus influenzae.


2. 2. Infección por Mycoplasma pneumoniae.
3. 3. Infección por Coxsackie B.
4. 4. Infección por el virus de la parotiditis.
Gráfico de respuestas
Comentario

Una elevación generalizada del ST hacia arriba es sugestivo de una pericarditis, sobre todo si
además es cóncavo. Por otra parte, sabe que la causa más frecuente de pericarditis aguda es
vírica (Coxsackie B), por lo que debe elegir la respuesta 3.

En cuanto al resto del cuadro clínico, no era una pregunta tan difícil. Prácticamente todos los virus
pueden justificar exantemas cutáneos, y los enterovirus (entre ellos el coxsackie) son también una
causa frecuente de meningitis vírica. El mismo virus, por tanto, puede justificar todo lo que nos
cuentan de este niño…(R3)

216. En relación con el examen neurológico, marque la CORRECTA:

!
!
!
!
1. 1. El reflejo de succión dura hasta los 2 años.
2. 2. La parálisis de Klumpke siempre se asocia con el síndrome de Horner.
3. 3. El síndrome de Horner: enoftalmos, miosis y disminución de apertura palpebral.
4. 4. El tono muscular siempre es normal en los prematuros estables.
Gráfico de respuestas
Comentario

Nunca debe de olvidar la clínica de un síndrome de Horner: miosis, enoftalmos, anhidrosis, ptosis
palpebral. La respuesta correcta es la 3.(R3)

217. Señale la afirmación CORRECTA en la técnica de biopsia del ganglio centinela en


cáncer de mama:

1. 1. No se debe realizar en pacientes con ganglios palpables.


2. 2. Se realiza mediante marcado con arpón de un ganglio axilar.
3. 3. Sirve para detectar los ganglios axilares sanos.
4. 4. Se realiza simultáneamente en ambas axilas.
Gráfico de respuestas
Comentario

La técnica del ganglio centinela sirve para localizar el ganglio (o grupo de ganglios) que recibe en
primer lugar el drenaje de un tumor, por lo que si los ganglios son palpables, esta técnica no sería
necesaria y, de hecho, estaría contraindicada. Por tanto, la respuesta 1 es correcta.

La respuesta 2 también es falsa ya que se realiza mediante la inyección de un radioisótopo y


posteriormente se localiza dónde ha migrado éste mediante una gammagrafía.

La 3 también es falsa porque detecta el primer grupo ganglionar de drenaje del tumor, ya sean
ganglios sanos o con metástasis ganglionares, y para diferenciar entre ambos es necesario
biopsiar el ganglio.

La 4 también es falsa ya que se realiza en la axila del mismo lado que la mama afectada.(R1)

218. Which of the following options would not be expected to be seen in a patient with
acromegaly?

1. 1. Sleep apnea.
2. 2. Rheumatoid arthritis.
3. 3. Acanthosis nigricans.
4. 4. Carpal Tunnel Syndrome.
Gráfico de respuestas
Comentario

Son complicaciones típicas de la acromegalia el SAHOS, el síndrome del túnel carpiano, la


hipertensión y otras enfermedades cardiovasculares, la resistencia insulínica (acompañada de
acantosis nigricans), el cáncer de colon y la hipercalcemia. Aunque pueden presentar artralgias, la
AR no es una complicación típica de esta enfermedad.(R2)

219. Paciente de 30 años que trabaja en una fábrica de corcho. Este año ha tenido dos
cuadros de fiebre, tos y disnea, que su médico ha tratado como neumonías. Actualmente

!
!
!
!
refiere tos seca y disnea de esfuerzo. En la exploración funcional respiratoria muestra:
CVF 58% del predicho, VEF1 60%, índice de Tiffeneau 80%, CPT 70%, DLCO 60%. La Rx
de tórax muestra un infiltrado reticulonodular difuso. El diagnóstico más probable será:

1. 1. Proteinosis alveolar.
2. 2. Asbestosis.
3. 3. Bisinosis.
4. 4. Alveolitis alérgica extrínseca.
Gráfico de respuestas
Comentario

La exposición al corcho debe sugerirnos una neumonitis por hipersensibilidad, o alveolitis alérgica
extrínseca. La suberosis es una forma de neumonitis por hipersensibilidad producida por la
exposición a los mohos de la corteza de corcho. Clínicamente, no difiere de otras neumonitis, y
desde el punto de vista funcional se puede ver un patrón restrictivo con disminución de los
volúmenes pulmonares y alteración de la capacidad de difusión. En la forma crónica, la radiografiía
de tórax puede mostrar un infiltrado reticulonodular difuso. En cuanto al tratamiento, es
fundamental evitar la exposición al antígeno. En las formas crónicas, suelen ser necesarios los
glucocorticoides.(R4)

220. A patient with a previous history of pancreatic carcinoma is hospitalized in order to


undergo Whipple surgery. The patient is hospitalized and presents fever 7 days after the
aforementioned surgical procedure. The patient is already eating but he still has a central
iv line in order to receive supplemental parenteral nutrition due to severe
hypoalbuminemia. Physical examination shows no abnormal findings and chest X-ray
film shows mild blunting of the left costophrenic angle. Laboratory studies show
leukocytosis with left shifting. 2 out of 3 blood cultures yielded positive results. Which of
the following is the most likely underlying cause of the patient's current status?

1. 1. Urinary infection.
2. 2. Wound infection.
3. 3. Intravascular catheter associated sepsis.
4. 4. Postoperative pneumonia.
Gráfico de respuestas
Comentario

Resulta más fácil resolver esta pregunta mediante el descarte de opciones.

R1. No nos hablan de sonda urinaria, ni tiene síntomas sugestivos de IVU.

R2. Las heridas infectadas suelen verse en la exploración, aparte de que producen dolor local,
etc…

R4. No existen síntomas respiratorios ni radiológicos sugestivos de neumonía. El pequeño


pinzamiento es un hallazgo absolutamente inespecífico que, además, podría deberse a la propia
cirugía y al encamamiento.

R3. La sepsis por catéter (que además insisten en que tiene uno puesto) sería, por tanto, la
respuesta más razonable. Hemocultivos positivos + vía central sin otra focalidad presente.(R3)

!
!
!
!
221. En la enfermedad de membrana hialina:

1. 1. Las manifestaciones comienzan a partir de la primera semana de vida.


2. 2. La puntuación del test de Silverman suele ser baja.
3. 3. Para su tratamiento suele ser suficiente con la administración de oxígeno en incubadora.
4. 4. Una de las complicaciones es la displasia broncopulmonar.
Gráfico de respuestas
Comentario

La opción 1 es falsa: la enfermedad debuta el primer día de vida. En cuanto a la opción 2, también
es falsa, dado que la hialina genera dificultad respiratoria importante, con gran componente de
retracción. La opción 3 es falsa: genera hipoxemia importante, que a veces sólo se controla con
intubación. La opción 4 es correcta: se puede complicar con DBP (por inmadurez, toxicidad por
oxígeno y barotrauma).(R4)

222. A 5-month-old infant is being evaluated at a routine visit in his local clinic for
vaccination. He has bilateral cataracts and congenital deafness. Cardiac auscultation
shows a continuous machine-like heart murmur. What is the most likely cause of these
patient's symptoms?

1. 1. Congenital Rubella.
2. 2. Congenital CMV infection.
3. 3. Congenital toxoplasmosis.
4. 4. Edward's syndrome.
Gráfico de respuestas
Comentario

Congenital rubella is a disease that has decreased in frequency thanks to vaccination programs in
developed countries. The classic manifestations are also known as Gregg triad: deafness, visual
abnormalities (most commonly congenital cataracts) and congenital heart defects. In this case, the
continuous murmur is highly suggestive of patent ductus arteriosus. Congenital CMV infection do
not induce heart abnormalities in the fetus.(R1)

223. Ante una paciente de 42 años que consulta por alopecia androide, hirsutismo de
aparición brusca, hipertrofia de clítoris, amenorrea, sensación de pesadez en hipogastrio
y urgencia miccional, debemos descartar de forma inmediata:

1. 1. Hirsutismo de origen periférico.


2. 2. Tumor ovárico funcionante.
3. 3. Tumor hiperfuncionante de hipófisis.
4. 4. Pseudohermafroditismo masculino.
Gráfico de respuestas
Comentario

El cáncer de ovario es un tema importante en el examen. Un aspecto bastante preguntado son los
distintos tipos histológicos.

La presencia de alopecia androide, hipertrofia de clítoris, amenorrea e hirsutismo de aparición


brusca son signos de virilización; y la sensación de pesadez en hipogastrio junto con urgencia
miccional nos orienta hacia una masa abdominal.

!
!
!
!
Por lo tanto, este caso clínico es muy sugestivo de tumor ovárico funcionante, y por eso es lo
primero que debemos descartar.

Lo más probable es que estemos ante un androblastoma, un tumor ovárico que reproduce
elementos testiculares. El 50% produce andrógenos (testosterona), responsables de la virilización.
Suelen ser benignos. Constituyen la primera causa de virilización de origen ovárico.(R2)

224. Señala la asociación INCORRECTA:

1. 1. Carcinoma supraglótico – Sensación de cuerpo extraño y disfagia


2. 2. Carcinoma dlótico – Disfonía precoz
3. 3. Carcinoma subglótico – Disfagia
4. 4. Carcinoma de laringe – Virus del papiloma humano
Gráfico de respuestas
Comentario

Los síntomas laríngeos son localizadores, presentando típicamente las lesiones subglóticas disnea
y estridor, y no disfagia, al no encontrarse o interferir en la vía digestiva. Todos los tumores de
cabeza y cuello (con excepción de los nasosinusales y de cavum) están relacionados con el
consumo de tabaco y alcohol. Otros factores de riesgo son los productos químicos o textiles.(R3)

225. A 3-year-old child who was recently diagnosed with whooping cough and is currently
recovering from the infection, is brought to the emergency department with
subcutaneous emphysema on the anterior chest wall and neck. What should be done
next?

1. 1. Place a thoracic tube.


2. 2. Perform a chest X-ray.
3. 3. Start amoxicillin.
4. 4. CT scan of the brain.
Gráfico de respuestas
Comentario

Perform a chest X-ray. Subcutaneous emphysema is the abnormal presence of air in the
subcutaneous tissue with consequent loosening of soft tissue. There are several causes that may
explain this condition. It can be spontaneous, or it can have a traumatic, iatrogenic or sometimes,
an unknown origin. A chest tube is only indicated if the subcutaneous emphysema is caused by a
pneumothorax. To make a proper diagnosis, a chest radiography is required.(R2)

226. Las alteraciones metabólicas tras una cirugía o un trauma incluyen todas EXCEPTO:

1. 1. Retención de agua corporal por efecto de la ADH.


2. 2. Retención renal de sodio.
3. 3. Glucogénesis.
4. 4. Reabsorción renal de bicarbonato.
Gráfico de respuestas
Comentario

La respuesta erronea es la glucogénesis, no se forma de novo, recuerda que el cuerpo pasa por un
periodo de estres, ya que en la reacción metabólica al trauma lo que se presenta es la
glucogenólisis.(R3)

!
!
!
!
227. Niño de 6 años que consulta por presentar en los últimos meses una lesión
blanquecina de 1 cm de ancho por 7 cm de longitud que afecta a la mitad derecha de la
frente, introduciendo en cuero cabelludo y provocando alopecia. Va creciendo
lentamente y es asintomática. A la exploración la lesión está dura, es móvil sobre el hueso
subyacente y está bien delimitada. ¿Cúal es el diagnóstico?:

1. 1. Liquen escleroso y atrófico.


2. 2. Esclerodermia lineal.
3. 3. Cicatriz queloidea espontánea.
4. 4. Vitíligo segmentario.
Gráfico de respuestas
Comentario
La esclerodermia es una enfermedad crónica de la piel, se caracteriza por la aparición de áreas
circunscritas o difusas, de color marfil, lisas y consistencia dura. Existen formas localizadas y
formas sistémicas. Los tipos cutáneos son la morfea y la esclerodermia lineal. Ésta última puede
extenderse a lo largo de un brazo o una pierna y lo más frecuente es que aparezca en la primera
década de la vida. Puede afectar la región frontal del cuero cabelludo extendiéndose hacia la frente
(en coup de sabre). Cuando se afecta una extremidad inferior puede acompañarse de espina
bífida, hemiatrofia, contracturas de flexión y desarrollo anormal del miembro. En el estudio
radiológico es posible encontrar melorreostosis (hiperóstosis de la cortical). El mejor pronóstico es
para las formas infantiles que afectan las extremidades ya que es posible la mejoría espontánea.
En la histología se observa un aumento de colágeno en la dermis y en las formas más avanzadas,
ausencia de las unidades pilosebáceas. Esto lleva a la pérdida de pelo en las lesiones y si éstas se
localizan en el cuero cabelludo a alopecia de la zona afectada.(R2)

228. Recién nacido de 37 semanas de edad gestacional y parto eutócico, en el que se


detecta al 2º día de vida una desviación de la comisura bucal izquierda hacia abajo
durante el llanto, con surcos nasogenianos simétricos. Al cesar el llanto, la facies es
simétrica. La abuela asegura que su hija presentaba una anomalía similar en la infancia.
¿Qué diagnóstico le sugiere?

1. 1. Síndrome de Moëbius.
2. 2. Parálisis facial derecha con afectación exclusiva de la rama mandibular.
3. 3. Hipoplasia del músculo depresor anguli oris derecho.
4. 4. Agenesia del núcleo derecho del VII par craneal.
Gráfico de respuestas
Comentario

Una pregunta sobre una patología poco importante. En la parálisis facial periférica se prodcuce una
desviación de la boca hacia el lado sano, no hacia abajo. Además, desaparece el surco
nasogeniano del lado afecto. En la entidad referida en esta pregunta (hipoplasia del depresor del
ángulo de la boca), situación bastante menos frecuente que la parálisis facial, simplemente se
registra una ausencia de la capacidad para la infradesviación de la comisura de la boca.(R3)

229. Lactante de 9 meses de edad, con tiempo de evolución de 16 días. Presenta


evacuaciones diarreicas sin moco ni sangre en pequeñas cantidades en número de 3 a 4
por día y sed normal. Hace dos días se agrega fiebre no cuantificada. Al exámen físico:
despierto, activo, mucosa oral húmeda y signo del pliegue negativo. ¿Qué tipo de diarrea
es la que presenta?

!
!
!
!
1. 1. Disentérica sin deshidratación.
2. 2. Aguda acuosa sin deshidratación.
3. 3. Crónica con deshidratación.
4. 4. Persistente sin deshidratación.
Gráfico de respuestas
Comentario

La diarrea se considera

- aguda < 2 semana.

- persistente de 2 a 4 semanas.

- crónica > 4 semanas.

la mucosa oral pastosa y el signo del pliegue son signos de deshdratació. Respuesta 4
correcta.(R4)

230. Paciente de 56 años, prostituta, ADVP y fumadora de 1/2 paquete al día, que acude
con disnea, embotamiento facial y hemorragias conjuntivales, que se ha ido instaurando
lentamente durante los últimos meses, hasta hacerse insoportable. Es VIH positiva y
reconoce haber padecido diversas ETS. La radiografía torácica es normal. Usted
sospecha:

1. 1. Mediastinitis crónica sifilítica.


2. 2. Adenocarcinoma pulmonar.
3. 3. Mediastinitis necrotizante aguda.
4. 4. Mediastinitis secundaria a síndrome de Boerhaave.
Gráfico de respuestas
Comentario

Un caso clínico relativamente complejo. Se trata de una paciente inmunodeprimida y con


antecedentes de ETS, que clínicamente presenta un síndrome de vena cava superior. La dificultad
del caso clínico está en reconocerlo, lo cual no es fácil. Sin embargo, teniendo en cuenta la clínica
y los antecedentes, puede intuir la respuesta correcta.

Al no haber tenido vómitos, se podría descartar el síndrome de Boerhaave. La mediastinitis


necrotizante aguda tampoco puede ser válida, por la lenta evolución del cuadro. La placa de tórax
normal va en contra de la existencia de masas pulmonares, y por lo tanto de adenocarcinoma de
pulmón. La tuberculosis es causa de mediastinitis crónica granulomatosa, pero habitualmente
cursa de forma asintomática. En cambio, la sífilis puede producir una fibrosis mediastínica que se
manifiesta como un síndrome de cava superior de evolución crónica. Dados los antecedentes de la
paciente y la clínica descrita, la principal sospecha debería ser la respuesta 1.(R1)

231. ¿Cuál es el síntoma o signo más frecuente del cáncer de mama?

1. 1. Telorragia.
2. 2. Mastodinia.
3. 3. Nódulo mamario.
4. 4. Retracción del pezón.
Gráfico de respuestas

!
!
!
!
Comentario

Pregunta muy sencilla sobre la clínica del carcinoma de mama. La forma de inicio más frecuente es
asintomática (diagnosticada por screening mastográfico), pero en esta pregunta se nos pide el
síntoma o signo más frecuente. Así, la opción 3 es la correcta, ya que la presencia de un nódulo o
tumoración es el signo más frecuente (80%). Recuerde también que el cáncer de mama suele ser
indoloro (opción 2 incorrecta). El resto de opciones (1 y 4) nos presentan tumores clícamente más
agresivos o en estadios más avanzados.(R3)

232. En un lactante se detecta a los 2 meses una masa cervical de aspecto quístico que
se comprime manualmente, no le duele y no está eritematosa. ¿Cuál es el diagnóstico
más probable?:

1. 1. Adenopatía cervical.
2. 2. Quiste sebáceo.
3. 3. Linfangioma.
4. 4. Quiste enterógeno.
Gráfico de respuestas
Comentario

Los linfangiomas son masas esferoidales, de aspecto quístico, blandos a la presión. Los quistes
son de consistencia elástica, y las adenopatías suelen estar induradas; ambas no desaparecen con
la presión.(R3)

233. Mujer de 25 años de edad, sin antecedentes personales de interés, pero con
antecedente familiar de cáncer de mama en madre y hermana, que requirieron
mastectomía, quimioterapia y tratamiento con hormonoterapia de forma coadyuvante. Se
le explica a la paciente que, dada la posible incidencia familiar del cáncer de mama, la
asociación entre los antecedentes familiares del cáncer de mama y el riesgo de
padecerlo, es necesario realizar un estudio genético que oriente la decisión terapéutica
en el caso de esta paciente asintomática. Uno de los siguientes genes implica un riesgo
de padecer cáncer de mama superior al 50% en las mujeres portadoras. Señale cuál:

1. 1. c-erb B2.
2. 2. BRCA 1.
3. 3. RET.
4. 4. K-RAS.
Gráfico de respuestas
Comentario

El BrCa1 y el BrCa2 son los genes implicados en más de la mitad de los cánceres de mama
hereditarios. No olvide el papel de Her 2 neu (c- erb B2) en el cáncer de mama: los tumores
metastásicos que expresan este gen pueden ser tratados con Trastuzumab, aunque en principio
son de peor pronóstico que los que no lo expresan.(R2)

234. En relación a la reparación de las hernias inguinales señale la afirmación


INCORRECTA:

1. 1. La técnica de Bassini aproxima la zona conjunta con el ligamento inguinal.


2. 2. La técnica de Bassini es adecuada para las hernias femorales.

!
!
!
!
El uso de material protésico promueve la formación de mayor cicatrización para reforzar
3. 3.
el cierre del defecto de pared.
Una indicación precisa para la hernioplastía laparoscópica es la recidiva repetida del
4. 4.
abordaje anterior.
Gráfico de respuestas
Comentario

Pregunta de alta dificultad sobre el tratamiento quirúrgico de las hernias inguinales que
eventualmente son preguntadas en el ENARM, ya que precisa saber los procedimientos
quirúrgicos que se deben aplicar en las diversas situaciones.

La técnica de Bassini es una de las variaciones de la herniorrafia o reparación anatómica de la


hernia, que consiste en la corrección de la hernia mediante sutura usando los propios tejidos del
paciente para la reparación.

Esto hay que diferenciarlo de la hernioplastia, que consiste en la reparación de la hernia con una
protesis de material sintético. Actualmente es el procedimiento que se realiza con mayor frecuencia
dados los excelentes resultados obtenidos (técnicas de Lichetenstein y de Rutkow).(R2)

235. ¿Cuál de las siguientes hormonas tiene mayor similitud con la HCG?

1. 1. GnRH.
2. 2. LH.
3. 3. TSH.
4. 4. PRL.
Gráfico de respuestas
Comentario

Pregunta difícil sobre las hormonas femeninas. Podemos tener dudas entre la FSH, LH y TSH, que
comparten la misma subunidad alfa. La diferencia estará en la subunidad beta.

La única que presenta cierta similitud en cuanto a la subunidad beta es la LH (opción 2 correcta).
Aunque no lo supiera, se puede sacar esta pregunta con las similitudes funcionales de la LH y
HCG. Recuerde que la LH es la hormona de la fase lútea, y la HCG es luteotropa, ya que es la
encargada de mantener el cuerpo lúteo durante el primer trimestre de la gestación. A la inversa,
recuerde que en el ciclo ovárico justo antes de la ovulación ocurre un pico de LH. En los ciclos de
estimulación ovárica utilizados en la reproducción asistida, precisamente empleamos LH
subcutánea. De esta manera, se podría intuir la veracidad de la respuesta 2. En cualquier caso, no
se preocupe si la ha fallado, la dificultad de esta pregunta es evidente.(R2)

236. RN de 1 semana de vida presenta candidiasis oral, lo CORRECTO es:

1. 1. Requieren manejo sistémico con anfotericina B.


2. 2. Tienen buena respuesta a la terapia tópica con nistatina.
3. 3. Requiere el uso de terapia tópica más uso de anfotericin B.
4. 4. Tratamiento con fluconazol.
Gráfico de respuestas
Comentario

En pacientes RN con candidiasis oral, el tratamiento se basa en nistatina tópica, por lo que la
respuesta correcta es la 2.(R2)

!
!
!
!
237. RN de 37 semanas de edad gestacional que, a las pocas horas del nacimiento,
comienza con dificultad respiratoria, taquipnea, quejido espiratorio y ligera cianosis. En
la radiografía de tórax aparece un aplanamiento de los diafragmas, líneas vasculares
prominentes y derrame en cisuras. La actitud terapéutica más apropiada sería:

1. 1. Ventilación asistida.
2. 2. Atmósfera enriquecida en oxígeno y observación.
3. 3. Antibióticos por vía parenteral.
4. 4. Administración de bicarbonato.
Gráfico de respuestas
Comentario
La modalidad de distrés respiratorio aguda de causa pulmonar que tiene como diana de afectación
a los recién nacidos a término o de los prematuros casi- términos es la taquipnea transitoria (SDR
tipo II o síndrome de Avery). Su expresión radiológica por antonomasia es la presencia de líquido
intersticial o en cisuras y el derrame pleural. Para tratarlo, simplemente se necesita oxígeno y
observar al niño.(R2)

238. ¿Cuál es el tipo de sangrado ginecológico más común que producen los miomas?

1. 1. Metrorragia.
2. 2. Polimenorrea.
3. 3. Menometrorragia.
4. 4. Hipermenorrea.
Gráfico de respuestas
Comentario

Debe dominar el significado de cada respuesta. Lo más característico es que los miomas,
especialmente los submucosos, produzcan un aumento en el flujo menstrual normal, más que
producir sangrados en otros momentos del ciclo. La respuesta 4 es, por tanto, la correcta.(R4)

239. Indique qué NO esperaría encontrar en un paciente con enfisema predominante:

1. 1. Hipercapnia.
2. 2. Pa O2 alrededor de 70 mm Hg.
3. 3. Volumen residual alto.
4. 4. Capacidad de transferencia de CO baja.
Gráfico de respuestas
Comentario

El EPOC (bronquitis crónica, enfisemia) ambas son enfermedades obstructivas podremos


encontrar en las dos un aumento del VR por atrapamiento aéreo que provocará el descenso de la
CV. Un hecho fisiopatológico clave para diferenciar el enfisema de la bronquitis es que en el
enfisema se produce un descenso de la capacidad de transferencia de CO, cosa que no ocurre en
la bronquitis. Las pruebas complementarias también nos sirven para diferenciar ambos tipos. La
gasometría del enfisema muestra alteraciones no muy pronunciadas (ligera hipoxia y casi nunca
hipercapnia) mientras que en el bronquítico estas son muy acusadas. La RX del enfisema muestra
hiperinsuflación (aplanamiento diafrágmatico, hiperclaridad retroesternal y retrocardiaca),
deficiencia periféricas de vasos, bullas y silueta cardiaca alargada. En el bronquítico la RX no tiene
nada característico pero es sugestivo encontrar cardiomegalia o engrosamiento de las paredes
bronquiales.(R1)

!
!
!
!
240. Mujer joven que 4 días después de ser intervenida por apendicitis aguda perforada,
comienza con dolor abdominal localizado en hipogastrio, fiebre, polaquiuria, disuria y
tenesmo rectal. En la exploración el abdomen es blando, depresible, doloroso a la
palpación profunda en hipogastrio y no hay signos de irritación peritoneal. En el tacto
rectal se aprecia un abombamiento doloroso en la cara anterior del recto. Ante esta
situación, hay que pensar que la paciente presentará con mayor probabilidad:

1. 1. Absceso perianal.
2. 2. Torsión ovárica.
3. 3. Absceso de Douglas.
4. 4. Infección urinaria.
Gráfico de respuestas
Comentario

En su conjunto y dado el contexto en el que se desarrolla el cuadro (apendicitis perforada


posteriormente operada) debemos pensar que se trata de un absceso intraabdominal de
localización pélvica (en este caso sólo la opción 3 concuerda con todo lo comentado). Recuerde
que su clínica consiste en fiebre persistente, que puede ser su único síntoma, taquicardia, dolor y
masa (en nuestro caso detectado por tacto rectal, a través del cual podemos acceder a la
palpación del fondo de saco de Douglas). Tanto la torsión ovárica como la salpingitis plantean el
diagnóstico diferencial con el abdomen agudo, pero no con un cuadro de carácter más insidioso
como son los abscesos. La infección urinaria puede presentarse con disuria polaquiuria y dolor
hipogástrico, pero no se palparía masa en la exploración y el contexto clínico nos hace pensar con
mayor probabilidad en la opción 3. La opción 1 queda descartada al tratarse de un cuadro
superficial que se visualiza externamente en la región perianal.(R3)

241. Los anticolinérgicos deben evitarse en pacientes de edad avanzada con enfermedad
de Parkinson por producir:

1. 1. Sequedad de boca.
2. 2. Estreñimiento.
3. 3. Retención urinaria.
4. 4. Cuadros confusionales.
Gráfico de respuestas
Comentario

Pregunta de dificultad moderada pero que puedes contestar con conocimientos de farmacología.
Los fármacos anticolinérgicos (trihexifenidilo, biperideno) producen dos tipos de efectos
secudarios: 1) efectos antimuscarínicos periféricos como sequedad de boca , estreñimiento,
retención urinaria, visión borrosa, trastornos de la sudoración y taquicardia y 2) efectos
anticolinérgicos centrales que pueden producir síndromes confusionales y alteraciones de la
memoria, por lo que se debe evitar su uso en ancianos (respuesta 4 correcta).(R4)

242. Mujer de 26 años, primigesta de 32 semanas, acude a urgencias por dolor cólico y
lumbalgia. No refiere pérdida de líquido. En la exploración no se observan sangrado ni
líquido amniótico en vagina. Se confirma latido cardiaco fetal normal y el ulrasonido
vaginal nos informa de un acortamiento cervical del 60%. En los primeros 10 minutos de
vigilancia cardiotocográfica presenta 3 contracciones. ¿Qué pensaría como primera
opción en esta paciente?:

1. 1. Antibióticos.

!
!
!
!
2. 2. Sulfato de magnesio.
3. 3. Corticoesteroides y tratamiento tocolítico.
4. 4. Ecografías seriadas.
Gráfico de respuestas
Comentario

Se trata de una gestante de menos de 34 semanas, con un acortamiento cervical del 60%. Aunque
no nos den la longitud exacta del cérvix uterino deberíamos sospechar que se trata de una
amenaza de parto prematuro, por lo tanto lo tanto lo más indicado seria la maduración pulmonar
con corticoides y el tratamiento tocolítico.(R3)

243. La ictericia fisiológica del recién nacido se caracteriza por:

1. 1. Iniciarse al segundo o tercer día.


2. 2. Acompañarse de hepatomegalia.
3. 3. Iniciarse nada más nacer.
4. 4. Producir prurito.
Gráfico de respuestas
Comentario

La ictericia fisiológica preserva al recién nacido del estrés que provocan los radicales libres de
oxígeno. Es debida a una hiperbilirrubinemia de predominio indirecto. Comienza alrededor del
segundo, tercer día de vida en los RNT y entre el tercer y el cuarto día de vida en los RNPT.
Alcanza un máximo en ambos casos entre el tercer y el cuarto día (12 mg/dl en los RNT, 14 mg/dl
en los RNPT).(R1)

244. Un paciente presenta dolores torácicos desde hace 4 meses. Si queremos descartar
el origen coronario de estos síntomas, la prueba diagnóstica, de entre las siguientes,
MENOS sensible, será:

1. 1. Talio-201 con dipiridamol.


2. 2. Ecocardiograma con dobutamina.
3. 3. Electrocardiograma basal.
4. 4. Electrocardiograma de esfuerzo.
Gráfico de respuestas
Comentario

La respuesta es la 3, pues en pacientes con angina estable, lo habitual es que el ECG basal sea
normal. Del resto de respuestas, la menos sensible es el ECG de esfuerzo o ergometría
convencional, que tiene un porcentaje no despreciable de falsos negativos. El resto de pruebas,
con ecocardiografía o medicina nuclear, mejorán la sensibilidad y la especificidad. Los isótopos son
ligeramente más sensbles que el eco de estrés, aunque este los supera en especificdad .(R3)

245. Mujer de 58 años, en tratamiento desde hace 12 años con indapamida por
hipertensión. Refiere presentar dolor centrotorácico opresivo, acompañado de disnea
suspirosa cuando realiza esfuerzos con los brazos, sube escaleras o se enfada. En
cambio, no lo presenta si camina. La exploración muestra un cuarto ruido y un latido de
la punta vivo y no desplazado como únicos hallazgos relevantes. El electrocardiograma
muestra un discreto crecimiento ventricular izquierdo con repolarización normal. A
propósito de este caso, puede afirmarse todo lo que sigue, EXCEPTO:

!
!
!
!
Está indicado estudio radiológico de columna cervical para descartar la cervicoartrosis
1. 1.
como causa de sus síntomas.
La existencia de una prueba de esfuerzo electrocardiográfica máxima normal permite
2. 2.
descartar razonablemente la isquemia miocárdica como causa de sus síntomas.
Seguramente tiene una hipopotasemia importante secundaria al tratamiento prolongado
3. 3.
con una tiazida.
La exploración y la clínica sugieren disfunción diastólica, que debe confirmarse con un
4. 4.
ecocardiograma.
Gráfico de respuestas
Comentario
Uno de las principales repercusiones orgánicas de la HTA es la cardiopatía hipertensiva que se
produce tras varios años de HTA, debido al aumento de la postcarga cardiaca. Consiste en una
hipertrofia concéntrica de la pared ventricular izquierda que determina un deterioro de la
distensibilidad de la pared ventricular con la consiguiente disfunción diastólica. Además la
hipertrofia de la pared ventricular incrementa las demandas miocárdicas de oxígeno, y junto con un
aumento en la incidencia de las lesiones coronarias que aparece en la HTA hacen que la HTA
pueda provocar también cardiopatía isquémica, por lo que también conviene descartar ésta en esta
paciente (opciones 2 y 4). Por la localización del dolor que presenta esta paciente en relación con
el movimiento de los brazos, también convendría descartar una artrosis cervical, como se reseña
en la opción 1. Por último, la opción falsa es la 3 porque, aunque la indapamida puede ocasionar
hipopotasemia, ésta no cursa con dolor centrotorácico y disnea, sino más bien con mareos,
calambres y debilidad muscular.(R3)

246. A young boy presents with a typical nephrotic syndrome, selective proteinuria and
no hematuria. A renal biopsy is obtained. Which of the following is the most likely
pathological result?

1. 1. Extracapillary proliferative glomerulonephritis


2. 2. Minimal change nephropathy.
3. 3. Renal amyloidosis.
4. 4. Acute poststreptococcal glomerulonephritis.
Gráfico de respuestas
Comentario

Sencilla pregunta de aspectos anatomopatológicos de GN que, por la frecuencia con la que


aparecen en el ENARM, es de las pocas que le podrían aparecer. Aconsejamos la realización de
una tabla que resuma los aspectos más importantes como la microscopía óptica y electrónica y la
inmunofluorescencia (IF), o en su defecto, utilizar la tabla de la sección de Anatomía Patológica del
Manual.

La nefrosis lipoidea es sinónimo de GN por cambios mínimos (OPCION 2 CORRECTA), cuya


clínica típica de presentación es el síndrome nefrótico en niños. Se utiliza el término de nefrosis
lipoidea para designar el aspecto de esta GN al microscopio óptico (MO), por la reabsorción de
lípidos en los túbulos (recordamos que en el síndrome nefrótico existe hiperlipemia y lipiduria). No
hay ningún otro hallazgo al MO. La IF es generalmente negativa. Finalmente recordamos que la
GNCM al microscopio electrónico muestra fusión de podocitos como único hallazgo. Si nos
hubieran preguntado la causa más frecuente de síndrome nefrótico en el adulto, la respuesta
hubiera sido la GN membranosa (aunque hoy se está igualando su frecuencia con la de la
nefropatía diabética).(R2)

247. Señale la opción FALSA acerca del tratamiento de las bronquiolitis:

!
!
!
!
Se debe mantener bien hidratado al niño. Lo mejor es que continúe con su alimentación
1. 1. vía oral, pero si esto no es posible se debe emplear sonda nasogástrica para siga siendo la
vía de alimentación más fisiológica.
La hipoxia se debe corregir con medidas de oxigenoterapia para mantener saturaciones de
2. 2.
oxígeno transcutáneas por encima del 95%.
La adrenalina es eficaz en esta enfermedad, ya que disminuye tanto el componente
3. 3.
inflamatorio (por vasoconstricción), como la secreción de moco.
La ribavirina no se debe aplicar porque no ha demostrado que sea eficaz ni para disminuir
4. 4.
el tiempo de ventilación mecánica ni la duración de la estancia.
Gráfico de respuestas
Comentario

Esta pregunta acerca de las bronquiolitis es útil porque nos resume su tratamiento. En cuanto a las
medidas de soporte es necesario mantener una nutrición adecuada con tomas pequeñas y
frecuentes evitando la sonda nasogástrica ya que favorece la obstrucción nasal y el reflujo. Se
mantener al niños bien hidratado ya sea vía oral o parenteral. Recuerde que hay un aumento de
las perdidas insensibles de agua asociadas a la taquipnea. El paciente hospitalizado debe recibir
oxigeno para mantener una saturación > 93-95%. En lo referente al tratamiento farmacológico, la
utilización de mantener saturaciones de oxígeno mayores de 95%.salbutamol en aerosol puede ser
beneficiosa en algunos pacientes a pesar de que el mecanismo fisiopatológico de la obstrucción
bronquial es el edema y las secreciones y no el broncoespasamo. El fármaco más eficaz es la
adrenalina ya que disminuye tanto el componente inflamatorio (por vasoconstricción) como la
secreción de moco. El uso se ribavirina (un agente antiviral específico) ha demostrado una leve
eficacia en los ensayos clínico. Se reserva para casos graves en pacientes de alto riesgo, pero no
se debe aplicar de rutina.(R1)

248. Masculino de 36 años, que acude a la consulta por clínica de infecciones de


repetición de vías respiratorias altas. Desde hace varios años, presenta varios episodios
de otitis y sinusitis al año, requiriendo con frecuencia tratamiento antibiótico. En los
exámenes de laboratorio realizada los niveles de IgG, IgA e IgM son normales, así como
las subpoblaciones linfocitarias. ¿Qué prueba realizaría usted para descartar una
inmunodeficiencia humoral?

1. 1. Subclases de IgG, IgG2 e IgG4.


2. 2. Subclases de IgA, IgA1 e IgA2.
Los niveles de inmunoglobulinas detectados con las técnicas realizadas hasta el momento,
3. 3.
descartan la inmunodeficiencia humoral sin necesidad de más pruebas.
Si existen rangos normales de subpoblaciones linfocitarias, no es posible sufrir una
4. 4.
inmunodeficiencia humoral.
Gráfico de respuestas
Comentario

Una de las inmunodeficiencias primarias humorales o de anticuerpos que debemos descartar ante
un caso clínico sugerente, es el déficit de subclases de IgG (IgG2 e IgG4) que nos puede pasar
desapercibido ya que generalmente presentan cuantificación de IgG total en niveles normales.(R1)

249. Señale la CORRECTA en relación al adenocarcinoma gástrico:

1. 1. El 75% aparecen en el cardias.


2. 2. El tipo difuso aparece en personas más jóvenes.
3. 3. H. pylori puede estar en relación con el tipo difuso.
4. 4. Las personas del grupo sanguíneo B tienen mayor riesgo.

!
!
!
!
Gráfico de respuestas
Comentario

En relación a la epidemiología del adenocarcinoma gástrico, su incidencia tiende a disminuir en las


últimas décadas. Parece que tienen mayor riesgo las personas con grupo sanguíneo A. Los de tipo
intestinal, que son más frecuentes, aparecen sobre todo en cuerpo y antro, y son los que se
asocian a infección por Helicobacter pylori. Mientras que los de tipo difuso, son más raros y se
pueden encontrar en pacientes jóvenes.(R2)

250. Embarazada de 38 semanas de gestación ingresa en trabajo de parto. En la


exploración se detectan unas lesiones ulceradas, agrupadas, en región vulvar, así como
adenopatías dolorosas bilaterales. Nunca antes había tenido episodios parecidos. Tiene
además febrícula y leve afectación del estado general. Señale la respuesta INCORRECTA
en relación con este caso:

Este caso sería tributario de una cesárea, aunque esta medida es más importante en las
1. 1.
recidivas que en la primoinfección herpética.
La afectación del recién nacido podría producir manifestaciones parecidas a la varicela
2. 2.
neonatal.
Es desaconsejable la evolución por vía vaginal, incluso si se administra aciclovir i.v.
3. 3.
durante el parto.
Si el recién nacido se contagia en el canal del parto, una posible manifestación es la
4. 4.
queratitis.
Gráfico de respuestas
Comentario

Se trata de una infección por el Virus Herpes. El serotipo 2 es el principal implicado,


relacionándose el contagio con el contacto con secreciones genitales infectadas al pasar por el
canal del parto. Por ello la cesárea electiva es el tratamiento de elección para evitar ese contacto y
ulterior contagio, fundamentalmente en la primoinfección herpética y con menos importancia en las
recidivas. Por todo esto la respuesta falsa es la 1.(R1)

251. A 58-year-
old man presents to the emergency department for "a racing heartbeat". On questioning,
it is known that the patient usually visits the emergency department complaining of

!
!
!
!
palpitations, dizziness and chest pain that spontaneously appear and disappear with no
recognizable trigger factors. The ECG findings are shown in the image. Which of the
following is the most likely diagnosis?

1. 1. Atrioventricular node re-entry tachycardia.


2. 2. Ventricular tachycardia in a patient with Brugada syndrome.
3. 3. Ventricular tachycardia.
Orthodromic re-entry tachycardia in a pacient with an accessory atrioventricular
4. 4.
conduction pathway.
Gráfico de respuestas
Comentario

Tema poco preguntado para fines del ENARM. El sustrato anatómico de la taquicardia por
reentrada ortodrómica es la existencia de un vía accesoria auriculoventricular. Lo más frecuente es
que se trate del haz de Kent, responsable del sindrome de preexcitación de Wolf - Parkinson -
White.

El sindrome electrocardiográfico de preexcitación está conformado por:

Intervalo PR corto (menor de 0.10seg)

Onda delta

Trastornos de repolarización.(R4)

252. Once the acute episode is successfully managed, which of the following options is
the most appropriate next step in management?

1. 1. Surgical treatment.
2. 2. Catheter ablation of accessory atrioventricular pathways.
3. 3. Pacemaker insertion.
4. 4. Implantable cardioverter-defibrillator insertion.
Gráfico de respuestas
Comentario
Para contestar esta pregunta debemos haber sido capaces interpretar el ECG y diagnosticar la vía
accesoria. La ablación es el tratamiento de elección en el WPW con taquicardias. En los casos
asintomáticos, el manejo es conservador salvo que se trate de profesionales de alto riesgo
(conductores, deportistas, etc.). No hay indicación de DAI y la amiodarona, a pesar de ser eficaz en
muchas arritmias, no es útil para taquicardias por reentrada.(R2)

253. Masculino con genitales externos ambiguos, teste unilateral, presencia de útero.
Debemos sospechar:

1. 1. Disgenesia gonadal mixta.


2. 2. Deficiencia de factor antimülleriano.
3. 3. Disgenesia gonadal pura.
4. 4. Déficit de 5-alfa reductasa.
Gráfico de respuestas
Comentario

!
!
!
!
Los trastornos de la diferenciación sexual son poco rentables para el examen. La disgenesia
gonadal mixta suele tener cariotipo 45XO/46XY. Presenta un testículo disgenético unilateral y una
gónada contralateral en cordón o bandeleta, defectos en la virilización y persistencia de los
conductos müllerianos. El sexo genético se determina en el momento de la concepción. El brazo
corto del cromosoma Y contiene el gen para iniciar la diferenciación testicular. Una anomalía en
este gen da lugar a la presencia de estrías gonadales o disgenesia testicular, y los genitales
internos y externos serán ambiguos, pues la secreción del factor inhibidor de Müller y de
testosterona está alterada (se desarrolla fenotipo femenino). En la deficiencia del factor
antimülleriano habría útero pero no genitales ambiguos, pues la virilización es normal. Por el
contrario en el déficit de 5- alfa- reductasa y en la alteración en la síntesis de T habría genitales
ambiguos pero no restos del útero. La disgenesia gonadal pura presenta a ambos lados una
gónada fibrosa y fenotipo femenino.(R1)

254. Un paciente cirrótico con antecedente de varios episodios de descompensación


hidrópica y varices esofágicas grado III, ingresa por distensión abdominal y edemas, sin
fiebre ni dolor abdominal, reconociendo que desde hace tres semanas no ha tomado el
tratamiento diurético. En la exploración física se aprecia: TA 110/60, afebril, palmas
hepáticas, ascitis moderada, circulación colateral, y edemas pretibiales. En los análisis:
tiempo de protrombina 58%, la albúmina es 3.2 mg/dl, y los leucocitos 5300/mm3. El
análisis de una muestra de líquido ascítico: albúmina 1.1 g/dl, y 180 neutrófilos /mm3. En
el cultivo de líquido ascítico creció S. aureus meticilín-sensible. Tres hemocultivos fueron
estériles. En este caso es correcto afirmar que:

1. 1. Tiene una peritonitis bacteriana secundaria con foco cutáneo.


2. 2. Puede evolucionar a una peritonitis espontánea.
3. 3. Presenta probablemente una colecistitis asociada.
4. 4. S. aureus es un contaminante.
Gráfico de respuestas
Comentario

Se trata de un paciente cirrótico con ascitis, sin clínica sugestiva de peritonitis bacteriana
espontánea (no tiene fiebre, dolor abdominal...). Los neutrófilos son menores de 250 neutrófilos
/mm3, pero el cultivo es positivo para S. aureus. El diagnóstico de sospecha es una bacterioscitis
monomicrobiana no neutrofílica. Esta entidad puede evolucionar posteriromente a una PBE, por lo
que lo correcto sería repetir un cultivo y recuento celular 48 horas después, y tratar si persiste el
cultivo positivo o si aparece clínica.

Tenga mucho cuidado con la respuesta 4. En la 4, se AFIRMA que es un contaminante, sin


contemplar ninguna otra posibilidad. Podría serlo, porque a veces coloniza la piel… Pero no puede
asegurarlo, como hace la respuesta 4. ¿Y los 180 neutrófilos que se han encontrado en líquido,
qué hacen ahí? ¿Son también contaminantes…? Sin embargo, la respuesta 2 sí es correcta. Dado
que el diagnóstico de sospecha es el que hemos dicho, existe la posibilidad de que el enfermo
evolucione a una PBE. Siendo un cirrótico con ascitis y neutrófilos en líquido ascítico, desde luego
tiene cartas para ello…(R2)

255. Masculino de 65 años acude a su consulta refiriendo pérdida de peso de


aproximadamente 8 kg en 2 meses con anorexia, náuseas y vómitos, y refiere en la última
semana emisión de heces claras (acólicas) y orinas oscuras (colúricas) con discreto
dolor abdominal. A la exploración presenta discreta ictericia cutánea y franca de mucosas
(escleróticas) con lesiones cutáneas de rascado. ¿Cuál es su diagnóstico de presunción?

!
!
!
!
1. 1. Colecistitis.
2. 2. Coledocolitiasis.
3. 3. Hidatidosis hepática.
4. 4. Cáncer de páncreas o periampular.
Gráfico de respuestas
Comentario

El cuadro de colestasis (ictericia, acolia, coluria, prurito) asociado a pérdida de peso, orienta
fuertemente a una obstrucción de las vías biliares de muy probable origen canceroso.(R4)

256. ¿Qué germen origina con más frecuencia una infección espontánea del líquido
ascítico?:

1. 1. Klebsiella.
2. 2. Streptococcus.
3. 3. Escherichia coli.
4. 4. Staphilococcus aureus.
Gráfico de respuestas
Comentario
Los gérmenes más frecuentemente responsables de la peritonitis bacteriana espontánea que se
produce en los pacientes cirróticos son los gramnegativos: de ellos, el más frecuente es la
Escherichia coli.(R3)

257. Una mujer de 46 años consulta por la aparición en la zona pectoral de una placa de
5 cm de diámetro eritematoescamosa, con un borde sobreelevado bien delimitado y
ulceración superficial. Usted observa otras dos similares muy pequeñas cercanas a ella.
La paciente refiere que la lesión más grande apareció hace más de un año y las otras han
brotado recientemente. ¿Cuál es su diagnóstico de sospecha?:

1. 1. Epitelioma espinocelular.
2. 2. Epitelioma basocelular superficial.
3. 3. Queratosis actínica.
4. 4. Psoriasis con intenso componente inflamatorio.
Gráfico de respuestas
Comentario

El epitelioma basocelular es el tumor maligno más frecuente en la especie humana, y en Occidente


su incidencia casi quintuplica la del cáncer de pulmón. La clínica de este tumor es característica.
Se debe sospechar su existencia cuando estamos ante una pápula de aspecto blanquecino y brillo
perlado que crece progresivamente y que puede llegar a ulcerarse en su centro, con formación
sucesiva de costras. Aparece por orden: cabeza, cuello, extremidades superiores y tronco.
Destruye por contigüidad y son excepcionales las metástasis a distancia. A diferencia del
carcinoma epidermoide, suele localizarse sobre piel sana. En el caso que comentamos, nos
presentan una variedad especial, la variante pagetoide o superficial- multicéntrica, que es
típicamente eritematodescamativa y suele localizarse en tronco.(R2)

258. En los pacientes con insuficiencia renal crónica terminal, se pueden evidenciar
varias anomalías en órganos y sistemas, EXCEPTO:

!
!
!
!
Puede existir neuropatía periférica, que suele ser una firme indicador para el inicio de la
1. 1.
diálisis.
2. 2. Hay una mayor incidencia de úlcera péptica.
3. 3. El prurito urémico mejora ostensiblemente con la diálisis.
4. 4. Las funciones de la hipófisis, tiroides y suprarrenales pueden ser normales.
Gráfico de respuestas
Comentario

Las complicaciones de la IRC son ubicuas, écheles un vistazo porque aunque son muchas son
sencillas. Recuerde que entre los trastornos del sistema nervioso está la polineuropatía urémica,
que al principio es sensitiva y si no se instaura la diálisis se convierte en motora (así es una
indicación relativa de diálisis). A nivel endocrino la hipófisis, el tiroides y las suprarrenales pueden
ser normales en algunos pacientes; en otros hay un déficit de la transformación periférica de T4 a
T3 y un déficit de LH y FSH e incremento de la PRL que puede llevar a impotencia. A nivel
digestivo aparte de la anorexia y el fétor urémico es más frecuente la úlcera péptica y la HDA, por
hipersecreción. El prurito es tan sólo una indicación relativa de diálisis y que no suele mejorar con
ésta.(R3)

259. Una vez instaurada la nefropatía diabética en estadio clínicamente reconocible.


¿Cuál es el factor de mayor relevancia para enlentecer la progresión?:

1. 1. Dieta hipoproteica.
2. 2. Disminución de la hipercolesterolemia.
3. 3. Control tensional.
4. 4. Disminución de la hipertrigliceridemia.
Gráfico de respuestas
Comentario

Esta pregunta es de cultura general médica por lo que NO SE DEBE FALLAR en el ENARM. Todas
las opciones pueden ser útiles para enlentecer la progresión pero el control tensional es la que se
ha mostrado MAS EFECTIVA en este aspecto. Un control estricto de las cifras tensionales reduce
tanto la lesión vascular como la hipertensión intraglomerular lo cual protege aún más al riñón.
Aunque sirve cualquier antihipertensivo, los IECAs y los ARA- II son los más efectivos (vasodilatan
la eferente) puesto que reducen la presión en el ovillo capilar disminuyendo significativamente la
proteinuria.(R3)

260. ¿Cuál de estos niños tiene el MENOR riesgo de mortalidad neonatal?

1. 1. 2,400 gramos, 36 semanas de gestación.


2. 2. 3,600 gramos, 35 semanas de gestación.
3. 3. 3,000 gramos, 38 semanas de gestación.
4. 4. 4,500 gramos, 42 semanas de gestación.
Gráfico de respuestas
Comentario

Muy fácil sólo tiene que identificar los mejores parametros. Tienen mayor riesgo de mortalidad
neonatal los prematuros (edad gestacional menor de 37 semanas), los bajos pesos para edad
gestacional (de forma aproximada, se considera bajo peso a aquellos que se sitúan por debajo de
los 2500 gramos) y los de peso excesivo (peso mayor de 4000 gramos). Simplemente, tiene que
seguir estas directrices y llegará con facilidad a la respuesta correcta, que es la 3 (a término y de
peso adecuado).(R3)

!
!
!
!
261. Respecto a las otitis externas, señale la asociación INCORRECTA:

1. 1. Otitis externa maligna - Virus herpes zóster.


2. 2. Otitis externa bacteriana - Otalgia.
3. 3. P. aeruginosa - Otitis externa difusa.
4. 4. S. aureus - Otitis externa circunscrita (forúnculo del CAE).
Gráfico de respuestas
Comentario

Esta es una pregunta importante que no debe fallar. Nos preguntan por el agente causal de la otitis
externa maligna o necrotizante. No se trata del VVZ, sino de la Pseudomonas aeruginosa que
causan un cuadro muy grave y de elevada mortalidad, típico de pacientes con diabetes u otro tipo
de inmunodepresión.(R1)

262. Todas las siguientes patologías se asocian a fontanela anterior amplia, EXCEPTO:

1. 1. Hipertiroidismo.
2. 2. Osteogénesis imperfecta.
3. 3. Raquitismo.
4. 4. Hidrocefalia.
Gráfico de respuestas
Comentario

La fontanela anterior o mayor tiene gran utilidad en la exploración física de los recién nacidos y
lactantes hasta los 9-18 meses de vida, fecha en que se cierra definitivamente. Servirá para
orientar desde el punto de vista diagnóstico, si está a tensión (hemorragia intracraneal,
hidrocefalia, meningitis); otras veces la encotraremos agrandada de forma inespecífica
(acondroplasia, rubéola congénita, raquistismo, hipotiroidismo, osteogénesis imperfecta).(R1)

263. A 10-day-old infant presents with blisters on his palms and soles along with sneezing
and watery rhinorrhea. He was a full term newborn with a body weight of 2.4 kg and body
height of 48 cm. Cutaneous pallor, hepatomegaly and 2-cm splenomegaly were observed
at birth. Laboratory studies show: 25,000 leukocytes, Hb 13.5 g/dl, platelets 80,000/mm3.
Urinalysis shows proteinuria and mild glucosuria. Which of the following options is
INCORRECT?

Dark-field microscopic study of the skin lesions may be helpful in order to confirm the
1. 1.
diagnosis.
2. 2. The diagnostic work-up should include skeletal X-ray examination.
A positive result in FTA-abs would be expected and it would remain positive for the rest
3. 3.
of the patient's life.
4. 4. RPR blood test result will remain positive despite appropriate antimicrobial therapy.
Gráfico de respuestas
Comentario

Tiene que sospechar que el paciente presenta una infección connatal cuando presente exploración
compatible (adenopatías, hepatomegalia, esplenomegalia), antecedentes de CIR simétrico y la BH
típico (anemia, trombocitopenia). Pero no se ha de quedar ahí. Para pensar en una lúes congénita
ha de aderezar este contexto con lesiones cutáneas de diversa naturaleza, participación mucosa
(coriza) y alteraciones óseas con periostitis. Y ha de tener claro que la IgM es de procedencia
estricta del niño (no ha pasado la barrera placentaria), que las pruebas treponémicas permanecen

!
!
!
!
positivas el resto de la vida en un niño infectado pero las reagínicas varían en función de la
actividad de la enfermedad y que el campo oscuro es uno de los métodos más útiles de
diagnóstico de la lúes en una muestra obtenida del pequeño.(R4)

264. Existen diversos factores o circunstancias que pueden favorecer la aparición de


hematomas en la herida quirúrgica. Señale cuál de los siguientes es el MENOS
importante:

1. 1. Disección subcutánea.
2. 2. Infección de la herida.
3. 3. Tos.
4. 4. Tratamiento antiagragante.
Gráfico de respuestas
Comentario

Requiere un simple razonamiento sobre los posibles factores que pueden dañar los capilares o
afectar el sistema hemostásico. Cuanto mayor es la disección subcutánea más vasos se ven
afectados. La tos supone una agresión para los pequeños capilares ya afectados por la cirugía. Es
obvio que la antiagregación plaquetar de la aspirina y más aún los trastornos de la coagulación
favorecen la aparición de un hematoma en la herida quirúrgica.(R2)

265. ¿Cuál es la afectación más frecuente en la rubéola congénita?:

1. 1. Coriorretinitis.
2. 2. Cardiopatía.
3. 3. Catarata.
4. 4. Sordera.
Gráfico de respuestas
Comentario

En la rubéola congénita, las lesiones que pueden aparecer pueden ser: las de cualquier TORCH
(antecedente de CIR, hepatoesplenomegalia, adenopatías, anemia, trombocitopenia), la tríada de
Gregg (hipoacusia neurosensorial - que es lo más frecuente- , alteraciones oculares y cardiopatía)
y lesiones óseas (recuerde que éstas no asocian periostitis).(R4)

266. Femenino de 55 años de edad, de sexo femenino, que refiere deseo imperioso de
miccionar y que no le permite inclusive llegar al baño. ¿Qué tipo de incontinencia urinaria
presenta?

1. 1. De esfuerzo.
2. 2. Mixta.
3. 3. De urgencia.
4. 4. Estructural.
Gráfico de respuestas
Comentario

Pregunta fácil.

La incontinencia urinaria de esfuerzo es la pérdida involuntaria de orina coincidente con un


aumento de la presión abdominal por un fallo en los mecanismos de resistencia uretral, ya sea por

!
!
!
!
hipermovilidad uretral o por insuficiencia uretral intrínseca.

La incontinencia de urgencia es la pérdida involuntaria de orina asociada a un fuerte deseo de


miccionar, que se debe a contracciones involuntarias del músculo detrusor. Suelo relacionarse con
un aumento en el número de micciones diarias.

Respuesta 3 correcta.(R3)

267. Acerca de la puntuación del Apgar:

1. 1. Es básico para el inicio de la reanimación neonatal.


2. 2. Permite determinar los pasos a seguir en la reanimación neonatal.
3. 3. Muestra la respuesta a la reanimación neonatal.
4. 4. Valora la frecuencia respiratoria.
Gráfico de respuestas
Comentario

La escala de Apgar valora y gradúa la salud del recién nacido. Consta de 5 categorías con 3
criterios cada una. La puntuación de cada categoría oscila entre 0 y 2, por lo que el rango total va
desde 0 a 10. Este test se valora en los recién nacidos al primer y quinto minuto de su nacimiento.
Si la puntuación total se mantiene inferior a 3 se recomienda realizarlo a los 10, 15 y 30 minutos.
Respuesta 3 correcta, ya que no determina los pasos a seguir en la secuencia de renaminación
neonatal.(R3)

268. ¿Cuál NO es factor de riesgo del cáncer de mama?:

1. 1. Edad.
2. 2. Nuliparidad.
3. 3. Radiación ionizante.
4. 4. Mastodinia en el último año.
Gráfico de respuestas
Comentario

Pregunta directa bastante fácil sobre el cáncer de mama. Sabiendo simplemente que el cáncer de
mama suele presentarse de forma asintomática en el 80% de las veces, y que no produce dolor,
accedemos directamente a la opción 4 como la correcta. Las otras tres opciones son factores de
riesgo. La edad (opción 1) es el factor de riesgo más importante. La nuliparidad (opción 2) y el
primer embarazo a una edad avanzada, y las radiaciones ionizantes (opción 3) también son
factores de riesgo. (R4)

269. Un paciente de 53 años, taxista, acude al servicio de urgencias con fiebre,


escalofríos, mal estado general y dolor hipogástrico. Sus signos vitales demuestran una
importante tendencia la hipotensión y los exámenes de laboratorio muestran leucocitosis
con desviación izquierda, discreta alteración de la coagulación y función renal
conservada. Refiere inicio brusco del cuadro, que se acompaña de molestias miccionales
tipo disuria, aunque ya llevaba unos días notando cierta polaquiuria. El tratamiento inicial
más adecuado será:

1. 1. Ciprofloxacino 500 mg v.o.


2. 2. Ceftriaxona i.v. + tobramicina i.v.
3. 3. Trimetoprim-sulfametoxazol v.o.

!
!
!
!
4. 4. Vancomicina i.v.
Gráfico de respuestas
Comentario
En un caso grave de prostatitis como el que se nos propone, el tratamiento se basa en
antibioterapia parenteral de alta eficacia para gramnegativos (quinolonas, beta lactámicos..) los
aminoglucósidos cuentan con un importante efecto sinérgico. La impermeabilidad hematotisular
prostática para el paso de antibióticos se altera con la inflamación y hace que puedan usarse
medicamentos que normalmente no alcanzarían concentraciones tisulares adecuadas.(R2)

270. En un paciente con tratamiento antiagregante, una de las siguientes pruebas estará
alterada:

1. 1. Tiempo de tromboplastina parcial activada.


2. 2. Tiempo de protrombina.
3. 3. Tiempo de hemorragia.
4. 4. Lisis de euglobinas.
Gráfico de respuestas
Comentario

Esta pregunta podría acertarse con bastante sencillez, si se conocen los siguientes conceptos
básicos.

- La hemostasia primaria (vasoplaquetaria) depende de la integridad de los vasos y las plaquetas.


Cuando falla, se producen manifestaciones como sangrados cutáneos o mucosos (petequias,
gingivorragias). En el laboratorio, se altera el tiempo de hemorragia (respuesta 4 correcta). Un
ejemplo de este grupo serían los sangrados por trombocitopenia, por la enfermedad de Von
Willebrand o asociadas al uso de antiagregantes plaquetarios.

- La hemostasia secundaria depende de la cascada de la coagulación. Cuando se altera, aparecen


hemorragias internas, como un hemartros o hematomas musculares. Analíticamente, se modifican
los tiempos de coagulación. El ejemplo más clásico es la hemofilia. Recuerde que las alteraciones
de la vía intrínseca alteran el tiempo de tromboplastina parcial activado, mientras que las de la
extrínseca alteran el tiempo de protrombina.(R3)

271. La operación de facoemulsificación de la catarata por ultrasonido, casi la única


técnica hoy utilizada, preserva y mantiene en el ojo:

1. 1. La cápsula anterior del cristalino.


2. 2. Las masas corticales del cristalino.
3. 3. Las masas nucleares del cristalino.
4. 4. La cápsula ecuatorial y posterior del cristalino.
Gráfico de respuestas
Comentario

Una vez que se extirpa la catarata, la lente intraocular se mantendrá apoyada sobre la cápsula
posterior del cristalino, que es lo que se preserva tras su extirpación, así como la cápsula
ecuatorial. Estos elementos son los que dan soporte a la lente. Recuerde que, a largo plazo, es
frecuente la opacificación de la cápsula posterior, que se trataría con el láser YAG.(R4)

!
!
!
!
272. Una paciente de 30 años, con un cuadro clínico de sangrado menstrual irregular,
dismenorrea, dispareunia, y una esterilidad de 3 años de evolución, presenta un estudio
hormonal normal, con USG transvaginal que informa de un útero normal y sendas
formaciones quísticas ováricas bilaterales de 4 cm con signos ultrasonográficos de
sospecha. Teniendo en cuenta el diagnóstico más probable, ¿cuál de las siguientes es
de elección para obtener el diagnóstico de sospecha?

1. 1. Ecografía abdominal.
2. 2. Ecografía transvaginal.
3. 3. Laparoscopía.
4. 4. Culdocentesis.
Gráfico de respuestas
Comentario

Ante un posible caso de Síndrome de Ovario Poliquístico, dada la edad de la paciente y la


sintomatología, en cuyo estudio, no tenemos las cosas del todo claras, con unos datos hormonales
que no están alterados, lo mejor que podemos plantear es la laparoscopía exploradora, para
realizar un diagnóstico de certeza y podremos tomar biopsias para un posterior estudio
anatomopatológico.(R3)

273. Alteraciones de asfixia neonatal:

1. 1. Prolapso de cordón.
2. 2. DM de la madre.
3. 3. Prematurez.
4. 4. Infección fetal intrauterina.
Gráfico de respuestas
Comentario

La prematuridad puede dar lugar a la enfermedad de la membrana hialina como consecuencia del
déficit de surfactante lo que puede conllevar a asfixia neonatal. Respuesta 3 correcta.(R3)

274. A 6-year-old boy is brought to the pediatrician with a 20-day history of progressive
visual loss in his right eye. He refers changes in sizes and shapes of objects in his visual
field. Ophthalmological examination confirms the loss of vision acuity and fundoscopy
shows a thinner area of the retina, just on the macular region, with hyperpigmented
edges, with no additional lesions. He has no relevant past medical history except mild
learning difficulties. What is the most likely diagnosis?

1. 1. Congenital Toxoplasmosis.
2. 2. Early onset retinitis pigmentosa.
3. 3. Perinatal citomegalovirus infection.
4. 4. Candida albicans infection.
Gráfico de respuestas
Comentario

Esta pregunta es bastante compleja. En primer lugar, ha de enfocar el caso clínico hacia una
afectación de la retina (por pérdida de agudez visual, adelgazamineto retiniano a nivel macular). La
respuesta no puede ser retinitis por C. albicans, pues en el examen del fondo ocular no se aprecian
exudados algodonosos. Tampoco parece plausible una infección connatal por CMV, pues en este
caso, si hay coriorretinitis también habría microcefalia y retraso mental profundo.(R1)

!
!
!
!
275. Al examinar a un neonato se encuentra ausencia de murmullo vesicular en un
hemitórax. La causa mas probable es:

1. 1. Síndrome de dificultad respiratoria.


2. 2. Atelectasia segmentaria.
3. 3. Neumotórax.
4. 4. Enfermedad de membrana hialina.
Gráfico de respuestas
Comentario

Al síndrome de distrés respiratorio del recién nacido se le conoce también como enfermedad de
membrana hialia, cuyo sustrato fisiopatológico es el déficit de surfactante pulmonar, con lo que la
uno y la cuatro quedan excluidas.

Una atelectasia segmentaria provocacia ausencia del MV en parte del hemitórax, mientras que el
neumotórax provoca ausencia de MV en todo un hemitórax. Respuesta correcta 3.(R3)

276. Una de las siguientes medidas está MENOS indicada en el tratamiento del dolor
crónico:

1. 1. Estimulación eléctrica de la sustancia gris periacueductal.


2. 2. Empleo de amitriptilina.
3. 3. Benzodiacepinas a dosis hipnótico-relajantes.
4. 4. Rizotomía posterior.
Gráfico de respuestas
Comentario

Pregunta de elevada dificultad y de poca trascendencia sobre el manejo del dolor crónico. Existen
varios fármacos que se pueden utilizar como adyuvantes de los analgésicos, como los
antidepresivos tricíclicos. Las benzodiacepinas se pueden usar durante cortos periodos de tiempo,
recomendando su retirada en 2 ó 3 semanas, ya que existe un riesgo importante de dependencia,
por lo que no se consideran útiles para el tratamiento del dolor crónico. En relación a los diferentes
métodos neuroquirúrgicos de control del dolor, existen varios procedimientos, entre los que
destacan:

- Cordotomía: dolor unilateral de tronco y extremidades.

- Mielotomía comisural: dolor bilateral de tronco y extremidades.

- Implantación de bombas de narcóticos a nivel epidural e intratecal: dolor oncológico.

- Estimulación de la sustancia gris periacueductal.

- Estimulación de cordones posteriores.

- Lesión DREZ: dolor debido a la avulsión de raíces.(R3)

277. En el caso de un CIR asimétrico o tipo II, ¿cuál es el primer parámetro


ultrasonográfico que se altera?:

!
!
!
!
1. 1. Longitud del fémur.
2. 2. Diámetros abdominales.
3. 3. Cantidad de líquido amniótico.
4. 4. Aparición de edema nucal.
Gráfico de respuestas
Comentario

El CIR tipo I, también llamado simétrico, tiende a reducir el tamaño fetal de forma homogénea. En
cambio, el CIR tipo II pueden entenderlo como un feto excesivamente “delgado”, es decir, lo
primero que se alteran son los diámetros abdominales, mientras que el resto de los parámetros
biométricos permanecen relativamente inalterados. Respuesta correcta 3.(R2)

278. ¿Cuál de las siguientes NO constituye una complicación de la asbestosis pulmonar?:

1. 1. Insuficiencia respiratoria crónica no hipercápnica.


2. 2. Cor pulmonale.
3. 3. Carcinoma broncopulmonar.
4. 4. Tuberculosis pulmonar.
Gráfico de respuestas
Comentario

La exposición al asbesto puede producir diversos efectos sobre el organismo: fibrosis pulmonar
(asbestosis), cáncer de pulmón y mesotelioma pleural o peritoneal maligno. La fibrosis pulmonar
por exposición al asbesto se manifiesta clínicamente igual que la fibrosis pulmonar de otras
etiologías, pudiendo llegar a presentar insuficiencia respiratoria y cor pulmonale. No existe mayor
riesgo de sufrir tuberculosis (sí en la silicosis).(R4)

279. ¿Qué contenido de sodio se recomienda que contenga la solución de rehidratación


oral en la fase de mantenimiento de una diarrea?:

1. 1. 40-60 mEq/l.
2. 2. 75-90 mEq/l.
3. 3. 90-110 mEq/l.
4. 4. 110-120 mEq/l.
Gráfico de respuestas
Comentario

Esta pregunta sobre la rehidratación oral, que es la que de elección, es difícil y no muy importante.
Puede ser útil recordar que en el tratamiento de la deshidratación hay una fase de reposición y otra
de mantenimiento. En la primera se puede usar la solución de Reposición de la OMS, pero su
contenido en sodio es elevado (Na 90 mEq/l). En países desarrollados las soluciones que se
emplean en ambas fases deben estar compuestas por sodio 50mEq/l, K 20mEq/l, glu 20 g/l y
sustancias tampón. En México, contamos con el sobre preparado de vida suero oral, que es
gratuito y está basado en las concentraciones recomendadas.(R1)

280. Un lactante de 9 Kg con diarrea, luce moderadamente deshidratado, el déficit de líquidos


probablemente sea del orden de:

1. 1. 1350 mL.
2. 2. 1900 mL.
3. 3. 450 mL.

!
!
!
!
4. 4. 900 mL.
Gráfico de respuestas
Comentario

En caso de deshidratación moderada, el déficit de líquidos es de aproximadamente el 10%, por lo


que 9 Kg x 10%: 900 mL. Respuesta 4 correcta.(R4)

281. ¿Cuál de los siguientes fenómenos dermatológicos que podemos encontrar en un


neonato NO debe sugerirnos una patología subyacente?

1. 1. Manchas hipopigmentadas geográficas.


2. 2. Ictericia precoz en las primeras 24 horas.
3. 3. Melanosis pustulosa.
4. 4. Manchas café con leche.
Gráfico de respuestas
Comentario

La melanosis pustulosa es una entidad relativamente frecuente y benigna. Típicamente las


lesiones tienen PMN y afecta a palmas y plantas.(R3)

282. Hombre de 46 años que presenta disnea de moderados esfuerzos desde hace cuatro
meses. Como sintomatología acompañante refiere sensación de palpitaciones
esporádicas desde hace más de un año y dolor torácico inespecífico que no se relaciona
con nada. En la exploración encontramos frecuencia cardíaca de 84 lpm, TA de 150/75
mmHg, pulso hiperdinámico, latido de la punta hiperdinámico en el sexto espacio

!
!
!
!
intercostal izquierdo desplazado lateralmente. Auscultación: ritmo sinusal, soplo
diastólico en tercer espacio intercostal izquierdo que aumenta con la realización de
ejercicio físico isométrico. ¿Cuál es su diagnóstico de sospecha?:

1. 1. Insuficiencia pulmonar.
2. 2. Estenosis mitral.
3. 3. Insuficiencia mitral.
4. 4. Insuficiencia aórtica.
Gráfico de respuestas
Comentario

Lo más importante de la insuficiencia aórtica (IAo) es que sepa reconocerla en forma de caso
clínico, las opciones de tratamiento médico y dominar las indicaciones de tratamiento invasivo.

El hecho fisiopatológico más importante es el volumen de sangre que regurgita desde la aorta al VI
durante la diástole.

Los pacientes con IAo crónica permanecen asintomáticos durante años, comenzando la clínica
cuando ya existe deterioro de la función del VI. Durante este período son frecuentes las
palpitaciones debidas a la percepción molesta de los latidos hiperdinámicos.

La disnea es el síntoma más importante y precoz que indica el deterioro de la función ventricular.
El dolor torácico puede ser por isquemia o latido hiperdinámico. Si es por isquemia, es más
frecuente por la noche y no suelen responder bien al tratamiento con nitratos.

La auscultación característica es: - disminución del componente aórtico del 2R; - soplo diastólico
que comienza inmediatamente después del 2R, más largo y más intenso cuanto más grave sea la
IAo.

El tratamiento médico consiste en diuréticos, digoxina y vasodilatadores.

Las indicaciones de Qx son pacientes asintomáticos con disfunción sistólica/ dilatación ventricular
izquierda, y paciente sintomáticos con IAo significativa. El tratamiento quirúrgico de elección es la
prótesis valvular.(R4)

283. Respecto a la ictericia neonatal, todas las siguientes afirmaciones son ciertas,
EXCEPTO:

1. 1. En los prematuros, la ictericia fisiológica aparece en las primeras 24 horas.


A los 15 días de vida, en un RN tanto si fue a término como pretérmino, la bilirrubina
2. 2.
indirecta debe ser <1 mg/dl.
3. 3. La ictericia por incompatibilidad ABO puede ocurrir en el primer hijo.
4. 4. La hipoglucemia y la acidosis aumentan el riesgo de kernicterus.
Gráfico de respuestas
Comentario

No debe de terminar de leer la primera respuesta para saber que es la mala (aún así debe leer
todas). La hiperbilirrubinemia es un método que, según teorías recientes, sirve para proteger al
neonato frente al estrés provocado por los radicales libres. Por este motivo, en mayor o menor
medida, como una defensa natural, todos los neonatos presentan ictericia fisiológicamente. Ésta
comienza en los RNT el

!
!
!
!
2º día y en los RNPT el tercero. Se prolonga aproximadamente hasta el día 7 de vida en los RNT y
un par de días más en los RNPT. A partir de entonces, la cifra de bilirrubina se normaliza. Las otras
opciones son correctas: la isoinmunización ABO no precisa sensibilización previa (se da por
anticuerpos naturales tipo IgG), y la permeabilidad de la barrera hematoencefálica es mayor en el
neonato gravemente enfermo (séptico, acidótico,...).(R1)

284. Mujer de 32 años, primigesta. En exámenes de laboratorio del primer trimestre


destaca en la serología negatividad para Toxoplasma (IgM e IgG). Señale la respuesta
INCORRECTA con respecto al manejo de esta paciente:

1. 1. Está indicado realizar serologías de control a lo largo de toda la gestación.


El riesgo de afectación fetal está en relación con el momento de infestación materna. Si la
2. 2. madre padece la primoinfección en los últimos meses de gestación, el feto se afecta con
menos frecuencia que al principio de ésta.
La paciente debe seguir durante la gestación una serie de recomendaciones higiénicas (no
3. 3. consumir carne o derivados crudos o sin cocer, evitar el contacto con gatos, lavado de
verduras...).
Si se produce afectación fetal, uno de los signos sugerentes sería la presencia de
4. 4.
calcificaciones intracraneales.
Gráfico de respuestas
Comentario

La toxoplasmosis es una enfermedad que puede ser crónica, ya que el toxoplasma tiene la
capacidad de acantonarse en las células musculares del organismo, poniéndose de manifiesto en
momentos de inmunodepresión. Sin embargo, en un adulto sano casi siempre es asintomática.

Este microorganismo es intracelular y precisa medios de cultivo específicos (por ello se usan más
las técnicas serológicas para su diagnóstico). En el recién nacido, la presencia de IgG
antitoxoplasma podría no tener importancia, dado que podría tener un origen materno (IgG
atraviesa la placenta). Sin embargo, la IgM anti-toxoplasma fetal sí que tendría un significado
patológico, porque la habría producido el propio feto.

En el embarazo, es característico que la afectación fetal es más grave cuanto más precoz sea su
instauración, y más probable cuanto más avanzada esté la gestación(respuesta 2 correcta).(R2)

285. La complicación crónica más común en la infección congénita por CMV es a nivel
de:

1. 1. Sistema nervioso central.


2. 2. Sistema hematológico.
3. 3. Sistema inmune.
4. 4. Hígado.
Gráfico de respuestas
Comentario

En la infección connatal por CMV se producen calcificaciónes a nivel periventricular. Además


corioretinitis y microcefalia. Respuesta 1 correcta.(R1)

!
!
!
!

286.! ¿En cuál de las siguientes circunstancias NO


se aconseja el tratamiento quirúrgico como opción inicial en el manejo de esta
fractura?:

1. 1. Fractura patológica.
2. 2. Lesión asociada de la arteria braquial.
3. 3. Angulación en el foco de fractura de 10 grados.
4. 4. Lesión asociada ipsilateral del cúbito y radio (codo flotante).
Gráfico de respuestas
Comentario
Cuando la angulación en el foco de fractura es de 10º, el tratamiento indicado es la reducción
cerrada y la inmovilización con yeso. Recuerde que el yeso colgante de Caldwell es útil en las
fracturas diafisarias de húmero por permitir la reducción “por efecto de la gravedad”, debido al peso
de la escayola. En un politraumatizado está indicada la cirugía de sus fracturas para reducir la
morbi-mortalidad y mejorar el manejo del paciente. En las fracturas patológicas, la cirugía ayuda a
estabilizar la lesión, permite la toma de muestra del tumor, y mejora las posibilidades de
consolidación que están mermadas por la patología de base. La lesión asociada de la arteria
braquial obliga a la síntesis de la fractura para permitir la reparación de la arteria con un hueso
estable, y evitar el riesgo de una nueva lesión de la arteria reparada. Por último, cuando existe un
codo flotante es necesaria la cirugía en las fracturas alrededor del codo para poder mover pronto la
articulación, recuperar la función, y evitar rigideces.(R3)

287. En la fractura que puede verse en la imagen n° 13, ¿cuál es el nervio que puede verse
afectado con mayor frecuencia?:

1. 1. Nervio musculocutáneo.

!
!
!
!
2. 2. Nervio radial.
3. 3. Nervio mediano.
4. 4. Nervio circunflejo.
Gráfico de respuestas
Comentario
En la imagen podemos observar una radiografía simple en proyección anteroposterior del codo. En
la región diafisaria y metafisaria distal del húmero se aprecia una fractura con un tercer fragmento.
El nervio que puede verse afectado con mayor frecuencia en este tipo de fracturas es el radial. La
lesión del nervio radial es la complicación aguda más frecuente en las fracturas de la diáfisis
humeral, y es más habitual en las fracturas oblicuas del tercio distal (fractura de Holstein-Lewis). La
lesión del nervio radial suele ser una neuroapraxia, que se recupera en unos tres a cuatro meses.
La lesión del radial, asociada a una fractura de húmero, no precisa cirugía salvo en los casos de
empeoramiento de sintomatología cuando se manipula la fractura (sugiere atrapamiento en el
foco), o en las fracturas abiertas.(R2)

288. Paciente de 70 años de edad, referida a su consulta por adenopatías generalizadas,


leucocitosis con linfocitosis absoluta, con linfocitos pequeños, de escaso citoplasma y
cromatina condensada en grumos, así como abundantes sombras nucleares. El
inmunofenotipo es: CD 5 +, CD19 +, CD 20 +, CD23 + y FMC7 negativo. En cuanto al
pronóstico de este paciente, todos los siguientes se asocian a una menor supervivencia
EXCEPTO:

1. 1. Presencia de hepatoesplenomegalia.
2. 2. Tiempo de duplicación linfocitaria >12 meses.
3. 3. Infiltración difusa de la médula ósea >70%.
4. 4. Deleción 11q.
Gráfico de respuestas
Comentario

Una pregunta sencilla sobre la leucemia linfática crónica, que puede resolverse por sentido común.
La respuesta incorrecta es la 3, puesto que esta enfermedad es una neoplasia maligna, por lo que
el pronóstico será peor cuanto más breve sea el tiempo de duplicación linfocitaria. Respecto a esto,
conviene recordar que la neoplasia con el tiempo de duplicación más breve es el linfoma de Burkitt,
donde son tres días.(R2)

289. Lactante de 2 meses de edad que hace tres días comenzó con rinorrea acuosa, tos
y estornudos. Desde ayer presenta taquipnea con 60 rpm, tiraje intercostal con aleteo
nasal. En Urgencias observan dificultad respiratoria con sibilancias inspiratorias y
espiratorias y algunos crepitantes bilaterales. Una Rx de tórax muestra hiperinsuflación
bilateral con una atelectasia laminar y corazón pequeño. Presenta: pH 7.24; pCO2 58
mmg; HCO3 21 mEq/l. ¿Cuál es el diagnóstico más probable?

1. 1. Bronconeumonía bilateral.
2. 2. Crisis asmática de origen infeccioso.
3. 3. Neumonitis intersticial.
4. 4. Bronquiolitis.
Gráfico de respuestas
Comentario

Pregunta de dificultad fácil-media en forma de caso clínico de bronquiolitis.

!
!
!
!
La bronquiolitis es una enfermedad viral que cursa con obstrucción inflamatoria de las pequeñas
vías aéreas. Se define como el primer episodio de dificultad respiratoria con sibilantes espiratorios
que tiene lugar en un niño < 2 años (el niño de la pregunta tiene dos meses), con síntomas de
infección de vía respiratoria de tipo vírica.

El virus sincitial respiratorio (VSR) es el agente causal más frecuente.

La fuente de infección suele ser familiar y la transmisión se produce por vía respiratoria.

Los niños mayores y adultos no presentan dificultad respiratoria a pesar de la infección, dado que
toleran mejor el edema bronquiolar. En efecto, la contribución de la vía aérea de pequeño calibre a
la resistencia total es mayor en los lactantes, y durante una infección por el VSR se produce una
obstrucción bronquiolar causada por edema, acúmulo de moco y detritus celulares, con el
resultado final de disminución del radio de la vía aérea y aumento reflejo de la resistencia al paso
de aire.

Es frecuente el antecedente de infección respiratoria leve unos días antes (hace tres días tuvo
rinorrea, tos, estornudos, etc.) para aparecer posteriormente tos, dificultad respiratoria e
irritabilidad.

Habitualmente no existe fiebre, pero puede haber febrícula. Debido a la taquipnea, el lactante
puede manifestar rechazo del alimento. En la exploración encontramos signos de dificultad
respiratoria como aleteo nasal, tiraje, etc. En la auscultación se oye una espiración alargada y
sibilancias. La disminución significativa de los ruidos respiratorios indica obstrucción casi completa
de las vías aéreas y constituye, por tanto, un signo de gravedad.

El diagnóstico es habitualmente clínico apoyado en la radiología. Se asume que bronquiolitis es


todo aquel primer episodio de dificultad respiratoria baja que sufre un lactante menor de dos años,
por otro lado, sano. La radiología muestra hiperinsuflación pulmonar. Se puede demostrar el virus
en secreciones nasofaríngeas por inmunofluorescencia y elevación de los títulos de anticuerpos en
sangre o cultivo.

Para el tratamiento es esencial que recuerde que los broncodilatadores son de muy dudosa
eficacia, puesto que la fisiopatología de la enfermedad no es la broncoconstricción (como ocurre en
el asma). Lo que más les beneficia es la administración de oxígeno. La adrenalina inhalada puede
ser útil al producir vasoconstricción y, por consiguiente, ayudar a disminuir el edema.

La ribavirina inhalada se reserva para casos graves. En resumen, recuerde que no están indicados
ni broncodilatadores, ni corticoides, ni antibióticos ni sedantes.

El principal diagnóstico diferencial de la bronquiolitis se debe realizar con el asma. El asma:

•! Es raro en menores de un año.


•! No hay infección previa.
•! Suele existir historia familiar.
•! Las crisis son repetidas (solamente se llama bronquiolitis al primer episodio de dificultad
respiratoria de un lactante menor de dos años).
•! El comienzo es brusco.

(R4)

!
!
!
!

Diagnóstico diferencial entre bronquiolitis y asma

290. Masculino de 65 años, fumador importante, con antecedentes de cardiopatía


isquémica que consulta por cuadro de 5 meses de evolución de dolor abdominal cólico
postpandrial en mesogastrio y pérdida de 10 kilos de peso debido a disminución de la
ingesta. La prueba diagnóstica que le confirmará su sospecha diagnóstica es:

1. 1. Gastroscopia.
2. 2. Pancolonoscopia.
3. 3. Enema opaco.
4. 4. Arteriografía.
Gráfico de respuestas
Comentario
El paciente que previamente ya presenta factores de riesgo cardiovascular presenta el dolor típico
de un cuadro de angina intestinal que aparece en el período postpandrial cuando se deriva gran
cantidad de flujo sanguíneo al sistema circulatorio intestinal. Estos pacientes presentan
disminución de peso como consecuencia de que el paciente tiene miedo de comer ya que ello le
ocasiona dolor. Por tanto la prueba diagnóstica de elección será la arteriografía que determinará la
presencia de estenosis parcial de los vasos intestinales como consecuencia del desarrollo de
placas de arterioesclerosis.(R4)

!
!
!
!
291. El signo o síntoma que NO es posible encontrar en la enfermedad inflamatoria
pélvica es:

1. 1. Dolor en fosa iliaca derecha.


2. 2. Cuerpo uterino agrandado.
3. 3. Fiebre igual o mayor de 38ºC.
4. 4. Elevación de la proteína C reactiva.
Gráfico de respuestas
Comentario

Tema muy importante para el ENARM. De los datos clínicos que se aportan, el único que no es
posible encontrar es el cuerpo uterino agrandado. El resto si es posible encontrarlos. Respuesta 2
correcta.(R2)

292. Femenino de 55 años que consulta por prurito y aparición de lesión eritematosa y
descamativa a nivel de pezón derecho que se ha ido extendiendo hacia la areola. En la
exploración se observa lesión eczematosa de aproximadamente 3.5 cm que abarca
pezón-areola derechos. No se palpan nódulos subyacentes ni se objetiva secreción por
pezón (ni espontánea ni a la expresión). Mama izquierda sin alteraciones en unidad
areola-pezón. No se palpan nódulos. Ambas axilas negativas. Señale la CORRECTA:

Debemos tranquilizar a la paciente y pautar tratamiento con crema de corticoides y


1. 1.
antihistamínicos.
2. 2. Es una localización poco frecuente de dermatitis atópica.
En la histología de la lesión podemos encontrar células grandes de citoplasma amplio y
3. 3.
claro típicas.
4. 4. Sólo sospecharemos malignidad si se asocia una lesión subyacente palpable.
Gráfico de respuestas
Comentario

Pregunta de dificultad media sobre la enfermedad de Paget de la mama.

La enfermedad de Paget debe considerarse como la extensión de un tipo de cáncer de mama


ductal a la epidermis suprayacente, afectando pezón y la areola. Es poco frecuente, representando
el 1-3% de todos los cánceres de mama. Clínicamente, consiste en una lesión eccematosa,
asociando también prurito. Por ello, el eccema es su principal diagnóstico diferencial. Una de las
características por las que puede distinguirse del eccema es por ser unilateral (el eccema suele
afectar a ambas mamas). Otro rasgo diferencial son los límites, que suelen ser difusos en el
eccema y mejor definidos en el Paget mamario.

Recuerde que, histológicamente, son características las células de Paget. Son grandes,
redondeadas, PAS- positivas, con núcleos de gran tamaño y sin puentes intercelulares que las
unan.(R3)

293. Paciente de 94 años de edad es traído a urgencias por presentar una evacuación
sanguinolenta, acompañada de otras posteriores con repercusión hemodinámica. En la
exploración física se aprecia palidez cutaneomucosa y soplo sistólico aórtico, siendo la
exploración abdominal normal. Se le practicó una fibrogastroscopia que resultó negativa,
por lo cual se practica una colonoscopia que detecta angiomas en araña a nivel del ciego
y colon ascendente. Con todos estos datos, ¿cuál sería su diagnóstico?

!
!
!
!
1. 1. Angiodisplasia de colon.
2. 2. Hemangioendotelioma difuso.
3. 3. Linfoma intestinal.
4. 4. Colitis isquémica.
Gráfico de respuestas
Comentario

La localización más frecuente de la angiodisplasia intestinal es el ciego, y se diagnostica mediante


colonoscopia, como en el paciente de la pregunta (respuesta 1 correcta).

El cuadro clínico típico suele ser un anciano con hemorragia digestiva baja de repetición. De
hecho, esta enfermedad es la causa más frecuente de hemorragia digestiva baja recidivante en el
anciano. Muchas veces la han relacionado con la estenosis aórtica, asociación que recibe el
nombre de síndrome de Heyde.(R1)

294. Todos los siguientes son factores de riesgo de cáncer de mama excepto uno,
señálelo:

1. 1. Antecedentes familiares de cáncer de mama.


2. 2. La terapia hormonal sustitutiva.
3. 3. El estatus social bajo.
4. 4. La menopausia tardía.
Gráfico de respuestas
Comentario
El factor de riesgo más importante de cáncer de mama es el factor genético, en aquellas pacientes
con el antecedente familiar de cáncer de mama, sobre todo en etapas anteriores a la menopausia.
Se han identificado dos genes: BRCA1 y BRCA2, que también están en relación con el cáncer de
ovario. Todos los demás son más discutibles, pero en cuanto al estatus social, es el alto el que
sería de riesgo, seguramente por el mayor diagnóstico al seguir más regularmente las estrategias
de screening.(R3)

!
!
!
!

295. Neonato de 8 días de vida, prematuro de 27


semanas de edad gestacional, ingresado en cuidados intensivos neonatales con
ventilación mecánica. Presenta distensión abdominal progresiva con irritabilidad y no ha
realizado evacuaciones en 48 horas. Se realiza radiografía de abdomen. Respecto a la
patología de sospecha, señale la respuesta INCORRECTA:

1. 1. Esta patología no tiene una alta mortalidad.


2. 2. La lactancia materna podría proteger de esta patología.
En los laboratorios podríamos encontrar descenso de leucocitos y aumento de reactantes
3. 3.
de fase aguda como la PCR.
4. 4. La prematuridad es factor de riesgo en esta patología.
Gráfico de respuestas
Comentario

Estamos ante un paciente con enterocolitis necrotizante, con factores de riesgo como prematuridad
e hipoxia. Las zonas isquémicas intestinales se sobreinfectan con gérmenes intrabdominales por lo
que solemos encontrar elevación de reactantes de fase aguda y en ocasiones leucopenia.

La lactancia materna es un factor protector de esta patología.

Es una patología con una alta mortalidad y gran morbilidad en la unidades de cuidades
intensivos.(R1)

296. Respecto al tratamiento del neonato de la pregunta anterior, señale la respuesta


CORRECTA:

1. 1. Se debe iniciar tratamiento con gentamicina y ampicilina.


2. 2. Se debe iniciar tratamiento con vancomicina y amikacina.
3. 3. Debe mantenerse la sonda nasogástrica cerrada.
4. 4. Debe realizarse cirugía urgente.
Gráfico de respuestas
!
!
!
!
Comentario

La radiografía muestra edema de asas y no se observa aire distal, pero no se observan signos de
neumoperitoneo ni de gas en la vena porta, por lo que la primera indicación sería tratamiento
conservador no quirúrgico.

Permanecerá en dieta absoluta, con administración de cristaloides o NPT y sonda nasogástrica


abierta.

El tratamiento antibiótico es empírico, pero ante la sospecha de una sepsis nosocomial deben
cubrirse gérmenes nosocomiales y sobre todo gérmenes anaerobios y gram- negativos. Respuesta
correcta 2.(R2)

297. Which of the following statements is FALSE regarding pancreatic cysts and
pseudocysts?

1. 1. Pseudocysts lack epithelial cells or endothelial cells in their "fibrous wall".


2. 2. Pseudocysts occur in non-functioning pancreas.
3. 3. The cysts communicate with the pancreatic duct.
4. 4. Pseudocysts are more common in chronic pancreatitis than in acute pancreatitis.
Gráfico de respuestas
Comentario

Los pseudoquistes son formaciones de contenido líquido no recubiertas del epitelio de los
conductos pancreáticos, que tampoco tienen una cápsula propiamente dicha y que se producen
por el acúmulo de secreciones pancreáticas tras agresiones diversas en el seno del tejido
pancreático, en su mayoría por pancreatitis. Su contenido es muy rico en enzimas pancreáticos.
Así pues, no es habitual la formación de estas colecciones en el seno de un páncreas no
funcionante, por lo que esa es la opción claramente falsa. El resto de las opciones expuestas son
verdaderas.(R2)

298. A 17-year-old female comes to her physician complaining of delayed-onset puberty.


She claims she hasn’t had menarchia yet. Physical examination shows: body height 1.49
m, wide neck, separated nipples and Tanner’s stage 1 breast and pubic hair appearance.
Mark which of the following tests will most likely yield the suspected diagnosis:

1. 1. Growth hormone levels.


2. 2. Brain MRI.
3. 3. Karyotype.
4. 4. FSH levels.
Gráfico de respuestas
Comentario
Karyotype. Turner syndrome is one of the most common chromosomal abnormalities
(approximately 1 in 2000 live-born female infants). DIAGNOSIS: A) Clinical appearance, B)
Karyotype and C) associated conditions. About 50% of affected girls have a 45,X karyotype; about
80% have lost the paternal X. Most of the other 50% are mosaics (eg, 45,X/46,XX or 45,X/47,XXX).
Among mosaic girls, phenotype may vary from that of typical Turner syndrome to normal.(R3)

299. En un embarazo fisiológicamente prolongado:

!
!
!
!
1. 1. Se puede detectar CIR tipo II.
2. 2. Presenta riesgo de distocia por peso fetal elevado.
3. 3. Presenta insuficiencia placentaria.
4. 4. Presenta oligoamnios.
Gráfico de respuestas
Comentario

Se considera un embarazo prolongado cuando dura más de 42 semanas. Un embarazo


fisiológicamente prolongado el feto es normal, pero de mayor tamaño, no hay signos de sospecha
de pérdida de bienestar fetal y puede dar problemas de distocia debido al tamaño del feto. En un
embarazo patológicamente prolongado es donde hay CIR, insuficiencia placentaria, signos de
sospecha de pérdida de bienestar fetal y oligoamnios.(R2)

300. En una mujer de 52 años, que lleva 14 meses en amenorrea y que acude a una
revisión ginecológica rutinaria, ¿qué datos en relación con la clínica o los exémenes de
laboratorio NO esperaría encontrar?:

1. 1. Valor de FSH de 28 mU/ml, con intensa clínica vasomotora y disminución de la libido.


Dificultad en la conciliación del sueño y episodios ocasionales y leves de enrojecimiento
2. 2.
facial y sudoración fría.
3. 3. Relaciones sexuales más dolorosas, con síntomas de prurito y escozor genital ocasional.
4. 4. Presencia de labilidad emocional, con algún sofoco ocasional.
Gráfico de respuestas
Comentario

Para el diagnóstico de la menopausia, además de los datos clínicos (amenorrea de un año,


sofocos, cambios en piel y mucosas, alteraciones de la sexualidad, trastornos del sueño, cambios
corporales,…), tiene mucha importancia la determinación hormonal que ya ha sido preguntada en
el ENARM y no debe olvidarla. Para confirmar la menopausia, los niveles de estradiol deben ser
inferiores a 20 pg/ml y los niveles de FSH deben estar aumentados (superiores a 40 mU/ml),
debido a que no existe retroalimentación negativa por los niveles disminuidos de estradiol. Por
tanto, la respuesta correcta es la 1 puesto que la FSH debe encontrarse elevada en la
menopausia.

Tal vez no te pregunten los niveles de FSH, pero si debes recordar que está elevada en la
menopausia.(R1)

301. Un niño de 7 años presenta fiebre, adenopatías retroauriculares y cervicales


posteriores, exantema maculopapuloso y en la BH: leucopenia con células
plasmocitarias (células de Turk) y linfoides reactivas. ¿Cuál es el diagnóstico más
probable?

1. 1. Sarampión.
2. 2. Rubéola.
3. 3. Escarlatina.
4. 4. Varicela.
Gráfico de respuestas
Comentario

La rubéola es una enfermedad infecciosa, producida por un Togavirus, cuyo período de incubación
es de 2-3 semanas. El pródromos consiste en un cuadro catarral leve, con fiebre baja o moderada,

!
!
!
!
conjuntivitis sin fotofobia y un enantema no patognomónico (manchas de Forcheimer). El signo
más característico de esta fase son las adenopatías retroauriculares, cervicales posteriores y
postoccipitales. Poco después, aparece un exantema, que es morbiliforme y confluyente en la cara.
Se resuelve a través de una mínima descamación. Es posible que aparezcan ciertas
complicaciones, como la artritis (la más frecuente), la encefalitis o la trombopenia.(R2)

302. A 61-year-old female patient comes to the physician because of a palpable nodule in
her breast, measuring 1.5 cm in diameter. Mastgraphy reveals a nodule measuring 9 mm
in diameter, with spiculated margins and clustered microcalcifications located in her right
breast. Thick needle biopsy reveals an invasive ductal carcinoma (grade I). Further
studies show that the lesion stains positive for estrogen receptors and negative for Her-
2. Which of the following is the most appropriate treatment?

Modified radical mastectomy, hormone therapy and chemotherapy, regardless of the


1. 1.
lymph node involvement.
Tumorectomy combined with sentinel node biopsy. Radiotherapy, hormone therapy and
2. 2.
chemotherapy according to the result of the sentinel node biopsy.
Lymphadenectomy. Chemotherapy according to the result of the lymph nodes biopsy.
3. 3.
Mastectomy and trastuzumab.
4. 4. Tumorectomy, radiotherapy, lymphadenectomy and danazol.
Gráfico de respuestas
Comentario

Pregunta que no se debe fallar de cáncer de mama.

La respuesta correcta es la 2, ya que al ser una neoplasia de <2 cm y probablemente localizada, se


tiene que realizar en primer lugar la tumorectomía y posteriormente un ganglio sentinela y
dependiendo de este resultado se determinará si es necesario o no la realización de radioterapia,
tratamiento hormonal y quimioterapia.(R2)

303. A 63-year-old male comes to the ER due to sudden onset of severe retrosternal and
upper abdominal pain. He has been vomiting for the past few hours after consuming
alcohol. He smokes 40 cigarettes a day, and has a history of alcoholic pancreatitis. His
temperature is 37.8ºC, BP is 127/75, pulse is 110/min, and respirations are 30/min.
Physical examination reveals palpable crepitus in the suprasternal notch. What is the
most likely diagnosis?

1. 1. Acute pancreatitis.
2. 2. Boerhaave's syndrome.
3. 3. Acute gastroenteritis.
4. 4. Lung perforation.
Gráfico de respuestas
Comentario
Boerhaave's syndrome. Esophageal rupture is a rupture of the esophageal wall. Iatrogenic causes
account for approximately 50-55% of esophageal perforations, usually due to medical
instrumentation such as endoscopy or paraesophageal surgery. In contrast, the term Boerhaave's
syndrome is reserved for the 10% of esophageal perforations which occur due to vomiting. The
classic history of esophageal rupture is one of severe retching and vomiting followed by
excruciating retrosternal chest and upper abdominal pain. Odynophagia, tachypnea, dyspnea,
cyanosis, fever, and shock develop rapidly thereafter. Subcutaneous emphysema (crepitation) is an
important diagnostic finding but is not very sensitive, and a pleural effusion may be detected.(R2)

!
!
!
!
304. A 55-year-old male comes to your office referred by his primary care physician for
study of the following blood count. Leukocytes: 40,000/mcl., 1% of blastic cells. Which of
the following is the most likely cytogenetic abnormalitie?

1. 1. t (15 ; 17).
2. 2. Alteración del 5q.
3. 3. t (9 ; 22).
4. 4. t (8 ; 14).
Gráfico de respuestas
Comentario

El diagnóstico más probable, dada la edad del paciente, el porcentaje de blastos y las
características de la biometría hemática, es leucemia mieloide crónica (respuesta 3). La alteración
del 5q aparece en SMD y LAM, t(15 ; 17) en LAM promielocítica, t (8 ; 21) en LAM- M2 y t (8 ; 14)
en LAL- Burkitt.(R3)

305. La presencia del reflejo de búsqueda en un niño obnubilado de 2 años indica:

1. 1. Reflejo normal relacionado con la edad.


2. 2. Hambre.
3. 3. Momento de destetar.
4. 4. Corteza frontal anormal.
Gráfico de respuestas
Comentario

Tema muy repetido cada año en el ENARM, debe dominarlo.

Reflejo de búsqueda: Si se presiona cerca de la boca del bebé, desplaza la cabeza hacia el lado
donde nota la presión. Desaparece a los 2 meses, por lo que presentarlo a los dos años de edad te
habla de un reflejo anormal y un posible daño en la corteza frontal.

Otros reflejos primarios son:

Reflejo de succión: Se desencadena un movimiento rítmico de succión al acercar a los labios del
bebé cualquier objeto. Desaparece a los 4 meses.

Reflejos de apoyo y marcha: Si se mantiene al niño de pie, en contacto con el suelo, sosteniéndole
firmemente con los brazos, se observa como se endereza y apoya los pies. En ese momento, si se
le impulsa un poco, va adelantando alternativamente uno y otro pie, de forma semejante a la
marcha. Ambos reflejos desaparecen a los 3 meses.

Reflejo de prensión palmar: Consiste en cerrar fuertemente la mano cuando se estimula la palma
del bebé al presionarla con algún objeto. Desaparece a los 6 meses.

Reflejo de prensión plantar: Consiste en cerrar los dedos del pie cuando se estimula el pulgar del
pie del bebé al presionarla con algún objeto. Desaparece a los 9 meses.

Reflejo de brazos en cruz (Moro): Cuando el bebé oye un golpe fuerte o experimenta un
inesperado cambio de posición, separa bruscamente los brazos, para después ponerlos sobre su
pecho. Desaparece a los 6 meses.

!
!
!
!
Reflejo Babinski: Si se le roza el empeine exterior de la planta del pie, de abajo hacia arriba, con un
objeto duro, se abren los dedos del pie en abanico. Desaparece a los 12 meses.

Reflejo cervical tónico-asimétrico: Cuando la cabeza del bebé se mantiene rotada hacia un lado, al
mismo tiempo, el brazo y la pierna correspondientes a ese lado cambian a extendidos y los otros
permanecen flexionados. Desaparece a los 4 meses.(R4)

306. ¿Cuál es el tumor benigno mamario más frecuente?:

1. 1. Adenoma.
2. 2. Papiloma intracanalicular.
3. 3. Fibroadenoma.
4. 4. Fibroma.
Gráfico de respuestas
Comentario

Pregunta muy fácil, en TUMOR MAMARIO BENIGNO MÁS FRECUENTE es el fibroadenoma,


respuesta 3 correcta.(R3)

307. Entre las manifestaciones clínicas de sífilis congénita precoz, señale cuál NO suele
aparecer en el recién nacido:

1. 1. Coriza.
2. 2. Hepatoesplenomegalia.
3. 3. Signos de infección del S.N.C.
4. 4. Ictericia.
Gráfico de respuestas
Comentario

En la sífilis congénita precoz, la manifestación más frecuente es la hepatomegalia, sin duda


alguna. Acompañan a ésta un exantema polimorfo (máculas, pápulas, ampollas) con repercusión
sobre la mucosa nasal (coriza o rinorrea) y lesiones óseas con periostitis, que derivan en lo que se
conoce como falsa parálisis (antiálgica) de Parrot. La afectación del SNC es más típica de la sífilis
congénita tardía.(R3)

308. Una paciente de 34 años de edad presenta un cuadro de cirrosis descompensada


con ascitis y encefalopatía. Se detecta virus de hepatitis C y un hepatocarcinoma de 3 cm
en el lóbulo hepático derecho. ¿Cuál es el tratamiento de elección?

1. 1. Resección del lóbulo hepático derecho.


2. 2. Resección limitada del tumor.
3. 3. Interferón.
4. 4. Trasplante hepático.
Gráfico de respuestas
Comentario

Pregunta muy típica sobre el manejo del cáncer de hígado.

La pregunta es un caso de un paciente joven con una hepatitis C muy descompensada con ascitis,
encefalopatía y con un hepatocarcinoma de 3 cm en el lóbulo hepático derecho. En pacientes con

!
!
!
!
un hepatocarcinoma menor de 5 cm o tres tumores menores de 3 cm sin evidencia de invasión
ganglionar o metástasis en un paciente no candidato a cirugía por insuficiencia hepática previsible
(estadios B y C de Child), se realiza trasplante hepático.

Resumimos el estado actual del tratamiento del hepatocarcinoma:

•! Trasplante hepático: tumor menor de 5 cm (o tres tumores menores de 3 cm) sin evidencia
de metástasis ni diseminación ganglionar en un paciente no-candidato a cirugía resectiva
por insuficiencia hepática previsible (fundamentalmente Child B o C).
•! Candidatos a cirugía: nódulos únicos, unilobares, resecables, sin enfermedad
extrahepática, con buena reserva hepática y sin evidencia de trombosis portal (invasión
vascular). El no cumplimiento de alguno de estos criterios contraindica la resección
quirúrgica.
•! Alcoholización con etanol percutáneo intratumoral, radiofrecuencia y criocirugía: resultado
similar a la cirugía en nódulos menores de 3-4 cm.
•! Embolización arterial: tumores en estadios intermedios, encapsulados y con aporte arterial
elevado. Puede ser paliativa o prequirúrgica.

(R4)

309. A 28-year-old man is brought to your office by his girlfriend, who is extremely
concerned about the patient's behavior in the past few weeks. He has not been getting
any sleep, but does not seem tired during the day. Instead, he has been planing to start a
new business with a man that he met last week at the casino, and he plans on investing
all his savings. She also explains that he has been dressing weird, for example, he wears
a suit and a tie but forgets to put on his shirt. When asked about his plans, he says the
business is going to be a great success, but his speech is hard to understand at times
and his ideas difficult to follow. His past medical history is unremarkable. The diagnosis
of bipolar disorder is made, and you explain to the patient the need for long-term therapy
with litium. What tests should be performed before initiating treatment with lithium?

1. 1. Creatinine and liver function.


2. 2. Creatinine and thyroid function.
3. 3. Liver function and thyroid function.
4. 4. Creatine kinase (CK) and complete blood count.
Gráfico de respuestas
Comentario
Creatinine and thyroid function. Lithium is the treatment of choice of patients with bipolar disorder.
The most common side effects seen on patients on lithium are: gastrointestinal (diarrhea, nausea,
vomiting), hypothyroidism, leukocytosis, acne, nephrotoxicity (nephrogenic diabetes insipidus) and
risk of fetal cardiac defects (mainly Ebstein's anomaly). Due to its many side effects, it is of
paramount importance to evaluate thyroid and renal function before prescribing lithium. Do not
forget to rule out pregnancy in case of young women.(R2)

310. Una de las siguientes afirmaciones sobre las anemias megaloblásticas es FALSA:

1. 1. Es característica la presencia de macroovalocitos.


2. 2. La LDH aumenta como consecuencia de hemólisis intravascular.
3. 3. Es frecuente la trombocitopenia.
4. 4. Es frecuente la leucopenia.

!
!
!
!
Gráfico de respuestas
Comentario

Pregunta de dificultad elevada, pero que debe conocer. La anemia megaloblastica se caracterizan
por tener en: a) sangre periférica: disminución de las tres líneas celulares (pancitopenia), hematíes
macrociticos, neutrófilos hipersegmentarios, y reticulocitos No aumentados (hiporegenerativa); y en
b) M.O.: Aumento del tamaño de los precursores, aumento del número de células (M.O.
hipercelular), y eritropoyesis ineficaz que eleva la LDH y la bilirrubina indirecta levemente y se
considera una hemólisis extravascular. Recuerde que en la anemia megaloblastica hay un
aumento de la HCM, pero no esta elevada la CHCM.(R2)

311. Señale la respuesta correcta respecto a la vacunación contra la infección por VPH:

1. 1. La población diana son mujeres entre 14 y los 16 años.


2. 2. La vacunación se realiza en cuatro dosis (0, 3, 6 y 12 meses).
3. 3. La vacuna tetravalente confiere protección contra los serotipos: 6, 11, 16 y 18.
4. 4. La administración de la dosis de recuerdo es subcutánea.
Gráfico de respuestas
Comentario

La vacunación contra VPH se realiza en niñas entre los 9 y 14 años, con 3 dosis intramusculares y
no hay (de momento) que administrar dosis de recuerdo. La vacuna tetravalente es la que protege
contra condilomas genitales.(R3)

312. Una mujer se encuentra en el sexto mes de gestación. La presión arterial es de 105/60
mmHg y el examen general de orina es normal. ¿Qué esperaría encontrar en el estudio
de laboratorio de esta embarzada?

1. 1. Hemoconcentración.
2. 2. Aumento del filtrado glomerular.
3. 3. Aumento del ácido úrico.
4. 4. Ausencia de variación del aclaramiento de creatinina.
Gráfico de respuestas
Comentario

Los cambios fisiológicos en e el embarazo en el ENARM es un tema bastante rentable.

Una pregunta muy sencilla sobre la fisiología de la gestación. El caso que nos presentan tiene una
presión arterial normal y no existen alteraciones patológicas en el sedimento urinario, por lo que
debemos pensar que se trata de una gestante sana. Durante el embarazo, el filtrado glomerular
aumenta de forma fisiológica, por lo que aumenta el aclaramiento de creatinina, eliminándose una
mayor cantidad de esta sustancia.

De todos los cambios que se producen en el organismo de la gestante, uno de los más importantes
es el aumento del filtrado glomerular.(R2)

313. Todas las siguientes respuestas son características de la esferocitosis hereditaria,


EXCEPTO:

1. 1. Descenso del CHCM.

!
!
!
!
2. 2. Disminución de la autohemólisis al añadir glucosa.
3. 3. Tratamiento con esplenectomía.
4. 4. Tratamiento con folato.
Gráfico de respuestas
Comentario

Se trata de una pregunta bastante asequible simplemente con recordar una de las características
típicas de la esferocitosis. Debemos tener siempre en mente que el CHCM en esta entidad está
aumentado, ya que el defecto radica en la membrana, produciéndose una pérdida de superficie,
con una producción normal de hemoglobina, por lo que aumenta la concentración (opción 1 falsa).
El resto de opciones son correctas: la prueba de hemólisis osmótica muestra la mayor fragilidad del
hematíe, que se ve reducida gracias a la administración de glucosa al medio. El tratamiento en
casos severos es la esplenectomía. También hay que recordar que la administración de folatos
trata de prevenir las crisis megaloblásticas debidas a la hiperplasia de médula ósea.(R1)

314. Los padres de una niña de 4 años le


consultan por haber sido remitidos por presencia de cojera en su hija. Refieren que la
paciente lleva así unas 8 semanas y que ha sido valorada por un especialista en
traumatología, quien ha realizado una resonancia magnética, encontrando en la
exploración un aumento de líquido sinovial, motivo por el cual se ha realizado una
artroscopia con extracción de un líquido cuyos resultados aporta, con 35,400 leuc/mm3,
glucosa de 60 y proteínas de 35, sin encontrar gérmenes en la tinción de gram. En la
exploración física se detecta un aumento de volumen de la rodilla izquierda, con
limitación en los últimos grados de extensión y un aumento de volumen con rubor, de la
tercera interfalángica proximal del cuarto dedo del pie derecho. En la exploración ocular
se detecta la anomalía que aparece en la imagen. Indique la respuesta correcta:

Se trata como primera opción de una artritis idiopática juvenil de inicio sistémico;
1. 1. debemos iniciar tratamiento con corticoides y, una vez descartadas otras causas, con
ciclosporina.
Como primera posibilidad, le parece probable una artritis séptica, por lo que precisaríamos
2. 2. ingreso de la paciente con tratamiento antibiótico y nueva artrocentesis con petición de
cultivo de hongos y de micobacterias.
Lo más probable es que la paciente tenga anticuerpos antinucleares positivos y presenta
una elevada posibilidad de que el cuadro ocular sea una uveítis crónica, por lo que
3. 3.
debemos solicitar una exploración e indicar tratamiento con ibuprofeno, corticoides y
metotrexato, y si no hay respuesta, incluir terapia biológica.

!
!
!
!
Se trata de una artritis idiopática juvenil pero, en este caso, por la edad de aparición, lo
4. 4. más probable es que se trate de una artritis poliarticular seropositiva, por lo que requerirá
tratamiento con metotrexato y etanercept, en la mayoría de los casos.
Gráfico de respuestas
Comentario

La dificultad de esta pregunta está en reconocer el tipo de artritis crónica juvenil que padece esta
paciente, que sería una forma oligoarticular precoz. Como sabe, ésta suele complicarse con uveítis
crónica, de ahí que podamos encontrarnos alteraciones en el iris como las que se aprecian en esta
imagen, sin que exista excesiva inflamación en el polo anterior del ojo, dado que se trata de un
cuadro crónico. En esta forma, tal como se explica en la respuesta 4, es muy frecuente la
presencia de anticuerpos antinucleares (ANAs). El tratamiento, al menos de inicio, consiste en
AINEs, corticoides y/o metotrexato. Cuando no existe una respuesta adecuada, durante los últimos
años se ha comenzado a utilizar el infliximab u otros anticuerpos anti- TNF, conocidos como
fármacos “biológicos” (respuesta 4 correcta).(R3)

315. An otherwise healthy 44-year-old woman comes to the emergency department with
complaints of chest pain and shortness of breath for the last 2 days. The pain is
aggravated when he inspires deeply. His vital signs are notable for mild tachycardia and
tachypnea. Physical examination and chest X-ray reveal no significant findings. Arterial
gasometry on room air shows: pH 7.50, PCO2 32 mm Hg, PO2 66 mm Hg, O2 saturation
91%. Which of the following is the most likely underlying mechanism?

1. 1. Dilated cardiomyopathy.
2. 2. Carbon monoxide poisoning.
3. 3. Ventilation-perfusion mismatch.
4. 4. Interstitial edema.
Gráfico de respuestas
Comentario

Es importante esta pregunta para recordar los mecanismos de hipoxemia. El cual el tipo más
frecuente es alteraciones en la ventilación - perfusión. Los más probable en esta paciente es que
se trate de un TEP. En ocasiones además de una alteración V/Q puede generar un defecto tipo
shunt, pero esto se presenta con TEP con mayor afección.(R3)

316. Es aconsejable prolongar el tratamiento antituberculoso hasta 9 meses en todas las


siguientes situaciones, EXCEPTO:

1. 1. Tuberculosis meníngea.
2. 2. Tuberculosis osteoarticular.
3. 3. Tuberculosis en el paciente VIH.
4. 4. Tuberculosis en la embarazada.
Gráfico de respuestas
Comentario

El tratamiento corto de la TB se suele recomendar en la mayoría de los casos ya que así se evita la
falta de cumplimiento. Pero existen casos en los que recomienda que el tratamiento sea más
prolongado: las tres primeras opciones son casos en los que la infección suele ser más grave y son
zonas a las que el antibiótico llega con cierta dificultad, por eso debe ser más prolongado. En el
caso del paciente VIH (aunque no cumpla criterios de SIDA) aunque muchos responden bien con
un tratamiento de seis meses, la respuesta debe ser prolongada porque a veces no alcanza el nivel

!
!
!
!
esperado. En la embarazada sólo debe prolongarse el tratamiento cuando la pauta elegida así lo
exija.(R4)

317. Paciente de 25 años de edad que refiere que


desde hace unos días le dule mucho el ojo derecho y tiene fotofobia. Al explorarle
encuentra Tyndall y el iris ofrece el siguiente aspecto. Con respecto al tratamiento de
esta paciente, señale lo CORRECTO:

1. 1. Corticoides tópicos y midriáticos.


2. 2. Corticoides tópicos y mióticos.
3. 3. Corticoides orales y midriáticos.
4. 4. Corticoides orales y mióticos.
Gráfico de respuestas
Comentario

Lo primero que debemos tener en cuenta al evaluar un caso clínico con imagen, como casi
siempre, es la información que procede del enunciado. La combinación de dolor ocular intenso y
fenómeno de Tyndall resulta muy sugestiva de uveítis anterior. Si además observamos la imagen
adjunta, se corrobora esta sospecha, ya que se trata de una pupila miótica y con sinequias. La
respuesta correcta, por lo tanto, es la 1. Por supuesto, no tiene sentido el uso de fármacos
mióticos. Lo que necesita el paciente es todo lo contrario, un midriático, para relajar el esfínter del
iris (lo cual tiene una función analgésica) y contribuir a romper las sinequias.(R1)

318. El paciente presentó en los meses sucesivos cuatro episodios más de inflamación
ocular. En esta situación, ¿qué prueba de las siguientes le parece más rentable?

1. 1. HLA B27.
2. 2. Colonoscopia.
3. 3. TC pulmonar.
4. 4. Antígeno carcinoembrionario.
Gráfico de respuestas
Comentario

La causa más frecuente de uveítis anterior es idiopática, pero la enfermedad que más se relaciona
con ella es la espondilitis anquilosante. Si nos cuentan un caso de uveítis de repetición,

!
!
!
!
deberíamos pensar en una posible causa subyacente que justifica la repetición del cuadro. Por otra
parte, si se trata de un varón joven, es el típico paciente en el que debemos descartar una
espondilitis anquilosante. Por ello, la respuesta correcta sería la 1. El HLA B27 se asocia
intensamente a esta enfermedad, hasta el punto de que más del 90% de los pacientes que la
padecen presentan este haplotipo.(R1)

319. En el caso previo se realizó estudio por el Servicio de Nefrología. Batería de pruebas
inmunológicas: ANAS, AntiDNA, antiENAS y ANOES negativos. ANCASc positivos. La
biopsia renal mostró granulomas necrotizantes intraglomerulares. ¿Cuál es el
diagnóstico?:

1. 1. Enfermedad de Good-Pasture.
2. 2. Glomerulonefritis por Ig A.
3. 3. Enfemedad de Wegener.
4. 4. Síndrome de Alport.
Gráfico de respuestas
Comentario

El tema de las vasculitis puede resultarle un poco complicado, pero en sus preguntas, ciertos datos
pueden hacerle identificar RAPIDAMENTE el proceso responsable. La clave son los
autoanticuerpos: los primeros nos orientan hacia un lupus, pero al ser negativos lo descartamos
(recuerde: ANA la deSMleNADa tiene lupus - - > antiDNA, antiSM, ANA). Sin embargo, los ANCAS
tipo c (antiproteinasa 3 con patrón difuso citoplasmático) son muy específicos y sensibles en la
granulomatosis de Wegener que es una vasculitis necrotizante granulomatosa con afectación ORL
(en la biopsia se ven granulomas), pulmonar (la biopsia ve tanto los granulomas como la vasculitis,
siendo, por tanto, la más rentable), renal (se ven datos de vasculitis en forma de GN siendo poco
frecuente ver granulomas), y otras zonas. Su tratamiento consiste en esteroides y
CICLOFOSFAMIDA. Presta atención a los efectos secundarios.(R3)

320. Paciente mujer de 45 años de edad, con Papanicolacu positivo a lesión intraepitelial
alto grado. ¿cuál es la conducta más apropiada a seguir?

1. 1. Biopsia dirigida por colposcopia.


2. 2. Crioterapia.
3. 3. Conización.
4. 4. Repetir el Papanicolaou.
Gráfico de respuestas
Comentario

Hay diferentes clasificaciones de la neoplasia intraepitelial: unas basadas en la citología y otras en


la histología; sin embargo, existe el consenso generalizado para la aceptación de las mismas. El
tratamiento de las lesiones de alto grado requiere una correlación citológica previa adecuada,
colposcópica e histopatológica, así como un manejo conservador escisional. El material requerido
es un cono cervical.(R3)

321. La vagina mantiene normalmente su humedad por lo siguiente:

1. 1. Las glándulas de su mucosa y la secreción cervical.


2. 2. Descamación epitelial y secreción de las glándulas de Skene.
3. 3. Las secreciones del cérvix y trasudado de la superficie vaginal.
4. 4. Presencia del bacilo de Döderlein y acido pirúvico.

!
!
!
!
Gráfico de respuestas
Comentario

La vagina no posee glándulas en su mucosa (respuesta 1 incorrecta). Su humedad procede de la


secreción del endocérvix, que sí posee glándulas mucosas, y del trasudado de líquido a través del
epitelio vaginal (respuesta 3 correcta).

-La respuesta 2 es falsa porque la descamación epitelial no da lugar a líquido, sólo a células
descamadas; y las glándulas de Skene desembocan en la salida de la uretra, no en la vagina; por
tanto no contribuyen al flujo vaginal.

-La respuesta 4 es falsa, porque el bacilo de Döderlein (Lactobacillus acidophilus) es responsable


de la acidez del flujo vaginal, pero no produce ese flujo.(R3)

322. Un recién nacido, hijo de madre diabética, no ha expulsado meconio tras 72 horas
de vida. ¿Cuál es el cuadro que con más frecuencia se asocia a esta situación?:

1. 1. Fibrosis quística.
2. 2. Síndrome de colon izquierdo hipoplásico.
3. 3. Atresia anal.
4. 4. Atresia duodenal.
Gráfico de respuestas
Comentario

El hijo de madre diabética es con frecuencia macrosómico, por la acción de la insulina sobre los
tejidos fetales, ya que actúa como factor de crecimiento. Aparte de la macrosomía, pueden
asociarse otras anomalías más o menos frecuentes, como son:

- Colon izquierdo hipoplásico.

- Estenosis subaórtica hipertrófica.

- Hipoglucemia.

- Policitemia.

- Mayor frecuencia de ictericia.

- Trombosis de la vena renal.

- Hipocalcemia.

- Hipomagnesemia.

- Hidramnios.

!
!
!
!
La clave de la pregunta, por tanto, es el antecedente de diabetes materna. Las otras enfermedades
también podrían justificar una lenta eliminación del meconio, pero no se relacionarían con la
diabetes.(R2)

323. Mujer de 32 años diagnosticada hace dos años de hipertensión pulmonar primaria,
en clase funcional II, con prueba a vasodilatadores pulmonares de acción corta negativa.
¿Qué tratamiento indicaría?:

1. 1. Adenosina i.v.
2. 2. Antagonistas canales de calcio.
3. 3. Prostaciclina.
4. 4. Warfarina (INR:2-3).
Gráfico de respuestas
Comentario

Con esta pregunta podemos repasar el algoritmo terapéutico de la HAP. En primer lugar se realiza
una prueba de vasorreactividad con fármacos vasodilatadores de acción corta (adenosina iv o
prostaciclina iv u óxido nítrico inhalado): a) Si la respuesta es positiva (reducción considerable de
las resistencias vasculares pulmonares de forma sostenida) el fármaco de elección es un calcio
antagonista (nifedipino o diltiazem) b) Si la respuesta es negativa (reducción significativa pero
transitoria o mínima- moderada) la elección del fármaco depende de la clase funcional; ante clase
funcional I- II pautaremos anticoagulantes orales (warfarina) y ante clase funcional III- IV
pautaremos prostaciclina iv. Señalar que en todo paciente podríamos pautar anticoagulante oral
con intención profiláctica de episodios trombóticos microvasculares y concretamente la warfarina
porque ha demostrado mejorar la supervivencia.(R4)

324. Paciente de 31 años, primigesta de 38 semanas de amenorrea que acude a consulta


por presentar desde el día anterior cefalea frontal que no cede con paracetamol. En la
exploración destaca la existencia de dolor epigástrico y TA de 150/100 mmHg.
Laboratorio: Hb 11.8 g/dl, leucocitos: 12,500/mm3, plaquetas: 100,000, ácido úrico: 8.2
mg/dl y creatinina: 1.4 mg/dl. Mientras está recogiendo en la historia todos estos datos,
la paciente presenta una convulsión generalizada. ¿Cuál de los siguientes pasos que se
refieren a continuación NO está indicado ante esta situación?

1. 1. Colocar a la paciente en una habitación oscura para minimizar los estímulos externos.
2. 2. Inyectar inmediatamente diacepam intravenoso a ritmo lento para yugular la convulsión.
Administración de furosemida de forma sistemática para evitar posibles complicaciones
3. 3.
como el edema agudo de pulmón.
4. 4. Terminar la gestación por la vía menos perjudicial en el plazo más corto posible.
Gráfico de respuestas
Comentario

Concepto básico en el bloque de Ginecología y en esta respuesta no debe saber solo sulfato de
magnesio hay que valorar otras cosas. En un caso de preeclampsia, no debemos administrar
furosemida de forma sistemática (respuesta 3 falsa). Piense que este fármaco, al ser un diurético,
disminuye la volemia y dificulta el intercambio úteroplacentario. Por este motivo, es preferible el uso
de algún vasodilatador arterial tipo hidralacina.(R3)

325. Niño, de 5 años de edad, que acude por presentar cuadro febril y odinofagia con
exudado blanquecino en ambas amígdalas que su pediatra trató con penicilina oral. Dos
días después, presenta petequias y púrpura palpables, principalmente en miembros

!
!
!
!
inferiores, así como artralgias en ambos tobillos que le impiden la marcha y dolor
abdominal de tipo cólico. Los estudios complementarios revelan 13 g/dl de Hb; leucocitos
10,500/mm3 con fórmula normal; plaquetas 485,000/mm3; tiempo de protrombina de 95%
y tiempo parcial de tromboplastina de 27 segundos (control 25 segundos). La causa más
probable de estos hallazgos es:

1. 1. Púrpura de Schönlein-Henoch.
2. 2. Mononucleosis infecciosa.
3. 3. Enfermedad de Kawasaki.
4. 4. Púrpura trombocitopénica idiopática.
Gráfico de respuestas
Comentario

Pregunta muy sencilla sobre una enfermedad típica en la infancia, que aparece posterior a cuadros
infecciosos de vías respiratorias altas, la púrpura de Schönlein-Henoch. Llama la atención la
presentación clínica de petequias, púrpura, artralgias, dolor abdominal, plaquetas normale sy
tiempos de coagulación normales. Respuesta correcta 1.(R1)

326. ¿Cuál de estos grupos NO pertenece a la familia Picornaviridae?:

1. 1. Poliovirus.
2. 2. Echo.
3. 3. Adenovirus.
4. 4. Rhinovirus.
Gráfico de respuestas
Comentario

Pregunta difícil si no estamos relacionados con la taxonomía viral básica.

Entre los picornaviridae encontramos dos géneros: los rinovirus y los enterovirus. Estos últimos
incluyen los Echo, los Coxsackie, los Poliovirus, el VHA y los "nuevos enterovirus". Todos ellos son
RNA virus. En cambio los Adenovirus son DNA virus y tienen su propia familia (Adenoviridae).(R3)

327. ¿Cuál es el tratamiento del chancroide?

1. 1. Penicilina Benzatínica 2 400 00 UI monodosis.


2. 2. Penicilina G 1 000 000 por 3 días.
3. 3. Ciprofloxacino 500mg bid por 3 días.
4. 4. Azitromicina 1gr monodosis.
Gráfico de respuestas
Comentario

El chancroide, causado por Haemophilus ducreyi, produce un chancro blando y doloroso, con
exudado, que suele aparecer tras tres dias del contacto sexual. Su tratamiento puede ser con
azitromicina o preferentemente con ceftriaxona.(R4)

328. A newborn infant undergoes a routine checkup. Physical examination is


unremarkable, except for a scrotal mass that transilluminates with light. His mother is
very concerned because she does not want surgery. What should you do next?

!
!
!
!
1. 1. Reassurance and observation.
2. 2. Testicular ultrasound.
3. 3. Aspiration of fluid.
4. 4. Surgery.
Gráfico de respuestas
Comentario
Reassurance and observation. This patient most likely has a hydrocele, a fluid-filled sac
surrounding a testicle that causes swelling in the scrotum. Hydrocele is common in newborns and
usually disappears without treatment during the first year of life.(R1)

329. Ante un paciente que presenta una herida por arma blanca penetrante en abdomen,
con evisceración de varias asas de intestino delgado, ¿qué actitud tomaría?:

1. 1. Observación durante 24-48 h.


2. 2. Laparotomía.
3. 3. Laparotomía, sólo si el enfermo presenta choque.
4. 4. Lavado peritoneal diagnóstico.
Gráfico de respuestas
Comentario

Pregunta muy sencilla, que no puede fallar, después de un trauma abdominal por arma blanca
penetrante con evisceración el tratamiento de elección es la laparotomía, ya que es una urgencia
médico-quirúrgica.(R2)

330. En lo que a la arteriosclerosis se refiere NO es cierto:

1. 1. Es la causa más frecuente de arteriopatía obstructiva crónica de los miembros inferiores.


2. 2. La afectación arteriosclerítica de los miembros superiores es muy rara.
3. 3. El ejercicio físico es un factor que ayuda a eliminar el colesterol.
4. 4. El aceite de pescado es muy aterogénico.
Gráfico de respuestas
Comentario
La aterosclerosis es un proceso degenerativo de la capa íntima de las arterias de mediano y gran
calibre, caracterizado por la formación de placas constituidas por la proliferación de células
musculares, tejido colágeno y acumulación intra y extracelular de lípidos. En cuanto a su
localización, las lesiones suelen estar irregularmente distribuidas; la aorta es la más precoz e
intensamente afectada y la zona de la salida de las ramas principales, afectándose más las arterias
de los MMII que las de los MMSS, en especial la femoral y la poplítea. Puesto que parece que la
proporción de grasas animales (ricas en ácidos grasos saturados) sobre las vegetales (ricas en
ácidos grasos insaturados) tiene un papel definido en la patogenia de esta entidad, las
recomendaciones terapéuticas aconsejan la disminución de la ingesta de grasas saturadas y
colesterol, sustituyéndolas por grasas insaturadas, como las del aceite de pescado.(R4)

331. ¿Cuál de los siguientes fármacos debería evitarse en el tratamiento la esofagitis por
reflujo gastroesofágico?

1. 1. Rabeprazol.
2. 2. Ranitidina.
3. 3. Almagato.
4. 4. Nifedipino.

!
!
!
!
Gráfico de respuestas
Comentario

Los calcioantagonistas relajan el esfínter esofágico inferior y producen reflujo.(R4)

332. A 36-month-old girl is brought to the pediatrician's office for a routine visit. The
doctor notices that the child's growth is markedly diminished according to height and
weight for her age (under the 3rd percentile). Physical examination shows diffuse
ecchymoses on her limbs. Ronchi are heared on lung auscultation. What is the underlying
cause of this patient's growth failure?

1. 1. Child abuse.
2. 2. Scurvy.
3. 3. Rickets.
4. 4. Pancreatic insufficiency.
Gráfico de respuestas
Comentario
Pancreatic insufficiency. The correct diagnosis would be cystic fibrosis. Cystic fibrosis (CF) is a
chronic, multisystemic disorder characterized by recurrent endobronchial infections, progressive
obstructive pulmonary disease and pancreatic insufficiency with intestinal malabsorption. Fat-
soluble vitamins, A, D, E, K are expected to be deficient if not adequately supplemented. Growth
deficiency is due to her disease and the frequent bruises are explained by the lack of vitamin K.
Supplements of vitamins and fats must be initiated immediately.(R4)

333. Paciente de 42 años con infección por el virus de la hepatitis C, presenta proteinuria
nefrótica, edemas, hipocomplementemia y crioglobulinemia. ¿Cuál es el diagnostico
histológico más probable?:

1. 1. GN mesangial IgA.
2. 2. GN membranoproliferativa tipo I.
3. 3. GN aguda postinfecciosa.
4. 4. Hialinosis focal.
Gráfico de respuestas
Comentario

Caso clínico de dificultad moderada que se puede resolver siendo sistemático en el análisis de los
datos. El nivel de complemento es un buen dato para empezar a descartar opciones, puesto que
es muy discriminante en GN. Como recuerda, las patologías renales hipocomplementémicas son:
GN aguda postinfecciosa (en este caso transitoria y activación por la vía alterna), GNMP o
mesangiocapilar (es su tipo I la activación es por la vía clásica y en el II por vía alterna), GNRP tipo
II, LES, shunt, sepsis, endocarditis y crioglobulinemia mixta esencial. Además, la enfermedad por
émbolos de colesterol también cursa con complemento disminuído. La GN asociada al VHC es la
GNMP (RESPUESTA 2 CORRECTA), que cursa con síndrome nefótico en el 66% de los casos,
pudiéndose añadir microhematuria, croiglobulinemia y fallo renal. Las etiologías que debe recordar
de la GNMP son: las enfermedades autoinmunes (LES, sarcoidosis, etc.), las infecciones (VHC,
VHB, endocarditis, parásitos, VIH, etc.), tumores (mieloma, leucemia linfática crónica, linfoma no
Hodgkin, etc.).(R2)

334. ¿Cuál de las siguientes lesiones asociadas y complicaciones de la luxación de codo


NO es frecuente que aparezca en un adulto?:

!
!
!
!
1. 1. Recidiva de la luxación.
2. 2. Calcificaciones heterotópicas.
3. 3. Fractura de la apófisis coronoides.
4. 4. Neuroapraxia del nervio cubital o del mediano.
Gráfico de respuestas
Comentario
Las luxaciones de codo pueden ocasionar secundariamente rigidez, calcificaciones heterotópicas,
fractura de la apófisis coronoides y de la cabeza del radio (en el momento de la luxación como
asociación de lesiones), y neuroapraxia de los nervios cubital o del mediano. La recidiva de la
luxación, frecuente en el caso del hombro, es excepcional en el codo y cuando existe suele
asociarse a graves lesiones de partes blandas e inestabilidad.(R1)

335. A 5-year-old Somalian girl is brought to the doctor's clinic by her mother presenting
with fever, vomiting and fatigability. She is refusing to eat. Vital signs are 39ºC
temperature, 70/50 mmHg blood pressure, 100 bpm. Hematocrit is 20% and reticulocyte
count 10%. The girl has not been adequately vaccinated. Which of the following could
have prevented this condition?

1. 1. Vaccination with a conjugated capsular polysaccharide.


2. 2. Bone marrow transplant.
3. 3. Prophylactic antibiotherapy.
4. 4. Vaccination with an attenuated virus.
Gráfico de respuestas
Comentario
Vaccination with a conjugated capsular polysaccharide. Haemophilus inflenzae type b,
meningococcus and pneumococcus can cause serious infections with bacteremia and sepsis, like
the case described. Conjugated capsular polysaccharide vaccines have reduced the incidence of
such bacterial infections in the past years.(R1)

336. La enfermedad de membrana hialina afecta principalmente al recién nacido:

1. 1. A termino.
2. 2. De madre hipertensa.
3. 3. De madre adicta a opiáceos.
4. 4. Prematuro.
Gráfico de respuestas
Comentario

La enfermedad de membrana hialina es la causa más frecuente de distrés respiratorio en el recién


nacido pretermino. Se debe a un déficit de surfactante, que no alcanza la superficie pulmonar hasta
la semana 34-35. Este déficit causa un aumento de tensión superficial y una tendencia al colpso
pulmonar.

En el bebé a término, hijo de madre hipertensa e hijo de madre adicta a los opiaceos, ocurre lo
contrario, se reduce el riesgo de sufrir enfermedad de la membrana hialina.(R4)

337. ¿Dónde se produce la 16-alfa-hidroxilación de la hormona dehidroepiandrosterona-


sulfato, paso necesario para la síntesis de estriol por la placenta?

!
!
!
!
1. 1. En el sincitiotrofoblasto.
2. 2. En las glándulas suprarrenales fetales.
3. 3. En las glándulas suprarrenales maternas.
4. 4. En el hígado fetal.
Gráfico de respuestas
Comentario

El estriol se utiliza como marcador de bienestar fetal. Para su producción, es necesaria la


integridad de tres órganos: el hígado, las suprarrenales del feto y la placenta. Para su síntesis,
inicialmente se ha de producir un precursor llamado dehidroepiandrosterona-sulfato en las
glándulas suprarrenales fetales, que a continuación sufre una reacción de 16-alfa-hidroxilación en
el hígado fetal. La molécula obtenida es aromatizada en la placenta, para dar estriol.

En cualquier caso, esta pregunta es mucho más difícil de lo que suelen exigir en el ENARM. Lo
primordial es que recuerde que es el FETO el que produce el estriol, y que un descenso de esta
hormona suele implicar algún tipo de anomalía (cromosomopatías, por ejemplo, como sucede en el
síndrome de Down).(R4)

338. Señale cuál de las siguientes NO es causa de fontanela anterior grande:

1. 1. Síndrome de Aport.
2. 2. Síndrome de Russell-Silver.
3. 3. Hipertiroidismo.
4. 4. Síndrome de Hallerman-Streiff.
Gráfico de respuestas
Comentario

La presencia de ensanchamiento fontanelar puede asociarse con diversas enfermedades que


cursen con osteopatía (raquitismo, síndrome de Aport - o acrocéfalosindactilia- , el sindrome de
Silver-Rusell - displasia fibrosa poliostótica con manchas café con leche y pubertad precoz- , la
osteogénesis imperfecta, síndrome de Hallerman, etc) y otras (hipotiroidismo, rubéola congénita,
hipertensión intracraneal). NO EL HIPERTIROIDISMO.(R3)

339. ¿Cuál de los siguientes tumores da generalmente metástasis óseas con imágenes
osteoblásticas?:

1. 1. Adenocarcinoma de mama.
2. 2. Adenocarcinoma de próstata.
3. 3. Linfomas.
4. 4. Adenocarcinoma de pulmón.
Gráfico de respuestas
Comentario
La imagen radiológica más frecuente de una metástasis ósea es la lesión lítica. El tumor que más
frecuentemente produce metástasis con imagen osteoblástica es el adenocarcinoma de próstata.
Otros tumores como el adenocarcinoma de mama y los linfomas pueden presentar un patrón mixto
osteolítico y osteoblástico.(R2)

340. Señale la opción INCORRECTA acerca de la epidemiología de las bronquiolitis:

1. 1. La inmensa mayoría están producidas por el VRS.

!
!
!
!
Cuando afecta a niños previamente sanos, la mortalidad ronda el 1%, pero en niños con
2. 2.
patología de base, como cardiopatía congénita o inmunodeficiencias sube hasta el 30%.
Presenta un patrón estacional típico, apareciendo al final del otoño y principio del
3. 3.
invierno.
Otros agentes etiológicos serían, en orden de frecuencia, Parainfluenzae, Influenzae y
4. 4.
Adenovirus.
Gráfico de respuestas
Comentario

De la epidemiología de las bronquiolitis lo más importante es saber la etiología más frecuente y su


típico carácter estacional. No preste atención a los porcentajes que le dan. El agente etiológico
más frecuente es el VRS, aunque otros virus como el Parainfluenziae, Influenzae y Adenovirus son
también causas frecuentes. Más raros son Rinovirus y Metapneumovirus y de forma muy
infrecuente: Mycoplasma pneumoniae. Recuerde que la mayoría de los casos se producen en
invierno o a principio de primavera (respuesta 3 falsa) cuando los agentes virales asociados a este
proceso son más prevalentes.(R3)

341. Paciente
tercigesta con antecedente de dos partos eutócicos en la semana 35 y 34 acude en la
semana 31 a urgencias del hospital por percibir contracciones. Exploración: cérvix
permeable 1 dedo, borrado 50%, consistencia media, posición media. Feto longitudinal
podálica. Se realiza RCTG que se muestra en la imagen. Longitud cervical por USG
transvaginal 20 mm. Test de la fibronectina positivo. ¿Cuál sería la actitud más
CORRECTA en este caso?:

1. 1. Mantener a la paciente en observación y reevaluar en 2 horas.


2. 2. Iniciar tocólisis con atosibán y maduración pulmonar con betametasona.
3. 3. Ya se ha iniciado el parto, dejar evolucionar parto vía vaginal.
4. 4. Ya se ha iniciado el parto, realizar una cesárea, ya que la presentación es podálica.
Gráfico de respuestas
Comentario

Debe dominar este tipo de preguntas de ginecología en el ENARM recuerde que son
aproximadamente 100 preguntas en este bloque. A pesar de que la dinámica uterina es de parto, el
cérvix todavía no lo es totalmente. Teniendo en cuenta que el embarazo todavía no ha llegado a su
semana 34, debemos tratar de frenar el parto e iniciar maduración pulmonar. A partir de la semana

!
!
!
!
34, no sería necesario utilizar corticoides para la maduración pulmonar fetal, pero antes de este
momento sí lo es, como en este caso. Por último, entre los betamiméticos y el atosibán, en la
actualidad se prefiere este último.

No olvide que el atosibán es un inhibidor competitivo de la oxitocina, de ahí su carácter tocolítico.


Es tan eficaz como la ritodrina (betamimético clásicamente utilizado), pero sus efectos adversos
son menores, por lo que resulta preferible.(R2)

342. Lactante de 3 meses de edad que acude a la consulta por presentar desde los 15
días de vida, regurgitaciones pospandriales en casi todas las tomas y de forma
esporádica, vómitos en mayor cantidad. Heces de características normales. Lactancia
artificial con fórmula de inicio y presenta un adecuado desarrollo ponderoestatural, sin
pérdida de peso. ¿Cuál entre los siguientes, es el diagnóstico más probable?

1. 1. Alergia a la proteína de leche de vaca.


2. 2. Estenosis pilórica.
3. 3. Reflujo gastroesofágico madurativo.
4. 4. Vólvulo intestinal.
Gráfico de respuestas
Comentario

Pregunta con un caso clínico típico de reflujo gastroesofágico madurativo o fisiológico.

La manifestación clínica más usual es la regurgitación en la mayoría de las tomas a las pocas
semanas de vida (la mayoría empiezan ya en la primera semana). Dado que el paciente presenta
un adecuado desarrollo pondoestatural, hablamos de un reflujo gastroesofágico fisiológico debido a
un esfínter esofágico inferior incompetente por inmadurez.

La estenosis pilórica es un cuadro mucho más agudo, con vómitos proyectivos no biliosos, que
comienza en torno a las tres semanas de vida con riesgo de deshidratación para el neonato.

La opción 1 hace referencia a reacciones adversas frente a la proteína de la leche de vaca, y por
ello no guardarían tan buena evolución: se afectarían el peso y la talla. En la intolerancia, es
común el hallazgo de heces con sangre. La alergia puede provocar urticaria, angioedema,
broncoespasmo, e incluso anafilaxia como forma grave.

Por último, el vólvulo intestinal, se presentaría como un cuadro obstructivo y clínica de peritonismo.
No permitiría una evolución tan solapada en el tiempo (hablan de dos meses y medio), sino mucho
más aguda.(R3)

343. Uno de los siguientes procesos cutáneos NO se ha relacionado en su patogenia con


los andrógenos:

1. 1. Adenoma sebáceo.
2. 2. Alopecia común.
3. 3. Seborrea.
4. 4. Hirsutismo.
Gráfico de respuestas
Comentario

!
!
!
!
El adenoma sebáceo es un tumor benigno compuesto de células sebáceas parcialmente
diferenciadas con distintos grados de maduración. Es un tumor poco frecuente. El tipo solitario
puede aparecer en cualquier sexo y es más frecuente en los ancianos. Cuando son múltiples se
asocian con el síndrome de Muir-Torre. Clínicamente es una lesión redondeada, elevada, sésil o
pediculada, de color piel o ligeramente amarillenta. La superficie puede ser verrucosa y
generalmente miden menos de 10 mm. La localización más frecuente es la cara o el cuero
cabelludo. El tratamiento de elección es la cirugía.(R1)

344. ¿Cuál de las siguientes NO es una forma de presentación del cáncer de mama?

1. 1. Eccema de areola-pezón.
2. 2. Inflamación generalizada de la mama.
3. 3. Adenopatía axilar.
4. 4. Nódulo con imagen mamográfica de calcificación semilunar o en “cáscara de huevo”.
Gráfico de respuestas
Comentario

La respuesta falsa es la 4 porque es una imagen mamográfica sugerente de benignidad. Las


calcificaciones que sugieren malignidad son: las microcalcificaciones, sobre todo si están
agrupadas y con características peculiares (amorfas, pleomórficas, lineales y ramificadas).

Las respuestas 1, 2 y 3 son formas de presentación del cáncer de mama; sin embargo, la forma
más frecuente es la presencia de un tumor o induración, que se palpa como un nódulo. La
inflamación generalizada de la mama es una forma de presentación cada vez menos frecuente y
que indica la existencia de un proceso muy avanzado (puede verse, de forma típica, en el
carcinoma inflamatorio, de muy mal pronóstico). El eccema de areola-pezón, presente en el 1-3%
de cánceres de mama, corresponde a la enfermedad de Paget de la mama.(R4)

345. El diámetro SOB (suboccipitobregmático) mide aproximadamente:

1. 1. 12 cm.
2. 2. 9.5 cm.
3. 3. 13.5 cm.
4. 4. 10.5 cm.
Gráfico de respuestas
Comentario

El diámetro suboccipito-bregmático es la distancia desde la región suboccipital hasta el bregma o


fontanela menor. El tamaño en los fetos a término es de aproximadamente 9.5 cm. Es el diámetro
anterioposterior presentado cuando la cabeza fetal está flexionada. Es más pequeño que otros
diámetros lo que justifica que la flexión de la cabeza favorezca la buena evolución del expulsivo
fetal. Recuerde que en los casos extremos de deflexión de la cabeza, como son la frente y la cara
mentoposterior, se contraindica la vía vaginal.(R2)

346. Lactante de 5 meses que presenta, desde 3 días antes, fiebre, rinorrea acuosa y
estornudos, comenzando el día de la consulta con tos y dificultad respiratoria. En la
exploración presenta taquipnea, tiraje subcostal, alargamiento de la espiración y
crepitantes y sibilancias diseminados. ¿Cuál es el diagnóstico que cree más probable
entre los siguientes?:

1. 1. Neumonía.

!
!
!
!
2. 2. Fibrosis quística.
3. 3. Bronquiolitis.
4. 4. Cuerpo extraño bronquial.
Gráfico de respuestas
Comentario

La bronquiolitis se define como el primer episodio de dificultad respiratoria con sibilancias, de


causa infecciosa, en un niño de menos de dos años de edad. Constituye la causa más frecuente
de enfermedad y de hospitalización en menores de un año.

El VSR es el agente causal más frecuente, seguido del metapneumovirus.

El cuadro típico es el de un lactante con infección de vías respiratorias altas en las 24-72 horas
previas, que comienza con dificultad respiratoria progresiva, tos seca, fiebre y rechazo al
alimento.(R3)

347. ¿Cuál de las siguientes afirmaciones respecto a la menopausia es FALSA?:

1. 1. Hay agotamiento de los folículos primordiales del ovario.


2. 2. La estrona pasa a ser el estrógeno más importante de la posmenopausia.
3. 3. La producción de progesterona es baja.
4. 4. La FSH y LH han descendido.
Gráfico de respuestas
Comentario

Pregunta bastante sencilla sobre la fisiología de la menopausia. En la postmenopausia se produce


un agotamiento de los folículos primordiales, con lo que la producción de estradiol disminuye
notoriamente, pasando la estrona a ser el estrógeno fundamental de la menopausia; esta
disminución de estradiol dispara el feed-back negativo de la hipófisis produciéndose así un
incremento de las gonadotropinas FSH y LH (opción 4 la que debemos elegir), el perfil hormonal
típico cuenta con aumento de estas. Al no haber ciclos ovulatorios, la producción de progesterona
será muy pequeña.(R4)

348. Señale lo CORRECTO en relación a la edad gestacional por examen físico del RN:

El test de Ballard considera el examen físico y neurológico del RN y puede hacerse a


1. 1.
partir de las 24 hrs de vida.
La edad gestacional por examen físico es menos exacta que la edad gestacional por FUR
2. 2.
en una madre que ha iniciado control prenatal al cuarto mes del embarazo.
La edad gestacional por Capurro tiene una variación de + ó – 2 semanas y puede hacerse
3. 3.
máximo hasta las 12 horas de vida.
4. 4. Ninguna de las anteriores.
Gráfico de respuestas
Comentario

El Método de Ballard Modificado utiliza siete signos físicos (piel, lanugo, superficie plantar, mama,
ojo/oreja, genitales masculinos, genitales femeninos) y seis signos neuromusculares (postura,
ventana cuadrada de la muñeca, rebote de brazos, ángulo poplíteo, signo de la bufanda, talón
oreja). El valor se compara con una escala de madurez que establece las semanas de edad
gestacional.

!
!
!
!
De acuerdo con los hallazgos se clasificarán de la siguiente manera:

· Pretérmino: De 28 a menos de 37 semanas, o de 10 a 30 puntos; se transfiere para continuar la


atención en el nivel hospitalario y/o terapia intensiva de acuerdo con su condición.

· Término: De 37 a menos de 42 semanas, o de 35 a 40 puntos; si las condiciones lo permiten,


pasará con su madre a alojamiento conjunto, e iniciará lactancia materna exclusiva.

· Postérmino: De 42 semanas o más, también de 45 a 50 puntos; debe observarse durante las


primeras 12 horas ante la posibilidad de presentar hipoglicemia o hipocalcemia. Pasado el periodo,
si sus condiciones lo permiten, pasará con su madre a alojamiento conjunto, e iniciará lactancia
materna exclusiva.(R3)

349. El signo de Cullen, de considerable valor diagnóstico, pero de aparición tardía, en la


pancreatitis aguda, se caracteriza por:

1. 1. Trombocitopenia.
2. 2. Derrame pleural.
3. 3. Infiltración hemática periumbilical.
4. 4. Esteatomecrosis cutánea.
Gráfico de respuestas
Comentario

En las pancreatitis agudas graves, fundamentalmente en las formas necrohemorrágicas, la


hemorragia retroperitoneal que asocia la necrosis pancreática puede ponerse de manifiesto de
varias maneras. En ocasiones, se produce un cuadro de hipotensión, pero otras veces se
acompaña de infiltración hemática tisular, que diseca el tejido celular subcutáneo y se manifiesta
con una coloración azulada o equímosis en los flancos (signo de Grey Turner), el área umbilical
(signo de Cullen) o del ligamento inguinal (signo de Fox). Ninguno de estos signos es frecuente;
pero, cuando aparecen, su traducción es ésta.(R3)

350. De las siguientes técnicas diagnósticas, ¿cuál se debe incluir obligatoriamente en el


estudio inicial de una pareja estéril?:

1. 1. Cariotipo.
2. 2. Biopsia endometrial.
3. 3. Test postcoital.
4. 4. Histerosalpingografía.
Gráfico de respuestas
Comentario

Dentro de las pruebas diagnósticas de la pareja estéril el cariotipo se realiza en caso de sospecha
de anomalías genéticas, la biopsia endometrial y el test postcoital son pruebas en desuso y no se
incluyen de forma rutinaria. La histeroscopía está indicada en pacientes con alteraciones en la
HSG, abortos de repetición o partos inmaduros.(R4)

351. En el recién nacido señale que reflejos deben estar presentes al nacimiento.

1. 1. Reflejo de marcha y de moro.


2. 2. Reflejo de moro y de succión.
3. 3. Reflejo de succión.

!
!
!
!
4. 4. Reflejo de paracaídas y de moro.
Gráfico de respuestas
Comentario

El reflejo de moro y de succión son los típicos que se deben explorar en el recién nacido y deben
estar presentes.(R2)

352. A 42-year-old woman with chronic C hepatitis virus infection complains of asthenia,
arthralgias, low-grade fever, abdominal pain, dyspnea at rest and paresthesias in her
upper limbs. Her body temperature is 37.8ºC, blood pressure is 140/95 mm Hg, pulse is
60/min and respirations are 20/min. Which of the following is the most likely diagnosis?

1. 1. Type 2 cryoglobulinemia.
2. 2. Waldeström's macroglobulinemia.
3. 3. Systemic lupus erythematosus.
4. 4. Chronic lymphocytic leukemia.
Gráfico de respuestas
Comentario
Type II cryoglobulinemia. Mixed cryoglobulinemia (MC) type II is a disorder characterized by
circulating cold-precipitable immune complexes composed of polyclonal immunoglobulin IgG and
monoclonal IgM rheumatoid factor (RF). The clinical syndrome of MC is characterized by a systemic
vasculitis involving mostly small, but sometimes larger vessels. CLINICAL MANIFESTATION>
Typical triad—purpura, arthralgia, and weakness. More severe cases may have neurologic and
renal involvement.(R1)

353. Lactante de 15 meses de edad que


acude a consulta por fiebre de 6 días de evolución de 39.5ºC asociado a exantema que se
muestra en la imagen ##. Se detecta en la región cervical derecha una adenopatía de
aproximadamente 2 cm de diámetro y en la cavidad oral, la lengua es de color rojo
intenso, además de molestias con la deglución. Se realiza un test rápido de
Streptococcus pyogenes que resulta negativo. Respecto a la patología que sospecha
señale la opción CORRECTA:

1. 1. Debe tratarse de una escarlatina con un falso negativo en el test rápido.


2. 2. Debe iniciarse tratamiento con ácido acetilsalicílico e inmunoglobulina intravenosa.

!
!
!
!
3. 3. Las lesiones de las manos son características de la infección por Coxsackie.
4. 4. El exantema no afecta a cara ni partes distales y apareció al quitarse la fiebre.
Gráfico de respuestas
Comentario

La pregunta hace referencia a una enfermedad de Kawasaki observándose en las fotos edema y
eritema en palmas. Descartándose otras patologías cumple los criterios diagnósticos: fiebre de más
de 5 días más 4 de los otros 5 criterios: A de Adenopatía mayor de 1.5 cm. B de Boca: lengua
aframbuesada e hiperemia faríngea sin exudado, D Distal: afectación de palmas y plantas. E de
exantema y también podría haberse presentado la C de Conjuntivitis bilateral no supurativa. Las
otras opciones hacen referencia a varicela, enfermedad boca-mano-pie (opción 3), exantema
súbito (opción 4). Por lo tanto la respuesta correcta es la 2, que es el tratamiento del Kawasaki.(R2)

354. Respecto a la patología del paciente de la pregunta anterior, señale la respuesta


INCORRECTA:

1. 1. Su etiología se relaciona con una respuesta inmunológica mediada por superantígenos.


2. 2. El tratamiento de primera línea es con aspirina a dosis antiagregantes y corticoides.
3. 3. Se produce una vasculitis sistémica de causa desconocida.
4. 4. Es frecuente que se presente con trombocitosis.
Gráfico de respuestas
Comentario

La enfermedad de Kawasaki es una vasculitis sistémica de origen desconocido, se cree es


producida por una infección por coronavirus New Haven, que desencadena una respuesta
inmunológica mediada por superantígenos, los cuales produce daño endotelial, pudiendo afectar a
las arterias coronarias.

En los exámenes de laboratorio se puede encontrar trombocitosis, además de un aumento en los


reactantes de fase aguda, así como leucocitosis.

El tratamiento se inicia con inmunoglobulina iv en los primeros 10 días para prevenir la aparición de
aneurismas, asociado a aspirina, en un inicio a dosis ANTIINFLAMATORIAS y posteriormente a
dosis antiagregantes.(R2)

355. Nos traen a un paciente a urgencias que ha consumido una droga que no se sabe
con certeza cuál es. El paciente se muestra desinhibido, irritable, con sensación de
grandiosidad, diciendo que ha creado un virus informático y que le van a pagar 1.5
millones de euros por el antivirus, con discreta inquietud psicomotriz y contusiones por
haberse peleado. Lo más probable es que haya consumido:

1. 1. Cannabis.
2. 2. Heroína.
3. 3. LSD (dietilamida del ácido lisérgico).
4. 4. Cocaína.
Gráfico de respuestas
Comentario

Merece la pena conocer algunas complicaciones de la intoxicación por cannabis, al ser la sustancia
ilegal más frecuentemente consumida. Aunque no es frecuente, puede producir episodios
psicóticos muy floridos. También puede producir ilusiones, desrealización y despersonalización con

!
!
!
!
disminución en la atención y concentración. Recuerde que los dos signos físicos más frecuentes de
la intoxicación son la inyección conjuntival y la taquicardia.

En esta pregunta, es posible que hayas considerado la cocaína. De hecho, es posible que la
cocaína produjese un cuadro como éste. Sin embargo, el consumo de hachís es mucho más
frecuente que el de cocaína. Por tanto, aunque este cuadro (grandiosidad, irritabilidad, etc.) sería
más típico de un cocainómano, es más frecuente verlo en el contexto del hachís, por su mayor
frecuencia de consumo. Dese cuenta de que el enunciado pide “lo más probable”.(R1)

356. La malaria es una enfermedad parasitaria poco frecuente en nuestro país, pero en
los últimos años estamos asitiendo a un resurgir de casos debidos a la inmigración y a
los viajes a países tropicales. En relación a la malaria, cuál de las siguientes afirmaciones
es FALSA:

1. 1. La gravedad de la infección está en relación con el grado de parasitemia.


2. 2. La infección por Plasmodium falciparum es la más grave.
3. 3. La malaria cerebral es una manifestación típica de Plasmodium vivax.
4. 4. El dato analítico más frecuente es una anemia normocítica.
Gráfico de respuestas
Comentario

La primera es cierta: más del 5% de los hematíes parasitados es un dato de mayor gravedad.

•! La segunda es cierta: la infección por P. falciparum es la más grave.


•! La tercera es falsa: el que produce afectación cerebral, que es muy grave, es el P.
falciparum.
•! La cuarta es cierta: el dato analítico más frecuente es una anemia normocítica, puesto que
se trata de una anemia hemolítica.(R3)

357. Señale la FALSA respecto a las posibles alteraciones de laboratorio en la infección


por leptospira en un inmunodeprimido:

1. 1. Bilirrubina elevada, a costa de la fracción conjugada, sobre todo.


2. 2. Elevación de la ALT y AST.
3. 3. Fosfatasa alcalina poco o nada elevada.
4. 4. Aumento de la CPK.
Gráfico de respuestas
Comentario

Pregunta muy difícil sobre aspectos de laboratorio de la afectación hepática de la leptospira en un


caso muy concreto, no se preocupe si ls ha fallado, no es un tema muy preguntado en el ENARM.

La leptospirosis produce un cuadro de afectación de mialgias con incremento de la CPK (opción 4)


y hepática que se caracteriza por el aumento de transaminasas (opción 2 correcta). A diferencia de
la hepatitis vírica aguda, en la leptospirosis es característica la elevación de la bilirrubina y de la
fosfatasa alcalina en suero (opción 3 falsa). En inmunodeprimidos puede cursar como una forma
muy grave llamada "enfermedad de Weil "con lesión hepática, diátesis hemorrágica e insuficiencia
renal que justificaría la elevación de la creatinina.(R3)

!
!
!
!
358. Regarding the diagnosis of pancreatic adenocarcinoma, which of the following
statements is true?

1. 1. Tumor markers and CT scan currently allow early diagnosis of pancreatic cancer.
The symptoms are very specific and highly suggestive of the diagnosis, especially in the
2. 2.
case of tumors located in the body and tail.
3. 3. Migratory thrombophlebitis is a very common sign of pancreatic cancer.
4. 4. Sometimes, neither laparotomy nor biopsy allow the diagnosis.
Gráfico de respuestas
Comentario

En el cáncer de páncreas no es infrecuente que nos encontremos ante un paciente que tiene una
masa pancreática con mayor frecuencia a nivel de la cabeza pancreática de la que no
conseguimos una clara definición histológica a pesar de la realización de biopsias previas a la
cirugía e incluso de biopsia tomadas en el mismo acto quirúrgico, siendo el cirujano el que en la
misma intervención decide realizar una resección incluso con una técnica tan agresiva como una
duodenopancreatectomía cefálica (Whipple) sin tener certeza anatomopatológica de que se trate
de un cáncer.

El resto de opciones son falsas: Los marcadores tumorales no permiten un diagnóstico de certeza
pues no son específicos y mucho menos dan un diagnóstico precoz. Los síntomas del cáncer de
páncreas especialmente los del cáncer de cuerpo y cola: hepatomegalia, dolor abdominal masa
palpable, e ictericia no son específicos. La detección de una masa no permite un diagnóstico de
certeza que es finalmente anatomopatológico. La tromboflebitis migratoria es un síntoma muy
infrecuente en el cáncer de páncreas y aparece asociado a otros cánceres.(R4)

359. Un paciente fumador presenta disnea con moderados esfuerzos, tos y expectoración
matutina durante más de tres meses al año. Se le realiza una espirometría en la que se
obtiene un volumen espiratorio forzado en el primer segundo (FEV1) del 45% del valor
predicho, y una relación FEV1 capacidad vital forzada (FVC) de 0.56. ¿Qué grado de
obstrucción presenta, según la iniciativa Gold?

1. 1. Grado I.
2. 2. Grado 0.
3. 3. Grado IV.
4. 4. Grado III.
Gráfico de respuestas
Comentario

EPOC es el tema más preguntado en el ENARM en el bloque de neumología.

Describen las características clínicas de un paciente EPOC y nos piden el grado de obstrucción:

Grado III corresponde a FEV1/CVF< 70%, FEV1< 50%, con o sin síntomas.(R4)

360. De las siguientes afirmaciones marque la respuesta CORRECTA:

1. 1. Las complicaciones del SNC en la asfixia son más frecuentes que a nivel renal.
2. 2. Cerca de 5 millones de RN mueren al año como consecuencia de la asfixia al nacimiento.
En RCP neonatal el masaje cardiaco se recomienda cuando la FC es menor a 60 latidos x
3. 3.
minuto.

!
!
!
!
4. 4. El empleo de medicamentos es infrecuente en RCP neonatal.
Gráfico de respuestas
Comentario

La respuesta correcta es la 1, ya que la asfixia lesionará con mayor frecuencia al SNC que al riñón.
El resto de respuestas son incorrectas.(R1)

361. Una paciente de 26 años acude a la consulta por infecciones urinarias de repetición,
siete en el último año, que su médico de cabecera ha ido tratando en todos los casos
según antibiograma, con buena respuesta. Aporta cultivos, varios positivos, para E. coli
y Staphylococo saprophyticus. En este caso:

Se debe dar el alta a la paciente, ya que su médico ya ha ido tratando el problema


1. 1.
correctamente, y así puede seguir haciéndolo en el futuro.
Se debe dar un ciclo de tratamiento intravenoso para lograr la esterilización completa del
2. 2.
tracto genitourinario.
3. 3. No se deben tratar las infecciones, ya que así sólo se perpetúa el proceso.
Probablemente esta paciente pueda beneficiarse de la administración de extracto de
4. 4.
arándanos rojos.
Gráfico de respuestas
Comentario

Ante una paciente con IVU de repetición se debe realizar un estudio urológico completo.
Evidentemente se requerirá tratamiento adecuado para todas las infecciones, pero no es útil dar un
ciclo intravenoso a no ser que el estado general de la paciente o la gravedad del proceso lo
requieran. En caso de IVU de repetición parece que la terapia con arándanos rojos puede mejorar
el curso de la patología.(R4)

362. Son factores de riesgo a considerar en el cáncer de endometrio en pacientes


posmenopáusicas, EXCEPTO:

1. 1. Sangrado postmenopáusico.
2. 2. Dolor pélvico.
3. 3. Diabetes mellitus.
4. 4. Hipertensión arterial.
Gráfico de respuestas
Comentario

La obesidad, la HTA, la hipercolesterolemia, la diabetes mellitus y la nuliparidad son factores de


riesgo para padecer cancer de endometrio.

El dolo pélvico no se asocia a cáncer de endometrio, ya que en la mayor parte de las veces es
asintomático, y, si presenta síntomas, suele ser en forma de sangrado.(R2)

!
!
!
!

363.! Paciente de 24 años que


acude por sensación de cuerpo extraño a la deglución. Se solicita una ecografía tiroidea
y niveles hormonales con TSH y T4L. ¿Cuál es la actitud a seguir según los resultados
de la ecografía y de laboratorio de la paciente?

1. 1. Si presenta normofunción tiroidea, no es necesario hacer PAAF.


Es imprescindible la realización de gammagrafía tiroidea para decidir si se debe realizar
2. 2.
PAAF independientemente de los resultados de TSH.
3. 3. La presencia de hipotiroidismo, apoya la realización de la gammagrafía.
Si la paciente presenta un hipertiroidismo subclínico, se puede realizar una gammagrafía
4. 4.
previa a la decisión de realización de PAAF.
Gráfico de respuestas
Comentario
El proceso diagnóstico ante un nódulo tiroideo, incluye la realización de unos niveles de TSH, una
ecografía cervical, siendo la punción- aspiración del nódulo con aguja fina (PAAF), la prueba que
ofrece la mayor rentabilidad diagnóstica. No obstante, en algunas circunstancias, la realización de
una gammagrafía tiroidea puede anteceder a la decisión de realización de PAAF sobre todo ante
aquellas situaciones en las que sospechemos patología autónoma tiroidea, es decir, bocio
multinodular tóxico ó adenoma tóxico. Por ello, ante un paciente con nódulo tiroideo e
hipertiroidismo ó hipertiroidismo subclínico (descenso en los niveles de TSH), es conveniente
comprobar con la gammagrafía si se trata de un nódulo hipercaptante, pues si es así, no
tendríamos que realizar PAAF para diagnóstico de enfermedad maligna, ya que la probabilidad de
cáncer en ese nódulo hipercaptante en paciente hipertiroideo, sería muy baja.(R4)

364. En la revisión de la paciente del caso anterior se detecta TSH 2.6 mUI/mL (N: 0.5-5
mUI/mL) junto con niveles de T4 libre normales. Se decide realizar PAAF, siendo el
resultado de esa punción: “aisladas células foliculares sin atipias, coloide y macrófagos.
Citología negativa para células malignas, sugerente de hiperplasia nodular”. ¿Cuál sería
la actitud más CORRECTA?

Tiroidectomía total con vaciamiento cervical de entrada, ante los hallazgos ecográficos y
1. 1.
el tamaño de la lesión.
Actitud expectante con ecografías cada 6-12 meses, valorando crecimiento y datos
2. 2.
ecográficos de mal pronóstico.
3. 3. Realizar gammagrafía para comprobar que se trata de un nódulo hiperfuncionante.
4. 4. Repetir la PAAF, dado que existen células foliculares en la punción.
Gráfico de respuestas

!
!
!
!
Comentario

Ante una PAAF con resultado de benignidad, como la que tenemos en la pregunta, se debe
realizar ecografías periódicas para valorar crecimiento del nódulo, aparición de datos ecográficos
de malignidad o factores de mal pronóstico (adenopatías, afonía…etc) que orientaran a
hemitiroidectomía diagnóstica junto con repetición de PAAF previamente. Durante el seguimiento
ecográfico se puede intentar tratamiento con levotiroxina a dosis supresoras, pero en ningún caso
antitiroideos. Dado que la paciente no tiene factores de mal pronóstico, signos sugestivos de
cáncer tiroideo, el tamaño del nódulo no es muy grande y los hallazgos ecográficos iniciales no son
del todo desfavorables (el único desfavorable sería la vascularización central del nódulo), en
principio la cirugía de entrada no estaría indicada, y si lo estuviera, en ningún caso se realizaría
vaciamiento cervical si no se ha comprobado la naturaleza maligna del nódulo. No haremos
gammagrafía ante una normofunción tiroidea y la repetición de la PAAF no habría que considerarla
dado que no tenemos un resultado no diagnóstico, si no uno de benignidad.(R2)

365. Which of the following is a necessary condition for the occurrence of cervical
squamous cancer cells?

1. 1. Human papillomavirus (HPV) infection.


2. 2. Smoking.
3. 3. Deficiency of alpha-1-antitrypsin.
4. 4. Early initiation of sexual intercourse.
Gráfico de respuestas
Comentario

Que la infección por HPV (sobre todo tipos 16 y 18) es el FR fundamental del ca. de cérvix es lo
más importante que deben saber de esta patología. Sobre el resto de las opciones, recuerden la
promiscuidad sexual y el inicio precoz de las relaciones sexuales, y también el tabaquismo (no
olvide el dibujo de la monja y la promiscua).

El déficit de alfa 1 antitripsina también se ha demostrado factor predisponente, aunque su


prevalencia es muy poco importante en comparación con la de los otros FR del cáncer de
cérvix.(R1)

366. Señale la INCORRECTA en relación al síndrome de Mallory-Weiss:

1. 1. Existe una historia de alcoholismo en un porcentaje alto de los pacientes.


2. 2. Suelen dejar de sangrar espontáneamente.
3. 3. El desgarro se localiza con más frecuencia en el esófago medio.
4. 4. La vasopresina intraarterial puede ser útil.
Gráfico de respuestas
Comentario

El síndrome de Mallory-Weiss se produce como consecuencia de una erosión o desgarro de la


mucosa esofágica distal de forma secundaria a vómitos de repetición. La expresión clínica suele
ser el sangrado digestivo, casi siempre en forma de hematemesis. Habitualmente cesa de forma
espontánea y raramente requiere cirugía o administración intraarterial de sustancias vasoactivas.
Suele asociarse a alcoholismo crónico.(R3)

!
!
!
!
367. El sangrado uterino disminuido en cantidad, que ocurre a intervalos regulares,
corresponde a:

1. 1. Menorragia.
2. 2. Metrorragia.
3. 3. Hipomenorrea.
4. 4. Oligomenorrea.
Gráfico de respuestas
Comentario

Hipomenorrea: disminución en la cantidad de la hemorragia < 20 mL.

Oligomenorrea: Disminución en el número de días de hemorragia < 3 días.

Menorragia: hemorragias excesivas o prolongadas, regulares.

Metrorragia: cualquier hemorragia vaginal, procedente del útero, no asociada al ciclo menstrual por
su ritmo o por la cantidad de flujo(R3)

368. ¿Cuál de las siguientes sustancias forma parte de la secreción biliar?

1. 1. Carboxipeptidasa.
2. 2. Tripsina.
3. 3. Elastasa.
4. 4. Lecitina.
Gráfico de respuestas
Comentario

Los principales componentes de la bilis son: agua (82%), ácidos biliares (12%), lecitina y otros
fosfolípidos (4%) y colesterol no esterificado (0,7%), así como alguna proteína (albúmina e IgA).
Además, la bilis es la vía de excreción de productos catabólicos (pigmentos biliares, metabolitos de
hormonas esteroideas, etc.), así como de muchos fármacos y toxinas.

Sin embargo, no era necesario que supiera que la lecitina es un lípido… Era suficiente con darse
cuenta de que las opciones 1, 2, 3 son ENZIMAS PANCREÁTICAS y, por consiguiente, no forman
parte de la secreción biliar.(R4)

369. Señale la verdadera sobre la afectación gastrointestinal en la esclerosis sistémica:

1. 1. Los síntomas por afectación esofágica ocurren en menos de un 20% de los casos.
2. 2. El reflujo gastroesofágico se debe a fibrosis del tercio distal del esófago.
3. 3. La disfagia es secundaria a pérdida de la motilidad esofágica.
4. 4. La afectación del intestino grueso se caracteriza por rectorragias.
Gráfico de respuestas
Comentario

La esclerosis sistémica es un tema poco preguntado en el ENARM y esta pregunta le ayudará a


sintetizar la afectación esofágica, que afecta a casi todos los pacientes, constituyendo la alteración
visceral más frecuente. La fibrosis va ocupando todas las capas en los 2/3 inferiores del esófago,
fibrosando la serosa, adelgazando la mucosa y atrofiando la capa muscular, afectando a su

!
!
!
!
motilidad; la prueba de oro es la manometría, que refleja el peristaltismo disminuído, no propulsivo
del bolo alimenticio, que tiene como consecuencia clínica la disfagia, por lo que la respuesta 3 es la
correcta. Es cierto que puede existir esofagitis por reflujo, pero por disfunción del esfínter esofágico
concretamente, y puede desembocar en una metaplasia del tercio distal esofágico (Barrett), Menos
importante es la afectación del resto de tracto intestinal, pero aplicando la misma lógica de
"peristalsis disminuída", podemos deducir que habrá estreñimiento y desarrollo de divertículos, y en
intestino delgado íleo paralítico con sobrecrecimiento bacteriano.(R3)

370. Masculino de 26 años con crisis


comiciales desde los 15 años en tratamiento con ácido valproico. Actualmente ingresa
en el Servicio de Urgencias por cefalea y disminución del nivel de conciencia,
evidenciando en el TC craneal un hematoma intraparenquimatoso temporal izquierdo. Se
le realiza la arteriografía mostrada en la imagen. Cuál sería la etiología más probable:

1. 1. Malformación arterio-venosa de arteria cerebral media.


2. 2. Disección arterial.
3. 3. Sangrado tumoral.
4. 4. Sangrado secundario a consumo de tóxicos simpaticomiméticos.
Gráfico de respuestas
Comentario

Aunque no tenga experiencia en la interpretación de una arteriografía cerebral, en esta ocasión no


lo tiene difícil. Solamente por el grupo de edad, deberías haber considerado una malformación
arteriovenosa. Por otra parte, observe que en las ramas más terminales se visualizan vasos
organizados en forma de ovillo. Si se tratara de un aneurisma, sería simplemente una cavidad
sacular, localizada casi siempre en zonas más proximales.(R1)

371. Tras diagnosticar angiográficamente al paciente de la imagen, cuál sería el


tratamiento de elección:

1. 1. Disección arterial, anticoagulación.


2. 2. Malformación arterio-venosa, cirugía.
3. 3. Aneurisma sacular de A. cerebral media, embolización.

!
!
!
!
4. 4. Ictus isquémico con transformación hemorrágica, antiagregación simple.
Gráfico de respuestas
Comentario

Siendo coherentes con el diagnóstico dado en la pregunta anterior, la respuesta correcta sería la 2,
cuyo tratamiento es quirúrgico.(R2)

372. Recién nacido hijo de madre con diabetes gestacional, que comienza con temblor
ante mínimos estímulos e irritabilidad desde el 2º día de vida. No existieron
complicaciones perinatales, siendo el test de Apgar 8/10. En los estudios de laboratorio
se encuentra una glucemia de 40 mg/dl y un calcio total de 7 mg/dl. Se inician aportes de
glucosa y calcio i.v. a pesar de lo cual persiste la sintomatología. Un nuevo examen de
laboratorio demuestra una glucemia normal y un calcio de 6.8 mg/dl. ¿Qué actitud le
parece más adecuada?

1. 1. Determinar el calcio iónico.


2. 2. Determinar calcio iónico, magnesio y fósforo.
3. 3. Realizar punción lumbar e iniciar tratamiento antibiótico.
4. 4. Realizar ultrasonido cerebral urgente.
Gráfico de respuestas
Comentario

Recuerde que el calcio y el magnesio son una especie de “hermanos gemelos”. Cuando una
hipocalcemia se mantiene a pesar del tratamiento con calcio, debe descartar una hipomagnesemia
asociada. Si ésta existe, tiene que corregirla, puesto que de lo contrario no solucionará el problema
del calcio. Por otra parte, no debe olvidarse del calcio iónico (indica más fiablemente la situación
del niño que el calcio total) y el fósforo.(R2)

373. Usted está atendiendo a un paciente masculino de 68 años de edad con fibrilación
auricular e intervenido en el pasado de hernia discal cervical. Presenta un dolor
abdominal intenso, de inicio agudo, a nivel periumbilical, que se ha ido haciendo difuso
y más constante. En la exploración comprueba que el abdomen está distendido y sensible
a la palpación, con los ruidos hidroaéreos normales. El tacto rectal no demostró sangre.
¿En cuál de los siguientes diagnósticos pensaría?:

1. 1. Angina abdominal.
2. 2. Trombosis venosa mesentérica.
3. 3. Embolia arterial mesentérica.
4. 4. Angiodisplasia de colon.
Gráfico de respuestas
Comentario

Se trata de un cuadro clínico típico de una a isquemia mesentérica aguda que, se caracteriza por la
aparición de dolor brusco con irritación peritoneal. Por isquemia se produce lesión de la pared
intestinal, cursando con dilatación de asas, leucocitosis con desviación, acidosis y por último,
perforación. Se produce con más frecuencia por una embolia en pacientes con fibrilación auricular
u otro proceso embólico. Con menos frecuencia se debe a una trombosis in situ de una placa de
arteriosclerosis o a bajo gasto, por tanto el método diagnóstico de elección será la arteriografía que
detectará un stop brusco de la vascularización a nivel de mesentérica superior fundamentalmente.
El tratamiento será la embolectomía o tromboendarterectomía pero en caso de gangrena o
perforación, resección del segmento afecto.(R3)

!
!
!
!
374. Con respecto a la hemorragia intracerebral hipertensiva señale la FALSA:

Suele ser causada por la rotura espontánea de pequeñas arterias penetrantes en zonas
1. 1.
profundas del encéfalo.
El cuadro suele producirse con el paciente despierto, y a veces cuando concurre alguna
2. 2.
situación estresante.
La técnica diagnóstica de elección es la TC. La RM es más sensible en las hemorragias de
3. 3.
fosa posterior, pero no suele ser necesaria.
El diagnóstico precoz y el tratamiento antihipertensivo agresivo son fundamentales y
4. 4.
pueden mejorar considerablemente el pronóstico de estos pacientes.
Gráfico de respuestas
Comentario

Pregunta de dificultad moderada sobre un tema muy importante en el ENARM; los EVCs. Los
procesos vasculares hemorrágicos representan un 15-20% y la HTA es el principal factor de riesgo
asociado. A diferencia de los EVC isquémicos, de instauración súbita, los hemorrágicos suelen
evolucionar en el transcurso de varios minutos y suelen acompañarse de cefalea, náuseas y
vómitos y se suelen presentar con el paciente despierto (respuesta 4 correcta). La hemorragia
intracerebral es preferentemente causada por la rotura de microaneurismas de Charcot-Bouchard
localizados en pequeñas arterias perforantes de zonas profundas del cerebro, fundamentalmente
en putamen, tálamo, cerebelo y protuberancia. En general, la prueba de neuroimagen de elección
en la patología aguda cerebrovascular es la TC, siendo la RM más sensible en las hemorragias de
fosa posterior. Sin embargo, el pronóstico en las hemorragias intracerebrales viene determinado
por el tamaño y la localización del hematoma y no por el momento del diagnóstico. La hemorragia
inicial produce un déficit neurológico determinado, siendo razonable el control de la hipertensión
arterial para evitar la progresión del mismo. En cualquier caso, el tratamiento antihipertensivo
nunca debe ser "agresivo" puesto que si se baja la tensión demasiado o muy deprisa, puede
provocarse una isquemia cerebral en torno a la cavidad hemorrágica.(R4)

375. A 32-year-old man with a history of ulcerative colitis treated with prednisone (40
mg/day) and mesalazine (4 g/day) comes to the ER with a history of global functional
decline accompanied by an increase in the number of stools (up to 10 per day, stained
with blood) and abdominal cramping. He is admitted to the hospital, where iv steroids (1
mg/kg/day) and fluid therapy are started. Thirty six hours later, his abdominal pain
worsens, and he develops abdominal distension and constipation. Physical examination
showed a body temperature of 38.7 ° C; pulse 124 beats per minute; a distended, tympanic
abdomen, with signs of peritoneal irritation. Lab tests show: WBC 17,000/dl. Radiographic
studies show a dilated transverse colon (diameter: 8 cm) without free peritoneal air. The
patient continues to deteriorate after 24 hours of observation. What is the most
appropriate treatment?

1. 1. Azathioprine.
2. 2. Cyclosporine.
3. 3. Emergency subtotal colectomy.
4. 4. Colonoscopy.
Gráfico de respuestas
Comentario
Emergency subtotal colectomy. The question describes a typical case of ulcerative colitis
complicated with toxic megacolon. The patient shows symptoms of severity, defined by True-Love
criteria: five or more bowel movements a day, <10 g / dl hemoglobin, > 90 bpm and> 37 ° C
temperature. A typical finding is dilation of the transverse colon > 6 cm in abdominal radiograph,
which is definitory for toxic megacolon. It can occur in any inflammatory disease affecting the colon,

!
!
!
!
although it is most common in ulcerative colitis. It is considered an emergency: it requires close
monitoring and intravenous fluids, steroids and antibiotics. If there is no improvement in 12-24
hours (as in this case) a subtotal colectomy should be performed, since the morbidity and mortality
in case of perforation is more than 20%.(R3)

376. La parálisis de Erb-Duchenne NO presenta:

1. 1. Aducción del brazo.


2. 2. Pronación del antebrazo.
3. 3. Desaparición del reflejo de prensión palmar en el R.N.
4. 4. Extensión del codo.
Gráfico de respuestas
Comentario

Existen dos parálisis braquiales en el recién nacido.

1) ERB DUCHENNE: raices (C4) C5 y C6: brazo en aducción y rotación interna, reflejo de moro no
presente o asimétrico, reflejo de presión palmar presente y se puede asociar a parálisis frénica.

2) KLUMPKE: C7, C8, (T1): mano caida con reflejo de Moro presente y prensión palmar abolida, se
puede asociar a síndrome de Horner.

Respuesta 3 incorrecta.(R3)

377. En relación con la HCG placentaria, NO es verdad:

1. 1. Se demuestra en sangre materna a los diez días del pico ovulatorio.


2. 2. Se alcanzan los máximos niveles a la 10-12 semana.
3. 3. Empieza a decaer en el tercer trimestre.
4. 4. Las concentraciones de HCG en placenta son similares a las séricas.
Gráfico de respuestas
Comentario

La hCG la hormona encargada de mantener el cuerpo lúteo. Tiene una función biológica parecida a
la LH. Se detecta en sangre materna tras la implantación, lo que puede emplearse para el
diagnóstico de embarazo. Sus niveles aumentan a lo largo del primer trimestre alcanzando el
máximo alrededor de la 10ª semana (50,000 mUI/ml), para disminuir a partir de este momento
(respuesta 3 falsa). Las acciones principales de esta hormona son:

- Función luteotrófica: sin duda la más importante. Mantiene el cuerpo lúteo las primeras semanas
hasta que la placenta es capaz de sintetizar progesterona.

- Esteroidogénesis: interviene en la síntesis de gestágenos, andrógenos (estimulación de la


secreción de testosterona por el testículo fetal, y de DHEAS por la suprarrenal), así como cierta
acción FSH-like a nivel del ovario fetal. También tiene una acción tirotrópica (TSH-like), lo que
justifica el aumento del tamaño del tiroides durante el embarazo.(R3)

378. Todas las siguientes pueden ser complicaciones de la fecundación in vitro,


EXCEPTO:

!
!
!
!
1. 1. Embarazo múltiple.
2. 2. Hiperestimulación ovárica.
3. 3. Infección pélvica.
4. 4. Hiperprolactinemia.
Gráfico de respuestas
Comentario

La fecundación in vitro consiste en que, tras inducir la ovulación, se recogen ovocitos mediante
punción folicular vía vaginal, normalmente con control ecográfico. Se ponen en contacto los
ovocitos extraídos con los espermatozoides y, una vez conseguida, se transfieren no más de tres
embriones. Los embriones no transferidos pueden ser criopreservados y transferirse en ciclos
posteriores.

Esta técnica incrementa el riesgo de gestación ectópica, abortos y embarazo múltiple, pero no las
malformaciones congénitas. Dada la manipulación que supone, es posible que se produzcan
infecciones, tal como comenta la respuesta 3.

El síndrome de hiperestimulación ovárica consiste en una respuesta excesiva del ovario a la


inducción de la ovulación, lo que produce una extravasación de líquido. Clínicamente cursa con
distensión abdominal, aumento del tamaño ovárico, dolor abdominal, vómitos, diarrea e incluso
ascitis. Suele resolverse en 1-2 semanas, pero en ocasiones pone en riesgo la vida del paciente,
complicándose con tromboembolismos, fracaso renal o distrés respiratorio del adulto.

Las opciones 1, 2 y 3 son, por tanto, asociaciones claras de esta técnica. Sin embargo, no existe
ninguna razón por la que pueda producir hiperprolactinemia (respuesta 4 correcta).(R4)

379. Un recién nacido a término nacido por cesárea presenta dificultad respiratoria,
taquipnea e imagen de "redistribución" de los vasos pulmonares, diafragmas aplanados
y líquido en las cisuras. No hay hipoxemia, hipercapnia, ni acidosis; ¿cuál es el proceso
más probable?

1. 1. Aspiración de meconio.
2. 2. Neumomediastino.
3. 3. Taquipnea transitoria del recién nacido.
4. 4. Síndrome de Wilson-Mikity.
Gráfico de respuestas
Comentario

La taquipnea transitoria es la modalidad de distrés respiratorio típica del RN término o casi término.
Para su génesis, se requiere además un desencadenante obstétrico (parto por cesárea o vaginal
rápido), que hace que el tórax del RN no sufra compresión en el canal del parto, con lo que el
líquido pulmonar no sería desalojado. Precisamente, ese líquido pulmonar en la radiología te debe
dar la clave diagnóstica, junto con la evolución favorable.(R3)

380. Mujer de 70 años que refiere dificultad creciente para levantar los brazos por encima
de los 90 grados y cefalea unilateral intensa que le interfiere el sueño. En los exámenes
de laboratorio destaca la presencia de anemia de 8.5 g de hemoglobina normocítica y
normocrómica con VSG de 112. La biopsia de la arteria temporal muestra una
disminución muy significativa de la luz vascular con intenso infiltrado de células
mononucleares y algún granuloma de células gigantes. El tratamiento de elección de esta
paciente es:

!
!
!
!
1. 1. Prednisona 10 mg al día.
2. 2. Ciclosporina 5 mg/kg/semanal.
3. 3. Ciclofosfamida 100 mg al día.
4. 4. Prednisona 1 mg/kg/día.
Gráfico de respuestas
Comentario

Ante una paciente anciana con clínica de dolor y rigidez en cintura pélvica y/o escapular
acompañado de elevación de VSG, sin clínica de miopatía ni otros síntomas, el diagnóstico debe
ser el de polimialgia reumática, y el tratamiento se realiza con corticoides a dosis bajas(10-20
mg/día) con una respuesta espectacular; sin embargo, cuando aparece clínica asociada de
sospecha de arteritis de la temporal, como cefalea o alteraciones visuales o claudicación
mandibular, el tratamiento debe realizarse con dosis altas de corticoides (1mg/kg/día) inicialmente
durante un mes para luego ir disminuyendo la dosis progresivamente, manteniendo tratamiento
durante al menos un año.(R4)

381. El efecto de Jod-Basedow observado tras la yodación en áreas endémicas:

1. 1. Consituye una clara contraindicación de la yodación profiláctica.


2. 2. Suele suceder en enfermos cuyo bocio era multinodular.
3. 3. Sólo se observa en zonas no endémicas.
4. 4. Es el nombre que recibe la enfermedad de Graves-Basedow en áreas endémicas.
Gráfico de respuestas
Comentario

El yodo es necesario para la formación de las hormonas tiroideas, y su metabolismo está muy
estrechamente controlado, hasta el punto de que la yoduria refleja la cantidad de yodo de que
dispone una persona. Sin embargo, el yodo en situaciones y dosis determinadas puede producir
efectos distintos a los fisiológicos. Un ejemplo es el fenómeno de Jod-Basedow que ocurre en
pacientes con bocio ante la administración de dosis elevadas de yodo y cuyo efecto es la aparición
de un hipertiroidismo que en el momento del diagnóstico tiene una gammagrafía hipocaptante. Otro
ejemplo es el Wolf- Chaikoff, que consiste en la inhibición de la organificación ante la
administración de dosis altas de yodo en pacientes con hipertiroidismo. Otro efecto es la inducción
de bocio o hipotiroidismo en pacientes susceptibles como lo son aquellos con enfermedades
autoinmunes de tiroides, reflejado en esta pregunta, a lo que se denomina mixedema por
yodo.(R2)

!
!
!
!

382. Paciente
primigesta de 35 semanas de gestación que acude a urgencias por percibir contracciones
durante la noche. Exploración cervical: cérvix formado, permeable 1 dedo. Se realiza
registro cardiotocográfico que se muestra en la imagen. ¿Cuál seria la actitud correcta
en esta paciente?

Reposo e hidratación en domicilio. Acudir si presenta contracciones más intensas y


1. 1.
regulares.
2. 2. Ingreso e iniciar tocólisis y maduración pulmonar.
Realizar una ecografía para valorar doppler en arteria umbilical ya que la variabilidad es
3. 3.
baja.
Finalizar la gestación si el Doppler en arteria umbilical está alterado ya que el RAF es
4. 4.
negativo.
Gráfico de respuestas
Comentario

Debe saber la conducta a seguir a la perfección. A partir de las 34 semanas de gestación, se


considera que el feto es maduro desde el punto de vista pulmonar, por lo que se dejará evolucionar
el parto en caso de instaurarse, sin necesidad de tratamiento tocolítico ni corticoterapia (respuesta
2 falsa). En cuanto al registro cardiotocográfico que nos presentan, puede considerarse normal, por
lo que se descartan opciones como la 3 y 4.

En casos como éste, es suficiente proceder con medidas conservadoras como la 1, con reposo
domiciliario, sin ninguna medida terapéutica adicional. En el caso de que se tratara de un cérvix de
parto, se esperaría a la evolución espontánea del mismo.(R1)

383. ¿Cuál NO comparte la subunidad alfa?:

1. 1. FSH.
2. 2. LH.
3. 3. HCG.
4. 4. ACTH.
Gráfico de respuestas
Comentario

Es importante que tenga claro este concepto porque en bastante preguntas, aunque no sea de
esta forma tan directa se refieren a él. La subunidad alfa es compartida por una serie de hormonas

!
!
!
!
que son: FSH, LH, TSH y HCG. Esta característica debe ser tenida en cuenta a la hora de ciertas
determinaciones de laboratorio y también es aprovechada en el plano terapéutico.(R4)

384. With regard to the treatment of COPD, which of the following statements is FALSE?

Smoking cessation is the only measure that prevents further progression of obstructive
1. 1.
symptoms.
2. 2. If acute infective exacerbation is suspected, empiric antibiotic therapy is indicated.
3. 3. Ipratropium bromide is an effective bronchodilator for chronic bronchitis.
Glucocorticoids are indicated in all cases in which the patient's symptoms do not improve
4. 4.
after treatment with bronchodilators.
Gráfico de respuestas
Comentario

Pregunta de dificultad media-alta que sobre el tratamiento del EPOC, tema que debes dominar
pues puede venir una pregunta en el ENARM. Lo primero que le debe llamar la atención son dos
fuertes afirmaciones que por método de examen puedes dudar de su veracidad: el dejar de fumar
es la única.... (opción 1) y los corticoides están indicados en todas... (opción 4). Seguro que pese a
ello la opción 1 no le dejará ninguna duda ya que lo habrá escuchado muchas veces. Sin embargo
la opción 4 sabemos que es mentira ya que los corticoides sólo están indicados en las
agudizaciones graves y en aquellos pacientes sintomáticos con respuesta terapéutica favorable
previa, enfermos con hiperreactividad bronquial y EPOC grave con reagudizaciones muy
frecuentes. El resto de las opciones no te deben plantear duda.(R4)

385. Un enfermo EPOC con diagnóstico de miastenia gravis recibe tratamiento


prolongado con inmunosupresores. En la Rx tórax de control post IQ por extirpación del
timo, se aprecia una densidad ovalada de diámetro mayor vertical en lóbulo superior
izquierdo, con pequeño menisco radiotransparente en su interior. Se compara dicha
imagen con Rx anteriores, observándose en estas una lesión de morfología idéntica, de
contornos muy definidos, pero de densidad aire en su totalidad, en la misma localización.
Se realiza una biopsia por BFC y se analiza en laboratorio, informándose como proceso
inflamatorio crónico con multitud de elementos filiformes tabicados y ramificados en 45º.
¿Qué diagnóstico sugiere?

1. 1. Ca broncogénico con calcio cristalizado.


2. 2. Infección por Histoplasma capsulatum.
3. 3. Absceso pulmonar por Actinomices israelii.
4. 4. Aspergillosis pulmonar.
Gráfico de respuestas
Comentario

Estrictamente, esta pregunta no está del todo bien formulada, porque el paciente no padece
exactamente una aspergilosis pulmonar, sino un aspergiloma. Por aspergilosis pulmonar, suele
entenderse una enfermedad invasiva por este hongo. Sin embargo, lo que nos describen es una
imagen redondeada con una semiluna aérea en la parte superior (menisco radiotransparente). Esta
imagen radiológica es muy característica del aspergiloma. La aparición de elementos filiformes
tabicados y con ramificaciones en 45º también va a favor de Aspergillus.(R4)

386. En una enferma de 40 años, con insuficiencia renal crónica secundaria a nefrectomía
bilateral y antecedentes personales de gastrectomía por úlcera péptica, apendicectomía
e histerectomía, ¿cuál sería el tratamiento sustitutivo más adecuado?

!
!
!
!
No es candidata a tratamiento sustitutivo por tener un pronóstico vital ominoso a corto
1. 1.
plazo.
2. 2. Hemodiálisis periódica.
3. 3. Diálisis peritoneal continua ambulatoria.
4. 4. Trasplante renal urgente.
Gráfico de respuestas
Comentario

Una pregunta de dificultad media sobre el tratamiento de la insuficiencia renal crónica.

Dado que se trata de una paciente joven, habría que ofrecerle un tratamiento sustitutivo para su
función renal, que será la hemodiálisis periódica, el trasplante renal o la diálisis peritoneal. En este
caso, por supuesto podría realizarse un trasplante renal, pero no es necesario realizarlo de forma
urgente (respuesta 4 falsa). Por otra parte, en esta paciente no podemos plantearnos la diálisis
peritoneal, puesto que tiene antecedentes de cuatro intervenciones quirúrgicas abdominales, por lo
que es de suponer que la capacidad de intercambio de su peritoneo no será óptima. Teniendo en
cuenta todo esto, la única opción posible es la respuesta 2.(R2)

387. Neonato masculino que, a las


pocas horas inicia vómitos con intolerancia de tomas. Usted analiza los antecedentes
perinatales. Es un embarazo no controlado, madre de 37 años, Gestas 2 aborto 1. En
urgencias de maternidad, a la madre se le hizo test rápido de VIH, siendo el resultado
negativo, estando pendientes resto de serologías. La ecografía preparto mostraba un feto
acorde a 36 semanas con polihidramnios. Afebril durante el parto, se le realizó
amniorrexis artificial, con parto vía vaginal, y Apgar 8/9 al nacimiento. En la inspección,
el neonato presenta rasgos faciales algo peculiares, aunque la facies está edematosa.
Destacan macroglosia, cierta hipotonía axial y hernia umbilical. En la auscultación usted
aprecia un soplo sistólico con pulsos presentes a nivel femoral. Dentro de las pruebas
complementarias se solicita una radiografía con la imagen de la figura. Señale la
respuesta INCORRECTA:

1. 1. Entre las pruebas realizadas, hay que despistar el riesgo infeccioso.


Estamos ante un caso de estenosis hipertrófica de píloro. Interconsultaría al cirujano para
2. 2.
intervención
3. 3. En la exploración física, el abdomen está excavado.
4. 4. Existe riesgo de hiperbilirrubinemia.
Gráfico de respuestas

!
!
!
!
Comentario

Nos encontramos ante un neonato que a las pocas horas de vida comienza con vómitos e
intolerancia oral. Dados los antecedentes y ante todo neonato con vómitos, es prioritario descartar
la etiología infecciosa (opción 1) y realización de radiografía de abdomen para despistar
obstrucción. En la figura se aprecia el signo de la doble burbuja, patente en la atresia duodenal.
Ésta cursa con vómitos biliosos desde el nacimiento debido a la obstrucción que generalmente
acontece en 2º porción duodenal; con interrupción del tránsito distal, por lo que el abdomen
aparecerá excavado (opción 3). El hallazgo de polihidramnios en la gestación es un antecedente
típico, presente en obstrucciones digestivas altas. Así nos encontramos ante una atresia duodenal.
Ello sumado al resto de hallazgos clínicos: macroglosia, facies peculiar, hernia umbilical, hipotonía
axial, soplo sistólico... y ante los antecedentes prenatales (madre añosa, no controlada), nos hacen
sospechar la existencia de un síndrome de Down. Recuerde que el síndrome de Down es causa de
CIR armónico y presentan riesgo de cardiopatía (CIV), riesgo de policitemia e hiperbilirrubinemia.
Para su diagnóstico es precisa la realización de cariotipo. La opción 2 es incorrecta ya que cursa
con vómitos en torno al mes de vida NO IMEDIATOS y en la radiografía sólo existiría dilatación de
la cámara gástrica (única burbuja).(R2)

388. Usted descartaría en este paciente:

Anomalía VACTERL: completaría el estudio con radiografía de tórax y ecografía cardíaca


1. 1.
y abdominal como estudios prioritarios.
2. 2. Síndrome de Turner: completaría estudio con cariotipo y ecocardiografía.
3. 3. Síndrome de Down: completaría estudio con cariotipo y ecocardiografía.
Rubéola congénita: realizaría estudio de placenta, serologías al neonato, estudio de fondo
4. 4.
de ojo y ecocardiografía.
Gráfico de respuestas
Comentario

Nos encontramos ante un embarazo no controlado en una mujer añosa, donde no se ha realizado
el triple screening ni amniocentesis. Ante los hallazgos neonatales: pequeño para edad
gestacional, rasgos faciales peculiares, macroglosia, hernia umbilical, atresia duodenal, hipotonía
axial, cardiopatía... lo más probable es que nos encontremos ante un S. Down (opción 3), cuyo
diagnóstico es mediante cariotipo. Será necesaria la realización de ecocardiografía para filiar el
soplo (recuerde la CIV como malformación más frecuente). El VACTERL (opción 1) es un acrónimo
de la asociación de anomalías vertebrales, anorrectales, cardíacas, fistula traqueoesofágica-
atresia esofágica, renales y radiales (limb); sin otros estigmas físicos. El S. Turner (45,X0) (opción
2) está presente en niñas, cursa con mamas separadas, pterigium colli, y es preciso descartar en el
periodo neonatal la coartación de aorta. La opción 4 rubéola congénita, tampoco cursa con atresia
duodenal, aunque recuerda que puede presentar cardiopatía, sordera, alteraciones oculares
(Tríada de Gregg) y microcefalia.(R3)

389. El anticonceptivo oral de elección en mujer con hemorragias leves intermenstrual


es:

1. 1. Monofásico de 15 mcg de etinilestradiol.


2. 2. Monofásico de 20 mcg de etinilestradiol.
3. 3. Monofásico de 50 mcg de etinilestradiol.
4. 4. Bifásico 40/30 mcg de etinilestradiol.
Gráfico de respuestas
Comentario

!
!
!
!
Anticonceptivos orales monofásicos con etinilestradiol 50 mcg tiene utilidad como ACO, hemorragia
uterina disfunción, regulación de ciclos opsomenorréicos o amenorrea; dismenorrea; síndrome de
tensión premenstrual; endometriosis; quistes funcionales del ovario, hirsutismo y acné.(R3)

390. La causa más frecuente de enfermedad vascular cerebral es:

1. 1. Embolismo cardíaco.
2. 2. Embolismo arterio-arterial carotídeo.
3. 3. Embolismo graso.
4. 4. Embolismo aéreo.
Gráfico de respuestas
Comentario

La mayor parte de las enfermedades vasculares isquémicas son originadas por la ateroesclerosis y
sus complicaciones trombóticas y tromboembolicas. Sin embargo, no debe olvidar que el corazón
es la fuente MÁS COMÚN de embolismo cerebral siendo algo menos frecuente el embolismo
arterio-arterial desde lesiones carótidas, vertebrobasilares o aorta ascendente.(R1)

391. El eritema toxico es:

1. 1. Más común entre los RN a termino que en los preterminos.


2. 2. Poco común antes de 5to día de vida.
3. 3. Usualmente asociado con elevación del recuento celular en sangre.
4. 4. Se manifiesta como un rash maculo vesicular.
Gráfico de respuestas
Comentario

El eritema tóxico consiste en la presencia de vesiculopústulas sobre una base eritematosa que
suelen respetar palmas y plantas, formadas por un infiltrado de eosinófilos, con tendencia a
desaparecer en la primera semana. Es más comun en los RN a término. Es una enfermedad que
no causa alteraciones sistémicas.(R1)

392. Todo lo siguiente puede ser causa de bronquiolitis, EXCEPTO:

1. 1. Virus sincitial respiratorio.


2. 2. Parainfluenza.
3. 3. Strpetococcus pneumoniae.
4. 4. Virus influenza.
Gráfico de respuestas
Comentario

La bronquiolitis está causada por un VIRUS por lo que el estreptococo neumoniae, que es una
bacteria, es la respuesta que debe marcarse.(R3)

393. Referente al síndrome bronconeumónico del lactante, son ciertas las siguientes
afirmaciones, EXCEPTO una:

1. 1. La fiebre puede faltar o ser muy escasa.


2. 2. El quejido respiratorio con taquipnea y tiraje intercostal hacen sospechar el diagnóstico.
3. 3. La percusión y auscultación del tórax son a veces normales o inespecíficas.
4. 4. La leucocitosis con fuerte neutrofilia es de gran valor diagnóstico.

!
!
!
!
Gráfico de respuestas
Comentario

La bronconeumonía hace referencia a la inflamación que se centra en los bronquíolos y que induce
la formación de exhudado mucopurulento que obstruye estas vías y provoca consolidación de los
lóbulos adyacentes. Los infiltrados difusos distribuidos en ambos campos pulmonares y
cambiantes según el momento son muy sugestivos.

La biometría hemática tiene utilidad limitada para el diagnóstico. De forma clásica se ha asociado a
la neumonía bacteriana con leucocitos neutrofilica. No obstante estos hallazgos son inespecíficos y
pueden aparece en víricas y faltar en bacterianas, sobre todo Mycoplasma y Chlamydia. La
leucopenia es rara y se asocia a mal pronóstico.

La clínica de las neumonías depende de la edad, patógeno y extensión de la enfermedad. En los


lactantes los signos pueden ser muy escasos.(R4)

394. Paciente de 55 años, obesa, hipertensa, con menopausia a los 51 años, que acude a
nuestra consulta refiriendo que en las dos ultimas semanas ha tenido un sangrado genital
en escasa cantidad. No refiere dolores pélvicos ni otros síntomas. A la exploración
encontramos una vagina y cérvix de características normales, sin objetivar sangrado ni
restos hemáticos en vagina y en el tacto vaginal no detectamos masas pélvicas
anormales. ¿Que actitud seria la más adecuada ante esta paciente?:

1. 1. No hacer nada puesto que el sangrado ha sido escaso y autolimitado.


2. 2. Realizar una citología cervicovaginal para descartar patología a ese nivel.
Solicitar un sedimento urinario puesto que podría tratarse de una infección de orina con
3. 3.
hematuria macroscópica.
4. 4. Sospechar patología endometrial y continuar estudiando a la paciente.
Gráfico de respuestas
Comentario

Ante una paciente con un sangrado postmenopáusico hay que sospechar que pueda presentar
patología ginecológica de origen uterino y aunque a la exploración no evidenciemos el sangrado es
necesario profundizar en el diagnóstico con otras pruebas complementarias. La realización de un
USG transvaginal y/o una histeroscopía diagnóstica para toma de biopsia son las pruebas que
mayor información nos van a aportar.(R4)

395. Al realizar un registro poligráfico a su paciente se observan ondas de 4 Hz de


frecuencia que ocupan más del 50% del trazado, junto con una actividad muscular mínima
y ausencia de movimientos oculares; señale la parasomnia que aparece típicamente en
esta fase del sueño:

1. 1. Enuresis.
2. 2. Terror nocturno.
3. 3. Bruxismo.
4. 4. Apnea del sueño.
Gráfico de respuestas
Comentario

Lo que nos están describiendo es un patrón EEG de ondas lentas (4 Hz), donde no existen
movimientos oculares…

!
!
!
!
Es decir, todo lo contrario a la fase REM, donde las ondas son rápidas (ritmo beta) y sí
encontraríamos tales movimientos. El patrón descrito es, por lo tanto, el de sueño profundo, cuya
parasomnia típicamente asociada es el terror nocturno.(R2)

396. En relación con el IFN-alfa como tratamiento de la hepatitis crónica por virus C,
señale la opción INCORRECTA:

La ALT se normaliza inicialmente en el 50% de los pacientes, aunque recaen con la


1. 1.
suspensión del tratamiento.
2. 2. La aparición de síntomas gripales es excepcional.
3. 3. La combinación con ribavirina es más efectiva.
4. 4. Su aplicación también resulta efectiva en pacientes VIH positivo con hepatitis C crónica.
Gráfico de respuestas
Comentario

El uso del interferón alfa es un componente fundamental de estos tratamientos. En el caso de la


hepatitis C, es cierto que el porcentaje de respuesta inicial es cercano al 50%, aunque muchos
recaerán cuando se suspenda. El uso conjunto con ribavirina aumenta el porcentaje de respuesta
significativamente (recuerda en relación a este fármaco dos cosas: puede provocar hemólisis y es
teratogénico). El uso de interferón disminuye el riesgo de hepatocarcinoma, y el uso en pacientes
VIH positivos también ha probado ser beneficioso. El cuadro pseudogripal que produce el
interferón es frecuentísimo (lo raro es no padecerlo), por lo que la respuesta 2 es falsa.(R2)

397. ¿Cuál de las siguientes situaciones NO supone una indicación primaria de la


ultrasonido obstétrico durante el primer trimestre de gestación?

1. 1. Definir la causa de un sangrado vaginal.


2. 2. Confirmar la edad gestacional.
3. 3. Valorar una sospecha de enfermedad trofoblástica gestacional.
4. 4. Diagnosticar un embarazo en mujer con amenorrea.
Gráfico de respuestas
Comentario

Una pregunta sencilla que podría acertarse por sentido común.

El estudio de un sangrado vaginal o el de un dolor pélvico agudo sería indicación de estudio


ultranosnográfico, para así filiar su causa (respuesta 1 correcta). También sería útil para confirmar
la existencia de enfermedad trofoblástica, puesto que ésta produce una imagen ecográfica
característica (copos de nieve, respuesta 3 correcta). Otra de las posibles aplicaciones sería el
estudio de la edad gestacional, que puede estimarse sobre la base de diversos parámetros
biométricos fetales, como la longitud cráneo-caudal (respuesta 2 correcta). Sin embargo, el USG
no se utiliza (o al menos no de forma rutinaria) para el diagnóstico de embarazo en toda mujer con
amenorrea (respuesta 4 falsa).(R4)

398. El principal problema clínico de un recién nacido posmaduro deriva de:

1. 1. Enfermedad de membrana hialina.


2. 2. Hipoglucemia.
3. 3. Sindrome de insuficiencia placentaria.
4. 4. Fibroplasia retrolental.
Gráfico de respuestas

!
!
!
!
Comentario

Se dice que un recién nacido es posmaduro si su edad gestacional excede las 42 semanas. El
mayor problema que presentan los posmaduros es la insufuciencia placentaria, que condiciona
potencialmente la aparición de cuadros de hipoxia, acidosis, pérdida de bienestar y sufrimiento
fetal agudo. Derivados de estos hechos puede aparecer, a su vez, cuadros también muy graves
como el síndrome de aspiración meconial.(R3)

399. Qué afirmación es INCORRECTA respecto a las variables pronósticas del cáncer de
mama:

El tamaño del tumor y el estado de los ganglios linfáticos axilares son factores de valor
1. 1.
pronóstico probado.
La recidiva del cáncer de mama es más probable en los tumores que poseen receptores
2. 2.
hormonales de estrógenos y de progesterona.
La sobreexpresión del gen c-erbB-2 (HER- 2/neu) puede ser indicativa de
3. 3.
resistencia/sensibilidad a determinados tratamientos.
4. 4. Elevadas concentraciones de PCNA (Ki67) indican peor pronóstico.
Gráfico de respuestas
Comentario

Pregunta fácil de uno de los temas primordiales de ginecología: el cáncer de mama.

En este caso, se nos pregunta la incorrecta acerca de los factores pronósticos.

El cáncer de mama es el cáncer más frecuente en la mujer. Los factores de riesgo más destacados
para padecerlo son la excesiva estimulación estrogénica (menarquia precoz, menopausia tardía, la
nuliparidad), los antecedentes familiares (BCRA1, BCRA2) y la patología mamaria previa.

Los factores de mal pronóstico más relevantes son:

•! Edad mayor de 35.


•! Tamaño tumoral mayor de 2 cm.
•! Afectación de ganglios axilares: se ha demostrado como el factor único más importante
para predecir la supervivencia global y libre de enfermedad. El número de ganglios afectos
es también pronósticamente muy significativo (respuesta 1 correcta).
•! Tipo histológico y grado de diferenciación: En general, el subtipo histológico de cáncer de
mama invasivo no es pronósticamente importante, pero algunos tipos especiales de
adenocarcinoma ductal infiltrante aparecen unidos a un riesgo muy bajo de recidiva:
carcinomas tubulares puros, carcinomas papilares puros y carcinomas mucinosos puros. El
grado de diferenciación tumoral sí que representa un factor pronóstico destacado.
•! Receptores hormonales negativos: la presencia de receptores hormonales es un factor de
pronóstico favorable. Tenga en cuenta que su presencia nos informa acerca de la
hormonodependencia del tumor y, por consiguiente, de la posibilidad de bloquearlos
reduciendo así la acción trópica hormonal (respuesta 2 incorrecta).
•! C-erb-B2 y p53: la sobreexpresión de c-erb-B2 conlleva peor respuesta a CMV y
tamoxifeno, y mejor respuesta a adriamicina (respuesta 3 correcta). P53 se considera un
regulador negativo del crecimiento celular, la mutación de p53 se considera un factor de
mal pronóstico por determinar pérdida de la función supresora, activación del crecimiento
celular y aumento del riesgo de progresión de la neoplasia.

!
!
!
!
•! Angiogénesis e invasión vascular: la presencia de mayor número de microvasos indica
peor pronóstico, porque aumenta la capacidad del tumor de producir metástasis a
distancia.
•! Agresividad local.
•! Gestación.
•! Otros: el antígeno Ki67 (PCNA) identifica las células proliferantes dentro de un tumor y, por
tanto, cuanto mayor es su presencia, más agresivo es el tumor. La positividad del Ki67 se
correlaciona con el grado de diferenciación tumoral, invasión vascular, metástasis en
ganglios linfáticos, y se relaciona inversamente con la presencia de receptores hormonales
(respuesta 4 correcta).(R2)

400. Señale el fármaco que incrementa el efecto hipoglucémico de la tolbutamida:

1. 1. Rifampicina.
2. 2. Fenobarbital.
3. 3. Tiazida.
4. 4. Warfarina.
Gráfico de respuestas
Comentario

El principal efecto secundario de las sulfonilureas (SFU) es la hipoglucemia. Las hipoglucemias son
menos frecuentes que con insulina pero mucho más severas y duraderas. La tolbutamida es una
SFU de primera generación, hoy en día ya no se usa por su larga vida media. Existen muchos
fármacos que pueden interaccionar con las SFU aumentando o disminuyendo el efecto
hipoglucemiante. Van a aumentar el efecto de las SFU el alcohol, el AAS, anti-H2, betabloqueantes
y anticoagulantes orales (warfarina). Entre los fármacos que disminuyen el efecto de las SFU
encontramos la fenitoína, rifampicina, beta- bloqueantes, esteroides, tiazidas y estrógenos.(R4)

401. Papá y mamá acuden muy ansiosos a tu consulta porque su hija pequeña, de 3 años,
todavía se orina de noche en la cama y de día a todas horas se encuentran "la braguita
mojada". Desde siempre les ha parecido que el pañal estaba permanentemente mojado,
pero ni ellos ni su pediatra le dieron mayor importancia en su momento. Refieren que su
hija mayor mostró control completo de los esfínteres desde los 2 años y medio y les
preocupa que su hermana se esté retrasando así. Controla perfectamente las heces. A
pesar de que no refiere disuria, su pediatra le ha realizado varios cultivos de orina que
han resultado negativos.

1. 1. Se trata de una situación fisiológica y se deberá tranquilizar a los padres.


Se trata de una enuresis nocturna y se deberá iniciar tratamiento con anticolinérgicos
2. 2.
nocturnos.
Se trata de una incontinencia continua y habrá que descartar la existencia de un uréter
3. 3.
ectópico.
Se trata de un síndrome enurético y se deberá remitir al paciente y a su familia al completo
4. 4.
a psiquiatría para iniciar una terapia conductual.
Gráfico de respuestas
Comentario

En este caso observamos una niña con constante incontinencia urinaria desde su nacimiento, lo
que debe orientar a la existencia de un uréter ectópico, causa más probable en niñas (respuesta

!
!
!
!
correcta la 3). No se trata de una enuresis nocturna (escapes con el sueño a partir de los 5-6 años
cuando se supone que el niño debería controlar sus esfínteres) ni de un síndrome enurético o
también llamado enuresis complicada (fugas nocturnas y diurnas por encima de esa edad).(R3)

402. Hombre de 45 años que tras infección de vías respiratorias altas presenta aparición
de lesiones nodulares, eritematovioláceas, en cara tibial anterior de ambas piernas. ¿Cuál
es su diagnóstico?:

1. 1. Eritema indurado de Bazin.


2. 2. Eritema nodoso.
3. 3. Paniculitis histiocítica citofágica.
4. 4. Vasculitis nodular.
Gráfico de respuestas
Comentario

Lo más probable en este caso es un eritema nodoso, por el antecedente de una infección previa y
por la localización de las lesiones en la cara anterior de las piernas, siendo dichas lesiones de tipo
nodular. La vasculitis nodular y el eritema indurado de Bazin, aparecen fundamentalmente en la
cara posterior de las piernas. No existe ningún dato que haga sospechar una panarteritis nodosa
(con vasculitis, habría mayor afectación), ni una paniculitis histiocítica citofágica.(R2)

403. Una de las siguientes lesiones elementales dermatológicas es bastante específica


de un cuadro cutáneo determinado:

1. 1. Pioderma.
2. 2. Queratosis.
3. 3. Poiquilodermia.
4. 4. Necrobiosis.
Gráfico de respuestas
Comentario
Pregunta compleja, de estudio en profundidad en Segunda Vuelta. La Queratosis es el aumento de
la capa córnea, que se produce en diversas patologías, no resultando característica de ninguna de
ellas. Lo mismo sucede con la Liquenificación (engrosamiento de la epidermis, con aumento de los
pliegues naturales y gran prurito, por rascado crónico) y la Poiquilodermia (hipo e
hiperpigmentación junto con atrofia cutánea y telangiectasias, producida por daño crónico),
ninguna de ellas es

!
!
!
!
específica. Podría existir duda entre el Pioderma y la Necrobiosis. El Pioderma es la aparición de
nódulos muy inflamados que drenan pus; se ve en determinadas formas de acné, el Pioderma
gangrenoso (igual, pero sin tendencia a la curación, sino a la ulceración profunda), que se asocia a
diversas patologías. En cambio, la Necrobiosis lipoídica es muy característica de la Diabetes
(respuesta 4 correcta). Son nódulos que crecen formando placas amarillentas con telangiectasias
superficiales, que pueden ulcerarse. Su evolución es independiente del control glucémico.(R4)

404. Masculino de 28 años,


que acude remitido a consultas de endocrinología al presentar en la química sanguínea
cifras de colesterol total: 436 mg/dl, HDL-colesterol: 42 mg/dl, LDL-colesterol: 335 mg/dl
y triglicéridos: 178 mg/dl. No presenta antecedentes personales de interés, y como
antecedentes familiares, su madre y hermano presentan hipercolesterolemia. En la
exploración física destacan los hallazgos observados en la imagen. Señale el diagnóstico
CORRECTO:

1. 1. Hipercolesterolemia familiar monogénica homocigótica.


2. 2. Déficit de lipoprotein-lipasa.
3. 3. Disbetalipoproteinemia familiar.
4. 4. Hipercolesterolemia monogénica heterocigótica.
Gráfico de respuestas
Comentario

Lo que podemos ver en la imagen no es otra cosa que xantomas, pero no era demasiado
importante saber reconocerlos, ya que el caso clínico lo dice todo. Es francamente difícil que una
persona de menos de 30 años tenga esas cifras de LDL colesterol sin que exista una causa
genética subyacente. De hecho, nos hablan de un problema parecido tanto en su hermano como
en su madre. Por este motivo, tendríamos que considerar las opciones 1, o 4, que son las que
podrían justificar estos hallazgos. Entre ellas, la opción 1 quedaría descartada, porque en ella
encontraríamos cifras todavía superiores. Recuerde que, en la herencia poligénica, aparte de una
susceptibilidad a la que predisponen múltiples genes, influyen factores ambientales, por lo que el
patrón de transmisión observado no sería tan estrecho.

Esperamos que nadie haya considerado las opciones 2 y 3... Si alguien lo ha hecho, debería
repasar el tema de las dislipemias, ya que en estas patologías hubiésemos encontrado
hipertrigliceridemia, y este paciente no la tiene.(R4)

405. Señale la opción CORRECTA en relación al diagnóstico realizado:

!
!
!
!
1. 1. Los fármacos de elección son los fibratos.
2. 2. Suele afectar a menos de un 10% de los familiares.
3. 3. Se suelen emplear en su tratamiento dosis elevadas de estatinas.
4. 4. El ezetimibe no tiene ninguna utilidad en esta patología.
Gráfico de respuestas
Comentario

Dado que se trata de una hipercolesterolemia heterocigota, la herencia debe ser autosómica
dominante... Y el tratamiento precisará fármacos dirigidos a disminuir el colesterol LDL.
Lógicamente, para esto no vamos a utilizar fibratos, cuya principal utilidad es disminuir los
triglicéridos. En cambio, las estatinas encontrarían aquí una clara indicación. Probablemente
tendremos que utilizar una muy potente. Recuerde que la de mayor potencia, hoy día, es la
rosuvastatina. Antiguamente era la atorvastatina, pero fue superada.(R3)

406. Uno de los siguientes no contribuye al diagnóstico de intersexualidad:

1. 1. Anamnesis.
2. 2. Examen clínico.
3. 3. Triple test (HCG, AFP, estrógeno no conjugado).
4. 4. Ecografía.
Gráfico de respuestas
Comentario

Los individuos con una discordancia entre los genitales externos, sexo cromosómico y gonadal se
clasifican dentro del grupo de trastornos del desarrollo sexual (TDS).

Para su diagnóstico, herramientas imprescindibles serían la anamnesis, el examen clínico y una


ecografía para valorar la relación fenotípica y gonadal del individuo.

La determinación de HCG, AFP y estrógenos no conjugados de la evalución gestacional por el


contrario, corresponde a la determinación de marcadores bioquímicos del primer y segundo
trimestre para el diagnóstico prenatal de cromosomopatías, por lo que, no sería útil para el
diagnóstico de la intersexualidad.(R3)

407. En relación al tratamiento y evolución de la GN por cambios mínimos, todos los


postulados son verdaderos, EXCEPTO:

1. 1. Las recidivas al disminuir o suspender la terapia se presentan en un 50% de los casos.


El 95% de los niños (pacientes menores de 16 años) desarrollan remisión completa de la
2. 2.
proteinuria a las 8 semanas de iniciado el tratamiento.
3. 3. No existe remisión de la proteinuria, sin tratamiento.
En caso de recidivas múltiples los pacientes pueden responder a terapias con
4. 4.
ciclofosfamida o clorambucil.
Gráfico de respuestas
Comentario

Esta pregunta se puede responder sin mayor dificultad si se tienen unos conceptos claros sobre el
pronóstico y evolución de la GN por cambios mínimos. El resumen es que el pronóstico es
excelente, pero son muy frecuentes las recidivas. Concretamente, en niños responden al
tratamiento con corticoides más del 95% de los pacientes, sin embargo, en adultos sólo lo hacen el
50%. El pronóstico a largo plazo es excelente, con altos porcentajes de supervivencia a los 15

!
!
!
!
años. Recordamos que la causa más frecuente de muerte es la peritonitis por neumococos. Sin
embargo, las recidivas son muy frecuentes, aproximadamente en el 50-60% de los casos, tanto en
niños como en adultos. Éstas se pueden tratar con inmunosupresores como clorambucil,
ciclofosfamida, ciclosporina y levamisol. La respuesta falsa es la 3, ya que existe remisión
espontánea en el 30-40% de los casos de aparición en la infancia, siendo las remisiones en
adultos menos frecuentes.(R3)

408. Señale qué alteración radiográfica NO se observa en las sacroilíacas de la


espondilitis anquilosante:

1. 1. Sindesmofitos.
2. 2. Esclerosis.
3. 3. Erosiones.
4. 4. Estrechamiento de la interlínea.
Gráfico de respuestas
Comentario
La sacroileitis es un hallazgo imprescindible para el diagnóstico de la espondilitis anquilosante. En
la fase inicial (sacroileitis grado I) se aprecia pseudoensanchamiento del espacio articular de la
sacroiliaca. Posteriormente la reacción del hueso origina estrechamiento del espacio articular con
erosiones y esclerosis (sacroileitis grado II). En la fase posterior llegan a establecerse puentes
óseos entre la vertiente sacra y la iliaca (sacroileitis grado III). La fase final (sacroileitis grado IV) se
define por la anquilosis total de la articulación. Los sindesmofitos, que son típicos de la
enfermedad, no se localizan en la sacroiliaca sino en las vértebras.(R1)

409. Paciente de 18 años de edad, secretaria, refiere que ha ovulado, todos los siguientes son
evidencias de que ha ocurrido la ovulación, EXCEPTO:

1. 1. Embarazo.
2. 2. Aumento de temperatura corporal basal.
3. 3. Catamenia.
4. 4. Endometrio secretor.
Gráfico de respuestas
Comentario

Pregunta fácil del ciclo menstrual.

Debe reconocer todo lo que sucede posterior a la ovluación, con lo que la respuesta 1, 2 y 4 son
posibles. La menstruación ocurre al principio del ciclo.(R3)

Obsérva la siguiente figura.

!
!
!
!

410. Paciente de 52 años, operada de colecistectomía electiva, 20 horas después de la


cirugía presenta temperatura de 38°C pulso de 110 y frecuencia respiratoria de 24, La
causa mas frecuente de la situación actual seria:

1. 1. Infección de herida.
2. 2. Flebitis.
3. 3. Atelectasia.
4. 4. Infección urinaria.
Gráfico de respuestas
Comentario

La causa más frecuente de fiebre postoperatoria en las primeras 24 horas son las atelectasias
(opción 3). Se deben prevenir mediante la fisioterapia respiratoria y controlando el dolor.(R3)

411. La neumonía lipoidea es una enfermedad ocasionada por:

1. 1. La infección vírica.
2. 2. Embolia grasa en pacientes politraumáticos.
3. 3. Aspiración repetida de aceites minerales (laxantes, gotas nasales).
4. 4. Déficit enzimático del metabolismo lipídico.
Gráfico de respuestas
Comentario
La neumonía lipoidea es una enfermedad pulmonar por depósito de lípidos, habitualmente tras
aspiraciones repetidas de aceites minerales (laxantes, gotas nasales o "comedores de fuego").
Clínicamente el paciente presenta disnea y radiológicamente es característico el patrón alveolar en
las zonas declives del pulmón. El tratamiento es de soporte y, en los casos graves, se han
empleado los esteroides.(R3)

412. El tratamiento de elección en una vaginitis candidiásica es:

1. 1. Metronidazol tópico.
2. 2. Metronidazol vía oral.
3. 3. Nistatina tópico.
4. 4. Clindamicina oral.
Gráfico de respuestas
Comentario

!
!
!
!
En caso de infección por Candida el tratamiento puede realizarse con nistatina o fluconazol.
Recuerde que el metronidazol y clindamicina son anaerobicidas y son de utilidad en Gardnerella.
Respuesta 3 correcta.(R3)

413. La precarga cardíaca aumenta cuando se produce:

1. 1. Una disminución del flujo sanguíneo coronario.


2. 2. Una disminución de la complianza venosa.
3. 3. Un aumento de la contractilidad miocárdica.
4. 4. Un aumento de la permeabilidad capilar.
Gráfico de respuestas
Comentario

La precarga se identifica con el volumen telediastólico ventricular. Simplificando, todo lo que haga
que los ventrículos “se llenen más de lo normal”, va a conseguir un incremento en la precarga lo
que dentro de los márgenes que establece la Ley de Frank-Starling conlleva un incremento de la
“fuerza de contracción” de la fibra miocárdica.

Así, para aumentar la precarga pueden ser útiles:

•! Incrementar la volemia (expansión de volumen).


•! Aumentar el tono venoso (contracción de la musculatura lisa que existe en la pared
venosa, lo que se puede identificar como un descenso en la “compliance” venosa).
•! Posición de Trendelemburg para incrementar el retorno venoso.
•! La contracción de los músculos de las extremidades que “comprimen” las venas
aumentando el retorno venoso (“bomba muscular”).
•! Disminuir la frecuencia cardíaca (para aumentar el tiempo de llenado ventricular).
•! Aumentar la fuerza de contracción auricular.
•! Mejorar la distensibilidad ventricular (con fármacos lusotropos como betabloqueantes y
calcioantagonistas).

La figura muestra un esquema de los determinantes de la función miocárdica.(R2)

Determinantes de la función cardíaca

!
!
!
!

414. ¿Cuál de las siguientes afirmaciones es cierta respecto al electrocardiograma en los


pacientes con EPOC?:

1. 1. Es normal en más de la mitad de los casos.


2. 2. La P pulmonale se observa en las derivaciones inferiores
3. 3. Las arritmias más frecuentes son las ventriculares.
4. 4. El bloqueo incompleto de rama derecha es infrecuente.
Gráfico de respuestas
Comentario

Esta es una pregunta de dificultad moderada sobre el EPOC pero que le exige conocimientos de
cardiología que hacen que la pregunta se complique más. Lo que debemos tener claro es que en
un paciente EPOC tiene lugar un proceso que comienza con el estado de hipoxia crónica, que
provoca un reflejo vasoconstrictor en el árbol vascular pulmonar que termina estableciendo unos
cambios irreversibles en la pared de los vasos. Esta es la causa de la hipertensión pulmonar que
desarrollan estos pacientes y es esta hipertensión la que somete al corazón derecho a una
sobrecarga de trabajo. En sus grados más extremos se llega a desarrollar cor pulmonale. Por ello
la mayoría tienen alteraciones electrocardiográficas, siendo consecuencia de esta sobrecarga del
corazón derecho: eje eléctrico a la derecha, P pulmonal que se percibe sobre todo en la derivación
II, bloqueo de rama derecha y arritmias auriculares (típicamente el flutter).(R2)

415. ¿Cuál de las siguientes manifestaciones obliga a descartar infección por VIH?

1. 1. Dermatomicosis extensas.
2. 2. Trombocitopenia autoinmune.
3. 3. Anemia ferropénica.
4. 4. Cáncer digestivo en persona joven.
Gráfico de respuestas
Comentario

!
!
!
!
La trombopenia puede ser una manifestación temprana de la infección por VIH. Habitualmente no
produce manifestaciones clínicas y mejora con el tratamiento antirretroviral. Cuando produce
clínica recuerda a la púrpura trombopénica idiopática (respuesta 2 correcta), con anticuerpos
antiplaquetarios circulantes. De hecho, en el capítulo de Hematología de nuestro Manual, se
explica que, ante una trombopenia de origen inmunológico, se debe descartar siempre la infección
por VIH, por esta razón. No obstante, en algunos pacientes puede apreciarse trombopenia por
efecto citopático directo del VIH sobre los megacariocitos, aunque esta situación es mucho menos
probable.

En cuanto al resto de las opciones, la que más dudas podría plantear es la 1. Existen muchas
dermatosis que son más frecuentes y/o graves en pacientes infectados por el VIH, como la
dermatitis seborreica, la foliculitis eosinofílica, la psoriasis, infecciones víricas (herpes simple,
herpes zóster, molusco contagioso). Sin embargo, las dermatofitosis no son una asociación típica
de este tipo de pacientes, por lo que esta opción no sería correcta.(R2)

416. ¿Cuál de las siguientes afirmaciones respecto al calostro es VERDADERA?

1. 1. Contiene más proteínas que la leche madura.


2. 2. Contiene más grasa que la leche madura.
3. 3. Contiene más lactosa que la leche madura.
4. 4. Contiene menos minerales que la leche madura.
Gráfico de respuestas
Comentario

El calostro es la secreción mamaria del final del embarazo, hasta los primeros 2-4 días después del
parto.

Contienemás proteínas y mineralesque la leche madura, es más denso que ella, y tiene menos
grasas e hidratos de carbono.

Además contiene una serie de factores inmunitarios importantes para el lactante; los de tipo
humoral, por tratarse de una secreción, son Igs de tipo A.(R1)

417. En los hijos de madres con infección genital por Ureaplasma urealitycum, ¿qué dos
patologías relacionadas con este micoplasma genital podremos encontrar con más
frecuencia?:

1. 1. Neumopatía crónica e infección del SNC.


2. 2. Neumopatía aguda y conjuntivitis.
3. 3. Sepsis e infección SNC.
4. 4. Neumonía aguda y neumopatía crónica.
Gráfico de respuestas
Comentario

El U. urealyticum provoca una corioamnionitis clínicamente silenciosa que aumenta el riesgo de


muerte fetal y prematuridad. Este agente patógeno puede aislarse en la tráquea, sangre y LCR de
los pequeños. Se especula sobre el papel de estos micoplasmas en el desarrollo de neumopatía
crónica; más constatada está su implicación en el desarrollo de meningitis hemorrágica.(R1)

418. Masculino de 34 años que ingresa por astenia intensa, fiebre elevada y artromialgias
de 1 mes de evolución a los que se añade dolor en región inguinal derecha que aumenta

!
!
!
!
con la deambulación. En la exploración física destaca dolor a la palpación en sacroilíaca
derecha y L5. En los exámenes de laboratorio presenta 5900 leucocitos con 1200
monocitos, hematocrito 44%, plaquetas 350,000, VSG 80, los hemocultivos fueron
negativos. Rx tórax y eco abdominal normales. Rx de sacroilíacas demuestra imagen
compatible con sacroileítis derecha. ¿Qué actitud le parece a usted la más CORRECTA?

1. 1. Administrar teicoplanina de forma empírica.


2. 2. Solicitar hemocultivos para Kingella.
Solicitar hemocultivos para microorganismos de crecimiento lento e instaurar tratamiento
3. 3.
empírico para Brucella.
4. 4. Solicitar serología para Borrelia burgdorferi.
Gráfico de respuestas
Comentario

Es importante acostumbrarse a sintetizar lo ESENCIAL de los casos clínicos, para que un exceso
de información no nos lleve en la dirección equivocada. En este caso podría ser algo así: cuadro
SUBAGUDO SISTÉMICO que acaba focalizándose en SACROILIACA derecha. Como ven, la
clínica es lo que comanda la orientación diagnóstica, el resto de laboratorios es relativamente
prescindible y no debe distraernos demasiado (si bien vale la pena tomar nota del "hemocultivo
negativo"). En base a este planteamiento, la actitud más acertada es la de la opción 3. No hay
motivos para sospechar Kingella o enfermedad de Lyme (opciones 2 y 4). Tampoco estamos
sospechando algún gram positivo (un S. aureus por ejemplo) que justificara de algún modo la
opcion 1.(R3)

419. Señalar cuál de las manifestaciones siguientes NO es típica de la sarcoidosis:

1. 1. Adenopatías hiliares.
2. 2. Adenopatías mediastínicas.
3. 3. Masas redondeadas intrapulmonares.
4. 4. Derrame pleural.
Gráfico de respuestas
Comentario

Tema poco importante para fines del ENARM. La radiografía de tórax casi siempre es patológica
en la sarcoidosis debido a la frecuente afectación pulmonar. Lo más frecuente y característico es el
aumento de los ganglios hiliares bilaterales que a menudo van acompanadas de aumento de los
ganglios paratraqueales derechos. La afectación parenquimatosa típicamente consisten en
infiltrados reticulonodulares difusos. Los nódulos pulmonares que se asemejan a la enfermedad
metastásica también pueden encontrarse aunque son raros. Hallazgos pocos frecuentes son las
calcificaciones hiliares en "cáscara de huevo", el derrame pleural, la cavitación, atelectasia,
hipertensión pulmonar y neumotórax.(R4)

420. Cual es la edad mas recomendada para el tratamiento de la criptorquidia.

1. 1. A los 6 meses.
2. 2. Entre 1 y 2 años.
3. 3. Después de los 5 años.
4. 4. Todos los anteriores.
Gráfico de respuestas
Comentario

!
!
!
!
Pregunta sencilla sobre el tratamiento de la criptorquidea. La respuesta correcta es la número 2:
entre 1 y 2 años, ya que antes del año podría descender el testículo y después de los 2 años se
incrementa el riesgo de neoplasia testicular.(R2)

421. En una paciente con diagnóstico de cáncer de endometrio, estadio II y grado de


diferenciación celular G2, ¿cuál es el tratamiento adyuvante de elección?:

1. 1. Quimioterapia.
2. 2. Radioterapia externa y quimioterapia.
3. 3. Radioterapia externa y braquiterapia.
4. 4. Quimioterapia y braquiterapia.
Gráfico de respuestas
Comentario
El estadio II del cáncer de endometrio, además del tratamiento quirúrgico que consiste en una
histerectomía radical con linfadenectomía pélvica, necesita como coadyuvancia, braquiterapia
localizada y radioterapia externa, siendo la quimioterapia poco relevante dada su baja respuesta en
este tipo de tumores. La quimioterapia se reserva para el tratamiento de estadios avanzados del
cáncer de endometrio junto con la radioterapia como tratamiento de rescate.(R3)

422. Paciente de 26 años, fumadora, sin


hijos, sin pareja estable, es remitida a su consulta de ginecología por citología positiva
para HSIL (lesión intraepitelial de alto grado). Usted realiza una colposcopía ante dicho
hallazgo, obteniéndose la imagen que se muestra. ¿Cuál es la conducta más indicada a
continuación?

1. 1. Tratamiento con antiinflamatorios locales.


2. 2. Estudio de extensión de la enfermedad.
3. 3. Biopsia de la lesión.
4. 4. Conización cervical.
Gráfico de respuestas

!
!
!
!
Comentario

Pregunta de dificultad media acerca del manejo de las displasias cervicales (simplemente
recordando el algoritmo se contesta la pregunta).

Ante una citología positiva debemos confirmarlo mediante una colcoscopía (tal como comentan en
el enunciado de la pregunta). La imagen corresponde a epitelio acetoblanco (mediante uso de
ácido acético las lesiones se tiñen de blanco, por lo que el siguiente paso será biopsiar la lesión e
estadificar (recuerde que entre los hallazgos colposcópicos anormales se encuentra el epitelio
acetoblanco, epitelio yodonegativo, cambios de coloración o leucoplasia, neoformación vascular y
superficie irregular con pérdida de epitelio normal). Lógicamente, no se realizará directamente una
conización sin confirmación histológica previa. La citología (y éste es el dato que nos dan en el
enunciado) es una técnica muy útil como screening, pero tiene sus limitaciones y, antes de tomar la
decisión terapéutica definitiva, necesitamos una muestra histológica (biopsia), que es una prueba
más específica, para asegurar el resultado anormal.(R3)

423. Con respecto a la patología que presenta la paciente ¿cuál de los siguientes
hallazgos NO sería sugestivo de patología cervical?

1. 1. Eritroplasia cervical.
2. 2. Áreas yodonegativas.
3. 3. Superficie ulcerada.
4. 4. Leucoplasia.
Gráfico de respuestas
Comentario

A continuación, le recordamos los hallazgos colposcópicos anormales:

1.- Epitelio acetoblanco.

2.- Epitelio yodonegativo.

3.- Cambios de coloración (leucoplasia).

4.- Neoformación vascular.

5.- Superficie irregular, con pérdida del epitelio normal, con ulceración.

La ectopia o eritroplasia cervical consiste en la presencia de epitelio cilíndrico por debajo del orificio
cervical externo, visible en la colposcopía desde la vagina, quedando en contacto con el medio
vaginal hostil. Aunque la mayor parte de las veces son lesiones asintomáticas, pueden originar
leucorrea y hemorragia postcoital. El diagnóstico se realiza mediante colposcopía, y pueden
eliminarse si producen clínica (a veces produce sangrado). No obstante, no se relaciona con
patología cervical maligna (respuesta 1 correcta).(R1)

424. La anhedonia total se relaciona fundamentalmente con:

1. 1. Depresión.
2. 2. Fobias sociales.
3. 3. Ataques de pánico.

!
!
!
!
4. 4. Personalidad antisocial.
Gráfico de respuestas
Comentario

Muy evidente; anhedonia es la incapacidad para experimentar placer y eso es típico de la


depresión.

Los síntomas de la depresión incluyen las alteraciones para experimentar placer que, pese a que
se pueden ver en otras enfermedades, cuando alcanzan un grado extremo (anhedonia total)
sugieren el diagnóstico de depresión. Además, en la manía y en la personalidad antisocial NO se
da este síntoma.(R1)

425. ¿Cuál de estas descripciones sobre el cáncer de mama y el oncogén Her-2 (neu) es
INCORRECTA?

1. 1. Se sobreexpresa en un 20-25% de los cánceres de mama.


2. 2. Los cánceres de mama que sobreexpresan el oncogén Her-2 tienen mejor pronóstico.
3. 3. La sobreexpresión de Her-2 se ha relacionado con resistencia al tratamiento antihormonal.
4. 4. Existen datos que señalan que se benefician del tratamiento con antraciclinas.
Gráfico de respuestas
Comentario

Este oncogén se sobreexpresa en un 20-25% de los cánceres de mama. Está asociado a un peor
pronóstico del mismo, entre otros factores porque se asocia a una peor respuesta a la
hormonoterapia. Responde y se beneficia al tratamiento con antraciclinas.

El trastuzumab (Herceptin) es un anticuerpo monoclonal empleado en aquellos pacientes que


sobreexpresan el gen c-erb-B2 (Her-2/neu), para el tratamiento del cáncer de mama
metastático.(R2)

426. En un paciente en el que sospecha reflujo vesicoureteral, ¿qué prueba diagnóstica


indicaría?:

1. 1. Cistrografía isotópica.
2. 2. USG.
3. 3. Cistouretrografía miccional.
4. 4. Cistoscopía.
Gráfico de respuestas
Comentario

La cistouretrografía miccional seriada (CUMS) sirve para descartar la presencia de reflujo


vesicoureteral. Está indicada en todos los niños y niñas menores de 5 años con infección del tracto
urinario, los afectos de una IVU febril (independientemente del sexo), en todo varón con una IVU y
en las niñas en edad escolar que hayan presentado dos o más episodios.(R3)

427. Las hemorroides se consideran de tercer grado cuando:

1. 1. Son sangrantes.
2. 2. Se prolapsan temporalmente en el momento de la defecación.
3. 3. Se prolapsan indefinidamente y han de reducirse manualmente.

!
!
!
!
4. 4. Se trombosan.
Gráfico de respuestas
Comentario

Las hemorroides internas se producen por dilatación del plexo venoso hemorroidal interno formado
por venas rectales superior y media.

Se clasifican en IV grados según la intensidad del prolapso.

El grado I permanece en recto y su tratamiento es conservador.

El grado II prolapsa a través del ano cuando el paciente puja, reduciéndose espontáneamente y se
trata con ligadura de vaina de caucho o bien esclerosis.

El grado III prolapsa cuando el paciente puja pero es necesaria la restitución manual y el
tratamiento es con ligadura de vaina de caucho.

El grado IV es el prolapso persistente cuyo tratamiento es la hemorroidectomía.(R3)

428. ¿Qué marcador nos indica una hepatitis aguda B?:

1. 1. AgHBs.
2. 2. AcHBc-IgM.
3. 3. AcHBc-IgG.
4. 4. AcHBs.
Gráfico de respuestas
Comentario
La presencia de AgHBs indica que el paciente tiene virus en su cuerpo pero son los anticuerpos
contra la proteína del CORE de tipo IgM los que nos indican infección aguda ya que los primeros
son positivos tanto en la hepatitis aguda como crónica.(R2)

429. Un paciente con EPOC moderado presenta, en los últimos 3 días, aumento de la tos
habitual y esputo purulento. Acude a Urgencias porque, en las últimas 12 horas, la disnea
se ha hecho de pequeños esfuerzos. Se le realiza una gasometría arterial que presenta
los siguientes valores: pH 7.29, PaO2 50 mmHg, PaCO2 50 mmHg. Usted decide su
ingreso. De las siguientes actitudes terapéuticas, ¿cuál NO estaría indicada de entrada?:

1. 1. Oxigenoterapia con fracciones inspiratorias bajas.


2. 2. Ventilación mecánica.
3. 3. Aminofilina intravenosa.
4. 4. Antibióticos de modo empírico.
Gráfico de respuestas
Comentario

No estaria indicada de entrada la Ventilación mecánica, pues es una Insuficiencia respiratoria


descompensada, sin perdida o deterioro de la conciencia, inestabilidad hemodinámica, en la que
aun tenemos margen de maniobra para intentar con oxigenoterapia a fracciones bajas controlar la
hipoxemia, intentando no aumentar mucho la retencion del CO2. Si la administración de oxígeno

!
!
!
!
no lograra un adecuado equilibrio entre la paCO2 y la PaO2 entonces si estaria indicada la
utilización del VM. El resto de fármacos se utilizan en las descompensaciones del EPOC.(R2)

430. En relación con las vacunas y el embarazo, señale cuál de las siguientes vacunas
podrían administrarse en la gestación en caso de ser necesarias:

1. 1. Sarampión.
2. 2. Fiebre amarilla.
3. 3. Rubéola.
4. 4. Hepatitis B.
Gráfico de respuestas
Comentario
Durante la gestación están contraindicadas las vacunas con virus vivos atenuados (sarampión,
rubéola, parotiditis y fiebre amarilla), mientras que están permitidas las vacunas de la gripe,
poliomielitis y hepatitis B si fueran necesarias, aunque no su uso sistemático. Las vacunas con
toxoides son seguras durante la gestación.(R4)

431. ¿Cuál de los siguientes tumores ováricos NO es tumor de las células germinales?:

1. 1. Tumor de células claras.


2. 2. Teratoma inmaduro.
3. 3. Tumor del seno endodérmico.
4. 4. Coriocarcinoma.
Gráfico de respuestas
Comentario

Recuerde los tres tipos principales de tumores ováricos:

-Del epitelio de superficie.

-Germinales.

-De los cordones sexuales-estroma.

La respuesta 1 es la única que no corresponde a un tumor germinal, pues el tumor de células


claras es un tumor del epitelio de superficie, junto con el seroso, el mucinoso, el endometrioide o el
tumor de Brenner.(R1)

!
!
!
!

432. After initial management, which of the following


is the most appropriate next step with this patient?

1. 1. Urgent MRI.
2. 2. Vascular exploration by a surgeon.
3. 3. Monitoring the patient by intensive care unit.
4. 4. Assessment of common peroneal nerve motor function by a neurosurgeon.
Gráfico de respuestas
Comentario
La complicación más temible de una luxación de rodilla es la lesión inmediata (laceración,
desgarro) o diferida (trombosis tras lesión de la íntima arterial) de la arteria poplítea. Es
imprescindible la realización de un estudio vascular (habitualmente con Eco-Doppler), y la
valoración por Cirugía Vascular de la pierna del paciente. Un porcentaje alto de casos de luxación
de rodilla tiene lesión arterial, y un porcentaje alto de lesiones arteriales no se recupera a pesar de
una cirugía de revascularización, necesitándose finalmente una amputación de la pierna.(R2)

433. Paciente que presenta desde hace algunos meses molestias vagas abdominales,
sensación de plenitud gástrica precoz, diarrea acuosa y debilidad muscular marcada. En
un ECG que se le realizó recientemente por los equipos médicos de su empresa como
estudio rutinario destaca la presencia de onda U y aplanamiento de la onda T. ¿Cuál de
las siguientes cree que sería MENOS probable en este caso?

1. 1. Masa pancreática detectada en la ecografía abdominal.


Hiperglucemia debida a la destrucción insular pancreática producida por la proliferación
2. 2.
de la masa tumoral.
3. 3. Presencia de metástasis al diagnóstico.
4. 4. Aumento de la concentración plasmática de VIP y volumen de heces de al menos 1l/ día.
Gráfico de respuestas
Comentario

Los tumores de los islotes pancreáticos es un tema poco preguntado. Basta con que sepa
reconocerlos en forma de caso clínico.

!
!
!
!
Aquí nos describen un caso clínico de vipoma.

El vipoma deriva de las células D pancreáticas y suelen ser de gran tamaño (responsable de la
sensación de plenitud gástrica), de comportamiento maligno y con metástasis en el 60% de los
casos al momento del diagnóstico.

Las manifestaciones clínicas de este tumor son diarrea secretora (cólera pancreático), debilidad,
hipercalcemia, hipofosfatemia, hipopotasemia e hipocloremia.

El tratamiento consiste en la extirpación quirúrgica; sin embargo, dada la alta frecuencia de


metástasis, a veces esto no es posible.(R2)

434. La forma más frecuente de obesidad en el niño se debe a:

1. 1. Síndrome de Cushing.
2. 2. Síndrome de Prader-Willi.
3. 3. Sobrealimentación.
4. 4. Tumor cerebral.
Gráfico de respuestas
Comentario

Esta pregunta sobre las causas de obesidad infantil es sencilla y no tiene especial importancia.

Algunas de las enfermedades asociadas a la obesidad infantil son los síndromes de Cushing,
Frölich (tumor hipotalámico), Prader Willi (hipogonadismo, manos y pies pequeños) si bien, éstas
enfermedades representan menos del 5% de casos de obesidad infantil. La mayoría de los casos
se debe a sobrealimentación.(R3)

435. Mujer de 52 años, cuya última menstruación fue hace más de un año, presenta una
hemorragia vaginal de varios días de evolución, sin ningún otro síntoma asociado. La
ecografía muestra un endometrio engrosado de 11 mm, sin áreas sólidas ni
hiperrefringentes. Se le practica una histeroscopia dignóstica con el resultado de
hiperplasia endometrial atípica. ¿Cuál es la conducta a seguir más adecuada?:

1. 1. Histerectomía simple.
2. 2. Histerectomía radical.
3. 3. Conducta expectante.
4. 4. Análogos de GnRH.
Gráfico de respuestas
Comentario
El caso clínico nos habla de una hiperplasia atípica lo cual implica que las células están
dividiéndose de una manera irregular, existiendo por tanto riesgo de degeneración maligna. De
hecho se considera la lesión premaligna del cáncer de endometrio. Su tratamiento debe ser
quirúrgico realizando histerectomía simple. No es necesaria la realización de una histerectomía
radical ya que ésta se reserva para el cáncer de cérvix o el cáncer de endometrio que invade el
cuello uterino.(R1)

436. A 5-year-old child is brought to the emergency department presenting with a 3-day
history of asthenia and vomiting. He also presented with low grade fever and one wet
diaper. Physical

!
!
!
!
examination shows 80/50 mmHg blood pressure, 95 bpm and a prolonged capillary filling
time. What should be done as a first-line measure?

1. 1. Intravenous infusion of glucose 5% .


2. 2. Blood transfusion.
3. 3. Infusion of 20 mL/kg of normal saline intravenously.
4. 4. Infusion of 20 mL/kg of 1/2 normal saline intravenously.
Gráfico de respuestas
Comentario
Infusion of 20 mL/kg of normal saline intravenously. Children are a group at high risk for
dehydration, which can have serious consequences. In this case, there is an identifiable cause
(vomiting) Basic treatment is rehydration with normal saline. The route of administration varies
depending on the severity. If dehydration is severe (as is this case) the route of choice is
intravenous.(R3)

437. Which is the current indication for treatment of multiple sclerosis with interferon
beta?

1. 1. Symptomatic treatment of outbreaks.


2. 2. Prevention of outbreaks in patients with relapsing-remitting clinical forms.
3. 3. Treatment for optic neuritis.
4. 4. Treatment of all clinical forms, but only in very advanced stages.
Gráfico de respuestas
Comentario

Pregunta directa sobre el tratamiento de la esclerosis múltiple y que le pueden preguntar en el


nacional.

El tratamiento de mantenimiento con IFN en la EM está indicado en las formas de curso remitente-
recurrente con el fin de disminuir el número y la intensidad de las mismas, aunque,
desafortunadamente, no modifica las secuelas ni la discapacidad previa.

En efecto, en la actualidad se dispone de interferón beta 1a y 1b, siendo este segundo más
inmunogénico (generación de anticuerpos neutralizantes) y peor tolerado al nivel de efectos
secundarios cutáneos.(R2)

438. Los cambios circulatorios en el embarazo normal incluyen todas, EXCEPTO:

1. 1. Aumento frecuencia cardiaca.


2. 2. Disminución resistencias vasculares.
3. 3. Disminución volumen sistólico.
4. 4. Aumento consumo oxígeno.
Gráfico de respuestas
Comentario

Pregunta sobre la adaptación cardiovascular de la mujer embarazada. Se produce un aumento de


la frecuencia cardiaca en un 15-20%. El gasto cardiaco se ve incrementado gracias al aumento de
la frecuencia cardiaca y del volumen sistólico que pasa de 66 a 86 ml por término medio; así, la
opción falsa es la 3. Las resistencias vasculares disminuyen gracias a la acción miorrelajante de la
progesterona y a la insensibilización a la angiotensina II que existe durante el embarazo. Como es

!
!
!
!
lógico pensar, habrá un mayor consumo de oxígeno, dado que el embarazo supone mayor
actividad a todo el organismo.(R3)

439. Uno de los siguientes hallazgos exploratorios precisa un estudio clínico completo.
Señálelo:

1. 1. Un RN con una frecuencia cardíaca de 130 latidos por minuto.


2. 2. Un niño de 2 años con una frecuencia cardíaca de 110 latidos por minuto.
3. 3. Un RN con un pulso de 260 por minuto, durante el llanto.
4. 4. Un niño de 2 meses, con una frecuencia cardíaca de 130.
Gráfico de respuestas
Comentario

Pregunta básca y bastnte rentable. Por motivos obvios, una frecuencia cardíaca tan alta ha de ser
estudiada: no puede corresponder a una taquicardia sinusal. La frecuencia cardíaca habitual en los
recién nacidos oscila entre 120 y 160 latidos por minuto; esta frecuencia cardíaca tan llamativa va
reduciéndose conforme el niño va medrando hasta alcanzar los valores del adulto a mediados de la
edad escolar.(R3)

440. El principal factor que determina la hipersecreción de las glándulas sebáceas es:

1. 1. Los niveles altos de estrógenos circulantes.


2. 2. La acción de la enzima 5-alfa reductasa sobre los estrógenos.
3. 3. Los niveles de LDH de la glándula sebácea.
4. 4. Los niveles de enzima 5-alfa reductasa en la glándula sebácea.
Gráfico de respuestas
Comentario

Pregunta muy compleja, de estudio en profundidad de Segunda Vuelta. La secreción de las


Glándulas Sebáceas se denomina Secreción Holocrina y se produce mediante lisis celular. Está
mediada hormonalmente por Andrógenos. En la célula, la enzima 5-alfa reductasa actúa sobre los
andrógenos (testosterona y dehidroepiandrosterona en el hombre y androstenediona en la mujer)
produciendo 5- alfa dihidrotestosterona, que es la sustancia que realmente activa la secreción. La
respuesta correcta es la 4, ya que esta enzima es la que actúa limitando la activación de la
secreción.(R4)

441. Todas las siguientes son causas de síndrome nefrítico agudo, EXCEPTO:

1. 1. Amiloidosis renal.
2. 2. Crioglobulinemia mixta esencial.
3. 3. Nefropatía mesangial IgA.
4. 4. Glomerulonefritis mesangiocapilar.
Gráfico de respuestas
Comentario

Los síndromes nefrológicos por sí solos, aportan pocas preguntas en el ENARM, pero si tenemos
en cuenta que nos ayudan a resolver gran parte de las glomerulonefritis y de las enfermedades
sistémicas, resulta ser un tema muy rentable. La lísta típica de síndrome nefrótico que debes
recordar es: GN membranosa, GN de cambios mínimos, nefropatía diabética, amiloidosis, GN focal
y segmentaria, GN membranoproliferativa en 2/3 de los casos. El resto de los casos probablemente
produzcan síndrome nefrítico, anomalías urinarias persistentes, IRA?(R1)

!
!
!
!
442. Una paciente de 52 años, fumadora, consulta por síndrome miccional. Refiere que
su médico de cabecera lleva más de dos años haciéndole urocultivos que siempre han
resultado negativos. Ha tomado anticolinérgicos sin mejoría y también varias pautas
antibióticas sin resultado. Además refiere polaquiuria muy importante y limitante con
nicturia de 5 veces cada noche y asegura estar desesperada. En este caso:

1. 1. La biopsia vesical aleatoria nos permitirá descartar un carcinoma in situ de vejiga.


Para el diagnóstico de esta patología es útil solicitar un cultivo específico de hongos en
2. 2.
orina.
3. 3. La sintomatología es totalmente específica y no precisa prueba alguna para el diagnóstico.
4. 4. La CUMS es de gran utilidad.
Gráfico de respuestas
Comentario

El cuadro descrito puede corresponderse a una cistitis intersticial pero como su diagnóstico es de
exclusión deberemos realizar biopsia vesical para descartar ca in situ y PCR para descartar TB
urogenital. En algunas pacientes con cistitis intersticial la biopsia muestra típicamente un infiltrado
intersticial de mastocitos.(R1)

443. Sólo una de las siguientes caracteristicas es propia del síndrome de Budd-Chiari:

1. 1. La mayor parte de los casos son de origen congénito.


2. 2. Su frecuencia es elevada en sujetos anticoagulados con dicumarínicos.
3. 3. Es una complicación frecuente en la cirrosis biliar primaria.
4. 4. Se acompaña de ascitis con alto contenido en proteínas (> 3 g/dl).
Gráfico de respuestas
Comentario

El síndrome de Budd-Chiari es una causa infrecuente de ascitis e hipertensión portal. Ésta es una
pregunta de dificultad media-alta.

Hemos de recordar que la ascitis que aparece en procesos que obstruyen el drenaje venoso
suprahepático (insuficiencia cardíaca congestiva, pericarditis constrictiva o el síndrome de Budd-
Chiari) se caracterizan por una elevada concentración proteica.

La causa más habitual es la trombosis de las venas hepáticas (generalmente en el contexto de una
policitemia vera), la hemoglobinuria paroxística nocturna o un estatus de hipercoagulabilidad
(incluida la toma de anticonceptivos orales); y, por tanto, el tratamiento suele incluir los
anticoagulantes.

En las oclusiones venosas hepáticas suele aparecer la hemorragia por varices con mayor
frecuencia que la ascitis.

La cirrosis biliar primaria afecta al tracto biliar, siendo rara la afectación vascular.(R4)

444. Respecto a la revascularización mecánica como tratamiento de la enfermedad


coronaria isquémica, ¿cuál de las siguientes afirmaciones es FALSA?:

1. 1. La mortalidad de la cirugía de by-pass coronario es inferior al 2%.

!
!
!
!
La angina desaparece en el 85% de los pacientes sometidos a cirugía de revascularización
2. 2.
coronaria completa.
Siempre que sea posible, se debe intentar la revascularización completa con injertos
3. 3.
venosos.
La estancia hospitalaria tras la ACTP es sensiblemente más corta que tras la intervención
4. 4.
quirúrgica.
Gráfico de respuestas
Comentario

Es una pregunta sencilla en la que lo único que tenemos que saber es que los injertos con arteria
son mejores. El flujo que proporcionan y las dificultades quirúrgicas son similares a las de los
injertos venosos. Sin embrago, los injertos venosos presentan un 10-20% de oclusión al año de su
implante. Los años posteriores, la incidencia de oclusión es del 2% anual. Los injertos arteriales de
mamaria interna, por el contrario, permanecen permeables en un 90% a los 10 años de la
intervención. En la práctica, se suele reservar la arteria mamaria interna [normalmente la izquierda]
para la anastomosis con la arteria descendente anterior, utilizando injertos venosos de safena para
el resto de vasos afectados. De todos modos, la tendencia debe ser a intentar revascularizaciones
arteriales completas [esto es, que todos los puentes se hagan con arterias], para lo que además de
las arterias mamarias internas, se suele utilizar la arteria radial. Por tanto, la respuesta 3 es la
falsa. El resto de opciones son correctas, y nos exponen la efectividad y relativo bajo riesgo de la
cirugía de revascularización. Es evidente que la estancia hospitalaria después de una ACTP es
mucho menor que en el caso de la cirugía de revascularización (esternotomía media y circulación
extracorpórea en muchos casos).(R3)

445. Dentro del tratamiento no farmacológico de la HTA lábil se incluyen las siguientes
medidas dietéticas, EXCEPTO:

1. 1. Dieta sin sal.


2. 2. Ejercicio físico isométrico.
3. 3. Suspender el tabaco.
4. 4. Suspender la toma de anticonceptivos orales.
Gráfico de respuestas
Comentario

Son medidas no farmacológicas que disminuyen la PA:

- Restricción de la ingesta de sal

- Disminución de peso

- Ejercicio físico aeróbico

- Evitar el estrés

La suspensión de fármacos causantes de HTA, como los anticonceptivos orales, también ayuda a
controlar la PA. La supresión del tabaco no reduce los niveles de PA, pero reduce el impacto de la
HTA sobre los órganos diana. El ejercicio isométrico (levantar pesas) no se recomienda como
método para tratar la HTA, pues mal realizado puede incluso incrementarla (al realizar
Valsalva).(R2)

!
!
!
!
446. Señale cuál de los siguientes antibióticos no ofrece cobertura frente a Pseudomonas
aeruginosa:

1. 1. Ceftazidima.
2. 2. Vancomicina.
3. 3. Piperacilina-tazobactam.
4. 4. lmipenem.
Gráfico de respuestas
Comentario

De las opciones que nos presentan, la vancomicina es la alternativa que no ofrece cobertura
alguna frente a esta bacteria. Debe saber que la vancomicina es útil frente a grampositivos,
especialmente para los que presentan problemas de resistencia a otros antibióticos. El resto de las
opciones son claramente válidas.

Respecto al ciprofloxacino, merece la pena comentar un aspecto. Este antibiótico ofrece una
cobertura menor que, por ejemplo, el imipenem o la ceftazidim. Pero es el mejor antibiótico por vía
oral frente al género Pseudomonas, por lo que podemos considerarlo útil.(R2)

447. La artropatía por depósitos de cristales de pirofosfato cálcico dihidrato se ha


asociado con:

1. 1. Hiperparatiroidismo primario.
2. 2. Hemocromatosis.
3. 3. Hipomagnesemia.
4. 4. Todas las anteriores.
Gráfico de respuestas
Comentario

Los pacientes con depósito de cristales de pirofosfato cálcico son en su mayoría ancianos, sin
ninguna enfermedad metabólica asociada. Sin embargo, cuando estos cristales se depositan en
personas por debajo de los 55 años, debemos descartar la presencia de una enfermedad que
favorece su depósito. Entre estas entidades se encuentran las respuestas que nos dan en la
preguntan, así como la hipofosfatasia. Los parámetros de laboratorio séricos que se recomiendan
para descartar estas enfermedades son la determinación de calcio, fósforo, fostatasa alcalina,
magnesio, ferritina y saturación de transferrina. También las displasias epifisarias favorecen su
depósito. Existen algunos casos de formas hereditarias donde se han encontrado defectos
genómicos en el cromosoma 5p y en el 8q.(R4)

!
!
!
!

448. A 35-year-old woman who is


a primary school teacher presents to her physician complaining of a 6-month history of
hoarseness. She says that the symptoms were first intermittent and improved during the
weekends but they have become permanent for the past few weeks. Physical examination
and laryngoscopy are performed. The image shows the patient's vocal cords. Which of
the following is the most likely diagnosis?

1. 1. Right vocal cord polyp.


2. 2. Bilateral Reinke's edema.
3. 3. Vocal fold nodules.
4. 4. Laryngeal papillomatosis.
Gráfico de respuestas
Comentario

Si es profesora de educación infantil, no le sorprenda que esta mujer tenga que forzar la voz en el
trabajo. Ya tenemos la pista fundamental para pensar en unos nódulos “de los cantantes” o
nódulos vocales, que incluso podemos ver en la imagen asociada a esta pregunta. La respuesta
correcta, por tanto, es la 3.(R3)

449. Cual de las siguientes patologías considera más improbable en una paciente que
refiere pérdida visual en su OD, y presenta un defecto pupilar aferente relativo en dicho
ojo:

1. 1. Obstrucción de arteria central de la retina.


2. 2. Obstrucción de vena central de la retina.
3. 3. Neuropatía óptica isquémica.
4. 4. Catarata.
Gráfico de respuestas
Comentario

La presencia de un defecto pupilar aferente relativo (DPAR, o pupila de Marcus Gunn), es propia
de patología de la vía visual anterior (nervio óptico y retina). El defecto pupilar aferente relativo se
explora iluminando las pupilas de forma alterna. Al pasar a iluminar el lado enfermo, las pupilas
experimentarán una dilatación paradójica. El DPAR es propio de las neuropatías, pero también la
patología retiniana cuando es severa puede producirlo. Sin embargo una catarata por densa que
sea, no es capaz de producir un DPAR significativo.(R4)

!
!
!
!

!
!
!
!

!
!
!
!

!
!
!
!

!
!
!
!

!
!
!
!

!
!
!
!

!
!
!
!

!
!
!
!

!
!
!
!

!
!
!
!

!
!
!
!

!
!
!
!

!
!
!
!

!
!
!
!

!
!
!
!

!
!

Potrebbero piacerti anche